Sie sind auf Seite 1von 665

ENGINEERING

MATHEMATICS -I
SECOND EDITION

P.B. Bhaskar Rao


M.Sc., Ph.D.
Retd. Professor, Former Chairman, Board of Studies,
Department of Mathematics
Osmania University
Hyderabad

S.K.V.S.Sriramachary M. Bhujanga Rao


M.A., M.Phil., B.Ed. M.Sc., Ph.D.
Professor & Head (Retd.) Professor, Dept. of Mathematics
Department of Mathematics University College of Engineering
University College of Engineering (Autonomous)
(Autonomous) Director of Centre for Distance Education
Osmania University Osmania University
Hyderabad Hyderabad

BSP BS Publications
4-4-309, Giriraj Lane, Sultan Bazar,
Hyderabad - 500 095 A.P.
Phone: 040 - 23445688
e-mail: contactus@bspublications.net
Copyright 2008, by Publisher

All rights reserved.

No part of this book or parts thereof may be reproduced, stored in a i


retrieval syste'm or transmitted in any language or by any means,
electronic, mechanical, photocopying, recording or otherwise without
the prior written permission of the publishers,

Published by :

BSP BS Publications
=:;;;;= 4-4-309, Giriraj Lane, Syltan Bazar,
Hyderabad - 500 095 - A. P.
Phone: 040-23445688
e-mail: contactus@bspublications.net
www.bspublications.net

Printed at
Adithya Art Printers
Hyderabad.

ISBN: 978-81-7800-151-7
Contents

CHAPTER -1
Ordinary Differential Equations of
First Order and First Degree ..................................................... 1

CHAPTER -2
Linear Differential Equations with
Constant Coefficients and Laplace Transforms ...................... 69

CHAPTER-3
Mean Value Theorems and
Functions of Several Variables .............................................. 111

CHAPTER-4
Curvature and Curve Tracing ................................................ 213

CHAPTER-5
Application of Integration to
Areas, Lengths, Volumes and Surface areas ........................ 313

CHAPTER-6
Sequences of Series .............................................................. 385 _

CHAPTER-7
Vector Differentiation ............................................................. 475

CHAPTER-8
Laplace Transforms ............................................................... 623
"This page is Intentionally Left Blank"
1
Ordinary Differential Equations of
First Order and First Degree

1.1 Introduction
Differential euqtions play an important role in many applications in the field of
science and engineering, such as (i) problems relating to motion of particles
(ii) problems involving bending of beams (iii) stability of electric system, etc. For
example, Newton's law of cooling states that the rate of change of temperature of
a body varies as the excess temperature of the body to that of its surroundings. If
8(t) is the temperature of the body at time 't' and 8 0 is the temperature of the room
de
in which the body is kept, then dt gives the rate of change of temperature with

time.
de
dt = K(8 - 8 0) ; K is constant

Similarly Newton's second law of motion for a particle of mass m moving in a


straight line can be written as
d 2x
m dt 2 =F
Where m is the mass, x is the distance of the particle at time 't' measured from
a fixed origin and F the external impressed force.
2 Engineering Mathematics - I

A differential equation is an equation involving an unknown function and its


derivatives. Ifthere is only one independent variable and one dependent variable the
equation is called (Ill ordinary differential equation.
If there are more than one independent variable the equation is called a partial
differential equation as this involves partial derivatives.
For example:

d3y dy
4 +3x _ y=e( .... (a)
3
dx dx

d3y J4 ( d2YJ8 (dY) 12 6 _ 8


(-dx;3- + -dx-2 + -dx + Y -x .... (b)

.... (c)

.... (d)

.... (e)

.... (t)

.... (g)

The first four equations (a), (b), (c) and (d) are ordinary differential equations and
the remaining three are partial differential equations.
Order 0/ a differential equation: The order of a differential equation is the order
of the highest ordered derivative appearing in the equation.
Degree 0/ a differential equation: The degree of a differential equation is the
power to which the highest ordered derivative appears in the equation after clearing
the radicals if any.
Ordinary Differential Equations of First Order and First Degree 3

In the above examples:


Example: 1.1(a) is a differential equation of order 3 and degree I.
Example: 1.1(b) is of third order and fourth degree differential equation.
Example: 1.1(c) is a second order, first degree differential equation.
Example: 1.1(d) is a second order, second degree ditferential equation.

1.2 Example
Formation of an ordinary D.E :
The differential equations ar~ formed by eliminating all the arbitrary constants
th~t areinvolved in the functional relationship between the dependent and independent
variables.
For example:
y = cx2 + c 2 where c is an arbitrary constant. .... (I)
To eliminate 'c': (only one constant)
dv
From(l) _0 = c.2x+ 0
dx
I (~V
c= -
2x dx
Substitution of c in (I) gives

y = _I dy x2+ _1_(dy )2
2x d\: 4x 2 d);

d
~ )2 + 2x 3 2d - 4x2y = 0
( dx dx
is the required D.E and y = cx2 + c 2 is called the solution of the D.E.
Note:
Depending on the number of constants in the given equation differentiate it as
many number oftimes successively. Then the elimination of the arbitrary constants
from the resulting equations and the given equation gives the required differential
equation whose order is equal to the number of constants.

1.3 Example
Eliminate the arbitrary constants a, b from xy + x 2 = aeX + be-X and form the
differential equation.
4 Engineering Mathematics - I

Solution:
The given equation is
xy + .x2 = ae-'" + be-x ..... ( I)
The number of arbitrary constants is two. Differentiating (I) w.r., to 'x' two
times successively.
dy _
x dx + Y + 2x = ae-' - be-x .....(2)

d2y ely dy
X ---2 + - + - + 2.1 = aex + be-x .... (3)
d-c dx dx
From (I), (2) and (3) el imination of a, b gives the D.E.
from (I) and (3) we get
d l )' 2dy . .
x --? + - - + 2 = xy + x 2 IS the requIred D_E.
dx- dx

1.4 Example
Form the differential equation by eliminating the constants a and b from
a.x2 + by =
1
Solution:
Differentiating
ax2 + by = I w.r.t 'x' .... (I)
dy
2ax+ 2by- = 0 .... (2)
d-c
Again differentiating wr.t., 'x'
d 2y dy dy
2a+2by - ? +2b-.- =0 .... (3)
dx- dx dx
Elimination of a, b from (I), (2) and (3) gives
x-? i -I
x yYl 0 =0
(Y.h + yl2 0

Expanding the determinant we get


2 y
x d y +x(d )2 _ y(dY)=o
2
dx dx dx
Ordinary Differential Equations of First Order and First Degree 5

1.5 Example
Form the differential equation by eliminating the constants from
y = a secx + b tan x
Solution
Given equation is
y= asecx + btanx .... (I)
Differentiating w.r. to 'x'
dy
= asecx tanx + bsec 2x .... (2)
dx

dy
= secx[a tan x + b sec xl .... (3)
dx

Further differentiation gives

d 2y
-,
2 = a sec x tan 2x + a sec3 x + h2sec 2x tan x
(X

2
I.e., d Y
--1 = asecx tan x
2 1
+ bsec-xtanx + bsec 2xtanx + asec 3x
dx-

d 2y
I.e., dx 2 = secx t'lnx(atanx + bsecx) + sec 2x(btanx + asecx) ..... (4)

Substituting
asecx + btanx =y from (I)

atanx + bsecx = --
(~) from (2)
and
secx
in (3) we get

dY)
[-f)
d2
=
(
secx tanx ~ + sec x(y)
--
2
dx secx

d 2y dy
i.e., - -2 - tanx - - ysec2x = 0
dx dx
6 Engineering Mathematics - I

1.6 Example
Form the differential equation of all circles passing through the origin and having <

their centres on the x - axis.


Solution
Take any tangent to the circle as y-axis, the centre lies on x-axis. Let 'a' be the
radius of the circle.
Then centre is (a, 0)
:. Equation of the circle is (x - a)2 + .r = a2 .... (1)

, x
x

y'

Fig. 1.1

Differentiating (I) w.r. to 'x'


dy
2(x - a) + 2y - = 0
dx
dy
x- a = -y-
dx
dy
a=x+ y - .... (2)
dx
From (1) and (2)

[x-(x+ y:lJ' y' =[x+ y:J


+

.r (:r +.r = x2 +.r (:r +2XY ( : )

2XY : + x2 - .r = 0 is the required D.E


Ordinary Differential Equations of First Order and First Degree 7

1.7 Example
Form the differential equation of all central conics whose axes coincide with the
axes of coordinates.
Solution
The equation of all central conics whose axes coincide with the axes is
ax2 + bl = 1 .... (1)
Differentiating (1) w.r.t., x
dy
2ax+ 2by - = 0 .... (2)
dx
Differentiating (2) w.r.t., 'x' again
2
d y
a+by - 2 +b -
y )2 (d
=0 .... (3)
dx dx
Eliminating a, b from (1), (2)(3)

x2 y2
dy
X y- o =0
2
y~+ ~
d 2 (d
dx
dx

dx
r 0

y
=> Xyd2; +x(d )2 _y(dY)=O is the required D.E.
dx dx dx

Exercise - 1(a)
1. Eliminate the arbitrary constants from the following and find the
corresponding differential equation :
(i) y = mx + c (m, c arbitrary constants)

(ii) y = a e2x + b e-2x (a, b arbitrary. constants)

d2
[ Ans : ~ - 4y - 0]
dx 2
8 Engineering Mathematics - I

(iii) y = ax + bx 2 (a, b arb. constants)

d2 dy
[ADS: x 2 ----?
dx
- 2x -d + 2y
x
= 0]

(iv) (x - h)2 + (y - kf = a2, (h, k a, b arb. constants)

(v) y = (a + bx)e-X , (a, b arb. constants)

d 2y 2dy
[ Ans: - 2 + - +Y = OJ
dx dx
(vi) y = a sin x + b cosx (a, b arb. constants)

d 2y
[ADS: - l
2 + Y = 0]
(X

2. Find the differential equation of all circles with centre on the line y = x and having
radius' I '.

3. Form the differential equation of all the circles with centre on the line y = -x and
passing through the origin.

[ADS. : (xl + y) (: -1) = 2(y - x) (x + y:)]


4. Find the differentiall equation of all the parabolas with vertex at the origin and foci
on the x - axis.
dy
[ADS: y -2xy dx = 0]

5. Find the differential equation of all parabolas with the origin as focus and axis along
x-axis.
?
dy dy -
[ADS: 2x dx + Y ( dx) -Y = 0]
Ordinary Differential Equations of First Order and First Degree 9

Methods to Solve

1.8 The differential equations of the first order and of the First
Degree:
1.8.1 Separation of Variables
Sometimes the differential equation
/

dy
dx = q(x, y)

can be written as
f(x) dx + g(y)dy = 0 .... (I)

if the variables can be separated.


Integration of (1) gives the solution of the equation.

i.e., Jf(x)dx+ Jg(y)dy=c

where c is an arbitrary constant.

1.8.2 Example
Solve et"tany dx + (1-~sec2ydy = 0
Solution
The given equation
e-'"tany dx + (1-e-'")sec 2ydy =0
can be rearranged as

e( sec 2 y
- - d x + - - dy=O
I-eX tany

2
eX Jsec y
Integrating --dx+ - - dy=c
J I_eX tany

-Iog( e-'" -I ) + log t~ll1Y = c

tany
i.e., -X-I = c or tan y = c(e X -I)
e -
10 Engineering Mathematics - I

1.8.3 Example
dy
Solve - = 1 + xl +
dx
Y + xly
Solution
The given differential equation can be written as

dy = (1 +xl) (1 + y)
dx
dy x3
-1- 2 = (1+xl)dx => tan-I y = x + - + C
+y 3
1.8.4 Example
dy
Solve dx - 2xy = x, where YCO) = 1

Solution
dy
- =x(1+2y)
dx
dy
I+2y =xdx

On integration
I x2
"2 Iog(1+2y) = 2 +c .... (1)

Giveny= 1 when x = 0
Substituting in (1)
1
"2 Iog(3) = 0 + C

C= ~ log3 = log(v'J)
Hence the solution is
2
"21 Iog(I +2y)= 2x + Iogv'J (i.e.,) log (1+2Y)
- 3 - =xl
Ordinary Differential Equations of First Order and First Degree 11

1.8.5 Example
dy
Solve - =(4x+y+ If
dx
Solution
dy
-= (4x + y + 1)2 .... (I)
dx
Substituting 4x + y + I = t in (I)
dy dt
4+- =-
dx dx

i.e.,
dy = dt -4
dx dx
dt
- -4 = t 2
dx
Integrating

I t
i.e., -tan-I-=x+c
2 2

tan-I (4X+;+ I) = 2(x + c)

The solution can also be written as


4x + y + I = 2tan(2x + c)
where C is an arbitrary constant.

Exercise -1(b)

1.9 Solve the Following Differential Equations

1. : =~Y [ADS: ~ + e-Y = c]

2. (2 - x)dy - (3 + y) dx = 0 [ADS (3 + y) (2 -x) = c]


12 Engineering Mathematics - I

[ ADs: yc = (a + x) (I - ay) ]

dy
6. (x-y)2 dx = a2 [ADs: (x - y) + log ( x- y-a) = x + c]
x-y+a

dy
7. - =(3x+4y+ 1)2 [ADs: 2(3x+4y+ 1)= .J3tan- 1 2.J3x+c]
dx

8. dy =(2x+y+ If [ ADs: 1 tan-I (2X +


.J2 .J2Y + I ) = x +c]
dx

dy
9. dx = tan (x + y) [ADs: log[sin(x + y) + cos(x + y)] = x - y + c]

10 dy = 2 [ADs: (x + 2y - 3) - 410g (x + 2y + I) =x + c ]
. dx (x+2y-3)

1.9.1 Homogeneous Equations


The differential equation of the form
dy f(x,y)
dx g(x,y)
where f, g are homogeneous functions of same degree in x, y is called a
homogenous differential equation.
Such a differential equation can be written as

.... (I)

Substitutingy = vx
dy dv
- =v+x-
dx dx
Ordinary Differential Equations of First Order and First Degree 13

The D. E (I) becomes

dv \11 (v)
v + x dx = ~(v)

dx ~(v)dv
-; + v~(v)-\lf(v) = 0

Integration yields the solution

J-
dx
+
J"'()~(v)dv( ) = c where v = -y
X v'" v - \11 v x
\
1.9.2 Example
dy
Solve (xl + I) -dx =xy

Solution

. dy dv
S U bstltuttngy = vx, -d = v + x -
x dx
dv xvx
v + x- = 2 2 2
dx x +v x

dv v
v+ x - =--
dx 1+ v 2

dv v
x- = - -2- v
dx 1+ v

dv _v 3
x-=--
dx 1+ v 2

dx JI
~+ JI-dv=c
J-+
X v3 v
14 Engineering Mathematics - I

v-2
logx + - + logv = e
-2
1
logxv- - = e
2V2

(y)
logx - ~
x 2y
X2
= e ~ 2ylogy - xl = 2ey

i.e., 2ylogy = 2ey + x 2

1.9.3 Example
dy y2
Solve x-+-=y
dx x

Solution
dy
xl-+y=xy
dx

i.e.,

Substitutingy = vx,
dy dv
-=v+x-
dx dx

dv
v+x - =v-vl
dx
dv
x - =-vl
dx
dx dv
-+- =0
X v2
Integration yields
V-2+1 1
logx+ - - =e ~logx-- =c
-2+1 v
Ordinary Differential Equations of First Order and First Degree 15

x
logx - - = c
y

ylogx -x = cy or ylogx = x + cy

1.9.4 Example

Solve [x+ YSin(Yx)]dx = xSin(Yx)dy

Solution
The given differential equation can be written as

dy x+ ysin(X)
= __ --,---.0.-,.-----.:-

dx xSin(j~)

dy dy
Substitutingy = vx ~ - = v + x. -
dx dx

dv x+ vxsinv
v+x - = ----
dx xsinsin(v)

dv l+vsin
v+x- = - - -
dx sinv

dv l+vsinv 1
x-= -v=-.-
dx sin v SI11 v

dx
sin v dv = -
x

Integrating
-cosv = logx + c

logx + cos(Yx) = c
16 Engineering Mathematics - I

Exercise - 1 (c)

1.10 Solve the Following Equations

dy y2
l. dx - xy-x2 [ADS : y = ceix ]

2. (2 - 2xy)dx = (x 2 - 2xy)dy [ ADS : xy(y - x) = c ]

3. 2xy + ~ - r) dy = 0 [ADS: r + y = cy ]
dx

[ADS: logx = 2tan- i (Yx) + c]

5. xdy - ydx = ~ x 2 + y2 dx

y
[ ADS: cos - = logcx ]
x

7. xcos(Yx) (ydx + xdy) = ysin (Yx) (xdy - ydx) [ADS: sec (Yx) = yxc]

-I(YI)
y~ x + y = cx.e
2 2 tan Ix
8. (ry - x 3 )dy - ~ + xy) dx = 0 [ ADS: ]

9. (r + y) dx = 2xydy ( ADS: (r - y) = xc ]
dy
10. X dx = y[logy - logx + 1] ( ADS : y = xecx ]

1.10.1 Non-homogeneous Differential Equations


The D.E of the form
dy = ax+by+c
.... (J)
dx Ax+By+C

where a, b, c A, B, C are constants, is called a non-homogeneous dif.ferential


equation.
Ordinary Differential Equations of First Order and First Degree 17

Case (i)
a b
If --::;:.-
A B
Substituting x = X + h, Y = y + K
where h, k are (constants) to be chosen so as to satisfy.
ah + bk + c = 0, Ah + Bk + C = 0
Solving these equations, values of h, k are obtained.
The given D.E then reduces to a homogeneous D.E.
dY aX +bY
dX AX +BY
which is then solved taking Y = VX and then
substitute X = x - hand Y = y - k in the solution.
Case (ii)
a b
If
A B
Then the differential equation will be of the fonn

dy _ (ax + by)+c
dx - m(ax+by)+C .... (2)

since Ax + By will be constant m times ax + by.


Now substitute
ax + by = t,
Differentiation gives

a+bdy=dt
dx dx
dt
--a
i.e., dy dx
-=--
dx b
D.E (2) then reduces to
dt
--a t+c
~=---
b mt+C
18 Engineering Mathematics - I

Then the solution is obtained by using the method of separation of variables.

1.10.2 Example
dy x+2y-3
Solve
dx 2x+ y-3

Solution
dy x+2y-3
= ... - (I)
dx 2x+ y-3
Substituting
x = X + h, y = Y + k
where h, k are chosen to satisfY h + 2k - 3 = 0 and 2h + k - 3 = 0
solving we get
h=I,k=I .... (2)
i.e., we take x = X + I, Y= Y + I
The D.E (I) reduces to
dY X-2Y
=---
dX 2X+Y
dY dV
Substituting Y = VX, dX = V + X dX

dV X+2VX
V+X-=---
dX 2X+VX

~
2
V+XdV =1+2V XdV =1-V
dX 2+V dX 2+V

2+v dv=dX
I-v X

dX +(1 +_3_) dv = 0
X v-I

10gX + v + 3 log(v-I) = loge


Ordinary Differential Equations of First Order and First Degree 19

v + log( v-I )3 + logX = loge


v-log(v-I)3 X = loge

~-X
3
Y V-x
X +10 )
X=loge

I.e., 10g(Y - X)3 + XY = Xloge


log(y - \ - x + \)3 + (x - \) (y - I) = (x -I) loge
log(y - x)3 = (x - 1) (loge - y + \)

1.10.3 Example
Solve (2x + 3y + \) dx + (2y- 3x + 5)dy = 0
Solution
dy 2x+3y+ 1
dx 2y-3x+5

Substituting x = X + h, y = Y = k, : = :

Choosing h, k so that 2h + 3k + 1 = 0, 3h - 3k - 5 = 0
we get h = 1, k = -1
The given differential equation reduces to
dV 2X+3Y
dX 3X-2Y
Substituting
Y=YX
dV dV
- =Y+X-
dX dX
dV 2X+3YX
Y + X dX = 3X -2YX

Y+X dV =2+3Y
dX 3-2Y

X dV = 2(1 + y2 )
dX 3-2Y
20 Engineering Mathematics - I

3-2V)dV= 2dX
( 1+ V2 X
Integrating

3 -dv
f 2v
- - --dv=2 f-+c fdX
1+ v 2 1+ v 2 X
3tan- 1(V) - log( I + v2) = 210gX + c

3tan 1 (~ )-IO~ X';/') ~ 210g)( + c


B~ X=x-h=x-I
Y=y-k=y+1

3tan-1 (~:: )-IOg[ (x -Il~: ~ + I)'] ~ 210g(x- I) + c


1.10.4, Example

dy =x-y+l
Solve dx 2x-2y

Solution

-dy = --'------'-'--
(x- y)+1
dx 2(x- y)

Substituting x - y = V

dY=I_ dV
dx dx

l_dV=V+I
dx 2V

dV_ V+l_V-1
-- I------
dx 2V 2V

2V
- dV=dx
V-I
Ordinary Differential Equations of First Order and First Degree 21

Integrating

2 JV-I+I
dV=x+c
V-I

2[V + 10g(V-l)] = x+c


2[x - y + log(x - y -1)1 = x+c
210g(x - y - I) = 2y - x + c
or (x - y -1)2 = e2j~x C

1.10.5 Example
dy 4x+6y+1
Solve
dx 2x+3y -5

Solution
(~V 2(2x+3y)+ I
-=-
d'( (2x+3y)-5

Substituting
2x + 3y = V

dy (N
2+3-=-
dx dx

dy
dx
=~[dV
3 Ix
-2]
The differential equation reduces to

~(dV -2)=- -2V + I


3 dx V-5

dV
-
6V+3
=2+ - -
IO-2V+6V+3
= ------
13+4V
dx 5-V 5-v 5-V

5-V
4V+13 dV=dx
22 Engineering Mathematics - I

3
Integrating fl 1-
J~
3
4V + 13
) dv = _ 4 Idx + e

33
v- -log(4V+ 13)=-4x+e
4

33
(2x + 3y) - - log[4(2x + 3y) + 13] = - 4xm + C
4
i.e., 4(6x+3y-e)=33Iog[4(2x+3y)+ 13]

Exercise -1(d)

1.11 Solve the Following Equations

dy y-x+5
\. - + -=-------
dx y+x+3

[Ans:tan- J (
y+4 1 y+4
1
x-I ) +"2 log[ ( x-I ) 2 +1 =log(x-I)+loge]

dy x+2y-3
2. dx - 2x+3y-5

[Ans: (2+Ji) IOg[y-1 -


x-I ,,3
~] - (2-Ji) log [y-I +
x-I ,,3
~]+ ,,3~ logx= loge]

3 _dy = _2y'------_x_-_4
. dx y-3x+3

[Ans: (x - 2)2 - 5(x - 2) (y - 3) + (y - 3)2 = e {2(y-3)-[5+h1{X-2)]}]


2{y -3)-(5 -h1(x-2

4. (2x+5y+ l)dx-(5x+2y- l)dy=O

[ Ans : (x + y) 7 = e ( x - y - ~ r]
Ordinary Differential Equations of First Order and First Degree 23

dy y-x+ 1
5.
dx y+x+5

y-2) I
2
[ADS : tan~l ( x _ 3 + log [(y - 2)2 + (x - 3)2] = loge]

dy _ 3y+2x+4
6. dx 4x+6y+5

[ADS: .7'j(2x+3y)- :9 (14x+2ly+22)=x+c]


7. (4x-6y-l)dx+(3y-2x-2)dy=O
3
[ADS: x - y + 41og(8x - 12y - 5) = c ]

dy x-y+3
8. dx = 2x -2y + 5

x
[ ADS: (x - y) + log[2 + x - y ] = - + C]
2

1.12 Linear Differential Equations

A differential equation of the from : + py = q where p, q are functions of 'x',


alone or constants is said to be a linear differential equation offirst order. Multiplying
both sides of the equation by el pta [called the integrating factor (I.F)] we get,

el ptlr dy + el ptlr py = q .e Iptlr .... (i)


dx

The left hand side is the differential coefficient of y. e l pcb:


(i) can be written as
1ptlr
d(y.e ) = qe 1ptlr

Integrating

which gives the desired solution.


24 Engineering Mathematics - I

Note:
In some cases a differential equation can be reduced to the linear form by taking 'y'
as independent variable and x as the dependent variable.
The D.E is written as

p" q I are functions of y or constants

eJ
Pldy
Now the I.F =

Solution is

x.e
JPldy
=
fqle Jpl"Y
+c

1.12.1 Example
dy
Solve (I + x 2 ) - + 2yx - 6x2 = 0
dx
Solution
Rearranging the given differential equation to the form
dy
dx + py = q
We have

dy 2x 6x 2
-+--y=--
dx I +X2 1+X2

2x 6x 2
Here p= I+X2 ,q= I+X2

Jpdx =e J I+x2
~dx
I.F= e
log(I+X2 )
I.F = e = (1 + x 2 )
Solution is given by

y(1. F) = fp(I.F)dx + c
Ordinary Differential Equations of First Order and First Degree 25

1.12.2 Example
dy
Solve xlogx - +y=2Iogx
dx
Solution
dy y 2
-+--=-
dx xlogx x

I 2
Here p= - - andq=-
xlogx x

Solution is

ylogx = J~ logxdx + c
(IogX)2
ylogx=2 +c
2

1.12.3 Example
dy Y (xsmx+cosx
xcosx -+ . ) =I
Solve
dx
Solution
dy xsin+ cosx
-+y.----
dx xcosx xcosx

xsin x + cosx I
p= ,q=--
xcosx xcosx
26 Engineering Mathematics - I

J ~Slll X~.COS~ dx
I.F = e XCDSX

= e(log(xsecx)

I. F = e1og(x sec x)

= xsecx
Solution is

y(xsec x) = f_l- x x sec x dx + c


xcosx

2
xysec x = Jsec xdx E c

xysecx = tanx+c

1.12.5 Example

Solve dy + 2ytanx = sinx given that y = 0 where x = ~,


~ 3

Solution

dy + 2ytanx = sinx
dx
p = 2tanx
q = sinx

ef
2 tan xd!:
IF =

~ = e210gsecx

ylF= JqxIF.dx+c

2
ysec 2x = Jsecx.sec xdx + c
Ordinary Differential Equations of First Order and First Degree 27

ysec 2x = Jsec x tan xlir + c


')
ysec-x = secx + (' .... (I)

Given that y = 0 when x = ;7j


1C
o= sec - + c => C = -2
3

Substituting c = -2 in (I)
ysec 2x = secx - 2
is the required solution

1.12.6 Example
dy
Solve (x + 2.v) -, = y
(X

Solution

X + 2.v = yd
y
dx

dx
--- =
dy
x
y
2.v
is in the form of

-1
P = - q = 2,1
I ' 1 Y
Y

y
28 Engineering Mathematics - I

x-1
y
= f2y 2 1
x-dy+c
y

x
- =y+C
Y

1.13.7 Example
dy
Solve (x + y+ I) - = 1
dx
Solution
dy
dx- X =y+l

PI=-lq\,=y+1

IF = e f-1dy
= e- Y
Solution is given by

x(IF) = fql(/F)dy+c

xe-Y = f(y + I)e-Y dy + c

i.e., or x +y + 2 = ce Y

Exercise 1(e)

1.13 Solve the Following Differential Equations


I. (I + y)dx = (t~n-Iy - x)dx
[ADS: xetan- I y = tan-Iyetan-Iy --etan- I y + c]
dy
2. cos2x - + Y = tanx
dx
[ADS: Y = ce-tanx - tanx -1]
Ordinary Differential Equations of First Order and First Degree 29

dy
3. x - +2y-x2 1og=0
dx

c I .r2
(ADS: y = -? +- x210gx --I
x- 4 16

dy n/
4. dx + ycot x = 4x cosec.x, if y = 0, when x = ~2

n2
(ADS: ysinx = 2x2 - -
2
5. yeYdx = (y + 2x&)dy
( ADS : xy~2 =c- e~Y I

6. (x + 31) dy = y
dx

3
( ADS : x = 21 + cy ]
dy
7. (xy -I ) 3 + y3 = 0
dx+ y

I
. I
I ADS: x = ce Y + - + II
Y

dy
8. - = x 3 - 2xy if y = 2 when x = I
dx
[ADS: 2y - x 2 + 1 = 4 el~~ ]

dy x+ ycosx
10. dx =
I +sinx

x2
[ADS: y(l+sinx) = c - -]
2
30 Engineering Mathematics - I

1.13.1 Non-linear Differential Equation of First Order


Ber noulli j. equation:
The differential equation of the form
ely
dx + PY = llyn .... (I)

where p, q are functions ofx alone is said to be a Bernoulli's differential equation.


Dividing (I) throughout by yn
dy
y-n_ + py-n+l =q .... (2)
dx
Substitutingynt-l(coefficient ofp) "" v
1 dy dv
(1-n) - - = -
y" dx dx
(2) reduces to
I dv
- - - +pv=q
(I-n)dl'
dv
- + (1-n)pv = (I-n)q
dx
which is linear in v.
The avove D.E can be solved by using the method given in 1.12.1 example.

1.13.2 Example

Solve dy - ytanx = ysecx


dx
Solution
dy
dx - ytanx = ysecx .... (I)

dy 1
y-2 -d - - tan x = sec x
x Y

Substituting

--
y =v,
Ordinary Differential Equations of First Order and First Degree 31

1 dy dv
+--=-
/ dx dx
(1) reduces to
dv
dx + vtan x = sec x

is linear in v.
Here p = tanx, q = secx

IF = e
fpd<
flanXd<
= e
= elogsec x

= secx
SolutIon of th [) E (I) IS therefore

v(lF) = Jq(IF}dx + c

v. secx = Jsecx.secxdx + C

2
vsecx.= J sec xdx + c

Substituting
1
v=--
Y

1
- - sec x = tan x + c
y

i.e., y(tan x + c) + sec x = 0

1.13.3 Example
Solve (3xy + .0) dx - 3i2dy = 0
Solution
dy
3x2 - - 3xy
dx
= .0
32 Engineering Mathematics - I

is in Bernoulli's fonn
1 dy _
"7 dx - xy - 3x 2

Substituting
-1
=v
y

D.E reduces to
dv I 1
---v=- .... (1)
dx xy 3x 2

which is linear in v
1 1
Here p=~, q= 3x 2

Jpdx
IF = e
= J!d\
e x

=x
Solution of the D.E (1) is

v(lF) = fq(lF)dx+c

-..!..x= f_l_. xdx + c


y 3x 2

x 1
--=-Iogx+c
Y 3

i.e., y(110g x + c) + x = 0
1.13.4 Example
dy
Solve dx + (2xtan- 1y -.x3) (1 + y) = 0

Solution
dy dv
1+ y2 dx = dx
Ordinary Differential Equations of First Order and First Degree 33

The given differential equation becomes


dv
- + 2xv=x3 .... (1)
d-c
which is linear in v
Here p = 2x, q = x 3

IF = e J
2xdx
=e
x2

Solution of the D.E (1) is

v(lF) = Jq(IF}dx+ c
Writing x2 = t
1
xdx = 2dt

2
v.e x = {e 2+c
r I dt

1.14.5 Example
dy
Solve tany dx + tanx = cosy cos3x

Solution
dy
tany d'C + tanx = cosy cos 3x

Dividing by cosy throughout


dy
secy tany dx + secy tanx = cos 3x

Substituting secy = v, we get


dy dv
secy tany dx d'C
34 Engineering Mathematics - I

dv
dx + v . tal1.X = cos3x .... (I)

is linear in v
Here p = tal1.X, q = cos3x

IF = f pdt = flallXdy = secx


e e
Solution of the D.E (I) is

v(IF) = fq(IF)dx + c
3
v. secx = fcos xxsecxdx+c

2
secy secx = fcos xdx + c

I +cos2x
secx secy =
I 2 +c

x sin2x
secx secy = -+--+c
2 4

1.13.6 Example

dy . . cosx
Solve -d = (SII1.X - smy) - -
x cosy

Solution

dy. ."
cosy dx + smycosx = SII1.XCOSX

Substituting siny = v

dy dv
cosy-=-
dx dx

The gIven equation reduces to

dv .
dx + vcosx = SII1.XCOSX .. .. (I)
Ordinary Differential Equations of First Order and First Degree 35

is linear in v
Here p = COSX, q = SIl1 X cos X

IF
~
= e
fpdr = e feo') '(tl\ =e
~1Il X

Solution of the D.E (I) is

v.e SlIlt =
J. xcosx.e m"'. I + c
SII1 (X

write sinx = t=> cosx dx = II in the RHS

siny e;lIlt = [te t - el] + c


sinyeSlI1\" = eSItlX [sinx-l] + c

Exercise - 4(f)

1.14 Solve the Following Differential Equations


I. (ylogx -I )ydx = x(~y

1
(Ans: - = 1 + logx + ex )
y

2.
dy .
- cosx + YSlnx =
r.::::::::
...; ysecx
dx

(Ans :2y y 12 -Jsecx = tanx + 2c )

( Ans : ~ = - sin 2x -sinx -~ + ce 2SIIl\" )


y- 2

dy tany
4. ---- = (I+x) ~secy
Ix l+x
[Ans: siny = (1 + x) (eX + c) )
36 Engineering Mathematics - I

dy
5. x - + ylogy = xyeX
dx
I ADs : xlogy = (x - I)e" +c J

dy 2 x3
6. 3-+--y=-)
dx x+ I y

x6 2x 5 X4
I ADs: (x + Iii = -
6
+-
5
+- +C
4
dy
7. + ytanx = isecx
dx
I ADs: cos2x = y(c + 2sin x) J

dy
8. 2 1
dx X Y +xy

1.14.1 Exact Differential Equations


Let us consider the differential equation Mdx + Ndy = 0 where M, N are functions
ofx,y.
If this equation is to be exact, then it must have been derived by directly
differentiating some function ofx,y.
Hence Mdx + Ndy = du, say .... (I)
But from differential calculus
au
du = -dr:+-dy
au
.... (2)
ax ay
From (I) and (2) we get

au N= au
M= ax' ay

aM
-=--
a2u
Now and -ax- - -ax-ay-
ay ayax
Ordinary Differential Equations of First Order and First Degree 37

aM
-
oN.IS tIle con d
= - ItlOn ..lor exactness.
oy AX
:. The differential equation Mdx + N~v = 0

.IS exact 'f aM aN


I -=-
oy ax
Then the solution is expressed in the form

(treatingyas (integrate w.r.t y those


constant integrate w.r.t x) terms that are independent of x)
Note:
IfN has no term independent ofx then the solution is

fMdx = e

1.14.2 Example
Solve (x + 2y - 3) dy - (2x - Y + \ )dx = 0
Solution
(x+2y - 3)dy - (2x - Y + I) dx = 0
M = -(2x - Y+ \)
N = (x + 2y - 3)
aM
-=\
oy
aN
-=\
ax
The given differential equation is exact
The solution is

- f(2x - y + \ )dx + fe x + 2y - 3) = (ry = e


_2X2 2i
- -2- + yx - x + 2 - 3y = e

=> I - x 2 + xy - x - 3y = e
38 Engineering Mathematics - I

1.14.3 Example
Solve (x 2+ y.)dr: + 2xy dy = 0
Solution
(x 2 + .V) dx + 2xy dy = 0
M=x2+y. N = 2xy
aM
-=2y
aM
- =2y
av ax
The given differential equation is exact
2
Solution is f(x + y2 }h + f2xydy = c

1.14.4 Example

Solve (I + e -,:~, )dx + /Y [I.'... ; 1 dy = 0

Solution

(I + e'x Y ) dx + e x'Y (
J -;;, ) dy = 0

x'y
M = J + e:t Y , N = e,i [
J- Y x)

aM
-=-
-x ,y'
e'Y
~ i
oM oN
oy ox
The given differential equation is exact
Ordinary Differential Equations of First Order and First Degree 39

Solution is

x
X + e Y (y) = c

Exercise - 4(g)

1.15 Solve the Following Differential Equations


1. (el' + 1)cosxdx + eJ'sinx~v = 0

lADs: (oY + 1)sinx = c I


2. (vcosx + siny + y)tb' + (sinx + xcosy + x)dy = 0
I ADs: ysinx + (siny + y)x = c I
3. (x 2 - ay)dx = (ax - ;l)(~v
ADS: x 3 + ),3 - 3ll.\y = C I
4. (ax + hy + g)d'C + (hx + hy + j)dy"= 0

I ADs.
ax
2
- 2 + (liy + g)x +
hi':' + fy = c I
.!.

5. (x 2 + 1- a2 )xdx + (x 2 -.v - P)ydy = 0

I ADS : x4 + 2x21 - 2a2x2 - l - 2b 2;l = 4c I


1.15.1 Integrating factors
If the differential equation Mdx + Ndy = 0 is not exact, it can be made exact by
multiplying it with some function of x, y. Such a function is called an integrating
jactor.
Rule!t' for fillt/illg the illtegrtltillg factors :
1. Integrating factors found by inspection:

Example
Solve x dy- ydx = 0

Solution
xdy- ydx = 0
40 Engineering Mathematics - I

Dividing by x2
xdy- ydx
---'-,=-=--
2
= 0
x

On integration

Yx =c
First method /0 find an integrating jac/or :
If the differential equation Mdx + Ndy is not exact, but is homogenous and

Mx + Ny 7:- 0, then the integrating factor is 1 . Multiply the differential


Mx+Ny
equation by IF. The DE becomes exact.
1.15.2 Example
Solve (x 2y - 2xy2)dx - (x 3 - 3x2y)dy =0
Solution
(x 2y - 2xy)dx - (x 3 - 3x2y)dy =0 .... (I)

The differential equation (I) is homogeneous


M = x 2y- 2xy N = - (x 3 - 3x2y)
aM aN
- =x2-4xy - =-3x2 + 6xy
oy ax
The DE is not exact and
Mx + Ny = x 2y 7:- 0
) 1
IF = Mx+ Ny - x 2 y2

Multiplying the DE by the integrating factor ~


x y

(
X2Y -2X
y2
2 2
)d _(X2 -3X3Y )dY
X 2 2
= 0 .... (2)
X Y X Y
Ordinary Differential Equations of First Order and First Degree 41

1 2 -x 3
write M = --- N =-+-
I Y X I y2 Y

aMI aNI
--=-=--
then ry ax y2

DE (2) is exact
Solution is

x
- - 210gx + 3logy + c
y

i.e.,

x
or

1.15.3 Second Method to Find the Integrating Factor


If the differential equation Mdx + Ndy = 0 is not exact and is of the form
j(xy)ydx + g(xy)xdy = 0
and Mx- Ny *- 0
1
then Mx _ Ny is an integrating factor

1.15.4 Example
Solve (x 2y2 + xy + 1)ydx + (x2y2 - xy + 1)xdy =0
Solution
(x2y2 + xy + 1)ydx + (x2y2 - xy + 1)xdy = 0 .... (1)
M = x2j3 + xy2 + y, N = x2y2 - x2y + x
Mx - Ny = 2x2y2 *- 0
1 1
Hence the IF- =--
- Mx- Ny 2x 2y2
42 Engineering Mathematics - I

Multiplying (I) by IF
2
(X y 2 +xy+ 1)+ (X 2
y 2 -xy+ I) = 0
2x 2y2 2x 2y2

2
I{ I 2} I{ II}
- y+-+-
x x 2y
dx+- x - - + -1
2 y xy-
dy=O .... (2)

DE (2) is exact
Solution of the DE (I) is

~ 'y+~+-!-tx+~ 'x_J.-+_I_ )dY = c


2 Jl x x- y r 2 Jl y xy
1

1[
2 xy
I] I
- xy + log x - - - -logy = c
2

1.15.5 Example
Solve (xysin xy + cos xy)ydx + (xysin xy - cos xy)xdy = 0
Solutiou
(xysin xy + cos xy)dx + (xysinxy - cosxy)xdy = 0 .... (1)
M = (xysin xy + cos x,v)y, N = (xysin xy - cos xy)x
Mx - Ny = 2xycosxy =F- 0

IF=---
Mx- NY 2xycosxy

Multiplyingthe DE by 2xycosxy

The DE reduces to

~(ytanXY+~)dX+~( xtanxy- ~) dy = c .... (2)


Ordinary Differential Equations of First Order and First Degree 43

which is exact (verify)


.. Solution is

1 1 1
-Iogsecxy + -Iogx - - log y = c
2 2 2

I.e., logxsecxy = 2c + logy. Taking 2c as log A


~ xsecxy = Ay

Exercise - 4(h)

1.16 Solve the Following Differential Equations


I. (x 3; + x 2y2 + xy + 1)ydx + (x 3; - xli - xy + 1)xdy = 0
1
r ADS: xy - - - 210gy = c I
xy
2. (xli + xy + I )ydx + (xli - xy + 1)xdy = 0
1
r ADS: xy + logx - logy - - = c]
xy

[ ADS: logx2 - logy - _I = c ]


xy
4. (x4y4 + x 2 i + xy)y dx + (x 4y4 - x 2i + xy)xdy = 0

I ADS : ~ x 2i - _I - logx - logy = c ]


2 xy

1.16.1 Third Method to Find the Integrating Factor


aM aN

If Mdx + Ndy = 0 is not exact and _a-=-~_ _ax_ is a function of x alone say fix),
N

then the IF =e
J
j (x)dr
44 Engineering Mathematics - I

1.16.2 Example
Solve (x 2 + y + 6x )dx + yxdy = 0
Solution
(x 2 + Y + 6x)dx + yxdy = 0 ..... (1 )

M = x2 + Y + 6x N = yx
aM aN
ay= 3y, fu = y2
aM aN
ay-ili 3y2 - y2 2
2
N y x X

is a function of x alone

IF = e f~
2tJx
= x2
Multiplying the given DE by x2
x2(x2 + y + 6x)dx + yi3dy = 0 ..... (2)
aMI
- =3x2y
ay

DE (2) is exact
Solutionis
4 2 2 2
f(x + x / + 6x 3 }tx + fy x dY = c

1.16.3 Example
Solve (x2 + y) dx - 2xydy = 0
Solution
(x2 + y)dx - 2xydy = 0 ..... (1 )
Ordinary Differential Equations of First Order and First Degree 45

M = .xl + .0, N = - 2xy


aM aN
ay = 2y, ox = -2y

aM aN
ax ax 2y+2y -2
N -2xy x

is a function of x alone

IF = e
f -~dx
x
1
=-
x2
MuItiplyingthe DE by
1
IF= -
x2

The given DE reduces to

[ l)
1+-
2
x
2y
dx,.--dy=O
x
..... (2)

aM, _ 2y oN, 2y
-a
y
--2'
X
-=-

(2) is exact
Solution of (1) is
y2 2y
Jl+-dx+
x
J--dy=c
2
x
y2
x- - =c
x

1.16.4 Fourth Method to Find the Integrating Factor

If the differential equation Mdx + Ndy = 0 is not exact and


aN aM
ax M By
1 is a
[

function of y alone, say fly)


46 Engineering Mathematics - I

Then IF =
e
f t(y)<ly

1.16.5 Example
Solve (xy + y) dx + 2(.x2.0 + x + y4)dy = 0
Solution
(xy + y)dx + 2(.x2.0 + x + y4)dy = 0
M = .xy3 + y N = 2(.x2.0 + x + y4)

oM
--- = 3xy + 1
oN
- =4xy2+2
8y ox

oN -8;
-a; OM] = (4xy2 + 2)-(3xy2 + I)
( M xi+y

is a function ofy alone

f~d)'
F=e Y =y

Multiplying the differential equation by I.F =Y


(xy4 + .0) dx + 2(.x2Y + xy + yS)dy = 0 ..... ( I)

oMI .,3 oNI .,3


--=4xy +2y - =4xy +2y
8y ox
The differential equation (I) is exact

Solution is f{xl + i }:Lx + 2 f(x 2 i + xy + yS) dy = 0


Ordinary Differential Equations of First Order and First Degree 47

1.16.5 Example
Solve (3x 2y4 + 2xy)dx + (2x 3_Y' - x2)(~V = 0

Solution

aN _ aM _ (6x /
2 -2x)-(12x 2y 3 +2xL 6x 2/ -4x -_3.
ax ay - l
3x 2 oJ- 2x - y{3x 2/ + 2x ) - Y
is a function of y alone

-J~ dy I
IF = e Y =-?
Y

Multiplying the DE by ~, the DE reduces to


y

2X) dx+ (3
(3x 2y 2+-y x2) dy= 0
2x y-7

aNI =6xy- 2x
ax i
:. DE (I) is exact

Solution of the DE is

2
x3 2 2 x
3-y+--=c
3 y 2
48 Engineering Mathematics - I

Exercise - 4(i)

1.17 Solve the Following Differential Equations


I. (x + y3 + I )dx + ydy = 0
2 3x
X e
2 x 3x 2 e 3x 3 e 3x
ADs: (ADs: - - - - e +--+(y + l)-=e I
3 9 933

2. (x 2 + Y + 2x)dx + 2ydy = 0

3. 2xydy - (x 2 + Y + I)dr = 0
I ADs: y - x 2 + I = ex I

5. (y4 + 2y)dx + (xy3 + 2y4 - 4x)dy = 0


2x
I ADs: xy + -) + Y.1 = e ]
y

1.17.1 Fifth Method for Finding Integrating Factor


If the differential equation
Mdx + Ntry= 0 .... (l)
is not exact and is of the form
xliyb(mydx + nxdy) + Xf y(pydx + qxdy) = 0
when a, b, r, s, m, n, p, q are constants
Then the IF = xlII
where h k are constants such that the equation (I) after multiplying with IF
becomes exact.
Ordinary Differential Equations of First Order and First Degree 49

1.17.2 Example
Solve xy\ydx + 2xdy) + 3ydx + 5xdy) = 0

Solution
xy3(ydx + 2xdy) + (3ydx + 5xdy) = 0 ..... (1 )

The IF = x"1
Multiplying (I) by IF = xhl it must become exact
(xh+1 1+1 + 3x" 1+1)dx + (2x ilj .? + 1+3 + 5xil f-/ I){~Y = 0 ..... (2)
is exact if

aM
- ay = (4 + k)xH1 .1+3 + 3x\k + 1)1
.

aN
- = 2(h + 2) xh+I /+ 3+5(h + 1)x"1
ax
For DE (2) to be exact

aM aN
ax ax

Comparing the coefficients on both sides


2h - k = 0 and Sh - 3k = -2

Solving h = 2, k= 4

Substituting h = 2, k = 4 in (I) DE (I) reduces to


(x 3y 8 + 3x2yS) dx + (2x4y1 + 5x3y 4)dy = 0 which is exact

.. Solution is
50 Engineering Mathematics - I

1.17.3 Example

Solve (3x + 2y)ydx + 2x(2x + 3y)dy = 0

Solution
(3x + 2y)ydx + 2x(2x + 3y)dy = 0 .... (1)

IF = 0/,
MUltiplying (1) by IF
(0/,+2 + 30+2 /,+I)dx + (2x h+3/, - 0+ 1/,+I)dy = 0 .... (2)

oM
- = 3(k + 2)0/,+1 + 2(k + 1) 0+2yK
ox
oN
- = 2(h + 3) x h+2/, ---(h + 1)0/,+ 1
ox
The DE(1) is to be exact

oM ON
ox ox
2(k + 1) = 2(h + 3) => h = - %k = -/i
Comparing the coefficients
k + 2 = - (h + J)

and 2(k+ 1) = 2(h + 3) => h = -,% k= -/i


Substituting h, k values in (2)

(
-5/
X /2 y/2
3/
+ 2x -1/
I II) ( II -II
12 yl2 dx + 2X 72 Y 12
-3/ II) dy = 0
_ X /2 y72 .... (3)

(3) is an exact DE
2 -2/ 3/ II II
Solutionis --x 13 yl2 + 4X 72 y72 = c
3
Ordinary Differential Equations of First Order and First Degree 51

Exercise - 4(j)
I Solve the following differential equations

I. x(3ydx + 2xdy) + 8y4(vdr + 3xdy = 0

4 2 10 +1
3x 3+l y 3 X 1 7
I Ans: ---y 3 =c]
4 10
- +1 - - +I
3 3

3. (y2 + 2x2y)dx + (2x 3 - xy)dy = 0


I I 2 3 3
( Ans : 4x 2 y 2 __ X 2Y 2 = ex ]
3

4. (2.~y - 3y4)dx + (3x 3 + 2xy3)dy = 0


- 36, 24 _10 ' 15
(Ans: 5x 13 y 13_12x I]y I] =c]

1.17.4 Applications to geometry, law of natural growth and Newton's law of


cooling:
When some action is applied on a quantity the changes and the action affects all
the parts equally. The rate of change depends on the original quantity.
For example the total population 'p' of a country increases with respect to time

't', say, then its rate of change with time is ~ . Under ideal condrtions the rate of
change of the population will be proportional to the total population at any given time
and is called the law of natural growth.
The growth of the population satisfies the differential equation
dp
- =kp
dt
where k > 0 is a constant
Solving the differential equation we have
p = cekl
dx
The rate at which a quantity x is decreasing is given by dt and this is negative.
52 Engineering Mathematics - I

This rate of decrease or decay is found to be proportional to x itself


dx
- =-kx (where k> 0 is a constant)
dt
is the law of decomposition
Solving the DE x = ce-kl
Newton s law of cooling:
Newton's law of cooling states that the rate of decrease of the temperature of
a body is proportional to the difference of the temperature 9fthe body and that of
its surrounding medium.
Let 0 be the temperature of the body at time 't' and 00 the temperature of its
surrounding medium.
:. Difference between the temperatures = 0 - 8 0 (0 > 00) and the rate of decrease
dO
of the temperature of the body is - dt' since the body is cooling oc.

de
Using Newton's law ofcooling - - oc(O - 0 )
dt 0

-dO
or dt = k(O - 00 ) where k> 0 is a constant of proportionality

dO
- = -k (0 - 0 )
dt 0

f~=-k
0-0
fdt+c
0

10g(0 - 00) = - kt + c or 0=0 o + Ce-kl


1.17.4 Example
The mass of crystalline deposit increases at a rate which"js proportional to its
mass at that time. The deposit has started around a crystal seed of 5 grams. Find an
expression of its mass at time 't'. Ifin 30 minutes the mass of the deposit increases
by'l' gram, what will be the mass of the deposit after 10 hours.
Solution
Let m be the mass of crystalline at time 't' then by law of growth

dm=k' dm
-=dt
dt ' km
Ordinary Differential Equations of First Order and First Degree 53

on integration
log m = kt + c .... (I)

initially when t = 0, m = 5
(1) => log5 = 0 - c
c=-log5
SubstitutilllJ; c value in (1)
log m = kt-log5

I.e., kt= 10g( ~) .... (2)

When t = 30 minutes mass deposit increases by , I ' gram


m = 5 + 1 = 6 grams
Substituting

t= "21 (hours), m =6

Substituting in (2)

log(1Y t= 109; .... (3)

Now to find the mass after 10 hours (i.e t = 10)


from (3) we get

IOg(H IO~ log 7~ log (7) ~ IOg(%)"


x

6)20
m = 5"5 ( grams
54 Engineering Mathematics - I

1.17.5 Example
The rate at whi~h a certain substance decomposes in a certain solution at any
instant is proportional to the amount of it present in the solution at that instant.
Initially, there are 27 grams and three hours later, it is found that 8 grams are lett.
How much substance will be left after one more hour.
Solution
If m grams is the amount of the substance left in the solution at time 't', then the rate

at which it decomposes is dm , which is proportional to m.


dl
By law of decay
dm
dt = - km (k> 0)

f~ =-k fdt+c

logm = -kt+ c .... (I)


Initially when t = 0, m = 27
From (1) we have
log27 = - k.O +c
=> c= log27
Substitution of'c' in (I) gives
log m = - kt + log27

10g(;; )= - kt .... (2)

It is given that m = 8 when t = 3


.. From (2)
8
log (2 ) = - k, 3
7

8
-k= log ( 27 )X
2
-k=log-
3
Ordinary Differential Equations of First Order and First Degree 55

Then (2) becomes

10g~ =IOg(%} ... (3)

when t = 4

4
III 2
log 27 = log ( "3 )

m=27 x (%r grams

16
m= 3 grams.

1.17.6 Example
The number x of bacteria in a culture grow at a rate proportional to x. The value
ofx was initially 50 and increased to 150 in one hour what will be the value ofx after

1
12" hour.

Solution
dx
-=/0:
dt

dx
- =kdt
x

logx = kt + c .... (I)

c is the constant of integration


when t= O,x= 50
.. log 50 = k.O + c
or c= log50
(1) => logx = kt + log50
56 Engineering Mathematics - I

x
log- =kl
50
x = 150, when 1= 1
150
.. log 50 = k.I

or k= log3
(2) then gives

x
log ( 5O) = flog3
3
we want to find x when I = "2

X 3
50 = (3)2

3
X = 50 (3)2 grams
1.17.7 Example
The rate of cooling of a body is proportional to the difference between the temperature
of the body and the surrounding air. If the air temperature is 20C and the body
cools for 20 minutes from 140C to 80C, find when the temperature will be 35C.
Solution
If 8 is the temperature of the body at time '1' then from Newton's law of cooling
-d8 de
-a(8-20) ~ - =-k(8-20)
dl dl

f~
8-20
= - k rldl + c
J'
log(8 -20) = - kt + c .... (I)

Initially when
t = 0, 8 = 140
log(8 - 20) = 0 + c,
Ordinary Differential Equations of First Order and First Degree 57

c = log( 120), (I) reduces to


log(140 - 20) = - kt + log 120
or +kt = log( 120) - 10g(S - 20) .... (2)
It is given that q = 80 when t = 20 minutes
k. 20= logI20-log(80-20)

I (120)
k= 20 log 60

I
k= -log2
20
Substituting in (2)
1
(2 0 IOg2)t= logI20-log(S-20)

It is required to find t when


0= 35c

0
20 10g( \25 )
logI20-log(35 - 20)
t=
I log2
-log2
20

10g(8) 2010g23 60 10g2 .


t= 20 - - = - - = - - =60mmutes
log2 log2 log2

Exercise - 4(k)
I. In a certain reaction, the rate of conversion of a substance at time "t' is proportional
to the quantity of the substance still untransformed at that instant. At the end of one
hour 60 grams while at the end offour hours 21 grams remain. How many grams of
the first substance was there initially?
( ADs: 85 grams approximately I
2. The rate of growth of a bacteria is proportional to the number present. If initially
there were 100 bacteria and the amount doubles in '1' hour, how many bacteria will
1
be there after 2"2 hours.

(ADs: 564 I
58 Engineering Mathematics - I

3. Under certain conditions cane sugar in water is converted into dextrose at a rate
which is proportional to the amount unconverted at any time. 1f75 grams was there
at time t = 0.0 and 8 grams are converted during the first 30 minutes find the amount
I
converted in 12 hour.

[ADS: 21.5 gms]

4. The rate of cooling of a body is proportional to the difference between the temperature
of the body and the surrounding air. If the surrounding air is kept at 30c and the
body cools from 80c to 60c in 20 minutes. Find the temperature ofthe body after
40 minutes.
[ADS: 48c J

5. If the air is maintained at 30c and the temperature of the body cools from 80c to
60c in 20 minutes. Find the temperature of the body after 40 minutes.
[ ADS: 48c]

6. The rate at which a heated body cools in air is proportional to the difference between
the temperature of the body and that of the surrounding air. A body originally at 80
cools down to 60c in 20 minutes the temperature of the air being 40c what will be
the temperature of the body after 40 minutes from the original temperature.
[ADS: 50c J

7. The rate at which bacteria multiply is proportional to the instantaneous number


present. If the original number doubles in 2 hours in how many hours will it triple.

[ ADS: 210g3 J
log2

8. Water at temperature 100C cools in 10 minutes to 80C in a room of temperature


25C. Find the temperature of water after 20 minutes.
[ ADS: 65.5c]

9. A cup of coffee at temperature 100C is placed in a room whose temperature is


15C and it cools to 60C in 5 minutes, find its temperature after a further interval of
5 minutes.
[ADS: 38.8c ]
Ordinary Differential Equations of First Order and First Degree 59

Orthogonal Trajectories

1.18.0 Definition (1)


A trajectory of a family of curves is a curve cutting all th.e members of the system
according to some law. For example a curve cutting a family of curves at a constant
angle is a trajectory.
Definition (2)
If a curve cuts every member of a given family of curves at right angles, it is called
an orthogonal Trajectory. The orthogonal trajectories of a given family of curves _
themselves form a family of Curves. If the two families of curves are such that
each member of either family cuts each member of the other family at right angles
then the members of one family are known as the orthogonal trajectories of the
other.
In two dimensional problems in the flow of heat, the curves along which the heat
flow takes place and the isothermal curves or loci of points at the same temperature
are orthogonal trajectories. In hydrodynamics, the flow of water from a lake into
narrow channel produces a family of streamlines which are orthogonal trajectories
to the curves of equal Velocity Potential. In the flow of electricity in thin
conducting sheets, the paths along which the current flows are the orthogonal
trajectories of the equipotential curves and vice - versa.

1.18.1 Orthogonal Trajectories: Cartesian Coordinates


Let f(x,y,c)=O ..... (1)

be a family of curves, where c is a parameter. We can form a first order differential


equation by eliminating 'c' from (1) \

i.e., F(X,y,:)=o ..... (2)

is the differential equation whose general solution is (1 )


If the two curves are orthogonal (curves intersecting at right angles) the product of
the slopes of the tangents at their point of intersection must be equal to -I.

Suppose (x, y) is the point of intersection of the curve (I) and its orthogonal
trajectory .
60 Engineering Mathematics - I

At this point slope of the tangent to the curve (I) is dy and -dx is the slope of
dx dy.
the tangent to the orthogonal trajectory. Therefore on replacing dy by - dx in (2)
dx dy
, the equation thus obtained is the differential equation of the family of orthogonal
trajectories of the family(l) .

. . ( xJI, - :)=0 ... (3)

is the differential equation of the system of orthogonal trajectories, and its solution
is the fami Iy of orthogonal trajectories of (I) .

1.18.2 Orthogonal Trajectories - Polar Coordinates


Suppose f(r,O,c) = 0 ..... (1)
is the family of curves where c is the arbitrary constant.

We can form a differential equation F(r,o, :~ )= 0 ..... (2)

of the family (I), after elimination of the constant 'c'.


Let be the angle between the radius vector and the tangent at any point (r, 0) .
on a member of the family of curves.
dO
Then tan=r- .... (3)
dr
Let I be the angle between the radius vector and the tangent at any point (lj, ~ )
on the trajectory.

Then tan;/,
dBI
= r,1 _
Y'I dr, ..... (4)
I

At a point of intersection of the given curve and the orthogonal trajectory

lj=r, 01 =0 Hence tan1 = -cot


From (3) and (4),
dOl 1 dr 1 dr dOl
lj dr = --; dO i.e., -; dO = -lj dlj
l
Ordinary Differential Equations of First Order and First Degree 61

Hence the differential equation of the orthogonal trajectory is obtained by


. dB fi 1 dr. J dB fi dr
Sll bstltutmg - r - or - - , I.e., -r- - or-
dr r dB dr dB

Therefore the differential equation F (r, B, _r2 ~~) = 0 ..... (5)

gives the differential equation of orthogonal trajectory of the family of cllrves(l)

Solution of (5) is the orthogonal trajectory of the family of curves.

1.18.3 Example
Find the orthogonal trajectory of the family of curves a/ = x 3 , where a is variable
parameter.

Solution:
Given family of curves is a/ = x 3
..... ( I)

Differentiating (I) w.r.t. 'x'; 2ay dy = 3x 2 ..... (2)


dx
Eliminating 'a' from (1) and (2)

. (X3) .y dy
.. 2 /
_
dx -3x
2

dy
2x =3y ..... (3)
dx

is the differential equation of the family (I). Now replacing : by - : in(3),

gives the differential equation of the orthogonal trajectory to (I) as -2x dx = 3y


dy
23 2
Integrating both sides -2~ = L + c . Therefore
2 2
2X2 + 3/ = c is the equation of orthogonal trajectory of (I)
62 Engineering Mathematics - I

1.18.4 Example
Find the orthogonal trajectory of the family of parabolas y2 = 4ax where 'a' is the
parameter.
Solution:
y2 =4ax ..... (1)

differentiating (I) w.r.t. 'x'; y dy = 2a ..... (2)


dx

Eliminating 'a' from (I) and (2). y2 = 2X(Y:)

dy
.. y=2x ..... (3)
dx
is the differential equation of the family (1)

Replacing dy by _ dx in(3)
dx dy

Y=2X( -: J ..... (4)

is the differential equation of the orthogonal trajectory to (I). Integrating (4)


fydy = -2 fxdx + c; 2X2 + y2 = c is the orthogonal trajectory of (I)
1.18.5 Example
2 2
Find the orthogonal trajectories of ~2 + ( = 1 whereA is the parameter.
a a +A
Solution:
X2 y2
-+ =1 ..... (1)
a2 a2 +A
Differentiating (I) w.r.t. 'x'; 2x2 + 22y dy =0
a a +A dx

..... (2)
Ordinary Differential Equations of First Order and First Degree 63

)
x- xy
Eliminating A from (I) and (2) -2- - - = 1
a a 2 --dy
dx
2
:. ( x - a 2 ) : = xy ..... (3) ---

is the differential equation of the family (I)


S ub stltutmg -dx
- fOor -dy.111 (3) ; (2
X -a 2)( -d- ) = xy
Y .
I.e,
dy dx dx

dY~-( x' x a
2
y )dx
is the differential equation of the orthogonal trajectory to (1)
2 2
Y
-=a logx---
2 X
Integrating we get
2 2+c
x +l
2
= 2a 2
log x + C is the orthogonal trajectory of the family of curves( 1)

1.18.6 Example
Find the orthogonal trajectories of the family of coaxial circles
x + l + 2gx + C = 0 where g is the parameter.
2

Solution:
Given x 2 + y2 + 2gx + C = 0 ..... (1 )
dy -
Differentiating (I) w.r.t. 'x' ; 2x+2y-+2g+0=O ..... (2)
dx
2
Eliminating 'g' from (I) and (2) x + y2 - 2x ( x + y : ) + c = 0

y2 _ x 2 _ 2xy dy + c = 0 ..... (3)


dx
is the differential equation of the family of (1 )

Substituting - dx for dy in (3) we get,


dy dx
dx
y2 _ x 2 + 2xy - + C = 0 ..... (4)
dy
64 Engineering Mathematics - I

Which is the differential equation of orthogonal traject{)ry. SimplifYing


dx 1 2 c+ /
2x---x = - - - ..... (5)
dy y y
is a linear equation of the Bernoulli's form (non - linear differential equation of
first order and first degree)

Substituting x 2 = t, 2x dx = dt in (5)
dy dy
dt t (c+ / )
---= ..... (6)
dy y y
is linear in t.
I
I --dy
:.I.F=e Y =-
1
Y
General solution of(6) is

t.! = -
y
f +yy2 .!dy+k
C

y
t C
-=-y+-+k.
y y
x 2 + Y - ky - C =0 ( since t = x 2 )
is the equation of the orthogonal trajectory of (I)

1.18.7 Example
Find the equation of the system of orthogonal trajectories of the parabolas

r = 2a , where 'a' is the parameter.


l+cosO
Solution:
2a = r (1 + cos 0) ..... (1)

~ a = rcos 0/2
2
2a = 2r cos 2 0/2
log a = logr + 21ogcosO/2
.Differentiating w.r .to '0 '

o=! dr + 2 (-sinO/2).(l/2)=O i.e,


r dO cosO/2
Ordinary Differential Equations of First Order and First Degree 65

1 dr
---tanB/2 = 0 ..... (2)
r dB
2 10 elr.
Substituting -r - for - m (2) we get
dr dB

1(
- -r-?dB)
r
-
dr
-tanB/ 2 =0

dr
- = -cotB/2dB ..... (3)
r
is the differential equation of the 0I1hogonai trajectory .
Integrating (3)
log r = -2 log sin B/2 + log c

log(~) = logsin 2
B/2

c . 2 / ( 1- cos B)
-=sm B 2=--'--------'-
r 2
:. 2c = r (1 - cos D) is the equation of orthogonal trajectory of the fami Iy of ( I )

1.18.8 Example
Find the orthogonal trajectory of rill = alii cosmB where 'a' is the parameter.
Solution:
Given rill = a cosmB
ln
..... (1)
mlogr = mloga+ log cos mB

Differentiating w.r.to 'B' ;


m -dr=
- 1 ( -msmmB
. )
r dB cosmB

~ dr = -tan(mB) ..... (2)


r dB
is the differential equation of the family of curves (I)
dr 2 dB .
Replacing - by -r - m(2)
dB d,.
66 Engineering Mathematics - I

~.(_r2 da) = -tan (rna)


r dr
dr = cot (rna)da ..... (3)
r
is the differential equation of the orthogonal trajectory.
Integrating (3)

'log r = J.-Iogsin (rna) + loge


rn
log rln = log e sin (rna) ; rill
lll
= e lll sin (rna) is the orthogonal trajectory of the

family (I)

Exercise 4 (s)

1. Find the orthogonal trajectories of the following family of curves, where a IS a


variable parameter.
(i) y=ax 2 2
[ADS: x +21 =e]

xy=a 2
2
(ii) [ADS: x - y-
?
=e ]
(iii) x 2 -xy+ l =a 2 [ADs: (x-y)=e(x+y)2]

2X2 + y2 = kx 2
(iv) [ ADs: x = - y2 log ( ; )]

(v) x 2 _ y2 = a2 [ADs: xy = e]

(vi) X2/3 + y2/3 = a2/3 [ADs: y4/3 _ X4/3 = e 4/3}

2. Show that the system of confocal and coaxial Parabolas l = 4a (x + a) is self


orthogonal.
3. 3
Find the orthogonal trajectories of the curves 3xy = x 3 - a , a being the parameter.

4. Prove that orthogonal trajectories of a system of circles x 2 + y2 - ay = 0 is


x 2 + y2 -bx =0
Ordinary Differential Equations of First Order and First Degree 67

5. Find the orthogonal trajectories of the following family of curves.


-
0'
--

(i) r=aB [ADS: r = ce 2 I


(ii) r =e aO [ ADS: (log)2 + B2 = c I
(iii) rn sinnB = a" [ADS: rll eosnB = e" I
2
(iv) reosB = asin B [ ADS: r = e (3 + eos 2B) ]

(v) r = a(l +eosB) [ADS: r = c(l-eosB) I


(vi) r = aeosB [ADS: r = esinO I
2
(vii) r2 = a eos2B [ADS: r ? = C- Sin 20 )
?
"This page is Intentionally Left Blank"
2
Linear Differential Equations with
Constant Coefficients and Laplace Transforms

2.1.1 Let

(i) the differential operator" ~" be donoted by 'D'


dx
(ii) PI' P2' P3' ............. Pn be either functions of x or constants
(iii) R be a function of x then the general form of a linear differential equation
(L.D.E) of order n is given by
D nY + PI Dn- I Y + P2 Dn- 2Y + ................. PrJ! -- R .... (I)
or simply (Dn + ppn-I + ............ +Pn)Y = 0, If PI' P2' ..................., Pn are constants
then (I) is called a L D E with constant coefficients.
Denoting the differential operator (Dn + PI Dn - I + .............. + Pn) by f (D),
(I ) can be written as
f(D)y = R .... (2)

2.1.2 Consld er the D.E, dy


dx + Py = 0

Separating the variables dy = - P(x) dx


y
70 Engineering Mathematics - I

Integration yields logy= fp(x)dx => y =Ce -fp(x)dx + logc ..... (3)

Solution isy= Ce-


fpdx
Suppose p is a constant = -m, say, then solution is y = Cernx
(i.e.) the solution of (D - m)y = 0 is y = Ce rnx .... (4)
2.1.3 In this chapter the attention is mostly confined to L.D.E.S with constant coefficients.
If R = 0 then equation (2) becomes
f(O)y = 0
We shall take, here afterwards, f(m) = 0 as the auxiliary equation, (A.E).
2.1.4 Consider a second order L.O.E
(D2 + a\ D + ~)y =0 ..... (6)
A.E. is m 2 + aim + a2 = o.
Let m\, m 2 be the roots of this equation.
Now four cases arise.
(i) m\, m2 are real and distinct
(ii) m\, m 2 are real and equal
(iii) m\, m2 are complex and distinct
(iv) m\, m2 are complex and equal
2.1.5 Case i : (6) can be written as
[0 2 - (m\ + m 2 ) D + m\m 2 ] y = 0 or (D - m2 ) (0 - m\) y = 0 .... (7) -
Call (0 - m l ) y = y
(7) now becomes (0 - m 2) Y = 0

From (4) it follows that Y = Ce D12X

From (7) (D - m\)y = Ce D12X


I.F. = e-rn\X

Solution is y . e-m\x = C fe(m,-m')x dx + C\ .... (8)

C C
y.e-rn\x= e(rnrrn\)x dx + CI' Call as C 2 .
m 2 -m! m 2 -m!
Linear Differential Equations with Constant Coefficients ... 71

(i.e.,) y = C2 e-1Il 2x + C, elll,x where C, and C 2 are arbitary constants


Similarly ifm" m2, .......... I11n are real and dinstinct roots off(l11) = 0 then
y = C, e'n,~ -I- C 2 el11 2x -I- .............. + C n elllnx .... (I)

(I) is the solution of f(O)y = 0 where C" C 2, ........... , C n are arbitrary constants.

2.1.6 (Case ii) : m, = m2,


From (8) it follows that the solution of(6) is given by

:. y = (Cx + C,) elll,x


Similarly if m, is repeated say 'r' times then solution of(D - m,Y y = 0 is
y = (C, + C 2x + C 3x2 + .............................. + c;-r-') elll,x ..... (II)
(II) is the solution corresponding to a root m, repeated r times

2.1.7 Case (iii) : Ill" m2 are complex, say (a if3)


Then solution of(6) is given by
y = A eta I~)X + B e(a- IfJJX where A and B are arbitrary constants
(i.e.,) y = Ae ax . e Vlx + Beax.e-1jJx = eax [A(cosjJx + isinjJx) + B (cosjJx' - isinjJx)]
= eax [C, cos/lr + C 2 sin,lk] ..... (III)
where A + B = C, and i(A - B) = C 2
(III) gives the solution corresponding to two complex conjugate roots (a ifJ)
2.1.8 Case (iv) : If (a ifJ) are repeated say's' times then the corresponding solution
(follows from case (ii) and (iii) is given by
y = ~[(C, + C 2x + C 3x2 + ........................ + CsXS- ' ) ] cosf3x
+ (d, + d 2x + d 3x2 + ....................... + dsxS- ' ) + sinf3x] .... (IV)

Example 2.1.9

d2 y dy
Solve - 2 +5-+6y=O
dx dx
Sol. A.E is m2 + 5m + 6 => (m + 2) (m + 3) = 0
:.m=-2,m=-3.
Solution is y = Ae-2x -I- B e-3x
72 Engineering Mathematics - I

Example 2.1.10
d2y dy
Solve dx 2 -11 dx +30y = 0

Sol. A.E is m2 - 11 m + 30 => (m - 5) (m - 6) = 0


Solution is y = Ae 5x + B e6x
Example 2.1.11
d2y dy
Solve2 dx 2 +5 dx -12y=0

Sol. A.E is 2m 2 + 5m - 12 => (2m - 3)(m + 4) = 0


m =3/2, m=-4

3
:. Solution is y = C 1 e2x + c 2 e--4x
Example 2.1.12
d 3y d2y
Solve -+4--6y=0
dx3 dx 2
Sol. m3 + 4m2 + m - 6 = 0
(m - I) (m + 2) (m + 3) = 0
:. Solution is y = cl~ + c 2 e-2x + C 3 e-3x
Exercise - 2(a)
Solve the following differences:
d 2y dy
1. -+2--3y=0
2
Ans : y = A~ + Be-3x
dx dx
2
d y _ 3y = 0 Ans: y = A~ + Be-X
2.
dx 2
d 3y d2y dy
3. -+2--5--6y=0
3 2
Ans : y = Ae-X + Be2x + Ce-3x
dx dx dx
d2 y dy
4. 9 -+18--16y=0
2
dx dx

d2y dy
5. -+3-+2y=O
2
Ans : y = Ae-X + Be-2x
dx dx
Linear Differential Equations with Constant Coefficients ... 73

Solved Examples
Example 2.1.13

dZy dy
Solve -z +6-+9y=0
dx dX'

Sol. A.E is mZ + 6m + 9 =
i.e. (m + 3)z = 0, m = -3,-3
:. Solution is y = e-3x (A + Bx)
Example 2.1.14

d3 dZ
Solve~+~=O
3 2
dx dx
Sol. A.E is m + mZ =
3
=> m (m
Z
+ I) =
:. m=O,O,-1
Solution is y = (A + Bx)e oX + C e-x = (A + Bx) + Ce-X
Example 2.1.15
d4y dZy
Solve - 4 + 18-
1 + 81y=0
dx dx~

Sol. A.E. is m4 -18m 2 + 81 =


:. (mz - 9) (m z - 9) =
:. m = 3, 3, -3,-3
Solution is y = (A + Bx) e-3x + (C + xD)e 3x = (C(x + Cz)e-3x + (C 3x -t- C 4 )e 3x
Exercise 2 (b)
d2y dy
I. Solve -Z +10- +25y=0 Ans : y = e-5x (Ax + B)
dx dx

2. Ans : y = (Ax + B)e2x + cex

3. Ans : y = (Ax + B) + (Cx + D)~

4. Ans : y = (M + Bx + C) e-t
74 Engineering Mathematics - I

Solved Examples
Example 2.1.16

d2 y dy
Solve -2 +4- +9y=0
dx dx

Sol. A.E. is m2 + 4m + 9 =0
m=-2i/5
a = -2 and f3 = /5
:. Solution is y = e-2x (Acos x /5 + Bsin /5 x)
Example 2.1.17

d 3y
Solve dx 3 + y= 0

Sol. A.E. is m3 + 1 = 0
:. (m + 1)( m2 - m + I) = 0

1+J3i
:.m=-I,m=--
2

Example 2.1.18
d4 y d3 y d2 y dy
Solve -+2-+3-+2-+y=0
dx 4 dx' dx 2 dx

Sol: A.E. is m4 + 2m 3 + 3m 2 + 2m + 1 = 0
(m 2 + m + 1)2 = 0

-1J3i -1J3i
:. m=
2 2
Linear Differential Equations with Constant Coefficients ... 75

Exercise 2 (c)

I. Ans: y = Acos J7x + Bsin J7x


d 2y dy x x
2. Solve2- 2 +4- +3y=O Ans : y = e-x (Acos.fi + Bsin .fi )
dx dx

3.

d4 y
4. Solve dx4 -64y = 0

Ans : y = C\e2x + C 2e-2x + e-x (C 3cosfix+c 4 sin fix)

+ ~(C3cos.J3x+C6sinfix)

2.2.1 Consider the O.E.f{O) y = R .... (I)

Operating both sides with f(IO) (called inverse operator)

1 I
we have f(O) [f(O)]y = f(O) R

I
or y = f(O) R

This solution is called the particular Integral of(l)


while the solution off(O) y = 0
is called the complementary function (C.F.) ..... (2)
Suppose y\ is C.F and Y2 is P.I for (I)
Thenf{O) y\ = 0 from (2)
andf{O) Y2 = R from (1)
Hencef(O) [y\ + Y2] = f(0) y\ +f(O) Y2
=O+R=R
76 Engineering Mathematics - I

Which shows thatYI + Y2 is also a solution of(l)'Yl + Y2 (i.e.) C.F. + P.I is called the
most general solution of(l).
I
2.2.2 Calculation of R
D-I11,

D _ nl, R = y, say, Operating both sides with (0-m 1), we get (0-m 1) Y = R

dy
(i.e.) dx - m1y= R
f-mjtU
I.F.lse =e-ml x
:. Solution is given by
y.e-mf = fRe-lIl/ dx+c

(i.e) y = Ce-ml X + ell/IX fRe-1Il IX dx = C.r. + P.I

P.I.=e mI X fHlIllx dx

If m
I
=O~
'0
R= fRdx

I
Thus the operator 0 stands for integration

I
Example 2.2.3 Find the value of - D x
-5

Sol. D 5 ~ Fe" fe".xdx ~ <' 0" [ - +(' m} e'+ xe;" - ~;'l


x I
-----
5 25

Example 2.2.4

1 I
Sol. eX
02 _50+6 = (D-2){D-3) .(eX)

I I
= (0-2) . (0-3)
(eX) I 3x -3x x dx
= (0-2) e e.e
f =
I
(0-2) . e
3x

2
I -- r -2x .exdx = -e
I - (eX) = - -I e 2x Je f
I 2 x e-xdx=-e
I 2 x.e- t
2 (0-2) 2 2 2 2
Linear Differential Equations with Constant Coefficients ... 77

Example 2.2.5
Solve (02+a 2) y = tan ax

Sol. A.E is m2 + a2 = 0 => m = ai

C.F. is y = C I cos ax + C 2sin ax

P.I =
I
tan ax = -
I
- I- - -I - J tanax
(0+ ai)(O- ai) 2ai [ O-ai D+ ai

2ai [e lax J e -a/\ tan axdx - e -IllX J eat" tan axdxJ

r alX
Je - tanaxdx =
J. . sinax
(cos ax -Ismax) - - dx = JSin ax- i
(l-cos ax)
dx
2

cos ax cos ax

cosax i isin ax
= - -- - -\og(sec at' + tan ax) + - -
a a a

an dx cosax i isinax
lilly
Je tanax = - - - + - \og(sec ax + tan ax) - - -
a a a

= (Cosax+aisinax) [-cos ax + isin ax -ilog (sec ax + tan ax]


II

[-\- icos ax log(sec ax + tan ax) + sin ax \og(sec ax + tan ax)]


a

\
:. P.l. = - 2
? [-2icos ax log(sec ax + tan ax)]
a-,

cos ax
- - - - log(sec ax + tan ax)
a

:. Most general solution is

. cos ax
y = C 1 cos ax + C 2 SIl1 ax - - - 2- log(sec ax + tan ax)
a
78 Engineering Mathematics - I

Example 2.2.6
Solve (02-50+6)y = ~
Sol. A.E. is m2-5m+6 = 0
C.F. is y = C)e 2x + C2 e3x
I eX
P.1. isy= 0 2 -50+6 (eX) = 2 (from 5.2.4)

:. Complete solution (C.S.) is y = C.F. + P.I.

Exercise 2 (d)
I I
I. 0 cosx 3. 0 (x 2 )

I
4. 0-2 sinx

x3 (2sinx+ cos x)
Ans: (I)+sillx (2) e-x (3) 3 (4)
5

Solve the following:

d2y dy
6. dx 2 - 4 dx + 3y = e2x

d2 y
7. -2 +y=5x+3
dx

d2 y dy
8. dx 2 - 3 dx + 2y = e-2x

9.

10.
Linear Differential Equations with Constant Coefficients ... 79

xsin ax
II. Ans : y = C I cos ax + C 2sin ax + C 2sin ax + - - -
a

cos ax
+ -2- log cos ax
a
12. (D 2 + 9) Y = tan 3x Ans : y = C1cos 3x + C 2sin 3x

cos3x
- - - log(sec 3x + tan 3x)
9

Methods of finding P.1.


We shall now consider the methods for finding P.1. where R is of some special form

2.3.1 Particular integral


Where R is of the form eax
Case (i) iff(a) =F 0
Since De ax = aeax ; D2e ar = a2 ear ...... Dn(eax) = an.ear
j{D) ear = (Dn + K1D n- 1 + K 2 Dn- 2 + ...... K,)e ax
= (an + K1a n- 1 + .......... + Kn)e an = j{a) ear

Operating with _I- on both sides


feD)

- I [f(D)e a,] = - (-)


I [f(a)elU]
feD) f D

(i.e) eax = 1 f(a) eax = f(a). [ feD)


feD) I e ax]

1 a, 1 at
(or) --e = --e
feD) f(a)

1 1 1
Note: If K is a constant then feD) (K) = feD) K.eax = f(m)K

3x I 3 I 3
for example -D-:02-_-2-D-+-l e 9 _ 6 + I = e x = "4 e x
80 Engineering Mathematics - I

2.3.2 Case ii) If./{a) = 0 then it is possible to write./{O) as <1>(0) (O-ay where <I>(a) i:- 0
Suppose r = 1 then
1 I 1
P.1. = - - eax = - - - - e(lX
f(O) O-a <1>(0)

1 1 1
- - eax = - - - - eax
O-a $(a) $(a) O-a

= <l>(la) eax Je~a\ .eaxdx = <I>~a) ea\dx = <I>~a) ell<.x .... (I)

1 ax 1 I ax 1
ifr=2then f(O) e = (0-a)2 <1>(0) e = <I>(a) (0-a)2 ea~

1 1
- - [xeax]
$(a) O-a

1 1 2
e ax- = - - e ax- -X
P.1. = f(O) <I>( a) . L2
In general, if (O-a) is repeated 'r' times then
1 I x2
P.1. = f ( D) eax = $( a) ea~. --;:!

Solved Examples
Example 2.3.3
d 2y dy
Solve - + 4 - + 5y = 13e3x
d~2 dx

Sol: (0 2 + 40 + 5) = 13e3x
A.E. is m2 + 4m + 5 = 0, giving m = -2 i
C.F. is y = e-2x (Acosx + Bsinx)
I
PI (13e3x)
.. = 0 2 +40+5

replace '0' by '3'


13e3x 13e3x eh
9+ 12+5 26 2
Linear Differential Equations with Constant Coefficients ... 81

h
e
:. Complete solution is y = C.F. + P.1. = e-2X(Acosx + Bsinx) + 2
Example 2.3.4
d2 y dy J. dy
Solve dx 2 + 4 dx + 5y = 2e-- x , gIven that x = 0, y = I and dx =-2

Sol. Given equation isJ(O)y = 2e-2x were f(O) = 0 2 + 40 + 5


:. A.E. is m2 + 4m + 5 = 0, giving m = -2i
:. C.F. is y = e-2x (Acosx+Bsinx)
I 2e- 2 ,
e 2x 2x
P.l. = 0 2 +40+5 2- = 4-8+5 = 2e-

:. Complete solution is y = C.F. + P.1. = e-2x(Acosx + Bsinx) + 2e-2x .... (4)


Now yeO) = I, (given)
Substituting in (4)
yeO) = I(A) + 2; A =-\.
yl(x) = e-2x(-Asinx + Bcosx) -2e-2x (Acosx + B sinx) -4e-2x

yl(O) = B-2A -4, yl(O) = -2 (given)


:. -2 = B-2A-4, => B = 0
:. y(x) = e-2x (-cosx) + 2e-2x

= 2e-2x - e-2x cosx


Example 2.3.5

d3 y d2 y dy
Solve - 3 + 2 -2 + - =e 2x
dx dx dx

Sol. A.E. is m3 + 2m2 + m = 0 ; m(m+ 1)2 = 0


m = 0, -I, -I.
C.F. = C1eOx + (C 2 +C 3x)e-X

I 2x
e 2x
P.l. = 03 + 202 + 0 e = 18
82 Engineering Mathematics - I

Example 2.3.6_
d2 y dy
Solve - , -3 - +2y=e-~
dx- dx

Sol. f(O) = 0 2-30+2


A.E. is m2 _ 3m + 2 = 0 => (m - 2) (m - I) = 0; m = 1,2
here f(l) = 0

I I I I I
.. (0-1) (0-2) eX= (0-1) (1-2) e = (0-1) eX=-eX
X
~ (using 5.3.2.(3)

:. Most general solution is


:. y = CteX + C2 e2x - xe n
2.4.1 R is of the form sin ax or cos a\"
0 2 (sin ax) =:' (~a2i~ sin ax

(02f (sin ax) = (_a2)2 sin ax

(0 2)3 (sin ax) = (_a2)3 sin ax

(02)n (sin ax) = (_a2)n sin ax

Hence j(02) sin ax = j(-a2) sin ax, provided f( -a2) :;i: 0

1 2' _ 1 2 .
j(O ) SIl1 ax - (,)j(-a )SIl1 ax
1 (0 ) 2
1 0-

2
Qr sin ax = 1(02 ) j(-a ) sin ax

I. sinax
1(02 ) SIl1 ax = f(-a 2 )

Similarly it can be shown that I 2 cos ax ax = I, cos ax


f(-O ) 1(-a-)
Linear Differential Equations with Constant Coefficients ... 83

Ifj{O) vanishes when 0 2 is replaced by _a2 then we write

I 1 .
1ar
j(O) [sin ax] = j(O) [Imaginary part of e ]

1 .
= I.P of - - e 1ar
j(D)
1 1 .
Similarly j(O) [cosar] = R.P. of j(O) e 'ar

Solved Examples
Example 2.4.2

d2 y dy .
Solve - 2
-3 - + 2y = sm3x
dx dx

Sol. A.E. is m2 -3m+2 = 0; m = 1,2

I. 1 . -1. (3 D - 7) .
P.I., (sm3x)= 9 30 2 (sm3x)= (3 7 (sm3x)= 9D 2 49 (sm3x)
0- - 3D + 2 - - + D + .) +

(3D - 7 ) . 9cos3x + 7 sin 3x


= -81-49 (sm3x)=--0-2=-_-3D-+-2-

Hence complete solution is


84 Engineering Mathematics - I

Example 2.4.3
Solve (D3 -D) Y = sillX
Sol. A.E. is m 3 -m =0
m=O, m =1
C.F. is C1e.x + c2eX + C3e-x = C 1 + C2e x + C3e-x
I.Sit1X I I
P.1. = D3 _ D = D2 _ 1 . D (sitlX)

I -cosx x
= - - - . cosx= = --SillX
D2_1 -1-1 2

The complete solution is

y = C + C eX + C e-x -~SillX
I 2 3 2

Example 2.4.4
Solve (D2 - D + 1) y = cos2x

1J3i
Sol. A.E. is m2 - m + 1 = 0 ~ m = 2

~~ fi fi
:. C.F. = e 2 (Acos-x + B sin- x)
2 2
I I 1 (3-D)
P.I. = D2 _ D + 1 cos2x = cos2x = cos2x = - cos2x
- 4- D+1 -3 - D 9 - D2

(3 - D )cos2x I
13 =- 13 (2sin2x + 3cos2x)

C.F. + P.1. = e 2
x
fi BSIl1-x
( Acos-x+
2
. fi
2
1 _-1 (2sin2x + 3cos2x)
13

Example 2.4.5
Solve D2(D2+9) = sin2x + 5
Sol. A.E. is m2(m 2+9) = 0 ~ m = 0, 0, + 3i
C.F. = (C I + C2x) + C3 cos3x + C4sin3x
Linear Differential Equations with Constant Coefficients ... 85

P.1. = D2 (D2 + 9) sin2x + (D2 + 9) D2

+ ~._I_(I)
2
sin 2x = _ sin 2x + 5x
-4(-4+9z) 9 D2 20 18

Example 2.4.6
Solve (D 4 -2D 3 + 2D2 - 2D + 1 ) y = eX + sin2(x/2)
Sol. A.E. is f(D) = D4 - 2D 3 + 2D2 - 2D + 1
m4 -2m 3 + 2m2 - 2m + 1 = 0
= (m-l )2 (m2+1) = 0
:. m= I, I,m=i,-i
C.F. is (C t +C 2x)eX + (C 3cosx + C 4sinx)
1 1
P.1. = - - eX + - - sin2(x/2)
f(D) f(D)

_1_ . 2 _ 1 . 2 _ 1
f(D)
Sin (x/2) - (? ) Sin (x/2) -
0
(D-It D-+l
?
(D-It D +1
(2 )

~~ [(0_1),1(0' + I) (I-COSX)] ~ ~ [-I -20~


(0' 1)(0' + I) COS X 1
1 1 1
= - - --.-- (cosx)
2 - 4D D2 + 1
1 lID
= "2 + "4 . D2 + 1 .
(D2) (cosx)

= ~+~._l_.~(-sinx)
2 4 D2 + 1 (-I)
1 1 1
= "2 + "4 . D2 + 1 . sinx
86 Engineering Mathematics - I

1 1 1
[)2 + 1 sinx = I.P of 02 + 1 ex = I.P of (D +-i)---:(-D----:-i) ex

I x
= I.P of 2i XCiX = "2 I.P. of -i[cosx + isinx] = -~COSX

1 1
:. P.1. = "2 - "8 x cos.\"

:. Ilence the complete solution is

2
1 x r 1 x
Y =(C +C I 2
x)~+C cosx+C sinx+ -.-e +---cosx
3' 4 2 2! 2 8

Exercise 2(e)
Solve the following

(1)

(2) (D2+D+ l)y = sin2\"

Ans: J3
e"r ( C,cos-x+ Co'S IJj - ('J
/ 1 - X -1 SIll '....
.... cos 2x+ 3' J X )]
2 - 2 13

3 x
(3) (D6 + 1) y = sin"2 x.sin "2

jj-, x . x .fh x . x
Ans. (C1COSX + C 2sinx) + e- 2 (C3cos"2 +C 4 SIIl"2)+ e2- (Cscos"2 +C 6SIIl"2)

x 1
+ -sinx+ -cos2x
12 126
(4) (D3 + 4Di y = Sin2x
xsinx cos2x
Ans. y = C 1 + C2 cos2x + C]sin2x --8- -16
I
2.5.1 P.1. is of form feD) xm

I
- - xm = [f(D)rl.x m
feD)
Linear Differential Equations with Constant Coefficients ... 87

Now expand [f(0)r 1 in ascending powers of 0 and retain as far as 0 111 and then
operate on xl11
Solved Examples
Example 2.5.2
Solve (0 2 + 50 + 6) y = x
Sol. A.E. is m2 + 5m + 6 = 0 => m = -2,-3
C.F. is C 1e- 2x + C 2e- 3x
2
1 1 (I 0 + 50)_1
P.1. = 0 2 + 5D + 6 x ="6 + 8 x

I 50 I 5 x 5
= - [1--] x= - [x--] = - ---
6 6 6 6 6 36
x 5
Solution is therefore)' = C e- 2x + C e-3x + - - -
I 2 6 36

Example 2.5.3
d3 v dy
Solve _ . - 3 - -2y=x2
dx' dx
Sol. A.E is m3 - 3m -2 = 0 => (m-2) (m+ 1)2 = 0
C.F. is C 1e 2x + (C 2 + C3x)e-X
2 3

P.1. = ,
x (1_(D -3D1,.2
2 .,
0- -30-2 2
1 0 3 -3D 02 -30 , .,
=-- [1+ +( t+ ....... ]x-
222

_ -I 3D 9 2 2 _ 1 2 6x 9 _ I 2
- - [ I - - + - D ] x ---[x --+-] - - [2x -6x+9]
224 2 224

Complete Solution is y = C 1e 2x + (C 2 + C 3x)e-X - ~ (2x 2 - 6x + 9)


4
Exercise 2(f)
I. Solve (03 - 302 +2)y = x
88 Engineering Mathematics - I

2. Solve (D2 + 2D + 3)y = x +.~


14-6x
Ans. y = e-x (C I cos fix + C2 sin J;:) +
27
3. Solve (D2 + 2D + I) Y = cos2x + x 2

Ans. y = (y + C e-> -~(3COS2X -


2 2sin 2x) + (x 2 - 2x)

4. (D2 - 4D + 4) y = x 2 + sin2x + e3t"


, 2X2 + X + 3 cos 2x 3
Ans. y = (C I + C2x)c~X + - - 8- -8- + e x

2.6.1
I
To find - - (e<txy) where Y is a function of x
feD)

D(elU"Y I) c<l\D(Y I) + aelU"Y I


= = ea\"(D+a)YI
Similarly D2(ea:.V I) = e aY (D+a)2y I
D3(elU"Y ) = e<tt"(D+a)3YI
1

Dn(elU"Y I) = ea\"(D+a)nYI
feD) (ea\v I) = e<tt"f(D+a)Y I

Calling f(D+a)Y I = Y, we get Y I = f(D+a) Y

I I I
feD) rea, Y] = elU"Y or elU" Y - - - elU"Y
f(D+a) f(D+a) - feD)

_I_ elU" Y = elU" I Y


feD) f(D+a)
2.6.2 Note

1. While finding _1- elU" when f(a) = 0 we can employ the above method for
feD)
Linear Differential Equations with Constant Coefficients 000 89

2. Further _1_,_ [cos ax or sin ax] when f( -a2) = 0 we can write


/(0-)

_1_2- [cos ax or sin at]


f(O)

1 \
= f(02) [R.P. or I.P of eaT] = R.P. or I.P. of e1a\". f(O + ia)2 .\

Solved Examples
2
dy dy
2.6.3 Solve: -2 - 2 - - 4y = tfcosx
dx dx
Sol. A.E. is m2 - 2m + 4 = 0
m= \ fJi
C.F. is eX(C I cosfJx+CzSinfJx)
1 1
P.1. = tf cosx = eX? cosx
0 2 -20+4 (0+\)"-2(0+\)+4
\
= tf 02 + 20 + 1- 20 _ 2 + 4 (cosx)

I ~X ) x cosx eo' cosx


= t;(cosx = e .
- -
2
=---
0 +3 _12 + 3 2
:. Complete solution is y = CoF. + P.1.
e'cosx
y = eX (C I cosfJx + C 2sin fJx) + 2

Example 2.6.4
Solve (02-2)y = tfcosx

Sol. A.E. is m2 - 2 =0 m= J2
C .F, is C Ie.fix + C 2 e~.fix

1 1
P.1. = (0 2 _ 2) tf cosx = tf. (0 + 1/ _ 2 cosx = tf. 0 2 + 20-1 . cosx

= tf. . cosx
20-2
(20+2) eX(-2sinx+2cosx) eX .
= tf . cosx = = -(Sin X - cosx)
40 2 -4 -4-4 4
90 Engineering Mathematics - I

eX
Y = CeJ2x +C e-- J2 + -(sinx-cosx)
I 2 4

Example 2.6.5

3
d d
Solve ---.E.
3
_7~ - 6y = e 2x + xe 2x
dx dx
Sol. A.E.ism 3 -7m-6=O:. m=-1,3,-2
:.C.F. = Cle-x + C2e3x + C3e-2 t"
(1+x) 2x
P.1. = 3 e
o -70-6
1
=e2x (I+x)
(0+2)3 -7(0+2)-6

= e2x ~------
3 2
0 -60 +50-12

_e2x 50 _e 2x 5
(1+-)(1 +x)= - [I+x+-]
12 12 12 12

_e 2x 17
= - [x+-]
12 12

x 3x 2x
e 2x 17
Y = C I e- + C2e + C3 e- - -
12
[x + - ]
12

Example 2.6.6

d3y d2y dy
Solve - -3 3 - +
2
3 - - y = eX(1 +x)
dx dx dx
Sol. A.E. is m3 - 3m 2 + 3m - 1 = 0 or (m - 1)3 = 0 or m = I, I, I.
C.F. - (C I xl + C2 x + C3 )eX
Linear Differential Equations with Constant Coefficients ... 91

P.1. = (0 _ 1)3 . (x + I) ~

e'.(x+l) 1 .1 x2
= (0+1-1)3 =~. D3 (x+ 1)=tf. 02 (T+ X
)

x3
X4
Complete solution is y = (C 1 x + C~ + C3)2
~ + ~ (24 + (;)

Example 2.6.7

d~y dy
Solve - , - 4- +y = e2x sin2x
de dx
Sol. A.E. is m2 - 4m + I = 0, m = 2 J3
c.F. = C,e(2+J3)( + C e(2 ,)1)(
z
I eh
PI - e2x sin2x = sin2x
.. - OZ-40+1 (D+2)2-4(0+2)+1

e 2x e 2x sin2x 1"
= -2
- sin2x = = -- eX sin2x
0 -3 -4-3 7
Complete solution is

Exercise 2(g)

I.

3x

Ans : y = (C 1 + C 2x) ~ + 8e (al - 4x + 3)

2. Solve (03 - 70 - 6) Y + e2x + x 2e3x


1 2 +-x+-)
Ans:y= C 1e-x +C 2e- 2x+C 3e 3x_e 2X_(x 5 169
12 6 72
3. Solve (0 2 - 20 + 4)y = eXcos 2x

Ans: y = ~(C, cos3Fx + C 2 sin3Fx)+le


x
-~sin 2x
92 Engineering Mathematics - I

4.

Ans. y = Clcosx + C2 sinx -~ ~(2cosx - sinx)


5

5.

13 C3sm-x
Ans.y= C le- x +e7i (C 2COS-X+ ' 13) - - 1 e 2x.(11cosx-3slnx)
.
2 2 130

2.7.1 To find I~O) [xv] where v is a function ofx


O(xv l) = xOv l + vI
02(xv l) = O(xDvl + vI) = x02v I + 20v I = x0 2v I + (0 2) vI
Similarly 03(xv l ) = x03v I + (03) vI
in general on((xv l ) = xOnv l + (on) vI
(here 1 indicates differentiation w.r.t '0')
Thus, f(O)(xv l ) =o.x(O)vl + fl(O)vl ..... (1)
now let f(O)vl = v
1
vI = I(O)'v

1 1
From(1) j(0) [x'/(D) v]=xv + fl(O)[/(O) v]

Operating both sides with I/O)

_1_ _ _1_ __1_ I _I_


x /(0) v - 1(0) (xv) + I(O)f (0) [/(0) .v]
Linear Differential Equations with Constant Coefficients""" 93

2.7.2. Note: For finding [xlll.(sinax or cosax)] when m > 1 the above method becomes very
lengthy. Hence we can take it as [Xlll. (R.P or I.P of e/a,"] and apply the method
(2.6.1 ).
Solved Examples
Example 2.7.3
Solve (D 2 + 9) y = xsinx
Sol. A.E. is m2 + 9 = 0 =>m = 3;
1 .) 1 I .
P. I. -2-(XSIllX = {x--,-.2D}.-,-.sll1x
="
D +9 D-+9 D-+9
[using the result of2.7.1. v = sitlX]
1 I. x. 1 2
= {x--,-2D} . - Stox= -Stox- - - - cosx
D" + 9 8 8 D2 + 9 8
x. 2 1 x. 1
= 8"
Stox - 8" . 8" . cosx = 8"
SltlX - 32 cosx

Complete solution is y = C1cos3x + C 2sin3x + ~ sinx - _I cosx


8 32
Example 2.7.4

d2 y
Find the solution of - , + 9y = xcos2x
dx-

Sol. A.E. is m2 + 9 = 0 => m = ; Jj

1
and P.1. = D2 + 9 (xcos2x)

1
= Real part of D2 +9 x(cos2x + isin2x)

= R.P. of -1- xe 2"IX = R.P. ofe2"IX 1 2"


x = R.P. ofe IX
1 x
D2 +9 (D+2i)2 +9 D2 +4iD+5

1+ D2 + 4iD )~t x= R.P. -e


21x 21x
= R.Pof -e ( (
1- D2 + 4iD) x
5 5 5 5
94 Engineering Mathematics - I

2lX
e 4iO 0 2
= R.P. of -5- [I ----+----]x
. 5 5
e 2lX 4i
= R.P. of -5- [x- 5]

I 4
= 5 [xcos2x + 5 sin2xJ
Most general solution is

Y = C ,cos3x + c 2sIn. 3x + -1 [xcos2x + -4 Sin


. 2X ]
5 5
Example 2.7.5
Solve (0 2 - 40 + 4)y = 8e2x x2sin2x
Sol. A.E is m 2 - 4111 + 4 =0 ~ m = 2, 2
C.F. is (C,x + C 2)e2x

I -- (0 -8 2)2 e2x . x-
')s,n2x - 8e2x 1 x2sin2x = 8e2x _1_ ') sin2x
P.. - (0 + 2 _ 2i 0 2 x-

= 8e2x [I.P of ~2 x 2 e 2ix ] = 8e2x[I.P. of e2ix . 1 x2 ]


(0-2i)

- 8 2x[I P f 2ix 1 x2 ]
- e . 0 e -4+4iO+02

-1 4iO+02
= 8e2x [I.P of e 2IX (4) [1 - 4 r I,)
x-

. 4iO+ 0 2
= -2e2x[I.P. of e 21x 1- 4 r l
x2

2 2
= -2e2x [J.P. of e 2ix [1 + 4iD + 0 + (4iO + 0 )2]x2
4 4
2
=_2e2x [J.P ofe2 ix [x 2 + 8ix+2 + 16i 02 (x 2 )]
4 4
2.<
= - ~ [J.P. of (cos2x + isin2x) (4x 2 + 8ix - 30)
2
-e 2.<
= -2- .J.P. of [4x2cos2x + 8ixcos2x - 30cos2x + i4x2sin2x - 8xsin2x - 30isin2x]
.
= -e2x[4xcos2x + 2x2sin2x - ISsin2x]
Linear Differential Equations with Constant Coefficients ... 95

General solution is y = C.F. + P.1.


Y = (C IX + C 2)e 2\" - e2~[2x2sin2x + 4xeos2x - 15sin2x]

Exercise 2(h)
1. Solve (0 2 -1) Y = x 2eos3x

1 26 6
Ans. y = Cle-' + C2e- x - 10 (x 2eos3x - 50 eos3x - "5 xsin3x)

2. Solve (0 2 +4)y = xsin 2x

X I
Ans. y = Cleos2x + C 2sin2x + "4 ["2 + sin2x]

3. Solve (04 + 202 + 1)y = x 2eosx

I I 3
Ans. y = (Clx + C 2 )eosx + (C 3x + C 4 ) sinx - 48 (x 4-9x2) cosx + x sinx 12
4. Solve (0 2 - 20 + 1)y = xe-'"sinx

I 4
Ans. Clcos3x + C 2sin3x + "5 (xcos2x + "5 sin2x)J
5. Solve (0 2 - 60 + 13)y = 8e3xsin2x
Ans. y = e3x[Clcos2x + C 2sin2x] - 2e3x xcos2x
-:::: xfj
6. Solve (04 + 0 2 + 1)y = e 2 cos (--)
2

-x
1 e2 1 1 1
+ C4 sin(-xfj)] + - [xcos(-xfj) + r::; xsin(-x.J3)]
2 4 2 -..13 2

d2 y
7. Solve - , + Y = e-'"sinx
dx-

I
Ans. Clcosx + C 2sinx - "5 e-'" (2cosx - sinx)
96 Engineering Mathematics - I

2.8.1 Cauchy's Homogenious Linear equation


Most general form of the homogeneous equation is
(xnon + K1x ll- 1 on-l + ..... + Kn- 1 xO + =X .... (I)

Where kl' k2' ........ kn are constants and 'X' is either a constant or a function of x
To solve such equations it is convenient to transform them into linear equations with
constant coefficients with the substitution x = eZ i.e. z = logx.

dy = dy dz = ~ dy .... (2)
d\: dz' dr x dz

I 'dy 1 d 2 y dy
= -l----t----] ( .: dz = x)
x x dz X dz2

__, d2y dy
- 2(d 2 - d ) ..... (3 )
x z z

..... (4)

d
By taking '0' to stand for dz'

dy dy
(2), (3), (4) ~ x dx = dz = Oy

..... (5)

dn
Xll -Z = 0(0-1) (D-2) ......... (O-n+l)y
dx"

By substituting (5) in the given equation (') it transforms into a linear differential
equation with constant coefficients.
Linear Differential Equations with Constant Coefficients ... 97

Solved Examples
2
d y dy
2.8.2 Solve x 2 m"2 -x d~" + 4y = 0
Sol. Writing x = el. (i) reduces to
d
[D(D-I) -D + 4tv = 0, where D = -
dz
(D2_2D+4)y=00
A.E. is m2 - 2m + 4 = 0 :::) m = 1 fi i

:. C.F. is given by y = (C 1cos,,'3z+C 2 sinfiz)e


I

substituting z = logt,
Y= Clcos (J3logx) + C~sin (J310gx) e logx
y = [C ICOS (fi log x) + C 2 sin (fi logx )lx

d 2y dv
2.8.3 Solve x2_~ - 3x-
" - 4y = 2x2 ..... (i)
dx- dx
Sol. Writing x = e Z (i) reduces to
[D(D-I) + 3D - 4lY = 2e 2/ ,
(D 2 - D + 3D - 4)y = 2e 21.

A.E. ism 2 +2m-4=0 ~m=-1 J5


C.F. is y = (C1e (I.JS)L + C 2e (h'5)L)

2 ")_ 2 2_ 2e 2z e 2z
P.1. = -D-2-+-2-D - 4 (e--) = 4 + 4 _ 4 e - = -4- = 2
Most general solution is

y = ( C e-(Il /"S)Z
1
+C
2
e (I JS)7 + e~z 1
98 Engineering Mathematics - I

2
2.8.4 Solve.xl --?-
d
d\'
2y = x 2 +-
X
1

Sol. Taking x = eZ equation (i) reduces to


[D(0-1)-21v = e 2x + e-Z
A.E. is m2-m-2 = 0 ~m = 2,-1
:. C.F. is y = C l e2x + C2e-z

1 I 1 1
P.1. = e2z + e--Z = e2z + e- Z
(D-2)(D+l) (D-2)(D+l) (D-2)(2+1) (-1-2)(D+I)

1 ~_ I __
---e-- - e-
3(D-2) 3(D+I)

=e2z I (I) e-3 (I)


3(D+2-2) - 3(D-I+l)

= .!. e2z . .!. (I) - .!. e-z .!. (1 )


3 D 3 D
1 _ I
= - ze2x - - ze-z
3 3
Complete solution C.F + P.1.
1
I.e. Y = C e2z + C e-= + - z( e2x -- e-Z )
I 2 3

I
Y = C1 + C e-Iogx + - (e2logx
e210gx 2 - e-1ogX)logx
3
1 1 1
Y = C x 2 + C - + - (x 2 - - )Iogx
I 2 x 3 x

Sol. Taking x = ez the equation reduces to


[D(O-I) (0-2) + 2D(0-1) +21v = 10ez + IOe-z
(D 3-D 2+2)y = IO(ez+e-Z )
Linear Differential Equations with Constant Coefficients ... 99

A.E. is m3-m 2+2 = 0


=> (m+ I) (m 2 -2m+2) = 0
(i.e.) m =-1, m = I i
:. C.F. is y = C1e-z +e-'(C 2cosz + C 2sinz)
10 IO z
P.1. = J , e= + D3 _ D2 + 2 e-
D' - D- +2
10e: 10e- z
+ - - - - :2 : - - - -
1-1+2 (D + 1)(D - 2D + 2)

= 5e-'z + 10 e -L = 5e -L + 2e -L . - -I - - I
5( D + I) (D - I) + 1
1
= 5e-z + 2e-z D (I) = 5e-z + 2e-z . z

Complete solution = C.F. + P.1.


= C1e-z + e=(C 2cosz + C sinz) + 5e-z + 2e-z . z logx
3
x

C1
= - + x(C 2cos(logx) + C3sm(logx)) + 5x + 2
x

d 2y dy .
2.8.6 Solve x 2 dx 2 + x dx + Y = logx sm(logx)

Sol. Writing z = logx the equation reduces to


[D(D-I) + D+ IlY = zsinz
(D 2 + l)y = zsinz
A.E. is m2 + 1 = 0 => m = i
C.F. is y = C1cosz + C2 sinz
1
P.I. = -2- zsmz
o +1
1 1
= I.P. of -2- ze 'Z = J.P. of e'z z
o +1 (0+i)2 +1

1
= I.P. of e'z ---:-,----z
0- -1+20i+l
100 Engineering Mathematics - I

= I.P. of e= I Z
D2 +2Di

= I P of e1z -
1 (
1+ -
0)'-1 Z
. 2Di 2i

= I.P. of e1z _I_(z_~)


2Di 2i

=IPof-- - - -Iz _e .i(z2


IZ
J
. 2 2 2i

= Imaginary part of [2 .(cosz +


-I . .
ISInZ)
[22 + 2.J]
Z U

Z2 I. z.
= - - cosz -- ZSInZ = - - (SInZ + 2zcosz)
2 4 4
Hence complete solution is

= C,cos(logx) + C2sin(logx) - lo;X [sin(logx) +2(logx) cos(logx)]

Example 2.8.7
Solve(x2 02-xO+4)y = cos(logx) + xsin(logx)
Sol. Taking x = eZ or Z = logx the equation reduces to
[0(0-1) -0 + 4lY = cosz + eZ sinz.

A.E. is m2 - 2m + 4 = 0 => m =1J3i


C.F. =ez (C,cosJ]z+C 2sinJ]z)
I I .
P.I. = 2 COSZ + 2
Z
e smz
D -2D+4 D -2D+4
Linear Differential Equations with Constant Coefficients ... 101

L
1 e
= --.cosz+ 1 smz
3-20 (D+I)--2(0+1)+4

(3+20) eZsinz 3cosz-2sinz eZsinz


---cosz+ - - = +--
9+4 3 13 3
The complete solution is y = C.F. + P.1.

. ~ 1 . xsin{logx)
= x[C1cos J3 (Iogx) + C2Slll v(logx) ] + 13 [3cos(logx) -2sll1(1ogx)] + 3

Example 2.8.8
Legendre's Linear equation:
An equation of the form
d ll dll-1y
(ax+b)11 dx~ + K1(ax+b)n-l dx n - I + ...... + koY = X
Where K's are constants and X is either a constant or a function of x is called
Legendre's linear equation. To solve such an equation we transform it into linear equation
with constant coeficients by the substitution.
ax + b = eZ or z = log (ax +b)
dy = dy dz = _a_ dy
Now
dx dz'dx ax+b'dz

dy dy d
(ax + b)2 - = a- = aOy where 0 = dz
dx dz

3
d
I

Similarly (ax + b)3 ---f


dx
= a3 D(O-l) (0-2)y

by substituting these values the given equation reduces to linear equation with constant
coefficients.
102 Engineering Mathematics - I

Solved Examples
Example 2.8.9
d 2y dy .
Solve (J+x)2 dx 2 + (I +x) dx + Y = sm2[log(l +x)] .... (I)

Sol. Write I +x = e => log( I +x) = Z


dy d2y d
(I+x) dx = Dy, (I+x)2 -? = D(O-l)y where D = -
dz
dx-

Substituting these values (1) reduces to


(D2_D+D+ I)y = sin2z; (i.e) (D2+1)y = sin2z
A.E. is m2 + I = 0 => m = i
C.F. = C ICOSZ + C2sinz
1 sin 2z sin 2z
P.1. = -2 - sin2z = -- = --
0 +1 ~4+1 ~3

IS = C .F + PIC
C omp Iete so IutlOn . = lCOSZ + C' sin32z
2Sll1Z - - -

. sin 2(1og(l + x))


= C1cos[log(l+x)] + C2slI1[log(l+x)]- "
.J

Example 2.8.10
d 2y dy
Solve (2x+3)2 dx 2 +6(2x+3) dx + 6y= log(2x+3)

Sol. Writing 2x+3 = eZ or z= log(2x+3)


the given equation reduces to [2 2.D(0-1) + 6.2D + 6]y = z

d
i.e. (4D 2+8D+6)y = z where D = -
dz

A.E. is 4m2 + 8m + 6 = 0 => 2m2 + 4m + 3 = 0


~2J2i 1
=>m= 2 =-l+J2i

C.F. is y = e-Z [Acos ~ z + Bsin ~ z]


Linear Differential Equations with Constant Coefficients 000 103

I 1 1 I 40 :m 2
Pol. = 1 z = - 40 2D2'Z = -6 [1+- +-3-rl z
40- +80+6 6 3
1+-- + ----
3 3

1 z
= -
6
r1--40
3
]z=
1 4
- [z- - ] = - --
6 3 6
2
9

Complete solution is y = C.F. + P.1.

=e-Z[ Acos r;::z+


1 B' z --
1 ] + -
SIl1-Z 2
..;2 ,fi 6 9

I 1 1 1 ')
= -2
x+..;2..;2
e
3 [Acos( r;:: log(2x+3+ Bsin( log(2x+3))] + -6 log (2x+3)- ::..
9

Exercise 2(h)
Solve the following differential equations
I. (x 202 - 3xO + 4) Y = 2x2
Ans. (C I + C2logx~ + x 2 (Iogx)2
2. (x 2D2 - 3xD + 5)y = sin(log(x
Ans. x 2(C Icoslogx + C2sinlogx) + (coslogx + sinlogx)
3. (x 2D2 - xD +2)y = xlogx
Ans. x(Clcoslogx + C 2sinlogx) + xlogx
4. (x 2D2-3xD+5)y = x 2 sinlogx

Ans. x2(C)coslogx + C2sinlog~) - ~ x2 1ogxcoslogr

2 2 (_,-s_in_l--,og~x-.:.)_+_1
5. (x D -3xD+l)y
= -
x

Ans. x2[C Icosh (filogx) + C.,sinh( J3log~) + _I + _1- [5sin(logx) - 6cos(\ogx)]


- 6x 61x

Ans.
104 Engineering Mathematics - I

8. (x20 2+xO+I)y= (I0gx)2 + sin(logx)


Ans. C I coslogx + C2sinlogx + (Iogx? -x[2coslogx - sinlogx]
d 2y dy
9. (2x-I)2 -2 + (2x-l) - +2y = 0
dx dx
C
Ans. y= C 1(2x-l)+ ~
2x+ I

d2 dy
10. (5+2x)2 dx; - 6 (5+2x) dx + 8y = 6x

Ans. y = C (5+2x)2+.fi + C (5+2xi t fi _ 3x _ 45


I 2 2 8
2
11. (2x+I)2 d y _ 6 (2x+l) dy + 16y = 8 (l+2x)2
dx 2 dx
Ans. y = C 1 + C2 log(2x+ 1) + [log(2x+ 1)2](2x+ 1)2
d2y dy
12. (2x+3) - 2 -(2x+3) --12y=6x
dx dx

Ans. C 1(2x+3)2 + Ci2X+3f~ - 2x + %


2.9.1 Linear Differential equations of second order

d 2y dy
The general form is dx 2 + P dx + Qy = R

where P, Q, R are functions of x.


Method of variation of Parameters
Let the C.F. of the equation
2
d Y + P dy + Qy = R
dx 2 dx
be y=Au+ Bv
where A and B are constants and u, v are two independent solutions-,of
d 2y dy
dx2 + P dx +Qy = R .... (I)
Linear Differential Equations with Constant Coefficients ... 105

be y = Au + Bv .... (2)
where A and B are constants and u, v are two independent solutions of

d 2y dy
- 2 +P-+QllJ= 0 .... (3)
dx dx .J'

such that 1I2 + PU I + Qu = 0 and v2 + PV I + Qv = 0


d 2u du
where tl2 = dx 2 ' ul = dx etc.

In order to obtain the solution of the equation (I) the arbitrary constants A and Bare
treated as arbitrary functions of x and are chosen in such a way that
y = A(x)u + B(x)v satisfies (1) .... (4)
Differentiation of(2) gives
= All, + lIAI + BVI + vB, (suffixes indicating the order of the derivative)
Now we chose A and B such that Alu + BI V = 0 .... (5)

So,

Again differentiating

d2y
dx 2 = (Au 2 + lIIAI) + (Bv2 + vIB,) .... (6)

substituting (4), (5) and (6) in (I) we get


[AlI 2 + BV2 + Alu, + Blv,] + P[Au, + Bvd + Q(Au + Bv] = R

(or) A(u2 + Pu, + Qlt) + B(v2 + PV I + Qv) + Allt, + BI vI = R

Alu, + Biv i = R (u and v are solutions of(3) .... (7)


Solving (5) and (7) we get
dA -vR dB uR
- =A = and - = B = ---
dx I uv, --u, v dx I 1/V, -II, v

integrating we obtain

A(x)= J -vRdx +CI;B(x)=


(uv, -1I,v)
J uR
lIV, -u,v
+C
2
.... (8)

where C I and C 2 are arbitrary constants


Substituting (8) in (4) we get the complete general solution of the equation (I)
106 Engineering Mathematics - I

Working Rule: First find two independent solutions 1I and v of(3). Then C.F. is given by
y = Au+Bv where A and B are arbitrary constants. Treating A and B as functions of x, we
have the solution of (1) as y = AI(x) 1I + BI(x) u where A(x) and B(x) are given by (8)

Solved Examples
Example 2.9.2

d2 2
Solve by the method of variation of parameters ~- y = --x
dx 2 1+c

Sol. A.E. is m2 - I = 0 => m = I


C.F. is y = A~ + Bc-x
Assuming A and B as functions of x such that the given equation is satisfied by
y = Ac-'" + Bc-X we have

dy dA dB
= AcX - Be-x + ~-+e-x- = A~ - Be--x .... (I)
dx dx- dx

dA dB
Choosing A and B so that ~~ + e--x dx = 0 .... (2)

.... (3)

Substituting (I) to (3) in the given equation, we get

dA dB 2
~- +e-x - =-- .... (4)
dx dx l+c x
dA -x
Solving (2) and (4) we get -dx = _c_ .... (5)
eX + I
X
dB C
and - =--- .... (6)
dx eX + I

= log (~+ I) -e-x-x


Integrating (5) and (6) A = _e- x + log( 1+e-') - x + C" B = -Iog( 1+~) + C 2
Linear Differential Equations with Constant Coefficients ... 107

Hence the solution is


y=[-e-x+log(ex + 1)-x+C,1~+[C2-log(1 +~)]e-X

(or) y=C,ex +C 2 e-x -1 +elllog(e'+I)-x]-e-'\'log(el+I)J

Remarks: If(i) I +P+Q=Otheny=~(ii) I-P+Q=Otheny=e-'x


and (iii) P + Qx = 0 then y = x are solutions of D2y + POy + Qv = 0
Example 2.9.3
Solve by the method of variat ion of parameters

d 3y d
- \ + (I - cOlt") ~ - cotx = sin 2x .... (I)
(ix" dx
3
d y dv
Sol: dx 3 + (I - cotx) dx - cotx = 0 .... (2)

C.F.P = I - cott Q = -cou comparing with original eqn.


:. I - P + Q = I - I + colx - cotx = 0
showing that v = e-X is a solution or (2)
To find another independent solution of(2)
let 11 = eO-x . w

By this substitution, the equation (2) reduces to

d 2 w dw 2 -x
-+-(l-cotx--e )=0
dx 2 dr c- x

or
d2w dw
- 2 = (l+cotx)- ~
i(~)
dw = I + cou
dx dx
dX

dw dw
log - = x + logsinx or - =eTsinx
dx dx

:. w= JeXsinxdx=- e; (cosx-sinx)

eX . I .
u = e-x [--(cosx-smx)] = - - (cosx - Slnx)
2 2
The second independent solution can be taken as cosx - sinx
108 Engineering Mathematics - I

Thus solution of(2) is


y = A (COS X - silu) + Be-x
For finding the solution of (I) we treat A and B as functions of x
i.e. y = A(x) (cosx - sinx) + B(x).e-x is the solution of (I) where A(x) and B(x) are
obtained by solving
(cosx - sinx) A I + e-x . BI = 0 .... (4)
and .... (5)
dA I
solving (4) and (5) ~ = -2 sinx

and -dB = ~--


sin2x
-ex
(I-cos2x)
--'-----'-
dx 4 4
integrating we get

A = - -I f'slflxdx+C 1 =--+C
cosx
1 .... (6)
2 2

and .... (7)

Hence the complete solution is

y = C 1(cosx - sinx) + C2e-x -~ (sin2x - 2cos2x)


10

Example 2.9.4
Solve by the method of variation of parameters
2
(I - x) d y + x dy _ Y = (I _ x)2
dx 2 dx
The given equation can be written as
d 2y x dy I
dx 2 + I-x dx - l-x Y = I-x .... (I)

x I
p= _ . Q= - andX= I-x
I-x' I-x
Clearly P + Qx = 0
. . d2y X dy I
Hence y = x IS a-solutIon of - 2 + -I- dx - --y = 0 .... (2)
dx -x I-x
Linear Differential Equations with Constant Coefficients ... 109

To obtain the second independent solution of(2)


take y = vx

d 2v dv x 2
Then (2) reduces to -'} + dx (-+-.1) = 0
dx- I-x x

d . dv dv x 2
- [-]+- [-+-] = 0
dx dx dx I-x x

d dv
~~ x 2 I-x-I 2 1 2
- - =- - - - =+( ) -- =+1----
d,,- 1- x x I- x x I- x x
dx

lo.g(:) =x+ log(x-I)-logx2

dv x-I 1 (-I)
- = -.e-t=
2
e-t [-+-, ]
dx x X x-

1
:. v = e"'" .-
x

The second independent solution is xv = e"'"


solution of equation (2) is Y = Ae"'" + Bx .... (3)
To find the solution of (I) treat A and B as functions of x such that
dA dB
e"'" - + x - = 0 .... (4)
dt" dx

dA dB
and, e"'" - + x - = I -x .... (5)
dx dx

dA dB
Solving (4) and (5) we get d; = -xe-X and dx = I

Hence the complete solution is y = A e"'" + B.x


= [C1+e-x(I+x)]e"'" + (x+C 2 ) x = C1e"'" + Cr + x 2 + (I+x)
110 Engineering Mathematics - I

Exercise (i)
Solve the following by the method of variation of parameters
d 2y
I. - + a2y = sec ax
dx 2

I
Ans. y = C,cos a:r + C'2sm x .
ax + -SIl1 ax + -cos ax Iog(cos a,)
a a

d2 y
2. dx 2 + Y = tanx

Ans. y = -[Iog(secx + tanx)] cosx + C,cosx + C2sinx

d2 y
3. --T
dx-
+ 4y = cosec 2x

Ans. y = C,cos 2x + C2sin2x + xcos 2x + sin 2~ log(sin 2x)

d2 dy
4. x2~-2x(l+x)
2
-+2(1 +x)y=x3
dx dx
2
x x
Ans. y = C, xe 2x + C r - 4-"4
3
Mean Value Theorems and
Functions of Several Variables

3.1.0 This chapter deals with


(i) Rolle's theorem
(ii) Lagrange's mean value theorem also called as first mean value theorem.
(iii) Cauchy's Mean Value theorem
(iv) Higher Mean Value theorems.
(v) Curvature (vi) Centre of curvature.
(vii) Evolutes (viii) Envelopes

3.1.1 Rolle's Theorem


Ifj{x) is (i) continuous in [a, b], (ii) differentiahle in (a, b)
and (iii)j{a) = j{b),
then there exists a'c'E(a,b) 3f'(C)=0
Proof: Suppose
(i) f (x) is a constant function throughout the interval [ a, b ],
then f' (x) = 0 VX E ( (I, h) Hence theorem is proved ...
(ii) f (x) is not a constant function in [ (I ,h ]. As f(x) is continuous in [a,b],
there exists a maximum value say, at 'c' (a ~ C ~ b) and a minimum value,
sayat'{f (a~d~h) for f(x) in(a,b).
112 Engineering Mathematics - I

f (c) 7:- f (d) and at least one of them is different from f ((l) == f (b)
Suppose f( c) 7:- f( a) and 'c + h' be a point in the neighborhood of 'c' ,.
/(c+II)- /(c)
then $; 0 when II> O. .... ( 1)
h

/(c+I1)- /(c)
and ;::: 0 when h < O. .... (2)
Iz
Further f(x) is differentiable in (a, h).

... / ,(c) == h~O


Lt /(c + h) - /(c)
h As h -t 0 we get from (I) and (2) that -

/(c+h)- /(c) /(c+h)- /(c) .


LI $; 0 and Lt ;::: 0 respectively.
h-~O h h~O h
i.e., 1'( c) $; 0 and 1'( c) ~ 0 simultaneously => 1'( c) == 0
Similarly the theorem can be proved when f (d) 7:- f (a)
3.1.2 Geometrical interpretation of Rolle's theorem
Let P and Q be two points on the curve y=f(x). AP==BQ ordinates f(a)=f(b and
the curve is continuous from P to Q. It can be shown that there is at least one
point on the curve y = f(x) between x = a and x = b at which the tangent to the curve
is parallel to x-axis. .

y
x=a y=a

P Q

f(a) .i(b)

x' 0 A B x

y'

fix) is a constant function


Mean Value Theorems and Functions of Several Variables 113

x=c
y

Q
f(a) f(b)
f(d)

o A B x
y'

j(c) andj(d) are both different fromj(a) = j(b)

x=c
y x=a x=b

f(a) f(c) f(d)

o A B x
y'

j(c) -:F j(a) andj(d) = j(a) = j(b)

x=a x=b
y
x=d

f(a) f(b)
f(d)

x' 0 A B x
y'

j(d) * j(a) and j(c) = j(a) = j(b)


114 Eng~neering Mathematics - I

3.1.3 Verify Rolle's theorem for / (x) = log { :;a:a:) }in ( a,b )
Solution:

Consider the function J (x) = 10g{ x(2 + ah)} in the interval ( a, b ).


x a+b

2
J(x+h)- J(x) I[ (x+h)2 +llh I x +abj
Lt = Lt
- log - og---
h~O h + h~O h ( a + b) ( x + h) ( a + b) x

~rlOg (x +_~h) + 2xll+h2 -log x+ hj


2
= Lt
+h~O hl (x 2 +ah) x

1
= Lt -1 [ log {'1+ 2xh + h 7 } -log {h}]
1+- = Lt [2X 1
--+O(h) ]
r"~O h x- + ab x +h~O x 2 + ab x
where O(h) indicates terms of order h and higher powers of h.

- ~
2x
I I( X) = 2
x +ab x
which indicates that I (x) is differentiable in ( a,b ) and hence continuous also.
2
(a +ab)
Further I (a ) = log ( ) = log 1 = 0
a a+b
2
(b +ab)
I (b) = log ( ) = 0. Thus I (a ) = 0 = I (b)
b a+b
All the conditions of Rolle's theorem are satisfied. Hence 3c( a < c < b) such
that I I( C) =0
. 2c 1 2
II ()
C =0 gIves 2 --=O~c =ab or c=ab
c +ab c
Clearly c = +ab is the G.M of a and b and so E (a, 1)
The theorem is thus verified.
Mean Value Theorems and Functions of Several Variables 115

3 .1.4 Ven'fy Rolle's t I1eorem '"lor the function


. j"() S1l1X.10
x = --- (0 ,Tl )
eX

Solution:

f(x) =~lIl<x in (O,Tl)


e

Lt
f(x+h)- f(x)
= Lt -1 [Sin(x+h) -Sinx]
-
h-... O h h--.O h e( ail) e'

e< [Sin ( x+ h) - e" sin x]


= Lt-
iI--.O h x
ex+1I .e

2
Sin(x+h)-Sinx{l+ h + h + .. ..1
= LI -
1 I! 2!
-----------X+-il~x--------~
f
}HO h e .f!

~ ~
1 2 cos ( x + ). sin ( ) - h {sin x + 0 ( h )}
=h--.O
Lt -
h
----~--~--~------------
ext-II

S1l1. (h)
Lt !h 2COS(X+!!').
=II--.O 2
( h ) -{sinx+O(h)}
2
f' ( x ) = cos x ~ S1l1 x
e
Thus f (x) is differentiable in (0, Tl) and hence continuous there.

Further f (0) = 0 = f (Tl) .AIl the conditions of Rolle's theorem are satisfied.
:. 3e( 0 < e < Tl) f'( e) = 0
such that

(i.e.,)
cose-sine
------ = 0 => e = -Tl (pnnclpal
.. value)
eC 4
and clearly e = Tl E (0, Tl) . Hence the theorem is verified.
4
116 Engineering Mathematics - I

3.1.5 Example
Verify Rolle's theorem for the function.f{x) = Ixl in (-1, I).
Solution
Here .f{x) = - x for -I < x < 0

= 0 for x = 0
=x for 0 <x < I
}(-I) = 1,.f{I)= I;
Hence .f{-I)=.f{I)

/'{x} =-1 for -1.:sx.:sO

/'{x} = 1 for

:. f'{x} does not exist at x = 0 and hence j(x) is not differentiable in (- 1, I)

.. Rolle's theorem is not applicable to the functionj(x) = Ixl in (-1, I).

y
y = f(x) = Ixl

------------------~~------------------x

y'

Fig. 3.1 (The curve is not smooth at x = 0).


Mean Value Theorems and Functions of Several Variables 117

Exercise - 3(A)

I. Verify Rolle's theorem for the following functions :


1. lex) = x (x + 3)e-xI2 m (-3.0) [Ans:c=-2]
2. j(x) = sin x m (O.n) [Ans : c = n/2]

3. j(x) = e" (sin x - cos x m (:.?~) [Ans : c = n]

4. j(x) = log (.'s;-


+6: m (2,3) [Ans : c = J6]

{mb+ na}
5. fix) = (x- a)m (x- b)n in (a, b), m > 0, 11 > 0 [Ans : c = ]
{m+ 11}

6. fix) = xl - 5x + 7 in (2, 3) [Ans : c = 5/2]

3.2.1 Lagrange's Mean Value Theorem (First mean Value Theorem):


Ifj(x) is (i) continuous in [a, b] and (ii) differentiable in (a, b) then ::3 at least one

"alue 'c' in (a, b) 3 f'{c) = f{bi- f{a)


-a
Proof: Define a new function j>x = fix) + A.\ ..... ( I)
where A is a constant 3 j>(a) = j>(b)
i.e., fia) + A (a) = fib) + Ab

i.e., A = _ [f{b)- }'(a)] ..... (2)


b-a

j(x) and Ax are continuous in [a, b]


Hence j>(x) is continuous in [a, b] ..... (3)
j(x) and A x are differential in (a, b),
Hence j>(x) is differential in (a, b) ..... (4)
andj(a) = j>(b) ..... (5)
118 Engineering Mathematics - I

(3), (4), (5) show that ~(x) satisfies all the conditions of Rolle's theorem.

3 atleast one value c in (a, b) ;) ~' = 0


~' (x) = f' (x) + A
~' (c) = f' (c) + A = 0
A = - f'(c) ..... (6)

From (2) and (6) it follows that

f'{c) = j'(b)- f{a)


b-a
3.2.2 Geometrical Interpretation of Langrange's Mean Value Theorem
P and Q are two points on the continuous curve y = j(x) corresponding to x = a
and x = b respectively.
.. P[a, j(a)], Q [b,j(b)] are two points on the curve.

... h . PdQ
SI ope on t he Ime JOll1lng t e pomts an IS
f{b)-
b_a
f{a) ' RIS a pomt
. h
on t e

curve between P and Q corresponding to x = c, so that f'{c) is the slope of the


tangent line at R [c,j(c)].
f{b)- f{a)
f'{c) = b_a means that the tangent at R is parallel to the chord P Q.

p
f(c) f(b)
f(a)

x' x=a x=c


0 x=b x

y'

Fig. 3.2

Hence this theorem tells that there is at least one point R on the curve PQ
where the tangent to the curve is parallel to the chord PQ.
Mean Value Theorems and Functions of Several Variables 119

3.2.3 Example
Verity Lagrange's theorem for the functionf(x) = (x - I) (x- 2) (x- 3) in (0, 4).
Solution
j(x) = (x - I)(x - 2)(x - 3) and a = 0, b = 4
j(x) = x 3 - 6 Xl + II x - 6 is an algebric polynomial and (0, 4) is a finite interval.
Hencej(x) is differentiable in (0, 4) and is continuous in [0,4] showing that the
conditions of Lagrange's Mean Value theorem are satisfied.
:3 atleast one value 'c' in (0,4), such that
f'{c) f{b)- f{a)
=
..... ( I)
b-a
j(0) = - 6, f(4) = 6
f'{x) = 3 x2 - 12 x + II
f'{c) = 3c2 - 12 c + 11

From (I) 3c2 _ 12 c + II = 6 - (- 6)


4-0
3c2 - 12 c + 8 =
62fj
c=
3

The point c = 6 2fj


3
W hICh CIear Iy )
IIe .111 ( ,4 .

3.2.4 Example
Verify Langrange's Mean Value theorem for the functionj(x) = ~ in (0, I) :
Solution
j(x) = ~ is differentiable in (0, I) and continuous in [0, I]
.. :3 atleast one value 'c' in (0, 1) such that
3 l'{c) = f{b)- f{a) ..... ( I)
b-a

'{ ) = f(l) - f(O)


3
f c 1-0 ..... ( 1)

j(0) = eO = I, f(l) = e

f'{x) = ~ gives f'{c) = e


C
120 Engineering Mathematics - I

e-I '
From (I) eC = -
1-0

c=log(e-I), thisc EO, I)

3.2.5 Example
Verify Langrange's Mean Value theorem for the functionj(x) = 5x2 + 7x + 6
in (3,4).
Solution
j(x) is an algebraic polynomial and the interval (3,4) is finite, j(x) is differentiable
in (3,4) and continuous in [3,4].
. . 3 atleast one value c r.(3, 4) such that
j'(e) = f(ll) - f(3)
..... (1 )
4-3
3 f'{c} = f{h}- f{a}
b-a

j(3) = 72,j(4) = 114, f'{x} = lOx + 7

114-72
IOc+7=
4-3
c = 3, 5 E, (3, 4)

Exercise - 3(8)

I. Verify Langrange's Mean Value theorem for the following functions :


I. j{x) = x(x - I) (x - 2) in (0, X)
2. j(x)=logxin(1,e)

3. j(x) = x 2 _ 3x _ I in ( -~ 1, I;)
4. j(x) = a2 - 7x + lOin (2, 5)
3.3.1 Cauchy's Mean Value Theorem
If two functionsj(x) and g(x) are (i) continuous in [a, b] (ii) differential in (a, b)
and (iii) g'(x) =F- 0 in (a, b) then 3 atleast one value 'c' in (a, b) 3
f'{c} f{b}- f{a}
g'{c} = g{b}-g{a}
Mean Value Theorems and Functions of Several Variables 121

Proof:
Define a new functionj(x) = j(x) + A g (x) .....( I)

Where A is a constant such that 4>(a) + 4>(b) ..... (2)

j(a) + A g (a) = j(h) + A g (h)

[j{b}- j{a}]
..... (3)
A=- [g{b}-g{a}]

j{x), A g (x) are continuous in [a, b)

Hence 4>(x) is continuous in [a, b)

j(x), A g (x) are differentiable in (a, h) ..... (4)

Hence 4>(x) is differentiable in (a, b) ..... (5)

and 4>(a) = 4>(b) ..... (6)

(4), (5), (6) show that 4>(x) satisfies all the conditions of Rolle's theorem

3 atleast one value c in (a, b) ;) 4>' (c) = 0

But 4>'{x} = f'(x} + Ag' (x)

4>'{c} = f'{c} + Ag' (c) = 0

/'{c}
=> A= - g'{c} ..... (7)

/'{c} /{b}- /(o)


From (3), (7) we get g'(c} = g(h}- /(o)

3.3.2 Example
1 1
Verify Cauchy's Mean Value theorem for j(x) = -? and g(x) = - in (0, h)
x- x

Solution

j(x), g(x) are differentiable in (a, b) and continuous in [a, b)


122 Engineering Mathematics - I

::3 atleast one value 'c' in (a, b) :)


f'{c) f{b)- f{a)
g'{c) = g{b)- f{a)

Here f'{x) = ~
X3

-I
g'{x) =7

-2
C 3
=--IT = 1
//c 2
b a

2 a+b
c ab
2ab
c = --b which is the Harmonic Mean of , a' and 'b'
a+
c E(a, b)

3.3.3 Example
Verify Cauchy's Mea~ Valve Theorem for fix) = e, g(x) = e-X in (3, 7)
Solution
fix), g(x) are differentiable in (3, 7) and continuous in [3, 7]

::3 atleast one value 'c' in (a, b) :)


f'{c) f{b)- f{a)
g'{c) = g{b)-g{a)
Here f'{x) = e
g'{x) = e-x

c = 5 E (3, 7)
Mean Value Theorems and Functions of Several Variables 123

Exercise - 3(C)

I. Considering the functionsj{x):::: x 2 , g (x):::: (x) in Cauchy's Mean value theorem for
(a, b) prove that 'c' is the arithmetic mean between a and b.
2. Verify Cauchy's Mean Value theorem for j{x):::: sin x, g(x):::: cos x in (a, b)
I
3. Verify Cauchy's Mean Value theorem for j{x):::: fx, &>(x):::: fx in (a, b)

3.4.1 Higher Mean Value Theorem with Lagrange's form of remainder


(Taylor's theorem with Lagrange's form of remainder):
If a functionj{x) is such that

(i) j(x); f'{x}, r{x} ..... fn - I (x) are continuous in [a, a + h]

(ii) fW{x} exists in (a, a + h), then :3 at least one number '8' between '0' and' I'

h hn - ' h"
3j{a + h):::: j{a) + ,f'{a} + ..... + - ( \"f(n-I)(a) + -F (ll + 9h)
1. n -I,. n!

Proof: Define a new function.

j{x):::: j{x) + (a+h-x) f'{x} + (a+h-xY r(x) + ...


J! 2!

(a + h - X ),,-1 /"-1 (a + h - x)"


+ (-I)
n . (x) + n., .A ..... (1 )

where A is a constant 3 cp (a) :::: cp (a + h)


2
h j" (a) + 2!
cp(a) :::: j{a) + 1! h fW (a)+

hn - I hn
..... + - (
I",,r-I(a) + - A ..... (2)
n- J! n!
and cp (a + h) :::: j{a + h) ..... (3)

but cp (a) :::: cp (a + h) ..... (4)

Thus j(a + h) :::: cp (a + h) :::: cp (a)


124 Engineering Mathematics - I

Hence using (2) we get

h h2
j(a + h) = lea) + - j(a) + - I" (a) + ..... .
J! 2!

..... (5)

j(x), I'(x) , I" (x ), ... .fn-I(x) and (a + h - x) (a + h - x)2 etc continllolls in


[a, a + 17] and differentiable in (a, a + h)

Hence ~(x) is continuous in ra, a + 17] and differentiable in (a, a + 17) ..... (6)

(4) and (6) show that ~(x) satisfies all the conditions of Rolle's theorem.

:. :3 atleast one value 'c' (a < c < 0 + 11) ;) ~' (c) = 0

Write c = a + e 17 where 0 < e < I

:. :3 e E (0, I) ;) ~' (0 + e 17) = 0 ..... (7)

Differentiating (I) with respect to 'x'

2 J
+ [(a+h-x)"J I" (x)- (n-IXa+h-xt- In-J/(x)j_ n(a+h-xt-
.,. (n-I) (n-I) F;'n! A ..... (8)

From (7) we get

(h - eh),,-J
~' (0 + e 17) = [rea + eh] =0
(n-I)
..... (9)
From (5) and (9) it follows that
2
j(a + h) =
hi' (a) + -,
j(a) + -1' h I "
(a) + ...
. 2.

(0 < e < 1)
Mean Value Theorems and Functions of Several Variables 125

The last term ~ .!' (a + 0 h) is called Lagrange'sform of remainder.


n!

3.5.1 Higher Mean Value Theorem with Cauchy's form of remainder


(Taylor's theorem with Cauchy's form of remainder):

If a functionf{x) is such that (i)f(x), f'{x) , r{x) ..... fll(X) arc continuous in
[a, a + 11] and (ii),f'1(x) exists in (a, a + h) then 3 atleast one number '0' between
'0' and' 1' such that
2
f(a + h) = I!
.f{a) + h f' () h '" (I) + ...
a + j!f

Proof: Define a new fUllction


(a+h-x) () (a+h-x)2 ()
~(x) = .f{x) + f' X + f" x + ....
I! 2! '

(a + h - x)" I
+ {n-l}! fll-l(x)+(a+h-x)A ..... (1)

where A is constant 3 ~ (a + h) = ~ (a) ..... (2)


From (1) ~ (a + h) = f(a + 11)
h h2 " h"-' ,
~(a) = f(a) + -1'.1
'I

.
{a} + -
2!
f {a} + ... + -(-1)'.1 11-1 (a) + II
n- .
A ..... (4)

From (2), (3), (4)


,J
h " n- f"
f{a + h) = .f{a) + -" j (0) + -, (a) + .......
. 2.

.. ... (5)

f(x), f'{x) , r(x) ..... jil- I (X) and (a + h - x), (a + h - x)2 ...... are continuous
in [a, a + h] and differentiable in (a, a + 1/)
126 Engineering Mathematics - I

Hence <I>(x) is continuolls in [a, a + h] ..... (6)


<I>(x) is differentiable in (a, a + h) ..... (7)
(2), (6) (7) show that <I>(x) satisfies all the conditions of Rolle's theorem.

:. 3 atleast one number '0' in between '0' and' I' 3 <1>' (a + 0 h) = 0 ..... (8)

Differentiating (I) W.f. to 'x'


<I>'{x) = I'{x) + [(a+h-x) IW{x) - f'{x)

(a+h-x f I"'{x)- 2(a+h-x) f"{x)]


[ 2! 2 + ... + ... +

llu1
+ [(a+h-x ) F(x)_{n_I){a+h-x)"2 f"'{X)] -A)
(11-1). (n -I).

(a+h-x)"-'
I.e., <I>'{x) = {n-I}. f"(x)-A ..... (9)

From (8) and (9) we get

{h-Oh)"-I
<1>' (a + 0 h) = {n -I}. fll (a + 0 h) - A=0

h,,-I (I - 0),,-1
.. A = (n-I)' fll(a+Oh) ..... (10)

From (9) and (10)


2
hi' h j.w
flo + h) = j(a) + -" (a) + -, (a) + ....
. 2.

h"{I-OY--1
The last term ( ) fll (a + 0 h) is called Cauchy's form of remainder.
n-I!
Mean Value Theorems and Functions of Several Variables 127

3.5.2 Alternate form of Lagrange's Mean Value theorem


If f(x) is i) continuous in the closed interval [a. a + h] and ii) derivable in
the open interval ( a . a + h ) then there exists at least one number B, 0 < B<1,
such that f(a+h)=f(a)+f'(a+Bh).
Proof: In ( 2.1 ) put b =a + h ,then from the proof of L . M . V theorem there
exists 'c' ,

such that
f'(c) = f(a+h)- f(a) ....... ( I)
a+h-a
writing c = a + Bh we have 0 < B < 1 and (I) becomes

f'(a + Bh) = f(a+ h)- f( a) => f( a + h) = f(a)+ hf'(a+Bh)


a+h-a
3.5.3 Example:
If f(x+h)=f(x)+hf'(x+Bh), O<B<l, find the value of B when f(x)
is any quadratic expression.
Solution:
Let f (x) = ax 2 + bx + c , then
f(x+h)=a(x+h)2 +h(x+h)+c = f(x)+h(2ax+b+ah) .......(1)
It is given that = f (x + h) = f (x) + hf' (x + Bh)
....... (2)
From ( I ) and (2) we get,
f'(x+ Bh) = 2ax +b +ah (i.e.,) 2a(x + Bh)+ h = 2ax +b +ah
.
(sincef'(x) = 2ax+b) => 2B = 1 or B= 1;2

3.5.4 Alternate form of Cauchy's Mean value theorem


If f (x) and g (x) are (i) continuQ,Us in the closed interval [ a, a+h] (ii) derivable

in the open interval ( a . a+ h ) and (iii) g' ( x) *- 0 at any point in the open interval
( a, a+ h ) , then there exists at least one number B , 0 < B < 1, such that
f'(a+Bh) f(a+h)- f(a)
g'(a+Bh) - g(a+h)-g(a)
Note: Lagrange's Mean Value theorem can be deduced from C . M . V theorem by
replacing g (x) with x in the interval ( a, b )
128 Engineering Mathematics - I

= b , g ( a ) = a and g X ) = 1
We get g ( b) I(

f'(C) f(b)-l(a)
-- = . which is L. M. V. theorem.
1 b-a
Example: Using Cauchy's Mean Value theorem, show that

sin b - sin a < b - a given that 0 < a < h < Jr


2
Solutio,,:
Taking f (x) = sin x and g (x) = x and applying C.M. V theorem we get that,

3e such that
sinb-sina
= cos e where 0 < a < c < b < -
Jr
b-a 2
.
Furt her cose < ] 111 (0 Jr)2
;- :. sin b - sin a <
b-a
l' .(I.e.,) Sin b '
-SIna < (b
-a)

3.5.5 Example:
I f f '( x) is continuous in [ a, b] and fll (x) exists in ( a, b ) then show that
I

f(b)-f(a)=(b-a)f '(a)+(b-a)2 fll(a) where a<e<b.


2!
Solution:

Choose a function g( x)
g( x) = h( x) _(b - X)2 h( a) where
such that
b-a
h(x) = f(b)- f(a) -(b -X)f'(X) ........(1)
Now g(a)=h(a)-h(a)=O and g(b)=h(b)-O=O from (I) and L.M.V
theorem.
g(a)=O=g(b) .
It is given that f'ex) is continuous in [ a, b] and differentiable inC I, b) .
Hence, so areh(x) and g (x) . :. g (x) satisfies all the conditions of
Rolle'stheorem.Hence 3e (a<e<b) 3g '(C)=0 ........(2)
g I ( X) = h' ( X ) + 2 (b - C! h ( a) from (I )
(b-a)
=-(b-X)f'1 (x)+ 2(b-X![f(b)_ f(a)-(b-a)/(a)]
(b-a)
Mean Value Theorems and Functions of Several Variables 129

Using (2) we get,

-C!
(b-c )f"( c) = 2( b [f(b) - f(a) -(b - a)f'( a)]
(b-a)
(b - a)2
or f(b)=f(a)+(b-a)f'(a)+ 2! f"(c) ........(3)
Taking (' = a + Biz where h = b - a and 0 <B<1

(3) gives f(a + h) = f( a)+ Jif'(a) + ~2! f'(a +Oh) where 0 < (1 < I

3.5.6 Example:
f (x), g (x) and h(x) are three functions derivable in ( a , b ), show that there
exists c in ( 'I, b ) such that
f'(c) g'(c) h'(c)
f(a) g(a) h(a) =0
f(b) g(b) h(b)
deduce L. M . V and C . M . V theorems.

Solution:
f'(x) g'(c) h'(c)
Consider the function ( x) = f (a) g ( a) h(a) =0 ........ ( I )
f(b) g(b) h(b)
Clearly (a) = 0 = (b)
Also f( x),g( x) and h( x) are all derivable in (a, b),

:. All the conditions of Rolle's theorem are satisfied. Hence 3c E (a,b) such
that ' ( c ) = 0
f'(c) g'(c) h'(c)
(i.e.,) f (a) g ( a) h(a) = 0 ........ (2)

f(b) g(b) h(b)


130 Engineering Mathematics - I

Taking g(x) =x and h(x) = I we have from (2)

f'(e) I 0
f(a) a 1 =O=>f'(c)(a-h)-[f(a)-f(h)J=O
f(h) b 1

f(h)- f(a)
(i.e.,) f'(e) = where e E (a,b) ........ (3)
h-a
(3) shows that L . M . V theorem can be deduced from (I) Taking only h (x) =I
f'(e) g'(c) 0
we have, f (a ) g ( a ) 1 = 0 where c E ( a, b)
j(b) g(b)
=> f'(e)[g(a)-g(b)J-g'(c)[f(a)- f(b)J=O
f'(c) f(b)- f(a) . .
=> -(-) = () () where c E (a,b) which IS C. M . V theorem.
g' e g h - g a
3.5.7 Example:
Apply Maclaurin's theorem with Lagrange's from of remainder for the function
2 2
f (x) = eX and show that 1 + x + ~ ~ e ~ 1 + x + ~ eX for every x ~ 0 .
t

2 2
Solution:
Maclaurin's theorem with L form of remainder is
2 n-I n
f(X) = f(o)+~r'(o)+~ f"(O)+ ... +1~_1 in-I) (0)+ ~~ f" (Ox)
~ ~ ~
2 3 n-I n
X + -X
eX = 1+ X + T=) + ...... + IX X Ox (SInCe
~ _ 1 +- e
. Dn ( eX) = eX)
~ lJ ~ ~
........ (1)
where 0 < (} < 1
Taking n = 2 in (1) we get
X2
eX = 1+ X + ~ eOx ........ (2)

For every X ~ 0 we have eOx ~ eX where 0 < (} < 1


Mean Value Theorems and Functions of Several Variables 131

1 1
x- Ox 1 x- x
1+x+-e S +x+-e
II ~
........ (3)
Further e
Ox
> 1 whenever x ~ 0 and 0 < 0 < 1
2 2
X X fh
l+x+-sl+x+-e
II II
........ (4)
From (2), (3) and (4) it follows that
2 2 2
1 X x Ox x x,
l+x+-s +x+-e =e sl+x+-e
II II II
3.5.8 Example
2 3
. Taylor's theorem to prove that loge ( 1 + x ) < 1- x + x
Apply 2 3 whenever x> 0 .

Solution:
According to Taylor's theorem
h h2 h"- I
I( a+h) = l(a)+11 1'( a)+ ~ f"(a) + ... + In-l f"- (a)+ R"
I

h" (1-0),,-"
where Rn = ~ I" (a+Oh) , 0<0<1
n- .p
h"
RII with Lagrange's form (n = p)is Rn = ~ f" (a+Oh)

Substituting a = I and h = x in (1)


2 n-I II

1(1 + x) = 1(1)+ ~f'(I)+ ~ 1"(1)+ ... + 1:-1 in-I) (1) + ~ I" (1 +Ox)
(_1)"-1 In-l
Taking I (x ) = log x, I" (x ) = n
X
3
x2 x
.. 10g(l+x)=O+x--+ 30<O<I(Takingonlyupto3terms)
II 3(l+Bx)
Foranyx>O, (1+0x1
132 Engineering Mathematics - I

3.6 Functions of Several Variables


3.6.1
Students are quite familiar with functions of a single independent variable.
Functions which depend on more than one independent variable are called
Functions of Several Variables.
(i) Volume V = nr2h ofa cylinder, r = radius, h = height,
curved surface area A = 2prh.
I
(ii) Area of a triangle A = "2 xy, x = base, y = altitude
(iii) Volume of a rectangular parallelopiped v = xyz, where x = length, y = bredth
z = height are all examples offunctions with more than one variable. (i) and
(ii) are examples offunctions oftwo variables and (iii) is a function of three
variables.
Let z =j(x, y) is a function of two independent variables. Here x, yare independent
variables and z is the dependent variable and let the function f(x, y) be defined in
a region R.
Suppose (x, y) be a movingpoint and (a, b) a fixed point in the region R ofthe xy-
plane. The point (x,y) may approach (a, b) along different paths (see figure) PI'
P2' P3 etc.

3.6.2 Concept of a Limit


Let (i) f(x, y) be defined in a region R
(ii) (x, y) tend to (a, b) along any path
and (iii) E < 0 be given.
Mean Value Theorems and Functions of Several Variables 133

If ::3 0 > 0 such that Ij(x, y) -II < E, V Ix - al < 0, Iy - bl < 0, thenj(x, y) is said to
tend to I.

We write Lt j(x, y) = 'I


x~a

y~b

or as Lt I(x, y) = I
(x,y)~(a,b)

3.6.3 Example
Lt x2 + y2
Consider x-~2 ---'--, it can be seen that if E = 0.01 (say) then
y->!
4xy

x2 + y2 5
4 xy
- -8 < E whenever Ix - 21 < 0.02, lv - II < 0.02 thus 0 = 0.02

satisfies
) )

x- +,v- 5 5
4xy -"8 < 0.0 I. Hence the desired limit is "8 .
Note: 1
Let Lt
,y)-->(a,b) .
f(x, y) = I and Lt
(X,Y)-'(a,b)'
g("(, v) = m

then (i) Lt [f g] = Im
(x,y)->(a,b)

(ii) Lt [fg] = /m
(x,y)->(a,b)

Lt I 1
and (iii) (x,y)->(a,h) g III

Note: 2

(x,y) LI
~ (a, b) exists iff LI
(x-~a) y~b
[ LI 1
I(x,b) == Lt [
(y~b) x~a
Lt f(x,h) 1
134 Engineering Mathematics - I

3.6.4 Example
~

x- + V
f(x, y) = . 2 ' find Lt as (x, v) ~ (2, I)
2x+ y .
Solution

LI [Lt
y~ x~2
2
x +y LI 1 [4
+ 5
2X+;2 . = y~ 4+ y2 =5=1
y]
..... (i)
2
x~2 y~l
Lt [Lt x
2x+ y2
+y ]
= x~2
Lt [X2 + I] 5
2x+1 =5=1 ..... (ii)

The two limits (i) and (ii) are equal.

'J
Lt x' + Y
Henee ---=-;;-=1
(x,y) ~ (2,1) 2x + y2

3.6.5 Example

J )
x- - Y'
f(x, v) = ? J find whether the limit exits as (x, y) ~ (0, 0)
. x- + y-

Solution

I,I L/ 2 2]
x - Y LI I[""' I,/ x] - Y 2]
Thus
X ~ 0 [Y ~ 0 J
x- + y-
') 7:-
y ~ 0 ly ~ 0 x- + y ? )

lIenee the limit does not exist.

3.6.6 Example

Lt ~
xy
')

Find (x,y) ~ (0,0) y- - x-


Mean Value Theorems and Functions of Several Variables 135

Solution
Puty = 11/X

2
Lt xy Lt mx
(X,y)~(O,O) / __ X2 X~O m 2 x 2 __ X2

It is clear that limits will be different for different values of m i.e., the limit depends
upon the slope of the path along which (x. y) approaches (0,0). Hence the limit does not
exist.

Exercise - 3(0)
I. Examine whether the following limits exist. Find them if they exist.
Lt x 2 ~ y2 + 4 Lt x 2 + 4y2
(i) (ii) ~ 2X2
(x.y) ~ (2,1) 3xy2 (x, J~ -~ (0,0) y2
4
Lt yx~2y Lt Y4
X ~
(iii) (iv)
(x.y) ~(2,2) xy~2x (x,y) ~ (0,0 ) x-J ~ y-J
J J
Lt x-y-
(v)
(x,y) ~ (0,0) x 2 + y2

3.7.1 Concept of Continuity


Suppose (i) Lt j(x. y) exists
(~,y) ~(a,b)

and (ii) Lt f(.~, y) = j(a, b)


(x,y)~(a,b)

thenj(x. y) is said to be continuous at (a. b).


Note: 1
Ifj(x. y) is said to be continuous at every point of a region R, then it is said to
be continuous in R.
Note: 2

Letj(x. y) and g(x, y) be continuous at (a, b) then f g,fg, and f (g -:t 0)


g
are all continuous at (~ b).
136 Engineering Mathematics - I

3.7.2 Example
Consider the function/ex, y) = x 2 + Y - 2x when (x, y) 7= (0, 0) and.l( I, I) =0
Lt f(x,y) = Lt [Lt (x 2 + y-J -- 2x) ]
(x,y) ~(I,I) x~ 1 y ~ I

=/(I, I)
Hence the function is continuolls at (1, 1)

3.7.3 Example
2 2
Consider the function f(x, y) = ~ y J when (x, y) 7= (0,0) and.l(O, 0) = 0
. . x + y-
Let (x, y) ~ (0, 0) along the path y = mx
Lt Lt 2 2
f(x,y) = ~y 2
(.Y,y) ~ (0,0) (x, y) ~ (0,0) x- + y

Lt x 2 m 2 y2
(x, y) ~ (0,0) x 2 + 1112 x 2

Ltm2x 2
(.r, y) ~ (0,0) I + m
--2 =
=.1(0 0)
,

lienee .l(x, y) is continuous at (0, 0)

3.7.4 Example

Consider the function.l(x, y) =


xy
x2 _ y2 (x, y 7= 0, 0), .1(0, 0) =
2
Lt xy Lt mx
(x, y) ~ (0,0) x 2 _ y2 (x, y) ~ (0,0) x 2 (I - m 2 )

along the path y = niX

= - - 7=
1-JIl
III
2
.1(0, 0) except when III =
Hence the given function is discontinuous at (0, 0)
Mean Value Theorems and Functions of Several Variables 137

3.7.5 Example
2 2
X Y
Consider the functionj(x, y) = (x, y) cf. (0, 0)
~X2 _ y2

= at (x, y) = (0, 0)
In this case it is convenient to introduce (polar co-ordinates). Substitutions x = rcosB,
y = rsinO.

x 2 y2 r4cos20sin2B
l"(2 + y2 - Jr2 (cos 2
0 + sin 20)

r'
-(sin 2 20)
4

= {x2 + y2 y2
4

Now
(x-, +y-, )3 2 ~ -E . I
provided /x/ < E 3
4 2
I I
thus when /x - 0/ < E
i3
and lY - 0/ < E3

..... (i)

Hence
Lt
(x,y)~(0,0)~X2 + y2
x 2 y2
---;===== =
frol11 (i)

=j(O, 0)
2 2
j(x, y) = Jxx Y+ y2
2- is continuous at (0, 0)
138 Engineering Mathematics - I

Exercise - 3(E)

I. Investigate the continuity of the following:


(i) j(x, y) = 2x + ),2 } (x, y) t= (2, 3)
=0 (x, y) = (2, 3) at (2, 3)
Ans: Not continllous
xy
(ii) j(x.y) = - - , (x, y) t= (0, 0) at (0,0)
~Xl + y2
=0 (x, y) = (0, 0)
Ans : Not continuolls.
x-y
(iii) j(x,y) = - - , (x, y) t= (0, 0)
2y+x
=0 (x, y) = (0,0) Ans : Not continuolls

3.8.1 Partial Differentiation


Consider z = j(x, y) where x and yare independent variables and z is the
dependent variable.
Keeping one of the two variables x and y as constant and allowing the other to
vary we get a partial derivative of 'z' with respect to the variable that is varied.
Keep 'y' constant and allow x to vary, then partial derivative of z w.r. to 'x' is

obtained and is denoted by az or f (x, y) or Dj{x, y) or 8f


ax x ax

az Lt j(x+Jx,y)- j(x,y)
So =
ax ax~o bx

Similarly az = Lt f(x,y+Jy)- j(.x:,y)


By ay~o by

Note: 1
z = j(x, y) represents a surface in the cartesian co-ordinates (x, y, z) system.
The section of the surface z = jex, y) with the plane x = k (parallel to yz plane) is a
curve. (Similarly the sections with planes parallel to xy - plane and zx - planes also
will be curves).
Mean Value Theorems and Functions of-Several Variables 139

Note: 2

az)
( Ox gives the slope of the tangent at (x, y, z) to that curve
x=k

[
~z 1 gives the slope of the tangent at (x. y. z) to the curve obtained as
~ y=k

the section of the surface z =f(x. y) with the plane y = k.

3.8.2 Example
z = x3 + x2y + I

(i) To find az , we keep 'y' as constant


ay
partial derivative w.r. to x of x 3 = 3x2
partial derivative w.r. to x of x 2y = 2xy
partial derivative w.r. to x of I = 0
az
.. ax = 3x2 + 2xy

(ii) To find az , we keep 'x' as constant


ax
partial derivative w.r., to y ofxJ = 0
partial derivative w.r., to y of x 2y = x2
partial derivative w.r., to y ofy2 = 2y
az
- =x2 + 2y
ay
3.8.3 Example
f= x 2 + 1- 2xy
af
- =2x-2y
ax
af
- =2y-2x
ay
3.8.4 Example
f= (x - y) (x + 2y)
140 Engineering Mathematics - I

This is a product, and the usual product rule applies


at' = (x - y) (1 + 0) + (x + 2y)( I - 0)
-'
ax
= x - y + x + 2y = 2x + y
aj
- = (x - y) (0 + 2) + (x + 2y) (0 - 1)
ay
= 2x - 2y - x - 2y = x - 4y

3.8.5 Example
y
Iff= x-2 , Find af and aj
x+y ax ay

Solution
Applying the Quotient rule
aj (x+ y)(I-O)-(x-2y)(I+O) 3y
ax (x + y)2 (x+ y)2

aj (x+ y)(-2)-(x-2y)(O+I) -3x


ay = (x + y)2
(x + y)2

3.8.6 Example

Iff = sin(ax + by), sino aj and aj


ax ay

aj
- = cos(ax + by) x -
a (ax + by) = acos(ax + by)
ax ax
a a
ay = cos(ax + by) x ay (ax + by) = bcos(ax + by)

3.8.7 Example
Consider f = ax2 + hxy - by

Then
aj
-
at
= 2ax + hy - ' = 2by + hx
ax ' ay

The expressionix = 2ax + hy is itselfa function ofx andy. We could therefore


find its partial derivative with respect to x or y.
Mean Value Theorems and Functions of Several Variables 141

(i) If we differentiate it partially w.r. to x, we get ~ [Of]


ax ax
and this is

a
ax-
2
a I]
[- , = - (2ax + hy) = 2a
ax
This is called the second partial derivative of 'f \V.r. to 'x'.

If we differentiate Of partially w.r. to 'y' we get !L (a~) and this is -


ax ay ax
ail
written as - -
ayax

Thus
al
- = 2ay + hy, -8f = hx - 2by
ax ay

Similar steps can be carried out with the expression for 8fay

a~ ~
2

ay
= (8f)
~ ay
= -2b

aI
axay
2
= al
ax
(8f)
~
= h
142 Engineering Mathematics - I

3.8.8 Example
2 2 2 2
al 8f a 1 a f a 1 a 1
Iff= ax3 + hx2y + bl, find - ' - ' - 2 ' - 2 , - - , - -
ax ay ax ay ayox axay
Solution

-
af =3ax2+2hxy~ -
8j ')
=hx2+3by
ax ay

a21 a2 f
In this example also, we see that - - = - ' -
ayax axay
3.8.9 Example
Iff= log(x2 + y), prove that

a21 a21
axay ayax
Solution
8j = 2x

a2f -4xy a2f -4xy


r
ayax = (x + y2 axay = {x + y2
2 2 r
axay ayax
3.8.10 Example

) au au
Ifu(x + y) =x2 + y~ then prove that ( - - - ) 2 =4( 1 - - - -)
au au
ax ay ax ay
Mean Value Theorems and Functions of Several Variables 143

Solution

Since u=
x+y
2
au x 2 +2xy- y2 . _au = _X +2xy+ y2
ax (X+y)2 'ay (X+y)2

au _ au 2 =[7
x- +2xy- y 2 +x-7-2xy- y-7]2
( ax ay ) (x+ y)2

..... (i)

..... (ii)

(i) and (ii) prove the result

3.8.11 Example

If z = 2(ax + byi - (x 2 + y) and a2 + b2 = 1, then prove that


a2 z a2 z
-2 +-2 = 0
ax ay
Solution
az
ax = 4(ax + by) . a - 2x

a2 z
- = 4a2 -')~
ax 2
az
ay = 4(ax + by) . b - 2y
144 Engineering Mathematics - I

3.8.12 Example

If z = f ( -Xl] aZ az
then prove that x - + 2y- = 0
y 8.:r Oy

Solution

x az + 2y az =f'(Xl][2X2 _ 2X2]=0 ..... (i)


ax ay y y y

Differentiating (i) partially w.r., to 'x' .


a 2z az a 2z
x-+-+2y--=0 ..... (ii)
ax 2 ax axay

Differentiating (i) w.r., to 'y' partially


a 2z az a 2z
x--+2-+2y-=0 ..... (iii)
ayax ax ay2

Multiplying (ii) by 'x' and (iii) by 'y' and adding

2
2 a z az a2z ::.2
u Z
a'Z ::.)
2 u- Z
x -+x-+2xy--+xy--+2y-+2y -=0
ax 2 ax axOy Oyax Oy ~l

az az
But x-+2y-=0
ax Oy
) 2 ::.)
2 a-z a z 2 u-Z
x -+3xy--+2y -=0
ax 2 axay Oy2

3.8.13 Example
If u = log(x3 + T + z3 - 3xyz) then prove tllat
Mean Value Theorems and Functions of Several Variables 145

Solution

...... (i)

au -3- - - - - - ( 3 x 1 - 3 yz)
Now ax x + y3 + Z3_ 3xyz

all
3 3 3 (3y2-3xz)
~y x + y +z -3xyz

au 1 ~
-3- - - - - - ( 3 z - -3xv)
az x + y3 + Z3 --3xyz .

au
- +- +-
au au 3x 2 +3yz-3 y 2 -3xz+3z 2 -3xy
= -----'--,-----'-------,.-------'-
ax ay OZ Xl +./ +Z3 -3xyz

3
(.: (x + y + z) (Xl + y2 + Z2 - xy - yz - zx)
x+y+z
= x 3 + y' + Z3 - 3xyz)
Hence from (i)

LHS = (! + ~ + ! J( x + : + z 1
=3(- 1
{x + y + Z )2
_ 1
{x + Y + Z )2
- 1
{x + Y + z f
1
-9
=--------:-
{x+ y+ Z)2
146 Engineering Mathematics - I

3.8.14 Example

r'
If II = en e 40 then find 'n' so that -au = -
1-a (r? au
-)
ao ,.2 ar a,.
Solution

au -_ nen 1
e
-r'
-40
+
On -40--
--I' (
.e .
~?
2
1
ae 4e

..... (i)

also

-1[4lt
== 2e r 20 + 3r-u
?]

..... (ii)

Equating (i) and (ii) we get

n -3
e 2e

-3
Hence n==-
2
Mean Value Theorems and Functions of Several Variables 147

Alternative method :

Taking logarithms on both sides


)
r-
log u = n log 0 - - ..... (I)
40
Diff. (I) partially w.r.t 8

I au 11 r2 au ( r2)1l
- = 11+- -
--=-+- ..... (2)
u ae 8 u0 2 ae 40 8
Diff.( I) partially w.r.to 'r'
I au - r
--=-=:)-=--
au - u. r
u ar 20 or 28
) au -11.,.3
r--=---
ar 28

- a( au) _-I [ u. 3r
I .2 - -- 2 +r .1(--1/1")]
-
ar ar 28 20

..... (3)

From (2) and (3) we get


r2 _3 r2
11+-=-+-
48 2 40
-3
.... 11=-
2

3.8.15 Example

?
If x- yY zZ = k then prove that at x = y =
a2 z
z, - - = [x log exr I
axBy
J48 Engineering Mathematics - I

Solution
Applying logarithms to.r y zZ = k, we get xlog x + ylog)' + zlog z = log k.
Differentiating this implicit function partially w.r., to 'x'.

x
1
x-+logx+ z.-+Iogz )az
z
-=0
ax
(I
az
(I + logx) + (1 + logz). ax = 0 ..... (i)

az (I + logy)
ax l+logz

az (I + logy)
/Illy =
ay 1+ logz
Differentiating (i) partially w.r., to 'y'

{1+logz )-a2Z
- +aZ[1
- - .az]
- =0
ayax ax.z oy
Substituting x = y = z, we get
a I =0
(I + logx)_z_+-
2

axay x

I I
-axay- - - x{1 + logx) - x{loge + logx)

or - (xlogex) I
xlogex

Exercise - 3(F)

x+4y OJ of -17y 17x


1. If f= 4x _ y' find -8 and 7) (ADS: (4x- y)2' (4x-- y)2 ]
x Y

az az
2. If z = ax2 + bxy + by/inti ax' ay (ADS: 2q.x + ~y, bx + 2by]
Mean Value Theorems and Functions of Several Variables 149

. OZ {)z
3. Ifz = (ax + by)(a\" - by), tind -ax '-0y

oz oz
4. Ifz=tan(ax+ by), find ox'ay [ADS: asec 2(a," + by), bsec 2(ax + by)

sin(ax + by) oz OZ
5. Ifz=
xy
find ox' OU.J'

a\"cos(a\" + by) - sin(a\" + by) bycos(a\" + by) - sin(ar + b Y2 )


[ADS: [ x2y , x2y I

6. If 11 = log(siny + ysinx), then prove that


a2 u
-----
a211
ax~v ~vJx

) )
x- - y-
2
ayax X + y-J

I XY
8. If u = tan . then prove that
~X2 + y2 + I

9. If u = --I .. x'-- prove that


ct 2 e 4c 't

au ) a2u
-=c---
at ax 2
10. If 11 = .I(x + Ky) + g(x - Ky) then show that

a2u = K2 a2u
ay2 ax 2
150 Engineering Mathematics - I

V z
II. If 1I =~ + - prove that
z x

x au + y ay + z au =: 0
ax . uy oz
12. I f II = log(x3 t- y3 -f :;3 ~3xyz), prove that
all au
-+-t----=:~--
UII 3
ax ~)I az x+y t- :;

I,,) I
13. I f 1I =.I(r) W Ilere r = .jx" + y + z- prove t HIt

x y z
14. [1' 11 = ~- + _.- -+ - - then prove that
y+z z+x y+z

lADs: -I)

16. If u = xyf (; l' prove that

au
x-- + y-- = 211
all
Dx . ~v
17. If x = rcosO, y = rsin0, then prove that
0 20 0 1 0
- , +-) =0
ox- ay
18. If 1-1 = logr, r = x + );J - x 2y - xy2, prove that
3

19. lf 11 = x 2y + J,2z + z2x , prove that


au+ -au+ -au= (x+ V+z-)~
-
ax ~ az .
Mean Value Theorems and Functions of Several Variables 151

20. If u = .x3 + i - 3ary then prove that


a2u a211

21. If u = e ax + by j(ax - by), prove that


au all
h-+a-=2abu
ax 0;
2
X y-) z-)
22. If - ?- + - 2- + - 2 - - = I , prove that
a- +u b +u c +u

(-au)2
ax
+ (au)2
-
0;
+ (au)2
-
az
- -2( allax au au)
x-+ y-+z-
By az
3.9.1 Composite Functions
If u is a function of two variables x and yare themselves functions of an
independent variable I.
au au ax au av
-=-.-+-.-'
then ..... (i)
at ax at ay al
Suppose now that 'u' is a function of the variables x and y, and x and yare
themselves functions of two other variables rand s.
au au ax au ay
-=-.-+-.-
then ..... (A)
ar ax ar ay ar
all au ax au 0;
-=-.-+-.- ..... (8)
'as ax as 0; as
II Ily if u is a function of rand s, where rand s are themselves functions of x and
ythen
au au or ax as
-=-.-+-.- ..... (C)
ax ar ax as ax
au au or au as
-=-.-+-.-
0; ar ay as ay
The second and higher-order partial derivatives of'u' can be obtained by repeated
application of the above formulae. Also, the formulae can be extended to functions
of three and more variabl~s.
152 Engineering Mathematics - I

3.9.2 Example
If z = fix, y), x = ell + e- v, y = e-l/ eV then show that
az az az az
---=x--y-
au av ax ay
Solution
az GZaX azOy az u az
-=--+--=-e --e
u
au ax .au Oy .ax ax . av .
az az ax az Oy
-=--+--=--e
az v az
--e
v

av fuav Oyav fu Oy.


By substraction

az - az)_( v)aZ
- - ("
e -e v)aZ
(- - - e +e -
11

au av ax ay
az y--
=x--- az
ax ay
3.9.3 Example
If u = j(x, y), x = rcos(J, y = sinO prove that

Solution
we know that

au = au. ax + all .ay = au cosO + au sin 0


ar ax ar Oy ar ax Oy
au au ax au Oy au . au
- = - . - + - . - = -(-rsll1 0) + -(rcosO)
as ax as ay ae ax ay

(-Or )2+2 ()2 =cos-O.()2


au lau-
au +2cosOsmO-.-+
-
. auau
1

rae ax axOy
Mean Value Theorems and Functions of Several Variables 153

. ~ au 2 .
SIIl-O(- ) +SIIl 2
au -~
0(-) . au au
-2cosOsIIlO-.-+ COS 2 0(-au )2
c3y ax ax ay c3y

3.9.4 Example
If u = j{x - y, y - z, z - x) prove that

au + au + al~ = 0
ax c3y az
Solution
Let X = x - y, y = y - z, Z = z -x then u = f(x, Y, Z)
all all ax au ay au az au au
-=-.-+-.-+-.-=---
ax ax ax ay ax az ax ax az
au au all au au az
lilly ay = ay - ax' az az ay
Adding all these gives
au + au + au =0
ax c3y az
3.9.5 Example
If z = j{u, v) Whereu x2 -y and v = 2xy
=< prove that
az y-=2(x
x-- az - y)az?
)-
(a) ax ay au

(b)

Solution
u = x2 - y, v = 2xy
au
ax =2x
- 'c3y au =-2y, av
ax =2y, av
c3y =2x
154 Engineering Mathematics - I

..... (i)

..... (ii)

From these, we get


at at 2 2 aZ
x - - y - = 2(x + y ) -
ax ay au

From (i) and (ii) we obtain

(-Jaz -
ax
7
az
+( -
By
)2 = {[ az az - az az-
4 x-+y- J? + [-y-+x- J? }
au av all av

3.9.6 Example
If x = reos Bandy = rsill B, prove the following:
ax ax . e.
-=-rsll1 ay = sine By = rsinE>
1. -=eose
as ' as ' ar 'as
ar x
,
ar y
,
as y as x
- =- -
2.
~X2 + y2 ~X2 + y2 x2 + y2
?
ax By ax x- + y-? '
By

3. (:r +(: r =1
Mean Value Theorems and Functions of Several Variables 155

Solution
I. We have x = reos O. Differentiating this relation partially w.r.t., rand w.r.t., '0,
we get
ox = cosO -o=r .
-
or ' ao -rsl110 ..... (i)

Similarly, the relation y = rsinO yields

oy = sin {} oy = reosO .... (ii)


or 'ao
2. From the relations x = reosO, y = rsinO, we get
x 2 +1=,2 ..... (iii)

y = tall 0 .... (iv)


x
Dr or
Differentiating (iii) paliially w.r.t., 'x' and w.1:l. ), 'we get -0 ;-0
x y
This gives
or x x or y y
ox r Jx 2 + y" ' ..... (v)

Differentiating again (iv) partially w.r.t., 'x' and 'y' we get

y( __1 ) = sec" 0 ao , ~ = sec" 0 ao


x2 ox x ~v

which gives

-ao = - -,
y = cos " 0 = --,
y =, y ,
..... (vi)
ox x r- x- + y

00 cos" 0 x x
-=--=- .... (vii)
oy x ,.2

3. From (v) we get


2
or)2
( ox +(or)"
oy r- r-
=X=< =<
+y2 =1
r2 ..... (viii)

(::l\(:r~1
156 Engineering Mathematics - I

4. From (v) we also get

..... (ix)

..... (x)

Adding (ix) and (x)

-+
a2 ,. a2r I
2 -=-
ax ay2 r ..... (xi)

5. From (ix) and (x), we get

..... (xii)

Differentiating ar = Y partially w.r.t., 'x' we get


ay r

So that ..... (xiii)

From (xii) and (xiii) we get that

..... (xiv)

6. From (vi) we get

Similarly from (vii), we get

..... (xvi)
Mean Value Theorems and Functions of Several Variables 157

Adding (xv) and (xvi) we get


2 ~
a 0 a-A 2xy 2xy 0
----:? + - 2 = -4- - - . 1 =
ax ~v r r

Note:
I. The variables (I', 0) related to (x, y) through the relations x = rcosO, y = rsinA are the
polar co-ordinates of the point whose cal1esian co-ordinates are (x. y).
2. In obtaining the resllits in (i) rand 0 are regarded as independent variables and
x and yare regarded as dependent variables.

3. In obtaining the results (ii) - (vi), x andy are regarded as independent variable~ and
rand 0 are taken as dependent variables.
4. After partial differentiation is carried out, the final expression for the partial derivatives
are to be expressed in terms of the independent variables.

. or ax ax ao . ar
5. From the expressIOn for - , - ; - , - , It may be observed that - is not the
ax ar as ax ax
.
reclproca I 0
fax ax.IS not the reclproca
or an d ae . I 0f ao
ox' I n Clact we note IJere

ar = ax and ax = 1'2 as . Similarly ar = 0' ay = 1'2 as .


ax ar ao ax 0' ar ' as 0'
6. The results (i - vi) arc useful in calculations involving polar co-ordinates.

Exercise - 3(G)

I. If u = x + y, v = xy and f = feu, v), show that


aj Of Of Of
x-+ y-=u-+2v-
ax 0' au av
2. If z = 1(r, y) and x = e" + e- X and y = e- II -ev show that
az az az az
---=x--y-
au av ax 0'
158 Engineering Mathematics - I

3. If z = ttu, v) where II '--= x2 - y2 and v = 2xy prove that


OZ UZ 2 2 ()z
x - -- v- == 2(x + y ) -
ox . oy OU

4. If II = (!"siny. v = e'cosy and z =.f(1I, v) prove that

5. If Z = /og(u 2 + v). U = eX' +/. V = x2 -f y, show that

!!..~ _ 2.\"(21/ 2 + I) OZ _ 4Y1l + 1


2

ox - /1
2+V ~v -
1/ + v

6. If w =.f(u. v) and u = x 2 -I, v = 2.\:v prove that


o2 W U-W
--+--=
)
J 4(.2
) .\ +Y
2) r
l--+--
J
o-W 1
;-,J
u-W
2)
olr ~v- OUov-

x V
7. Ifu = -;V=-'-- and w =zandf=}(u, v, ll~ show that
z z
Of Of Of Of
x-+y-+z-=w-
ox oy oz ow
8. If z = }(x, y), x = 11 cosh v, y = 1I sinh v prove that

9. If z = }(u, v), It = Ix + II~Y, l' =- ~v - I11X, show that

a-J +
- z 0 2z (/ 2
ox ~2
2
- = +111 2)(2
0 Z+0-
- 2Z
0,,2 ov 2
1
10. If z =f(u, v), U = x2 -I, v = 1- x 2 show lhat
OZ oz
x-+ y-=O
~v ox
Mean Value Theorems and Functions of Several Variables 159

II. If x = ell /e/1/V.)' C~ ell sec)' fino the vallie of

( x~l!-ax + .}' Dy(~)' 1x (x ax'


vv + }'-~l
~v
lADS: 0 I

(
. II == 2
Ilmt I Iog(y-? -. x-)
) an d v::= Sill
. I( yx II
12. If w = .f(x2 - y, )~ .. z2, 22 -- x 2) prove that
-.!. ow + _~_ ow + _~ all' == 0
x ax )' ~y z Dz
13. If w = j(x, y), 11 0= e\ v ~ eY show that
D2 w 3 2 \1'
--:= ---.(111')
3x~v ouvv
14. If x::= -!vw,y::= j;;,z = -J;;;, Prove that
o<p 0(1) o<jl o<jl o<jl o<jl
x-+ v---+z--=u-+v--+w-
ax . ~Y oZ all ov all'
where <jl is a function of x, y, z.

3.10.1 Homogeneous Function


Definition
A functionj(x, y, Z, II .. .) is said to be homogeneous in x, y, Z, 11 ..... if it can be

written in the form xl1 g(Yx '!-,!~


x X
..... ) and is said to be of degree 11.

Ex. I. z = sin-I (~ ) + tan 1(~) is homogeneolls and is of degree '0'. Since

Z=xO[sin-I(~J+tan I:]

2 + 2
2 Z = x y .
IS IlOmogeneous an d'IS 0 f'd egree Isll1ce
'
x-y
160 Engineering Mathematics - I .

3. ~ = x 3sin(~) + A)!ZIOg(;1+ XZ2 tan I(~ 1

~ x'ISin(~)<~lot H~r tan'(; lJ


llence ~ is of degree' 3' and homogeneous in x, y, z.

3.10.2 Euler's Theorem


The following theorem, known as the Euler's theorem gives a very lIseful formula
for a specific combination of partial derivatives ofhomogeneolls functions.
Theorem :
ffz is a homogeneous function ofx andy with degree '11' then
oz
x-+ y-=nz
oz
ox . ~y
..... (i)

Proof :
Since Z is a homogeneous function of degree '11' we can write 'z' in the form.

z= xllf(~)

Now ~: = Il - ~) + x l /(:)( ~;)


I1X
1f
(

= nxlllf(~)_xll 2y/(~)

and

xoxoz + yoz = f( xy) -x" 'y/ (y)


~y
I1X"
x
+ x" 'y/ (:Ix)
x

= nxllf(:Ix)

=nz [.:Z=Xllf(;)]
Mean Value Theorems and Functions of Several Variables

uz
x-+ V-=IIZ
oz
Thus oX . ~V

3.10.3
Ir z is a homogeneous functions of x and y with degree 'n' then
2 J )
]OZ o-z 20-Z ( )
X - , + 2xy - - + y -i = 11 II - 1z ..... (ii)
ax - uxoy uy
Pr(}(l:
Diflerentiating (i) pUl1iaily w.r., to x, we obtain
02 Z OZ 02 Z OZ
x--+-+
2
y--=ll--
ox ox oxoy ax
02 Z 02 z UZ
I.e., x-) + yx--=(n-I)- .... (iii)
ox' oxoy ax
Differentiating (i) par1ially w.r., to 'y', we obtain
02 Z 02 Z oz
y-+x--=(n-1)- ..... (iv)
oyJ oyox oy
Multiplying (iii) by 'x' and (iv) by 'y' and adding we get

02 Z
? 02 Z ) 02 Z (OZ Oz)
x--? +2xy--+ y-? =(n-I) x-+y- =n(n-l)z
ox' oxuy oy' ox ry

3.10.3 General Formula


The Euler's theorem may be generalized to homogeneous functions of more.
than two variables. For a homogeneous function j(x, y, z) of degree 'n' in three
variables, the theorem can be read as :

x oj + y OJ + Z oj = nj
ax ry oz ..... (v)

In general, for a homogeneous functionj(xl ....x,,) of degree 'n' in 'k' independent


variables x\ .... x k the theorem reads
OJ OJ oj
xl - +x2 - + ..... +xk -
aXl aX 2 ax k =l1j ..... (vi)

The proofs of (v) (vi) are left to the student


162 Engineering Mathematics - I

3.10.4 Example

.
1/ 1I :~. cosec
-[;-;+JY]
I if;: + if;; ,thcn prove that x au au __ ~
ax + y ay - 6 tanu

Solution
We note that from the given cxpression for' u' we can write

cosecll
;-;+JY
= "il
.
;1.: f (say)=: ..... (i)
'Y.\" +vY

Then

:. fis a homogeneous function of degree ~, Hence by Euler's theorem

W WI. I
x-+ y-=- j =-cosecu ..... (ii)
ax Oy 6 6

a a I
x-(cosecu) + y-(cosecl~ =: -cosecli
I.e.,
ax ay 6
Mean Value Theorems and Functions of Several Variables 163

Carrying Ollt partial differentiation, we get

I
--cosecu 1
au a 6 II
x-+ y-=-=----=--tanu
ax ~v - cosec II. cot u 6

x-
au + yau- = - -1 tanlt
ax ry 6

3.10.5 Example

Ifu= j o( -,-,-
x y z J prove that x-+y-+z-=O
au au au
yzx ax ry az

Solution

11= o(xy
j -,-,- z)
y z x

all au au
x-+y-+z-=j 0'( -----+---+-
x z x y y z J
ax ry az y x y z z x

=0

3.10.6 Example
au all
If u = sin 1; + tan -1 ~ prove that x ax + Y ry = 0

Solution

u = f(x. y) is of degree '0'


By Euler's theorem
164 Engineering Mathematics - I

au ali
x-+ y-=o.u=o
ax 0'
. ,x ,y
Sill - = v, tan - - =w
Alter:
y x
u=v+w
au avow -::::-+-
-=-+-
au av Ow
ax ax ax' ay 0' 0'
x . 1 av
-:::: Sill V, - = cosv-
y y ax
y - y ? aw
- = tan w, -} = sec- w.-
x x- ax
-x av I
--:::: cosv- - = sec"w.-
) aw
y2 0" x 0'

=0

3.10.7 Example

If ZI =
X3 + 1'3)
tan' - __ prove that
( x+y 0 -

au au .
x - + y - = s1112u
(i)
ax oy
2 a-u a-u? au. .
J 2 ?

(ii) X - 2 +2xv--+ y-} =sm4u-sm2u


ax . axay ay-
Solution
Proof:
For the given expression for 'u' we have
x3 + y3
tanu = ----'-- = f, say
x+y
Mean Value Theorems and Functions of Several Variables 165

Ilere ~f' is a homogeneous functions of degree'2'

By Euler's thcorem
at" of
.\"-'
Ox +-
Oy .1' 7f'
= 1l.J =~ --:

I.e., X~(lanl/) I~ y~~~(tanll)= 2tanll


ax ~v

Carrying oul partial derivatives, we get

?
:"\
vII ?
all
xsec-II-;-~ + ysecII - 2tanll
vx ~v

au
x - + y~~~
all 2 tan 11
= --,,- ceo
.
S1l1211 ..... (i)
aX (~y sec ~ u

Next dirterentialing bL)th sides of the resull (i) partially \Y.r., to 'x' and 'J" wc get
alII all alII
x ~-, + ~- + v .~-- =-- 2 cos 211 -
Ou
..... (ii)
ax- D.\ . axD)' Dx

a"11 D) II UII all


x -~ + )' - , + -~ = 2 cos 211 ~~ ..... (i i i)
(~VaX ~v' (Iy ~v

Multiplying the result (ii) by x, the result (iii), by 'y' and adding the results we get

a~1I + y'
x', ~-,- ,a"II +
~-:,~
'1
~xy-- =
a"II (2cos211 ~-l (all all)
x - + y---
(1.\'- ay OXG)' Dx ~v

= (2cos211 -I ) sin211, (from i)


= sin411 - sin211

3.10.8 Example

Solution

Let and
166 Engineering Mathematics - I

Then
v and ware hOl1logeneous./llllclioll.\' (!ldegree '2 '. Applying 3.10.3 to 'v' and
'w' we get

..... (i)

!Illy ..... (ii)

(i) - (ii) gives

Note:
Letj(z) be a homogeneous function ofx and y of degree Il ; then from Euler's
theorem
('JI"
x_:'l
a" =I/f
+ y_:'l
ax Dy

.' f)z , az . oz OZ Ilf


i.e., x..f . ax + xi ay = 1?/ => X ax + y ay = 7 ..... (i)

DifTerentiating (i) w.r., to y'partially

a'z aZ }I--=Il
J

\'-+-+
. ax 2
a'z [j")
Dx . oxoy
'
J
--JJ'fj'" aZ
/2 -ax
j ..... (ii)

Differentiating 0) w.r., to 'y' partially

a z
2
a z az
x--+y-+-=n
. axay ay2 ry
2
[/2/2-jt' 1-az
ay ..... (iii)

(ii) x + (iii) Y gives

= Il[j '2 ~ f( ][x.az + y_~~] ..... (iv)


j2 OX~}I
Mean Value Theorems and Functions of Several Variables 167

Using (i) in (iv)

..... (v)

llere

3.10.9 Example
If z = log(,3 + .vI __ x 2y __xy2), prove that

a=
x-+ v- =3
oz
(i) ax . ry
Note: (i) and (iv) can be llsed as formula in such cases
Hence f = t!, f' = c=, f" = e= and 1/ =3

From (i) of3.1 0.8 note az


x-+ v--=3
u=
Dx . ~v

1 ) ,
lO-Z az 2a'z "
Form (v) on. 10.8 note x - , +2xv--+y -,=3.2--9=-_,
ax' . axoy ~v'

3.10.10 Example

Verify Euler's theorem for the function 1/(x, y, z) = Jx',z+y;


Solution

1I is a homogeneolls function of degree -=J.


2

..... (i)

uu ,
2u- = -z(x- + v-) . . 2x
, _,
So that 0;'" .
168 Engineering Mathematics - I

au ) ).)
2u~=-z(x- + y) - .2y
oy

gives au y~+z-=-
x--+ au ou
ax oy oz
-z[
211
) I )
x- + y-
1
2
U I
= - - - -:;: ----1/ by (i)
211 2
Hence the result

3.10.11 Example
J J 3
X +Y - Z of Df of
If f = cos I I then show that x - + }'- + z-'- = -cotf
"X 1 -t )'4 - Z4 AX . ~v oz
Solution

..... (i)

So that coif= 11

From (i) we see that 11 is a homogeneous function of degree I.


By Euler's theorem
all
x~+ y~+Z--=-ll
all au
...... (iii)
ax ~ az
Form (ii), we find that

au = -sin l. Of, ou = -sinf i?l, au = -sin fo.l


ax . ax oy ~v oz . oz
Using these and (ii) and (iii), we get

-Sill .f[ x~
ox
of + z-'-
Of + y-'-
ay
Of) = cos j'
oz
Of Of Of
x-- + y - + z~ = -cotf
ax ~v oz
Mean Value Th.eorems and Functions of Several Variables 169

3.10.12 Example
,
. 2
1all au2 2
,all ...
' ,]1
x' + y3
Fmd x- - , + 2xy-- + y- - , ,If u = sm-- I
ax' axoy ~~!. [Xl + y2
, ,

Solution
,
x3~ + y31 ]~
Taking z = , , then z = sinu =.f(u) say z is a homogeneous function or
[
Xl + y2

I
degree - - = n
12
From Euler's theorem
au all j(u) I sin II 1
x - + y - = 11-,- = - - - - = --tan II
ax ay f (ll) 12 cosu 12
, 2 1 . 1 ) ,"

a'u .... a 11
2 2 a-lI ( I) f 11- f -I
Again x -+.ay--+y -,=1111- - , - ,
a.'/ axay ay' f f 3

= (-~)(-
12
~
12
-I)tanu - _I_(-tan u).tan
144
2
1I

{-:./ = COSll, (' = -sin 11)

tanu 1
= --(13 + tan-u)
144

3.10.13 Example
2
_ Y )a u a 2II 1
1 a-lI
If u = tan' ( -=--- ) ,find x' --+ 2xy--+ y--2
X ax axay ay

Solution

Let z = 2:. degree of z is 0 and z = tanu


x
az
x-+ y-=o
az az 1 au af.
- =sec'u-
,all
-=sec'u-
ax ay ax ax oy ~v
170 Engineering Mathematics - I

..... (i)

x 02~ + au + y 02U = 0
..... (ii)
. uxoy oy ~l

(i) x + (ii) y gives

) 0\' OlU
x--+2xv--+ y-=- x-+ y - =0
, 0\. (au au)
0'(2 . ax~y oy' ax oy

Exercise - 3(E)
I. Verify Euler's theorem for the following functions:

xy
(a) z=-- (b)
x+y

1 X} - Y1 )
(d) z = SIl1
( x-y

( e) _. --I X
3})
Y 2
+ y-?+ z-?) 1
Z-SIl1
(
X
3
+Y
3
(I) Z = (x 2

2. If z = xyf (~) then show that x OZ + Y OZ = 2z


y ax ~

3. Show the following by using Euler's theorem:


x3 + / OZ OZ 5
(a)lfz= . thenx-+y-=-z
~x+ y ax ~Y 2

x 3 + y3 OZ OZ
(b) Ifz= log ,then x-+ y-=2
x+ y ax oy

x-y 3 3)
87 OZ
( x-y
(c) Ifz=sin- I - - - - then x--=' + y - == 2 tan z
ax ~y
Mean Value Theorems and Functions of Several Variables 171

4. If z = tan-I ( x + /
J

lx+y
l' then x uz + y OZ = sin 2u
8x '~y

5. Ifz = sec~l[x2 + yll, then x a::: + yUZ =cotz


x+ y ox ~

6. If z is a homogeneous function of x and y with degree' 11' and z = f(u), prove that

x-+
all au
y-=n-'--
(u)
ux . ~ /(11)

7. If z = x 3
. (y)
1 SIll-1 - ,show that
a 2
x 2 --T a 2
z + 2xy--
11 fiz
+ y2 -2- = 20z
x ax- axay ~y

8. If z = tan-I ~ X4 + y4 , show that

az oz
(i) x - + y - = sm2z
.
ax oy
ii)
,a
x- -
2
)
z
+ 2xy
0 2z
- - + v-
, 2z . a .
-) = s1I14::: - Sill 2z
(
ax' ax~v' ~v'

az 8z I
(i) x-- + y - = -tanz
ax ay 2

1
10.lfz=tal1- 1 [;l+YSin- [.:l show that

\ \-
x2 + y2
II. If z = cosec-I then show that
172 Engineering Mathematics - 1

a2 z 0-" z 2 0-" Z
X
2
-2
ax
+ 2xy - - + y ---:;-
aX~Y ay
tan zll31 J
= - - -+-tan- z
]

12 12 12

12.lfz=tan- 1 x +y 2 2) ,then show that


( x-y

2 2 2
"a z a z )a z
13. If z = log rand ,2 = x 2 + y then show that x- - 2 + 2xy-- + y - " + 1 == ()
ax axay ay

14. If z = sin-I
X4I
!
+ y4I
!
j then show that
r x5 _ y5
') 2 2
2 a-za z )a z 1 2
X - - 2 +2xy--+ y - , =--tanz(tan z-19)
ax ax~ ay 400

~ x 2+ y2] ;\2;\
2uZ uZ2 ; \2UZ
2
15. If z = log ( x+y find the value of x GX2 + 2xy axay + y ay2

3.11.1 Jacobian
Ifu and v are functions of two independent variables x and y then the determinant

au au
ax ~ i.e.,
ux uy
Ov Ov v, vy is called the Jacobian of u, v with respect to x and y
ax au
and is denoted by

a(u, v)
a(x,y) or .J(~)
x,y

If u, l~ ware functions of three independent variables x, y, z then the determinant


Mean Value Theorems and Functions of Several Variables 173

lI( UI' 1I,

V( vI' Vz

W( Wy lV,

is called the Jacobian of 1I. V, w with respect to x. )~ z and is denoted by

a{/I, v, w)
or )(U,\" w)
a{x,y,=) x,y,z
Similarly

aUI uU I (J/I~
a{U p 1l2uJ _ aX I aX 2 ax"
a{X I'X 2.....x,,) - all" OU" au"
oX I aX2 ax"

SOME PROPERTIES OF JACOBIAN

3.11.2 Property
a(u,v) a{x'Y)_1
a(x,y)' a{u, v) -

where J = a{lI, vI and _ a{x,y)


.J - 0(11, v}
a{x,y)
Proof:
Let u = u(x. y) and v = lI(x. y) so that u and v are functions of x and y. Then
differentiating these partially each with respect to II and v, we get

I := au . ax + all . 0'
ax all ay au
174 Engineering Mathematics - I

av ax av ay
O= - . - + - . -
ax au ay au
1= av .ax + av .ry
ax av ay av

au all ax ax
Now
a(lI, v) a(x,y) ax oy x all av
a(x,y)' a(u, v) av av ry ay
ax ay au av

au -au
-
ax ry
-
ax ay x au all
av av ax ry
ax ay av av
(Interchanging rows and columns in 2 nd determinant)

ali ax au oy au ax au ay
-.-+-.- --+--
ax au oy au ax' av ay' au
av ax av ry av ax av ay
-.-+-.-- --+--'
au au ry au ax' av ~v' av

=I~ ~1=1 Proved

3.11.3 Property
a{u,v) _ a{u,v) a{r,s)
a{x,y) - a{r,s)' a(x,y)
where u, v are functions of r, sand r, s are functions of x, y.
Proof
By differentiation offunction ofa functions, partially; we get
au au ar au as
-=-.-+-.-;
ax ar ax as ax
Mean Value Theorems and Functions of Several Vanables 175

ou
-=-
or oy OS
au -+--'---_.
~ or .~ as oy , ..... (i)

oval'
01'
-=~- - + avos
-_.
ox or .ox os ox '
Ov 01' or 01' as
-=- -+--_.
and ~y or .oy os ~y ,
all au (lr (Is
=?----=
o(u, v) a{r,s) (lr os (Ix (Iy
(1(r,s) a(x,y) uv elv x us us
i"lr us (!y. ilyl
Oll or as
all
or os x ax ax
ov -oV (II' as
or as oy oy
(Interchanging rows and columns of the second determinant)

allor ou os
-.-+-.- -.-+-.-
ouor Oll as
or ox os ox or oy os oy
Ov or OV os ov or ov as
-.-+-.-- -.-+-.-
or ox os ox or oy as ~
ou -ou
ox ~ _ o(lI,v) using (i)
ov Ov - o(x,y)
- -
ox ~
3.11.4 Property
If u, v are functions of two independent variables x and y thell u, v, are

independent if
oo((U,V))oFO.
x,y
Otherwise they are dependent. Similarly if II, v, W

. . . . o(u, v, w) 0
(functions of x, v, z) are II1dependent If and only If ( ) oF
a x,y,Z
176 Englneenng Mathematics - I

Theorem
If the functions 11 1, 1I::...... 1I11 of the variables xI' x 2 .... xn be defined by the
relations Il, ==.ll'",), 112 =flx ,. x 2), 113 = .li x ,. x 2 x 3 ) ..... 1/11 .I;,<x,. x 2 ..... x). cc

Then
O(lI p 1/2 .....U,,} OU, UU] OU 3 OU II
o(x"x2.....x,,) - ox, . oX1 'a~~ OXII

We know that

Similarly lor

Oil, Oil 2
ox, ox,
Oil, 0111

0(U"l/2 .. ..U,J _ aX 2 oX1


Then
o(x, 'X 2 ..... X II )
all, ~1I2 aU Ii

OXII OXII OXII

OU, Oll] OU"


ox , ox , ox ,
Olll. Ollil
0
oX 2 oX1 OU, Olll OU II
ox , . oX 1 OXII

o o

3.11.5 Example

_ o(x,y)
If x = 112 -vl . Y = 2uv ,. tllld --(--)
0 1l,V

Solution

ox
We have x = u2 - v2 ~ -ox =
2lI. --=-2v
all OV
Mean Value Theorems and Functions of Several Variables 177

cYv 0)1
v = 2uv => --- = 2 v -"-- = 211
- V I I ' 01'

ox AX
V{X,y) _ or ()I~ ~ 1211 - 21'1 := 4(u-
+)1'- )
O(II.~) - VI' ~F 21' 211
au VV
3.11.6 Example

If x ~ reose. y ~ rsintJ find - (- ) lllld-(--)


V(x, y) a( t, )
, V r,O a x,y
Solution
x = reosq, y = rsinO

(It
~ = cos()o
=> (1,.

-ax = -rsiIlO, ov = rc()sf)


---
of) 0(1

ax ax
o(x,y) _ or ao _Icoso -rsinol
Then ~v
. = r(cos 2 0 + sin 20) = r
0(1',0) - ~v - sinO r Sill 0
or ao
Again 12 = x 2 + y2 ; 0 = tal1- 1 ~
x
or
2r-- = 2x
ox
(l() -- y -y
ax r+y
0 0 0
r-

or +x
- = -
ax I'

Dr y (1() -y -y
ay ,. ilx x2 +)'2 r
)
178 Engineering Mathematics - I

x x

lar ar x
a(,., 0) ~ ax ~}! Y
aCy,y) ~ ao ao -- ~ y
lay vy
3.11.7 Example

11
If x = uv, y = - then show that .1./ ::.: I
v
Solution

We have
ax
-=V,
at
-=11
all al'
~F ay
---
11
= )
VII v aI' v

VU DII
But
')
xy 2u- = )I, 2u-=x
lr = ax . ay

v
, x
=-, 2v
av =!... 21'-=
aI' x
and y ax y' ay
--?
y.

y x
II" lly 2u 211
J =
Vr v)' I ~x

2vy 21:J!

X X X

4uvy 411vy 2uv Y

I? V
=---v-
211v 2u
Mean Value Theorems and Functions of Several Variables 179

- 3.11.8 Example

Prove that JJ = 1 for x = e V secu, y = e V tanu


Solution
ax = e secu tanu, -=
ax e V secli
all av
oy 0; v
and = eV sec 2 u; - = (' tanu
au av
x" = "
e secutanu
v
e secu
=
2"
-e secu = -xe
I'

Yv I I'
e sec-
)
II e" tan 11

Now sec 2u - lan 2 1I = 1


x 2 e- 2v - ye- 2v = 1

eVtanu Y
e2v = x 2 - y, also ---
e" secll x

. y
SlIlli = -
X

U
-1
= Sill -;
(Y) an d v '21 Iog(x = 2
- Y-))

Du y all
,
i1x X~X2 - /
(Jy X~X2 -y 2
av x Dv y
1 ? ?
ax x- - y ? '
i1y x- - y-

Y
lIx lIy X~X2 _ y2 ~X2 _ y2
J
Vt VI' X Y
x2 _ y2 x-1 - 1
y-
180 Engineering Mathematics - I

, \' I
JJ =xe ,-=1\'
xe

3.11.9 Example
VIV Wlt lIV I I I O(X, y, z)
If x=-,y=--,z=-- t Jen SlOW tlat - ( - '-)=4
II v W 01l,V,\I'

Solution

ax ox oX VI\' lI' v
2
Oll ov all' u It 1/
o(x,y,z) _ oy oy oy 11' Wit II
2
0(1/, V,-H') Oil ov ow V V v
OZ oz oz V II uv
011 OV ow IV IV

- VlI' Wli ltV

VlI' - Wli ltV '-2 '-2 ' 2


II v W
VlI' lIW --l/V

-I
-I
-I

=-1(1-1)-1(-1-1)+ 1(1+1)=4

Exercise - 3(1)

I. If y + x + Z = 1I, Z + Y = 1Il: Z = lIVW, find o(x,y,z) [ADS: (1I


2
v) J
o(u, v, w)
Mean Value Theorems and Functions of Several Variables lSI

o(u, v, w)
2. If II = x+ 2/ - Z2, V = x2yz, W = 2X2 -xy find at the point (1,-1,0)
o(x,y,z)
Ans:19
3. If x==rcosB,}J==rsinB, find ~(x,Y) and !JS!~~~ and show that
. 8(r,0) 8(x,y)
o(x,y) o(r,O)
--'--. = 1 (Hint :relcr 3.6.6)
o(r,O) o(x,y)
XOXI X X, XIX)
I ,y, = - - - ,show that ----'----'--== 4 o(YI,VnY,)
4. If YI = - --- ,Y) = ---
XI - x2 - x, 0(X I ,X2,X3)

O(YI' Y), y, )
5. If YI ==1-XI'Y2 =xl (l-x2 ),y] ==xlxl(l-x,),provcthat - = (1)3
- XI 2 x"
0(X I ,X2 ,X,)
6. If x=a(coshu)cosv,y=a(sinhll)sinv, then show that
2
o(x, v) a
--,-'- = - (cosh 211 - cosh 2v)
0(11, v) 2
7. If II = /(x l ), v = (x l , x 2 ), W = If/(x!, X 2' Xl) then show that
Oell, v, w)
=---
011 Ov uw
0(X I ,X2 ,X3 ) i
ox
' oX ox}
2

8. If X
. 1')
= r S1l1 r.
do
cos 'I', 0 . do
Y = r Slll 0 I oC X, y, z) ). 0
S1l1 or, Z = r cos - S lOW that - - - = ,.- S1I1
o(r,a,)
ox ox uy
or 00 o sinOcos reosOcos - r sin Osin
o(x,y, z) ay oy uy
Solution: = -- = sin Osin r cos Osin rsinO eos
o(r,O,) or UO tJ
cosO -rsine o
OZ OZ oz
--
or oe o
= sin ecos[ 0 +,.2 sin 2 Oeos ] + r coseeos.r sineeosecos +
+( -r sin esin )[ -r sin 2 Osin - r cos 2 esin ]
== r2 sin 3 Oeos 2 + r2 sin Ocos] Oeos 2 +,.2 sin 3 Bsin 2 + r2 cos 2 Osinesin 2
== r2 sin 3 0 + r1 sin () cos 2 () = ,.2 sin O.
182 Engineering Mathematics - I

3.12.0 JACOBIAN OF COMPOSITE FUNCTIONS:


Let (u,v) be functions of x,y where x,y themselves are functions of r,e .Then 1I,V arc
. e 8(1l, v)
. fi ..
composite unctlOlIls of r, and = 8(u, v) . 8(x,y) .
(from 3.6.3)
8(r,B) 8(x,y) 8(r,B)
Note 1: Ifu,v are functions ofx,y we may regard x,y as functions ofu,v.
e
Taking r = ll, = v in the above result, we have
au
-
8u
8(1l, v) 8(x,y) 8(u,v) 8u 8v 1 0
--- = =1
8(x,y) . 8(1l, v) 8(1I, v) 8v av 0 1
811 8v
8(1l, v) 8(x,y)
.. = 1 => .1.1 1 = 1
8(x,y) . 8(1l, v)

where J = 8(1l,:i,J = 8(x,y)


I

8(x,y) 8(1l, v)

8(u, v, w) 8(u, v, w) 8(x,y,z)


Note 2: = . where u,V,W are functions of x,y,z and x,y,z
8(r,e,) 8(x,y,z) 8(r,e,)
are functions of r, B, .
Also J./' = 1; where J= 8(1l, v, w) , .II = 8(x,y,z)
8(x,y,z) 8(u,v,w)

3.12.1 Example: If II = xyz, V = x 2 + y2 + Z2, W = x+ y+ z find .I = 8(x,y,z) .


8(1l, v, w)
I 8(1l, V, w)
Solution: we first evaluate .I = , since u,v,w are explicitly given as
8(x,y,z)
yz zx xy
functions of x,y,z. .II = 2x 2y 2x = -2(x - y)(y - z)(z - x)
1 1 1
Hence from the relation that .1.1
1
= 1, we have
J = 8(x,y,z) = -]
8(u, v, w) 2(x- y)(y-z)(z-x)
Mean Value Theorems and Functions of Several Variables 183

Exercise - 3(G)
) 1 C(ll, v) 3
I. Ifu=2xy,v=x--y, whcrex=rcosO,y=rsmO show that ---=-4,..
o(r,O)
2. If x = ;;;;,y = Jwu,z =-r;;;;
where II = r sin OcosrjJ, v =,. sin Osin rjJ, W = rcosO,
v(x,y,z) 1 2 0
show that = --,. Slll
a(r,O,rjJ) 4
ll+V _ v(u,\') (1I-V)2)
3. If x = ltV )' = -- fll1d - - - (A liS : --'------'-
, 1I-V' o(x,y) 411v
yz zx xy 8(x,y,z)
4. II II = - V = - HI = - prove that =
x ' y , z 8( ll, V, lV) 4
x +Y I D( ll, 0)
5. 1I1I=--,O=tan-l(x)-tan-- (y) show that =0.
1- .xy v( x, y)
1 v(u,V,lI')
6. If lI=x--2y,v=x+y+z,w=x-2y+3z show that ----=1Ox+4
8(x,y,z)

3.12.2 Jacobian of implicit functions


Let 1I1' 1I2' 1I3, be implicit functions of x,, x 2' x3 so that
jJu" 1I2' 113' x" x2, x3 ) = 0, r= 1,2,3
Then oj; + a.t; .aU I + a.t; .aU 2 + a.t; .aU 3 = 0 and so on
aX I aUI oX I oU 2 aX I aU 3 oX I
a(.t; ,j~ '/1) 0(11" u), uJ
then
-a(u"U 2 ,1I))' a(X I ,X2 ,X3 )

I aj~ . all,. I oj~ . OUr I a.f! . Ollr


all,. ax! au, oX2 all, oX3
= I af2 . OUr I of2 . all,. I a.f~ . all r
Ollr ax! OUr aX 2 au,. aX 3
I a.t:l }u r I 'fJj~ . OUr I af3 . au,.
au,. ax! au,. oX2 all, aX3

(by the rule of prodllct of determinants)


184 Engineering Mathematics - I

_ aJ; W, _ ~t;
ax, aX2 ax)
_ ~f2 _ aj~ - aj~
au, QU 1 au)
0f, aj; 0f,
ax, aX2 ax)

= (_ I)' a(J;,j2''/;)
a(x"X2 ,X3 )
aCt; ,j~,f,)
Hence
a(u" 112' uJ _(_1)3 a(;;~-x~~~,"J
a(x, x2 ,xJ - ~V;,fI,lj)
, a(U"l/2,lIJ
(The result if obvious)
3.12.3 Example

x y z
If u = r:--:;' v = r:--:;' W = r:--:; prove that
v'1-r2 v'1-r2 v'1-r2

a(u, v, w) _ 1
a(x,y,z) - (l-r2 y;
where -? = x2 + r+ z2

Solution
we have II (1-/ 2 ) =x2
=> II(u, v, W, x, y, z) = u2 (I _x2 - r _z2) - x2 = 0
Similarly ./; == v2 (1 - x 2 ~ r - z2) - r = 0
13 == w2 (1 - x 2 ~ r- z2) - z2 = 0

.... (i)

(see 3.13.2)
Mean Value Theorems and Functions of Several Variables 185

J
- 2u 2 x- 2x - 211-Y - 2112 z
aCt; ,.I~ ,.I;) _ -2v 2 x -2v 2 y-2y - 2v 2 z
8(x,y,z)
-2w 2 x - 2w 2 z -2z
)
- 211'- y

?
u2 + 1
)
u- 11-

= (-2x) (-2y)(-2z) v-
) 2 2
v +1 V

w2 +1
) )
w- 11'-

0 u-?
)
-8xyz -I v-
0 -I w2 +1

by C 1 -C 2
C2 -C 3
= -8xyz(u2 + v2 + w2 + I)

X2 y2 Z2 ]
= -8xyz --J +--- 2 +--? + 1
[ I-r- I-r l-r-
? ') ') ,.,
x- + y + z- + 1- 1'-
=-8xyz J
1-r-
-8xyz
=--2 ;also.
I-r
2u(l-x2 - / _Z2) o o
8U;,.I;, .I;) o 2u(l_x2 - 0
= y2 _Z2)
8(ui' v2 , w3 )
o o 2u( 1- Xl _ yl _ Z2 )

8l1vw(l- Xl _ y2 _ Z2 )3

8 x . Y . z (l_r2)3=8~yz(l_r2)112
~ JI-? .J1-r 1
_. 8(11, V, w) 1)3 -8xyz ~ I
Hence trom (I), we have - - - - = (- - - , 8 (I 2)1/2 = r:-?
8(x,y,z) I-r- xyz -r ',"1_1'2

X Y Z
Cor: Given U=- V=- W=-
k' k' k

where k = VII - X
2
-
' f iIn d ,(x,y,z)
Y, - Z-, in terms ofk. lAns: k
S
]
(u, v, w)
186 Engineering Mathematics - I

Exercise - 3(H)

8(x,y,z) 2
I. Ifx+y+z = u ,y+z = uv, z = uvw, then prove that ----'----=--~ =- II V
8(u, v, w)
~)

8(u,v) x--y-
2. Ifx + l + u _v = O,uv + xy = 0, prove that
2 2 2

8(x,y) u 2 + v 2
3. If u1 =x1 +X2 +x3 + x,pU 1U 2 =x2 +X3 +X4 ,1lIU2113 =x4 then prove that
8(X p X2 ,X3,X4 ) 3 2
=1I1 .il2 113
8(u l , 112 , u3' 114)
4. If u 3 +V3 +W3 =X+ y+z,u 2 +V2 +W2 =X3 + / +Z3,
8(1I,v,w) (y-z)(z-x)(x-y)
t Ilen SlOW
I t Ilat = ---"'----'---'..-----'-----'-----'----"--
8(x,y,z) (v- w)(w-u)(u - v)
3.7.4 Use of Jacobians in determining functional dependence and independence of
functions.

Let u and v be two functions of x and y connected by the relation v = f(u), then
we say that u and v are functionally dependent. We shall prove that the condition for
functional dependence is

o(u, v) = 0
o(x,y)
Consider w = v - feu) = 0

Now w is a function of u and v where u, v are functions of x and y.


w is a composite function of x and y
ow ow au Ow Ov
-=-.-+-.- ..... (i)
ax ax ax ov ax
ow ow au ow Ov
-=-.-+-.- ..... (ii)
ax au ax ov ax
But w considered as a function of x, y is identically zero.

Ow
i.e., -=0
ax '
Mean Value Theorems and Functions of Several Variables 187

From (i) and (ii) we get


Ow all ow ov
-.-+---.--=0 ..... (iii)
au ax av ax
Dw
--.--+-.-=
all ow av 0
and ..... (iv)
au ~v D1' ay
Ow aw
Eliminating -a--o
11 I'
from (iii) and (iv) we have

all aI'
ax ax =0
all -av
Oy ~v

au au
--
ax ay
ou au =0
ax Oy

a(u,11=o
=> a(x,y)
The concept of functional dependence can be extended to any number of
variables. Thus if u, 1\ 11' are functions of x. y, z the 1I, v, w will be functionally
dependent (i.e., there will exist a relation between ll. 1\ 11') if

Q~!_,V, w) = 0
a(x,y,z)

3.14.5 Example

Are x -I- y - z, x - y + z, x 2 + y2 + z2 - 2yz functionally dependent? I f so, find


a relation between thelll.
Solution
Let lI=x+y-z ..... (i)

1'=x-y-l-z ..... (ii)


11' = Xl -I- I -I- i2 - 2yz ..... (iii)
188 Engineering Mathematics - I

\
o(U, v, w) = \
-\ -\
Then o(x,y,z)
2x 2y-2z 2z-2y

= -\ -\ =0
2x 2y-2z 2y-2z

(since 2nd and 3 rd columns are identical)


:. u, v, ware functionally dependent
To find the relation between u, v, w we notice that, from (i) and (ii)
11 + v = 2x, U - v = 2(y-z)
From (iii)

u2 + v2 = 2w, the required relation


3.14.6 Example
If 11 = x + Y + z, v = x 2 + y2 -I- z2, W = x 3 + i
+ z3 - 3xyz then prove that
11, l~ ware not independent and find the relation between them.

Solution
.. ti c: . Id d . o(u, v, w) 0
CondItIOn or lunctlOna epen ence IS ( ) =
o x,y,z

Now o(U, v, w) 2x 2y 2z
o(x,y,z) - ? ? 2
3x- -3yz 3y- -3zx 3z -3xy

=6 X
2
x - yz
Mean Value Theorems and Functions of Several Variables 189

0 0
=6 x-y y-z z
)
(x- y)x+ y+z y - z(x + y + z) z- -xy

o o
= 6(x - y) (y - z) I z
x+ y+z x+ y+z z -xy

=0

11, V, IV are functionally dependent. We shall now find the relation between them

we have w = (x + y + z) (.x2 + I + z2 - xy - yz - zx) ..... (A)


Now u2 - V = (x + y + z)2 - (x 2 + I + z2)
= 2(xy + yz + zx)
We write (A) as

W=U
[v -U-V]
--
2

Thus

3.14.7 Example

x+ y xy
Test whether u = - - , v = ( )2 are functionally dependent and if so, find
x-y x-y
the relation between them.

Solution
Treating'lI, vas functions of x, y the condition for functional dependence is

a(u,v) =0
a(x,y)

x+y xy
11=--, V=-~-c-

x-y (x- y)2


190 Engineering Mathematics - I

{x-y)'I-{x+y)'1 -2y
We have ux = (X_y)2 = {X_y)2

(X- y).(I).{X+ y){l) 2x


II == --'---~---'--'---'--~'--'--'--

y (X_y)2 {x-yf

V = {x- y)2 y-2{x- y).l.xy


x (x _ y)4

_ {(X- y)y-2xy)
- (x- y)

xy - y2 - 2xy _ - y{y + x)
{x- y)3 - (X- YY
X(X+ y)
lilly Vy={
x-y )3

a{u, V) lIx ll"


a(x,y) = Vx Vy

=UV - v v
x y x y

'-2y 2 .x{x+ y} + y{X+ y!. 2x J =0


(X- y) (X- y) {X- y} (X- y)-

. U, v are functionally dependent


We shall now find the relation between them
x+y
We have u=--
x-y

x-y
x+ y u

By componendo and dividendo


x 1+u
-=- A
Y I-li
Mean Value Theorems and Functions of Several Variables 191

o X x 1+11
Y
Now
xy
v = (x - y)' = (;
V

1 r y l-lt
by (A)

'V=C~:~2 J
u2(4v+ 1) = I
The required relation of functional depedence between lI, v.

3.14.8 Example
Examine for functional dependence of
u = sin-I x + sin-1y

v=x~l-y2 +y,JI-x 2
and find the relation between them, if it exists.
Solution

We have u =-===
, ~1_x2

lilly

a{u, v) 1I ( It )'

a{x,y) Vx Vy
192 Engineering Mathematics - I

:. ll, V are functionally dependent we shall now determine the relation between u and v

Let A=sin-1x,B=sin-1y; =>x=sinA y=sinB

The given function can then be written as u = A +B


2 2
v = sin A)1-sin B +sin BJl-sin A = sin(A+B)= sinu

Hence the required relation of functional dependence between u and v is v = sinu

3.14.8(a) If u = eX sin y, v = eX cos y, show that u,v are functionally independent

au au
-
X X
a(u, v) ax ay e smy e cosy
Solution: The Jacobian = _ex * o.
a(x,y) av av eX cos y _ex sin y
ax ay
:. x, v are functionally independent.
. x-y x+z
3.14.9 VerifY whether u = - - , v = - - are functionally dependent, and if so find
x+z y+z
the relation.

Solution: Treating u,v as functions of x,y regarding z as an absolute constant, the


condition for functional dependent in

a(u, v) =0
a(x,y) ,
au
Now - =
y + z au 1 av 1 av =
- = - - - and -=~-,
x +z
ax (x+z/' ay x+z ax y+z ay (Y+Z)2
Mean Value Theorems and Functions of Several Variables 193

y+z
---
B(u,v) (x + Z)2 X+Z
= =0
B(x,y) 1 x+z
y+z (y + Z)2
u, v are functionally dependent.To lind the relation between u and v, we eliminate 'x'
between the given relation viz:
x-y x+z
u = ------------- (A), v = ---------------(B)
X+Z y+z
liZ+Y
from(A) ux+vy = x-yo =>(I-u)x=uz+y => x = - -
1-11
liZ +Y
+z
(y+Z)
Substituting for 'x' in 'B', v = 1-11 =
y+z I-l/' y+z I-u
Hence the required functional relation between 'u' and 'v' is v( I -u) = I.

Exercise - 3(1)

I. Verify whether the following are functionally dependent and if so, lind the relation
between them.
x- y x+ y
(a) II = - - , V = - - (Ans: u( I+v) = 2)
x+y x
x-y
( b) u = - - , v = tan -\ x + tan -\ y (Ans: v = tan
-\
II)
I-xy
(c)u=x-y,v= xy (Ans: 4v + 1I
2
=1)
x+y (X+y)2
2. Prove that the following functions are not independent.
(a) II = X + 2y + Z, v = x- 2y + 3z, W = 2xy - 2z + 4yz - 2Z2
(b) II = x+ y- z, v = x- y+ z, w= x + l + Z2 - 2yz
2

3
(c) u = x + y + z, v = xy + yz + zx, w = x + l + Z3 - 3xyz

3. If u = x + y "v = y + z w = y(x + y + z) , show that u,v are connected by a


Z x xz
functional relation.
194 Engineering Mathematics - I

x-y
4. Examine for functional dependence between u = - - , v = tan-I x - tan -I y.1f
l-xy
dependent, find the relation. ( Ans: u=tan v)

5. Prove that functions II =Y + z, v = x + 2z 2 , W = x - 4yz - 21 are functionally


dependent and find the relation between them. (Ans: V=W+2l?)

3.15.1 Maxima and Minima


The method of finding the maxima and minima of functions of two and three
independent variables is discussed in this topic.
The maximum or minimum values are also called Stationary or extreme or turning
values. For finding these values we use Taylor's theorem for functions of two
variables.
3.15.2 Taylor's theorem for a function of two variables:
We know that Taylor's theorem for a function f(x) of a single variable 'x' is

f(x+h) = f(x) + hj\x) + ~~ /I(X)+ ...... .


....( I)
Now, consider f(x,y), a function of two independent variables. Keeping 'y' constant
and following (I) we get
. a h a
j(x+h,y+k) = f(x,y+k)+h-f(x,y+k)+---J j (x,y+k)+ .........(2)
2 2
.

ax 2! ax-
Keeping 'x' constant and by applying (I), we get
a. ea
f(x+h,y+k) = f(x,y)+k-j(x'Y)+---2 f(x,y)+ ...... .
2

ay 2! ay
.... (3)
Substituting (3) in(2) we get

f(x+h,y+k) = [
. a. ea
j(x,y)+k a;f(x'Y)+2T ay2 f(x,y)+ ...... .
2
]

2
+ h- a. e a f(x,y)+ ...... .]
a [ f(x,y)+k-j(x,y)+---)
ax ay 2! ay
2
112 a [ aj. a
2
e
+ 2T al f(x,y)+k iY (x,y)+2T al f(x,y)+ ....... +..... .
]
Mean Value Theorems and Functions of Several Variables 195

Substituting (iii) in (ii) we get


2 2
f(x + hy + k) a k a f(x,y)+ ....]
=
[f(x,y)+k-f(x,y)---2
ay 2! ay

2
a [ f(x,y)+k-f(x'Y)+---2
+h- a a f(x,y)+ .....] e
ax ay 2! ay
2 2 2
h a [ f(x,y)+k-f(X'Y)+---2
+---2 a a f(x,y)+ .....] e
<2 ax ay < 2 ay

+ .....

[ aj 2 2
k a f + .....] + h-+hk--+
= f(x,y)+k-+--2
a f .....] [aj 2
ay < 2 ay ax axay

+[~ a
<2 ax
2
{ +. . .]
Thus f(x + h. y + k) = f(x, y) + (h! + k ~ )f
1
+~hax+kay
(a a)2 f+ ..... (iv)

Expression (iv) if known as Taylor's expansion for functions of two variables.


Writing x = a, y = b expansion about (a, b) is given by

a a)2
If(a+h,b+k)=f(a,b)+ ( hax +kay ira,b)

[a a
1 h-+k- ]2 i((jb)+ .....
+-
1! ax ay ,
Now if we call h = x - a, y - b = k

we get f(x,y) =f(a, b) + ((x-a)! +(y-b)~)i((j'b)


196 Engineering Mathematics - I

a a )2 fiab) + ..... .
1 ( (x-a)-+{y-b)-
+-
2! ax ay'
..... (v)
Writing a = 0 = b, we have

f(x, y) = j(0, 0) + (x ~+y; )./(0,0) + ...... .. ... (vi)

This is Maclaurin's series for two varialbes.

3.15.3 Example

Solution

We have f(x, y) =- ~2 + y2 => f{I,I) = e 2

2+ 2 2
Ix (x, y) = 2xe x Y => Ix (I, I) = 2e

X2 + y2 2 2 7
fyy (x, y) = e + 4Y eX +Y- => fyy (I, I) = 6e2
Hence putting these values in second form of Taylor's theorem, we get
1
e,2+y2 =e2[l +2(x-I)+2(y-I)]+ 2! [6(x-I)2+8(x-I)(y---I)+6(y---If+ .... ]

3.15.4 Example
Expand ~ siny in powers of x and y as far as terms of third degree
Solution
f(x, y) = ~siny => j(0, 0) = 0
Ix(x, y) = ~siny ,=>Ix(O, 0) = 0
J;,(x, y) = ~co,';y =>J;,(O, 0) = 1
Mean Value Theorems and Functions of Several Variables 197

lxlx. y) = ~siny -=>lxx(O, 0) = 0


lx/x. y) = ~C()sy -=>fxy(O, 0) = I
/y/x. y) = ~siny -=>/y/O, 0) "= 0
lxxi'" y) = ~siny -=>j~x..l0' 0) = 0
lxx/x. y) = ~sillY -=>j~-X/O, 0) = 1
j~J'Y(x, y) = ~siny -=>fxy/O, 0) = 0
fy;./x. y) = ~C()sy -=>1;'JY(x, y) = -1
Then by Taylor's theorem we have
f(x. y) = j{0, 0) + [xlx(O, 0) + y1;. (0, 0)]

I
+ 21 2 .2
[xfxx (0,0) + 2xylxy (0,0) + yjXy(O, 0) ] + ......

J J J
x- y- x-
= 0 + x(O) + y( 1) + - (0) + xy( 1) + - (0) + - (0) + .....
226

x y
=y+xy+---+ .....
2
i
2 6

3.15.5 Example
j(x. y) = x 2y + 3y - 2 in powers of (x-I) and (y + 2) by Taylor's theorem
Solution
j(x, y) = f(a, b) + [(x-- a)lx (a, b) + (y - b)fy (a, b)]

I
+ 21 (x - a)2 fxx(a, b) + 2(x -a) (y - b)j~ (ab)

+ (y - b)2.fyy (ab) + ..... .... (i)


Here a = 1, b = -2
f(x. y) = x 2 + 3y - 2 -=> j{ 1, -2) = -10
fJx.
x y) 2xy -=> J,x (I, -2) = -4
/y(x. y) =x2 + 3 -=> 1;,(1, -2) = 1+ 3 = 4
lxix. y) = 2y -=> lxx(\' -2) = 2(-2) =-4
fxy(x. y) = 2y -=> fxy(\' -2) = 2( 1) =2
j~(x. y) = 0 -=> j~(1, -2) = 0
198 Engineering Mathematics - I

11)=0
Jrxx.\ (x'J/ --- Jrxxx (I ' -2)=0
---,I'

' /x Y')/
j xxy\" = 2 => j'xx)'(I , -2) = 2

f' /x y,)/ = 0
, AYY\"
=> j'XJ'}'(I , -2) = 0
' /x Y')/ = () => j')')'(I , -2) = 0
j .Y.ly\"
Substituting a = -1, b = 2 and above values in (i) we have
x 2y + 3y - 2 = - 10 -4(x -I) + 4(y + 2) - 2(x-l;2
+ 2(x- -1) (y + 2) + (x - 1)2 Cv + 2)
3.15.6 Example

Evaluate log[(l.03) I] + (0.98) '4 -1] ,approximately.

Solution

Then f(x + h. y + k) = IOg( (x + h)~ + (y + k)~ - I)

Assuming x = 1, y = 1, II = 0.03, k = - 0 .02

We get F(x+h, y + k) = IOg[(l.03)~ +(0.98)~ -IJ


oF OF) .
=
ox oy [
F(x, y) + h-+ k - approximately ..... (A)

I 2
3 1 --3/4
of -x of -y
3 4
1 I I
ox oy - -

x 3 + y4 _I x 3 + y4 -1

= F(x, y) + h (OF)
ox + k [OF) .
oy approximately
Mean Value Theorems and Functions of Several Variables 199

~ IOg[ (1.03)~ + (0.98)~ -I]

= F(I, I) + 0.03 (aF) + (_O.02)[aF)


ax (1,1) ay (1,1)

~O.03 ll+I-IJ
r j to.oJ ~ 1~O.005
ll+I-IJ
approximately

(-: F(l,I) = 0)

Exercise - 3(J)

I. Obtain the expansionllsing Taylor's theorem of the following:

I. xy2 + cos(xy) at (I'~)

2. xY at (I, I) lIpto second term

I ADS: xY = 1 +(x -I ) +(x -I) x (y -I ) + -21 (x_I)2 I

3. sinx. siny in powers of (x -~) and (y -~)


( ADS. ~+~(x_~)+~(V_~)_~(X_~)2
2242' 444

4. eax sinby at (0, 0)


( ADS: hy + xyah + '" )
200 Engineering Mathematics - I

3
. ( x+ y)
5. Prove that sm(x +y) =x +Y- + .....
<3
2
6. Show that eY 10g(1 + x) = x + xy - ~ approximately
2

7. Expandf(x, y) = sinxy in powers of (x -1) and (y - ~) upto second degree term.


2
[ ADs I - -1t (x - 1)2 - -1t(x -1 \ y - -1t) - -1 ( y - -1t)2 + ..... I
8 2 2 2 2

3.16.7 Maxima and Minima


The function F(x, y) is maximum at (x, y) if for all small positive or negative
values of hand k; we have
f(x + h, y + k) - F(x, y) < 0 .... (i)
Similarly, F(x, y) is minimum at (x, y) iffor all small positive or negative values
of hand k; we have
F(x + h, y + k) - F(x, y) > 0
Thus ifF(x, y) has a minimum or maximum value then [F(x + h, Y + k) -F(x, y)]
keeps the same sign for all small positive or negative values of hand k.
Now using Taylor's expansion, we get

F(x + h, y + k) - F(x, y) = haF aF)


-+k -
( ox
ay

2 2 2
-f -2-(h2 a F +2hk a F +k2 a F)
+ 21 ax 2 axay 0'2 + ..... ..... (iii)

Assuming h, kto be sufficiently small; the sign of the expression on the left can
aF aF
be made to depend on that of h ax + k 0'

Hence the necessary condition for f(x, y) to be a maximum or minimum is that


Mean Value Theorems and Functions of Several Variables 201

of of
- = 0 and - = 0
ox oy
of of
I.e., p = 0 and q = 0 where p = ox' q = oy

2 2 2
then (iii) ~ f(x + h y -I- k) - j(x y) = -
I[ ,0 F 0 F , 0 F]
11- --) + 2hk-- + K- --) + .....
. '2! or ox8y oy-
I
= -
2!
r rh 2 + 2shk + tk2 ]

~l
02F 2
s=-- z= 0
ox8y' ~y-

=r- [ h-, s t ,]
+ 2hk-+-K-
2! r r

=-
r
2!
s -
r )
[( h+-k )+ (2
- s )
,
k-
-,-k- +1-)
r- r-
II
=~[(h+~k)2
2! r
+(rt-.~.2)k:l
r-

It has the same sign for all h, k if and only if (rt - s2) is positive.
When (i) (rt - s2) positive,j(x, y) is maximum for negative 'r' and minimum
for positive 'r'.
(ii) (rt - s2) is negative, we have neither a maximum nor minimum and
such stationary points are called saddle points.
Solvingp = 0, q = 0, we get the extreme points
Extreme value :
.I(a, b) is said to be an extreme value ofJ(x, y) if it is either maximum or minimum.
3.16.8 Example
Find the maximum and minimum values of x 3 + 3xl- 31 + 4.
Solution
We have J(x, y) = x 3 + 3xl- 3x2 - 31 + 4
fx = 3x2 + 31- 6x, fy = 6xy - 6y, f)y = 6x- 6, 1;)' = 6y
202 Engineering Mathematics - I

We now solve
Ix = o,f;, = 0 simultaneously
I.C., x2+y~2x=0 ..... (i)

and xy ~Y = 0 ..... (ii)


xy ~ y = 0 => y = 0 or x = I
From x 2 + y ~ 2x = 0 we get
when y = 0, .~ ~ 2x = 0
x = 2 or x = 0

when x= l,y~1 = 0 i.e.,y= I or~1


Thus the points are (0, 0), (2, 0), (I, I), (I, ~ I)
(a) For x = 0 , y = 0 , r =j'xx = ~6 ' s = 0 ') t = ~f)
rt ~ s2 = 36 ~ 0 > 0

j(x, y) is stationary at x = 0, y = 0
But r=/xx = ~6 <0
fix, y) is maximum at x = O,y = 0 and the maximum valuej{O, 0)
=4
(b) Por x = 2 , .y = 0, r =/xx = 6 ' s =/.xy = 0 , t =j'n' = 6
Again rt ~s2 > 0

j(x, y) is stationary at x = 2, Y = 0
But r =/xx = 6 > 0
j{x, y) is minimum at x = 2, y = 0 and minimum value j(2, 0) = 0
(c) At x = I, Y = I or at x = I, Y = ~ I
rt ~ s2 > 0 and we can reject these points as they are not stationary points.

3.16.9 Example
Discuss the maximum and minimum of
f('(,y) = x2 ~ Y + 6x -- 12
P = 2x + 6, q = ~2y, r = 2, s = 0, t = ~2
p = 0 gives x = ~3, q = 0 ~ y =0
Mean Value Theorems and Functions of Several Variables 203

Stationary point is (-3, 0); and here r = 2 >0; and


rl-s~ == 4-0 == 4 > 0
) ) - 1,...
x- -.V + 6x - .!. is minimum at (-3,0) thc minimum valuc is
j'(-3,O)==-21.

Exercise -3(k)

I. Find the maximum and minimum valucs of:


I. x' + 3x/ - 3/ + 4 jAns : (0,0) max. its valuc 4,(2,0) min. its value 'W]
....
')
x'+3xy ) - 3.c)-3Y) +
7 [Ans :max . value 7 at (0,0) min.value '3' at (2,0))
3. x~ + / +6x+ 12 [Ans: mill. value '3' at (-3,O)j
3
4. x + x/ + 21x -12x2 - 2./ l Ans : max.value 10 at ( 1,0) min. value -98 at (7,0) 1
5. X4 +x/ + / jAns: minimum value '0' at (0,0)1
6. X4 + / _ x2 _ y2 +1

lAns: max.value I at (0,0) min.value Y2 at t(Hlr pOints( ~ ~)]


7. X
) 0 3
Y -x Y -x Y
2 ~ 3
jAns: max.valuc --at -,- ] 1 (1 1)
324 2 2
) ) ) 3
8. Discuss the stationary values of II == x-y-3x- - 2y -4y + [Ans: u is a
maximum at(O,-I). The maximumvalue is 5, u is neither maximulll nor minimum
at(4,3)
9. Find the maximulll and minimum values ofthc following functions.
(a) x'/(12-3x-4y) jAns:maximulllvalucat(2,1)]
(b) Xl + / - 3axy( a > 0) [Ans: minimum value is _(/' at (a,a)1
1

(c) xy(a-x-y) (a>O) [Ans:maximum value is ~ at (a/3,a/3)]


27
10. Determine the values of x,y for which the following functions are maximum or
mllllmum ..
(a) x ' y 2(1-x- y) l Ans: maximum for x = 112 , Y = I/3 j
(b) (x 2 + y2)" -2a\x~ - /) [Ans: minimum for x == a,y = OJ
(c) sinx siny sin(x+ y) [Ans : maximum for x = y = 7[ /3]
(d)a[sinx+siny+ sin(x+y)] [Ans: maximum for x = y =7[ /31
(e) x/(3x+6y-2) [Ans: minimum at x = y = 1/6J
204 Engineering Mathematics - I

Lagrange's method of undetermined multipliers:

3.17.0Theorem: Find the maximum and minimum values of f(x"X 2'X3' ......xlI )
where x" x2 ' x 3 ' XII are connected by the following m equutions.
, (x, ,x2 ' x,J= 0
2(X"X2' ....... .xJ = 0

m(X, ,X2 ,. .... .xll ) = O.


Proof: Let u = f(xl'x2, .........x,J .Since n variables are connected by the m given
relations, only n-m of the variables are independent.

The maximum and minimum values ofu can be found by the method of Lagrange's
method of undetermined Multipliers .For u to be max or min of u , du = 0
au au au
i.e. -dx, +-dx2 + ........ +-dxn =0
ax, aX2 aXil

also from, we have


..... ( 1)
multiplying the above equations by 1, ~, ~, ....Am respectively and adding, we get

au a, a2 am ) ( au a, al am )
( -+~-+~-+ ........ +AI/I- dx, + -+~-+~-+ ........ +AI/I- dx2
~ ~ ~ ~ ~ ~ ~ ~
+.................................... .
au a, ~ a2 1 am) _
+ ( -+~-+"'2-+ ........ +AI/I-- dxn -0. . .... (2)
ax" aXil aXil Ox ll

The values ~,~, .... Am are at our choice .We can, therefore choose them so as to
satisty 'm 'linear equations .
.. .. (3)

I(is immaterial which n - m of the n variables are regarded as independent.

Let these bexl/I+"x"H2' .......x".Then since the n - m quantities dxm+"dxm+2, .......dx" are
all independent, their coefficient must be separately zero. This gives additional equations.
Mean Value Theorems and Functions of Several Variables 205

..... (4)

au + A, atA + ~ a2 + ........ + Alii am = 0


axil aXil - axil aXil
Now we have m+n equations in all and given relations , = 0, 2 = 0, .......III = 0
The equations (3) and n-m equations (4) are sufficient to determine the m + n
quantities

A,,~, .... AII/ and x"x2 ' x lI


for which maximum and minimum values of u
are possible .The m multipliers are called Lagrange's Multipliers.
3.17.0 (a)Theorem: If u=<p(x,y, z), j; (x,y, z)=O., .t;(x,y, z) =0 discuss the
maximum and minimum values ofu
Proof: For a maximum or minimum, we have du =0
af" af" af"
I.e. -'-' d" + -'-' dy + _0_' dz = 0
ax ay oz

Also from j; = 0 andJ; = 0


FaJ; dx+ aJ; dy+ a.t; dz=O '" .. ( 1)
ax iY az
aJ; dx+ aJ; dy+ aj; dz = 0
ax iY az
Multiplying these equations by 1, A" ~ respectively and adding

a + A, aJ; + ~ aJ;) dx + ( a + A, a.t; + ~ aj ;) dy + l( a + A, a.t; + ~ a.t;) dz = 0


( ax ax ax liY iy iy fu fu fu

... ~ ... (2)

The lagrange's multipliers A" ~ are choosen such that


206 Engineering Mathematics - I

~ 01;
o +:1/11 ()I; + /l, = 0 .... (4)
oy oy - ~v
then equation (2) becomes
o +:1/11 ~/~ + /L1) ~/; =0 . (5)
()z oz - oz
These equations (3) ,(4), (5) and given 1;<'t,y,z)=0, .I;(x,y,z)=O are
sufficient to find ~, ~ ,x,y,z which give maximum and minimum values of u ..

3.17.0(b) Theorem: If u = (x,y, z) and f(x,y,z) = 0 find the maximum or


minimum values ofu.

Proof: For a maximum or a minimum II ,we have du = 0


o o o
I.e. ~.dx+~.dy+~.dz =0 .... ( I)
ox oy oz
and
from given f(x,y,::) = o.
of ol ~l
- .dx + -.Jy + -.Jz = 0 .... (2)
ox oy OZ
Multiplying (1) by 1 and (2) by A and adding

o + A of)dx+( o + A ol)dY+(o'+ A ol)dZ = 0 .... (3)


( ox ox oY oy OZ oz
Lagrange's multiplier A is choosen such that
o +A ~l =0
ox ox
.... (4)

and o +A ~l =0 .... (5)


Oy oy
The equation (3) gives
o +A ~l =0 .... (6)
oz oz
The equations (4),(5),(6) and 1 (x,y, z) = 0 enable us to tind the values of A "x,y,z for
which u is a maximum or a minimum.
Mean Value Theorems and Functions of Several Vanables 207

Exercise - 3(1)

3.17.1 Example
Find the point upon the plane ax + by + cz = p at which the function
$ = x 2 +1+ z2 has a minimum value and find this minimum $.
Solution
p-ax-hy
ax+by+:::=p ~:::= ~---~ ..... (i)
e
)

Again $= x~
') . .2
+ .V"" + z-
J
= r
') J
+ ), +
( 17 - axc- bV)-
Then for min./max., we have

ox = 2.x -
0$ 2a (p _ ax - by) = 0
e2
..... (ii)

~$ = 2y- 2~ (p _ ax - hy) = 0 ..... (iii)


uy e-

x p-ax-hy
From (8) ~-= )
a c-

y p-lLr-hy
~-= .
h c2
x bx y
~-=-'--~ y=-
a . (lb
Now, substituting this value ofy in (ii) we obtain
2
X) =0
x- 2a ( P - l I Xb- -
e 1I

c 2x - (lP + a2x + b2x = 0


x( a 2 + b2 + ( 2) = (lP
ap bx bp
x = )2 ) and Y = - =) ) )
a +h- +c a a- +b- +c-

Then,

and
208 Engineering Mathematics - I

Hence 3 a min, when

-x =-y =----:---=-::----c-
2
p
2 2
a b a +b +c

=~[p_
2
Also Z = p-ax-by a p
c c a2 + b2 + c2

I.e., Z P
=
Hence 3 a min. at
xy Z P
-==-
-
abc 2
a +b 2 +c 2
a 2 p2 + b 2 + p2 +C2p2
(a 2 + b2 + c2 r
4> . =.xl + y2 + z2 = ----'---,---...:...--.---:'--
mm

3.17.2 Example
Divide 24 into the three parts such that the continued product of the first, square
of the second and cube of third is maximum.
Solution
Let 24 be divided into parts x, y, z then x + y + z = 24 ..... (i)
Given 4>(x, y, z) = x 2 y2z
then 4>(x, y) =.xl;? (24 .- x - y) from (i)
or 4>(x, y) = 24x3y2 - x4y2 - x 3y3
Differentiating, we get

p = ~! = n.xly2 - 4~y2 - 3x2y3

a 4>
r =-?
2
= 144xy2 - 12x2y2 - 6xy3
ax-
Mean Value Theorems and Functions of Several Variables 209
- - --- - - - - - - - - - - - - - - - - - - - - - - - - - -

and S =-02~
- = - (o~
oxoy oX ~Y
- a 1 = 144x2y-? - 8x3y - 9x 2y-?
.

For a maximum value of ~(x, y)we have

a~ = 0 => 72x 2y"2 _ 4x 3i - 3x2y3 = 0


ox
=> x 2i (72 - 4x - 3y) = 0
=> 72 - 4x - 3y = 0, x = 0, y = 0 ..... (ii)

and o~ = 0 => 48x3.v -


~
2x4y - 3x3 v2
.
= 0

=> x 3y (48 - 2x - 3y) = 0


=> 48 - 2x - 3y = 0, x = 0, y = 0 ..... (iii)
Solving (ii) and (iii) we get x = 12 and y = 8 etc
At (12, 8), we have
r= 144, 12(8)"2- 12(12)2(8)-6(12)(8)3
= 12(8)2 (144 - 144 - 48) = 12(8)2 - (-48)
r=48(12)3_2(12)4_6(12)3.8
= (12)3 (48 - 24 - 48)
s = 144( 12)2 . 8 - 8( 12)3 . 8 - 9( 12)2 (8)2
== (12i. 8 (144 -96 -72) == (12)2.8(-24)
:. rt - s2 =12. (8)2 (-48). (12)3 (-24) -l (12)"2.8. (-24)]2
== (12)4.8 2 .24 (48 - 24)

== (12)4.8 2 .242> 0;
Since rl - s2 > 0 and r < 0, therefore ~ (x, y) is maximum at (12, 8).
Putting x == 12, and y == 8 in (i), we get z == 4
The values of x, y, z are 12, 8, 4 respectively. This is the division of 24 for
maximum ~(x, y, z).

3.17.3 Example
A rectangular box, open at the top, is to have a volume of 32 c.c. Find the
dimensions of the box requiring least material for its construction.
210 Engineering Mathematics - I

Solution
Let I, band h be the length, breadth and height of the box respectively. Then,
wc have
v = Ihh = 32 ; surface = 2(1 + h) h + Ih = S (say)
32
S = 2(1 I- b)h + Ih and b = - .....(i)
III

32) h + I (32)
s = 2 ( 1+- - = 2111 + -64 +-
32
III liz I h

Now ~~ = 2/-(:;)

For maximum and minimum ofS, we get

as =0= as
al ah

as = 0 ~ 2h _ 642 = 0 h = 32
0/ 1 ' 12

and as = 0 ~ 2/- 32 = 0
ah h2

and 82~ = +2
8h-
so 'S' is minimum for 1= 4,b = 4,h = 2.
Mean Value Theorems and Functions of Several Variables 211

I 1 1
3.17.4 Example: If u=a'x"+h'/+c3z1 where -+-+-=1 show that the
x y z
a+h+c a+h+c a+h+c
stationary point of u is given hy x =- - - , y = ,,"" - - - -
a h c
Solution: For a stationary value of 'u', du =0
1
I.e. a ' 2xdx+h'-2ydy+c 2:::dz =0 ..... ( I )

Also we have the given relation ~~ + l + l = 1


x y :::
1 1 1
-) dx+-) dy+-) dz = 0
x~ y .. ~

..... (2)
Equating the coefficients of dx , dy ,dz from (1)+ A(2) separately to zero
I A I A 1 A
We get a x + -) = 0, h x + -) = 0, c x + -, = 0 .
x- y --
1
:. a x ' =h ' /=c ' z'-=(-A)
or ax = by = cz = (-At] = k(say)
x = k I a; y = k I h; z = k I c
..... (3)
. j'or x, y, Z
SU hstltutll1g .
In -
1 + -1 + ~~1 = I
X Y z
abc
-+-+-= 1
k k k
:. k = a+h+c
..... (4)

Substituting in (3) we get


a+b+c a+b+c a+b+c
X= ,y= , z = - - - for which u is stationary.
II b c

Exercise 3(M)

I. Find the maximum and minimum distances from the ongll1 to the curve
5x" + 6xy + 5 y2 - 8 = O. [Ans : max. 4, min .16,6,3]
2. Fin~the dimensions of the rectangular box, with out a top of maximum capacity
whose surface is 108 Sq. inches. rAns:6",6",3" J
212 Engineering Mathematics - I

3. If f = u 1 +j'1 +W1 where 11 + v + w = 1, show that f IS stationary when


1
U=V=W=~
3
4. Use Lagrange's method of multiplies to determine the minimum distance from the
origin to the plane x + 2y + 3z = 14. [Ans: Jl4"1
5. Find the maximum value of / = xyz when xy + yz + zx = k. [Ans: (k / 3)3/?]
6. If the distance from the origin of any point p(x,y, z) on the plane
3x+2y+z-12 = is P =~X2 + y2 +Z1 ,find the minimum value of p(use the
Lagrange's method).
7. Find the maximum volume of a parallelepiped inscribed III a sphere
2
x-, + y-) +z 2 = r.
'
[Ans:
8r ]

3J3
8. Find the volume of the largest rectangular parallelepiped that can be inscribed in
, , ) h
d x- Y z- 1 8a c]
t I1e e II IpSo! - 2 +-) +-) = . [Ans: 3 (:;3
a b- c- -V j

9. Given x+y+z=k,findthemaximumvalueof XII,yll,ZY

aa./3II.rY.k<at/l+Yl 1
[Ans:
[ (a+/3+rt l /l+ Y J

10. If r2 = x
2
+ l + Z2 and x+y+z =30 find the values of x,y,z for which r IS a
mll111nUm [Ans: x = y = z = 10]
11. Find the dimensions of a rectangular box without a top of maximum capacity
whose surface area is 108 square inches. [Ans: 6,6,3 inches]
12. Find the dimensions of a rectangular box with open top, so that the total surface
area S of the box is a minimum, given that the volume 'V' of the box is constant.
fAns: x = y = 2z = (2\,)1/3]
13. Show that the rectangular solid of maximum volume of that can be inscribed in a
sphere is a cube
14. I f the total surface area of a closed rectangular box is 108 sq.cm. Find the
dimensions of the box having maximum capacity [Ans: .Ji8,.Ji8,.Ji8]
15. Perimeter of a triangle is constant, prove that the area of this triangle is maximum
when the triangle is equilateral.
16. If u=(x,y)where/(x,y)=O, find the maximum and minimul.1 values of 11
using Lagrange's multipliers Method
4
Curvature and Curve Tracing

4.1.1 Curvature
The curvature of a curve (bending of a curve) varies from point to point on the
curve.
Let P be a point on the curve and Q be c.l point nearer to P, are OQ = 0 s
Let tangents at P and Q make angles, \.11, and \.11 + 0\.11 with the X- axis.

LT R T' = d\jJ is the angle through which the tangent at P turns as a point moves
along the curve form P to Q through a distance os along the curve and hence 0\.11
depends on the arc length os.
0\.11 is defined as the total bending or total curvature of the arc P Q.
0\.11
8s is defined as the average curvature of the arc P Q.

Now os ~ 0 as Q ~ P

Lt ~\.11
&s~O uS
= dd\.l1 is defined as curvature of the curve at the point P and is denoted
S

by K.
d\.l1
Thus K=-
ds
214 Engineering Mathematics - I

x' 0 T

Fig. 4.1

Radius of Curvature
The reciprocal of the curvature of a curve at any point' P' is called the radius
(?f curvature at that point and is denoted by 'p'.
ds
Thus p = -d is called the radius of curvature of the curve at the point P.
\1' .

4.1.2 Theorem
If 'r' is the radius of a circle then the radius of curvature of the circle is same as
its radius.
Proof:
Let 'c' be the centre and 'r' be the radius ofa circle. P be any point on it and Q
be a neighbouring point, arc PQ = ds.
The tangents at P and Q make angles \If and \If + 8~, respectively.
From the figure

LT R T' = 8 \If

LP R Q' = 180 - 8\1f


LP C Q' = 8\jf, from the sector PCQ.

8s
~ = r and as Q ~ P, 8s ~ 0
u ~,
Curvature and Curve Tracing 215

Lt 8 \1' = ~
8s
01->0 r

1
I.e.,
P r
p=r

x' o x

y'

Fig. 4.2

4.1.3 Cartesian form of the Radius of Curvature


Let y = j{x) be the Cartesian ~quation of a curve and tan (\lJ) be the slope of the
tangent to the curve at a point P.
dy
tan \lJ = dx

Differentiating w.r to 'x'

We know that

ds
dx
216 Engineering Mathematics - I

... ,.. ,

1+(ddxy )2

[I+(~~)' I+(~)' 1
p=

3/
. _I[ + y( ) '2 dy d 2Y
I.e.,p- whereYI = -d 'Y2=-2
y~ X dx
is the radius of curvature of the curve (Cartesian form).

4.1.4 Parametric form of Radius of Curvature


Let x = fil), Y = get) be the Parametric form of the equation of a curve.

-d'( = f\t() -dy = g ,(t )


dl ' dl
i.e., x' = /'(t), y' = gl(t),

dy
dy
-=-
dl
dx dx
dt
'., -,
dy y'
- = -I .
dx X
Curvature and Curve Tracing 217

Differentiating w.r. to 'x' on both sides

,{2y " .- yx
X 'Y '"
--=
{X')3
3/

dy d'y
2 values in p =
Substituting -d ' - I
[l+(~)T
d2
X ex . Y
dx 2

p=
[l+(Hr
x'y" - y'x"
-
'----'..,..

{X')3

[(XI)2 + {y')2 y~
P= x'y" - y'x"

I dx I dy
where x =- y=-
dt ' dt

is the parametric form of the radius of curvature,

4.1.5 Polar form of Radius of Curvature


Let P (r, 8) be a point on the curve r =f(O). If '0' is the pole, 0 X is the initial
line.

LXOP =0
The tangent at P makes an angle \I' with the initial line.
$ is the angle between the radius vector OP and tangent PT.
218 Engineering Mathematics - I

From figure \If=e+cj>

d\lf de d<l>
-=-+-
ds ds ds

1 de d<l> de
-=- +-.-
p ds de ds

!
p ds
[1
= de + d<l>]
de
..... (1 )

Fig. 4.3

We know
ds
r2+ (dr)2
- ..... (2)
de de

de
and tan = r-
<I>
dr
r
i.e., tan cj>-=-
dr
de

Differentiating w.r. to 'e' on both the sides


Curvature and Curve Tracing 219

2 d~
(1 + tan <1 de =

d~
de

d<l>
de

The value of
I+ ~~
de
I + r' + t~
r2 + -
r-~~
(dr)2
r

de

r2 +2(_dr)2 _rtl_2_r 2
d,h de de
1+ = --'-----'------:---
+(-:r ..... (3)
-'I'

de r2
ds , d~
Substituting the value of de' I+ de from (2), (3) in (I)
220 Engineering Mathematics - I

is the polar form of radius of curvature.

4.1.6 Pedal form of Radius of Curvature


Let P be any point on the curve. the tangent P T and P makes an angle \jJ with
the initial line OA, LP 0 X = e (see the figure in 4.1.5)
\jJ=e+~ ..... ( I)
Differentiating w.r. to s
d\jJ de d~
-=-+-
d\' ds ds

I I de d~ dr
- = -.r- + - -
p r ds dr cis

I I d~
- = - (sin~) + - cos~,
p r dr

We have sin~=r-, -
de I = -
d~] ,cos~=-
I [ sm~+rcos~-
. dr
~ p r ~ ~

I d(r sin ~)
p = r sin ~
p r dr

_ = ~dp :. p =
dr
r-
p r dr dp

where p is the perpendicular distance from pole to the tangent.

4.1.7 Examples

2
Prove that the radius of curvature of the curve y = a cosh (x/a) is L
a

Solution
y = a cosh (x/a)
Curvature and Curve Tracing 221

dy I
- = a sinh(x/a)-
dx a

d 2y I
-2 = cosh (x/a)-
dx a

dy d 2 y ( dy)2l~2
[ I + ~j;
Substituting the values of - , - - ? in p = d2
dx dx- Y
dx 2

?
P =1[ + sinh - x / a
]3 2
cosh{x / a ).~
p = aeos h2(x/a)

p ~ a( ~:), as y ~ cosh(x/a)

p~ ( : )
4.1.8 Example
Prove that for the rectangular hyperbola xy = c2 the radius of curvature at any point
r3
is given by p = -2 where 'r' is the distance of the point from the origin.
2c

Solution
Differentiating xy = e 2 w.r. to 'x'

dy
x - +y=O
dx
222 Engineering Mathematics - I

.. dy y .
d 1+
2 [ ( ~b:dy)2l~~
Substltutmg for -d ,--? m P = /2
X dx- ( Y
dx 2

p=

as ,.2 = x 2 + rand xy = c 2

4.1.9 Example
Prove that the square of the radius of curvature at any point on the parabola
y = 4 ax varies as the cube of the focal distance of the point.
Solution
From y=4ax ..... (1 )

dy = ~
dx f~
d 2y I -3/
- = - lax /2
dx 2 2 -va.
Curvature and Curve Tracing 223

Hence p=

x' o x

y'

Fig. 4.4

p=
-1 r -~~
-~"a x
2

p = 1a(x+a)~ ..... (2)

The focal distance of the point p (x, y)

sp= ~(x-aY +(y-oy


= ~X2 +a 2 -2ax+ y2

= ~ x 2 + a 2 - 2ax + 4ax using(l)

sp= ~(x+aY
SP = focal distance = (x + a) ..... (3)
224 Engineering Mathematics - I

From (2) and (3)

-2
P = ~ [Focal distance]3/2

4
p2 = - [Focal distance]3
a
p2 00 = cube of the focal distance.
4.1.10 Example
Show that the radius of curvature at an end of the major axis of the ellipse
Xl y2
a2 +---;;: = 1 is equal to the semi latu~-rectum of the ellipse.

Solution

i ..e., ..... (1 )
Differentiating (1) w.r. to 'x'
dy
dx

d y 2 b 2fy-xdyl
dx
dx 2 = - --;; y2

/
Curvature and Curve Tracing 225

y I

x' A'(-a,O) o (0,0) A(a,O)

y'

Fig. 4.5
d 2y
--
2 ----
dx a
b2
2
i
( using ax'2 + b2 1
y' = I

p=
[1+(:)']"
d 2y
dx 2

Hence p=
[ l+---~
h"
4
a i
r; (a 4i+b 4x2)
..... (2)
_b 4 a 4b 4
2
a i
The coordinates of the ends of the major axis are

A (a, 0) and A' (-a, 0)


Taking the end A (a, 0) we have from (2)

b2
pl(a,o) = -;; = Semi latus-rectum of the ellipse.
226 Engineering Mathematics - I

4.1.11 Example
J J' 5<
Prove that p at any point of the curve x;; 3 + y/3 =a 3 is three times the
length of the perpendicular from origin to the tangent at that point.
Solution
The parametric equations of the curve are
x = acos 3e, y = asin3e
dy
dy de 3a sin 2 e cose dy
-------,,---- - - = - tan e
dx = dx - 3a cos 2 e sin e dx
de

d 2y de
and --
2
= -sec2e-
dx dx
= - sec2e / (-3acos 2 e sin e)

3a cos 4 e sin e

Hence gives

p=
(I + tan 2e f2
I
4
3a cos e sin e
p = 3a cos e sin e ..... ( I)
The perpendicular distance from the origin to the tangent
dy
y-x- 2
a sin 2e - cos e(- tan e)
p = ---;===dx==
I+(:J 2
JI + tan e

P = a sin e cos e ..... (2)


From (1)&(2) p=3p
Curvature and Curve Tracing 227

4.1.12 Example
Find p at any point for the cycloid
x = a (8 - sin 8)
y = a(1- cos 8) and also find p at 8 = 90
Solution
dy
dy _ de _ a(sin8) dy 8
- = cot-
dx - dx - a(l-cos8)' dx 2
de
2
d y 2 8 1 d8
and - 2
= cosec - - -
dx 2 2 dx
1 28
d 2y -2cosec 2 -1
2
dx = a(1-cos8) ,

.. p=
[1+(:)'r
d 2y
p=
( at
1+cot
-1
2
2

4a sin4 ~
2
dx
2
8
p = - 4a sin
2
at 8 = 90, P = - 4a sin 45 i.e., p = - 212 a
4.1.13 Example
Find p at (r, 8) for the curve r = a (1 + cos 8) and also find at 8 = ~.
Solution
r = a (1 + cos 8)
dr
de = r) = a(- sin 8)

d 2r
- 2 =r =acos8
de 2
228 Engineering Mathematics - I

[a 2(1 + cos e)2 + 2a 2 sin 2 e + a 2cos e(1 + cos e)]

[2a 2 (1 +cose)7i]
2
[2a .2cos
2
~~]7i
= .iacos e/
3a .2cos ~~
2 2 2 2 2 3 /2
a + 2a + 3a cose

at e=~
4 2 r;:;
P = -acos 45 = -,,2a
3 3

4.1.14 Example
Find p for the curve r'" = if! cos me
Solution
r'" = if! cos me
mlog r = m log a + log (cos m e)
1 dr 1
m -- = (- m sin m e)
. r de cos me
r) = -r tan me ..... ( I)

ar
r2 = -r m sec2 me - tan m e. de

r2 = - rm sec2 me + r tan 2 me ..... (2)


From (1)

3
= {r 2+ r2 tan 2me)2
r2 + 2r2 tan 2me + mr2 sec 2me - r2 tan 2me
Curvature and Curve Tracing 229

r
{m + J)cos me
r am
m
{m+J( m
a
For the curve ,-In = d" cos m e

P= (m + J)r m- 1

4.1.15 Example
Find the radius of curvature of the curve p2 = ar
Solution
p2 = ar ..... ( 1)
Differentiating w.r. to 'r'

2p dp = a
dr
dp a
dr 2p

p = r dr = r. 2p
dp a
From(l)
2r
p=-j;;;
a

4.1.16 Example
1 J J r2
Find the radius ofcurvature at the point (p, r) on the ellipse -2 = -2 + -b2 - ~b2
P a a
230 Engineering Mathematics - I

Solution

Differentiating

2 dp -2r
p3 dr a 2b 2
2
dr a 2b
=> r-=--
dp p3

4.1.17 Example
Find the radius of curvature p of the curve ,2 cos28 = if
Solution
,2 = cos28 = a2 ..... ( I)
Taking logarithms on both sides we get
2 log r + log cos 2 8 = 2 log a
Differentiating w.r.to '8'
1 dr
2 -;: d8 - 2 tan 28 = 0

d8
r-=cot28
dr
d8
tan <\> = r dr = cot 28

tan <\> = tan (~ - 28 )


1t
<\>=2- 20

We have P = rsin<\> => p = r sin (~-28)


P = r cos 28
a2
Substituting cos28 = -2 from (1)
r
Curvature and Curve Tracing 231

dp = ~
dr ,.2

dp ,.
p=r-=---)
dr -(r

4.1.18 Curvature at Origin


(i) When y can be expanded in powers ofx by some algebraical or trigonometrical
method from the equation of the curve as

y = px + -
qxl
+.... then p = -
dyl d2y
q = --)
2! dx x=O dx- x=o
y=o y=o

3
.. (I + p).i
p (at ongll1) = --'----~
q
(ii) Newtonian Method: If a curve passes through (0, 0) and the tangent at (0,0)

IS X - axis, then we have x = 0, y = and d,Y = at origin.


{X
The expansion of y by Maclaurin's theorem reduces to
Xl
Y = (yo) + (YI)O + 2 (Y2)O


Taking upto terms of (x 2 )

Y= + . + x
x2
q - +...
2!
Dividing each term by x 2 and taking limits
2y
q= L t -
x->o x 2
232 Engineering Mathel\latics - I

3
., (I + p)2
We have P = (at ongln) = -'-------=--<--
q
as p = 0 at (0,0)
2y
and Lt 2
q = x~Ox

I
and P (at origin) =
q

p (at origin) = Lt
x~o
(~)
2y
(iii) Similarly, if a curve passes through (0, 0) and the tangent at (0, 0) is Y-axis
2
then p (at origin) = Lt (Y2 )
x~o X

(iv) Curvature at pole: If the initial line is the tangent to the curve at the pole
2
X2 ) r2 cos 8 (.: x = r c.os 8)
then p= Lt - = Lt
x~o ( 2y x~o 2rsin8 y=rsm8

r 8 2
P = Lt ---cos 8
e~o 28 sin8

8
- Lt - r )
p - e~o ( 28
(as Lt - .- ~ 1.cos28 ~ I)
e~osm8

4.1.19 Example
Find p at the origin of the curve y4 + xl + a (xl + y) - a2y = 0 by Newtonian
method.
Solution
Equating to zero the terms of the lowest degree in the equation of the curve
- a2y = 0
=? y= 0
i.e., x - axis is the only tangent at origin
Then 'p' at (0,0) when X - axis is the tangent is given by
x2
p= Lt- ..... (1)
e~o2y
Curvature and Curve Tracing 233

Dividing the given equation by y


x-J x2
I + x -
y
+ a. - + ay =
y
a2

Let x~O
So that y~O

2
x
and - =2p
y

a
From (1) o + 0.2p + a2p + a.O = a2 => 2 a p = a2 => p = "2
4.1.20 Example
Find p at the origin for the curve x = a (8 + sin 8), y = a (I - cos 8) by Newtonian
method.
Solution
x = a (8 + sin 8), y = a( 1 - cos8) is a cycloid
X-axis is the tangent to the curve at (0, 0)

p at (0, 0) = Lt (~l
x---+o 2y

2
Lt a (8 + sin 8)2
0---+0 2a(1- cos 8)

a (8 + sin 8)2
L t - -'-----'-- (Of the form %)
0---+02 l-cos8 '

= Lt a (8 + sin 8)2(\ + cos 8)


0---+02 sin 2 8

= Lt !!.(~+ 1)2 (1 + cos 8)


0---+02 sm8

a 8
= -2 (1 + 1)2 (I + 1) using Lt - = I
0---+0 sin8

a
p= - [8]
2
P = 4a
234 Engineering Mathematics - I

4.1.21 Example
Find pat (0, 0) for the curve 2x4 + 31 + 4x2y + x Y - y2 + 2x = 0 by Newtonian
method.
Solution
Equating to zero the terms of the lowest degree in the given equation of the
curve 2 x = 0 => x = 0
y-axis is the tangent to the curve at (0, 0)
2
P at (0, 0) = Lt L
x->o2x

So that ..... ( I)

Dividing the given equation by 'x'


2 y2
2x3 + 3y2. L + 4x y + Y - - +2 = 0
x x
Taking x~O,y~O

2.0 + 3.0 (2p) + 4.0 + 0 - 2 P + 2 = 0; p = 1

Exercise - 4(A)

I. Find p for the following curves :


n
1. y = 4 sin x - sin 2x at x = 2"

(Ans:
515 ..
-I
4

3J2a
[Ans: - - ]
16
Curvature and Curve Tracing 235

5. E +JY =J; at %,~


a
lADs: J21

6. x = a (0 - sin 0), y = a (I - cos 0) at 0 = 1t


lADs: - 4 al
7. x = a sin 2 t (I + cos 2t)
Y = a cos 2 t (I - cos 2 t) at any point 't'
lADs: 4 a cos 3 tl
8. x = a (cos 0 + 0 sin 0)
y = a (sin 0 - 0 sin 0)
lADs: a 01
9. ? = a 2 cos20

a2
[ADS: );]

10. r = ~ (\ + cosO) at (~,~)


J2
[ADS. -3a ]

an
lADs: {n + 1)rn-1 1

4.2 Centre of Curvature, Circle of Curvature and Evolute


1. Definition: The centre of curvature at a point 'P' of a curves is the point 'C'
which lies on the positive direction of the normal at 'P' and is at a
distance 'p' (in magnitude) from it.
2. Definition: The circle of curvature at a point' P' of a curve is the circle whose
centre is at the centre of curvature 'C' and whose radius is 'p' in
magnitude.
236 Engineering Mathematics - I

x' o
y'

Fig. 4.6

4.2.1 Centre of Curvature


Let P (x, y) be any point on the curve and C (X, Y) be the centre of curvature at
'P' which is on the normal PC at P. Then PC =: p = radius of curvature at 'P'. The
tangent P T at P to the curve makes an angle \jJ with X - axis. P B, C A are
perpendiculars to X - axis, and P R is perpendicular to C A.
L.PTX=L.RPT=L.PCR=\jJ
I
cos \jJ = - -
sec \jJ

I
(I + tan 2 \jJ2 )2"

..... ( I)

dy
dx ..... (2)
Curvature and Curve Tracing 237

and we have ..... (3)

x = 0 A = 0 B - A B = OB - PR
(i.e.,) x = x - P C sin \jJ (From ~ P R C, PR = PC sin \jJ)
X = x - p sin \jl

From (2) and (3)

X=

d~[1 +(tfl)2]
dx dx
X=x-
d 2y
dx 2
Y=CA=RA+RC=Y+PCcoo\jJ
From (I) and (3)
238 Engineering Mathematics - ,

x' o x
y'

.. The coordinates of the centre of curvature of a curve

at p (x, y) is C

2 2 2
_ dy _ d Y . [ _ Yl (I + Yl ) 1+ Yl ] ,
Denoting Yl - d 'Y2 - - - 2 we can WrIte C x ,Y +
x dx Y2 Y2

C (X, Y) is the centre and 'p' is radius ofthe circle of curvature


.. The equation of the circle of curvature is
(x - xi + (y - Yi = p2
4.2.2 Example
Find the centre of curvature of the curve
Y = x 3 - 6x2 + 3x + 1 at (1, -1)
Solution
dy
- = 3x2 - 12x + 3
dx
Curvature and Curve Tracing 239

dy _
--6
dX(I,I)

=-6
dX2 (1,_1)

X= X -
YI (I +
2
yn I+ YI2
and Y = Y + - -
Y Y2
(-6XI+36) 1+36
X= I- and Y = I + - -
-6 -6
-43
X = - 36 and Y = -
6

-43)
The coordinates of the centre of curvature are ( - 36, -6-

4.2.3 Example
Find the centre of curvature for the cycloid
x = ace - sine), Y = a(1 - cos e)
Solution

dy = a sin e _ cot e/
dx a(l-cose) - 12

d 2y 2 e 1 de 1 1
-=cosec -.--=- x---,------,-
dx 2 2 dx 2 2 e
Sin -
a(l - cos e)
2

. 4
4aSIn
e
--
2
240 Engineering Mathematics - I

4
4a SIO
e
-
2

. 4e e
4 aslO -cot--
X= ace - sine) + _ _-'2"'--------=-2
. e
SIO-
2
X= ace - sine) + 2a sin e
X = ace - sine)
2
1+ YI
Y=y+--
Y2

(l+cot2E!)
Y=a(l-cose)- 12

. 4
- 4 a SIO
e
-
2

Y = a(l- cose)- 4a sin 2 ~


2
Y = a(l - cose) - 2a(1 - cose)
Y = -a(l - cose)
.. The coordinates of the centre of curvature are
[ace + sine), - a(1 - cose)]

4.2.4 Example
Find the centre ofthe curvature at any point (x, y) on the eIlipse.
Solution

..... (1 )
Curvature and Curve Tracing 241

Diff. (I) w.r. to 'x'

2b 2x + 2Q2y dy = 0
dx

..... (2)

..... (3)

Using (I) (2) and (3)

X= x _ YI (1 + yO
Y2

[~: (a' - h'), -;.' (a' - h' ij is the centre of curvalure,

4.2.5 Example

Find the circle of curvature of the curve fx + fY = fa at the point (~, ~)


Solution

..... (1)
242 Engineering Mathematics - I

Diff (1) w.r. to 'x'


1 -1 -I 1 --I
1
dy
-x 2 +_y2 -=0
2 2 dx
dy JY ..... (2)
dx--.r;

and
:10/ ai
14' /4
= ~1 ..... (3)

, I ~-I ----..jy-x
..;x-y dy C 1 ~-I]
d 2y [ 2 dx 2
dx 2 = X

d 2y 4
..... (4)
dx 0/ (// a
/4' )14

p = [1 + y~ = [1 + 1]% = ~ ..... (5)


Y2 i .fi
a

a a
is the radius curvature of the curve at ("4'"4 )

X= x _ YI (1 + YI2 )
Y2
a 1(1+1) 3a
X=-+--=-
4 4 4
a

(1 + y2)
Y=y+ 1
Y2
a (I + 1) 3a
Y="4 + -4- = 4
a
3a 3a)
Coordinates of the centre of curvature ( 4' 4
Curvature and Curve Tracing 243

Equation of the circle of curvature of the curve (x - X)2 + (v _ y)2 = p2

3a)2 ( 3a)2 _
( X -4- + y -4- -
(~)2
Ji

Exercise - 48

I. Find the coordinates of the centre of curvature of the curve y = x J at the point

7 31
IAns: - - )
64' 38

2. Show that the centre of curvature for the curve y =


2
x x+ 9 at (3, 6) is (3, I;)
3. Show that the centre of curvature at the point 't' on the ellipse x = a cos t,

y=bSllltlS
. .
[( a2 , )cos-I,- ()
a - b- a- - b 2) .
1
b Slll-'
1
1
4. Find the centre of curvature at (
3;1, 3;1) of the folium x 3 +I = 3a xy

21(1 21a
IAns: - - --I
16 ' 16
5. Find the centre of curvature at the point '0' of the curve
x = (I - aO) cos 0 + a sin 0
y = (I - aO) sin 0 + a cos 0
IAns : a sin 0, - a cos 01
6. Find the coordinates of the centre of curvature of the cycloid x = a (0 + sinO),
y = a( I - cos 0)
(Ans: a (0 -sin 0), a (I + cos 0)
2
7. For the curve y = 111X + ~ show that the circle of curvature at the origin is
a
x2 +I = a 2 (I + 1112) (v - I1lx).
244 Engineering Mathematics - I

4.3
4.3.1 Evolutes
Definition .' Corresponding to each point on a curve we can find the curvature of
the curve at that point. Drawing the normal at these points we can find
centre of curvature corresponding to each of these points. Since the
curvature varies from point to point, centres of curvature also differ.
The totality of all such centres of curvature of a given curve will define
another curve and this curve is called the evolute of the curve.
Thus the locus of centres of curvature of a given curve is called the
evolute of that curve.
Notes:
(i) Elimination of x, y from the coordinates of the centre of curvature (X, Y)
gives an equation in terms of X, Y which is called evo/ute of the given
curve y = fix).
(ii) When the equation of the curve is given in the parametric form (say t)
elimination of '1' from the coordinates of the centre of curvature (X, Y)
gives an equation in terms of X, Y which is called evo/ute of the curve
x = fit), y = get).

4.3.2 Example
Find the evolute of the parabolay = 4 a x
Solution
Differentiating y = 4ax w.r. to 'x'

2ydy = 4a
dx
dy Fa ..... (1)
dx = Fx
d y
2
-Fa
dx2 =-3- ..... (2)
2x2

X= X _ YI (1 + yl2 )
Y2

X=x
-~(1
Fx
+~)
x
-Fa
3
2x 2
Curvature and Curve Tracing 245

X=3x+2a ..... (3)

1 + y2
I + Cl )
X
Y = Y+ _ _I = Y+ ~----=c'-
y)
I-Fa
,
2x 2

..... (4\

Squaring
4
y2 = -.x3
a

X-2a
From (3) substituting x = --3- in (5)

y2= _ 4(X-2a)'
a 3

27 ay2 = 4(X - 2 a)3

27 a r = 4(x - 2 0)3 is the evolute of the curve.

4.3.3 Example

Solution

..... ( I)

Differentiating (I) with respect to 'x'


dy
2b 2x + 2a2y - = 0
dx
2
dy = -b x
..... (2)
dx 02y
246 Engineering Mathematics - I

_b 2 a 2b2
= a 2 y2 a 2;.

d 2y _b 4
dx2 = a 2y 3 ..... (3)

X = (I
x _ YI + yl2 )
Y2

x
X = x - -4 -2
(a4y2 + b4;x2)
a b
Substituting c?y2 = a 2b2 - b2x 2 from( I)

..... (4)

b4 x 2
1+4""2
ay
=y+
_b 4
a
2
i
Curvature and Curve Tracing 247

Y = y - ~ (0 4; + b4x 2 )
a 2b 4
Substituting b2x 2 = 02b 2 - a 2; from (I)

Y = y -+ a b4
[a 4; + b
2 2
a (a 2 -;)]

a 2 _b 2
Y = y3 ..... (5)
b4
Eliminating x, y from (4) (5)

(aX)32+ (bY)}2= (a 2 - b 2 )2:I is the evolute of the given curve.

4.3.4 Example
Show that the evolute of the curve

2 2 2 2 2 ~
X3 +y3 +a 3 = is(X+Y)3 +(x-Y)l =2a 3
Solution
The parametric equation of the curve is x = acos 3S, y = a sin 3S
dy
dy = de 3 a sin 2 ScosS
dx dx - 3a cos 2Ssin S
de
dy
- =-tan S
dx
d2y de
- 2 =-sec 2 S -
dx dx
248 Engineering Mathematics - I

30 cos 4 esin e

..... ( I)

(I + tan 2
e)
1

..... (2)
From (1) and (2)
(X + Y) = a(cose + sine)3 and X - Y = a(cose - sine)3
') I '), ~ /

{X+yt3 +{X-Y)"'3 = 2a 3
J J 2
(x + y)~ + (x - y)~ == 2a:1 is the evolute of the given curve.

4.3.5 Example
Show that the evolute of the curve
x = a(cose + sine), y = a(sin e - e cose) is
x 2 +y=a2
Solution
x = a(cose + sine) and y = a(sin e - e cose)
Curvature and Curve Tracing 249

Differentiating w.r.t 0

dx .. dy .
-;-0 = a(- Sll10 + Sll10 + OcosO), -'- = {{(cosO - cosO + 0 SIIIO)
a 10
dx dy.
dO = aO cosO, dO = aO SII10

dy aOsinO
dx (JOcosO
ely
-, = tan 0 ..... ( I)
(X

d 2y
..... (2)
dx 2 - (l0 cos'O

X = x- .
y{l+y2)
1 . 1 =
. tan{l+tan 2
a(cosO + 0 S1l10) _ ----"-------:-----"--
0)
Y2 I
aOcos 1 e
X = a cos 0 ..... (3)
I+ y2 (I + tan 2 0 )
y = y + --'_I = a(sin 0 - 0 cosO) + I
Y2
aOcos '0
Y = (I sin 0
Eliminating '0' from (3) & (4)
X2 + y2 = (/2

:. x 2 + 1= (12 is the evolute of the given curve.

4.3.6 Example

Show that the evolute of the curve x a( cos t + log tan ~) ,y asin
= = t is

x
y = a cos h
a
Solution

x = aCcost + log tan '2t ) y = 1I sin t


250 Engineering Mathematics - I

dy t
1
-dx =a -smt+--.sec
t
r. t 1 -dy =acost
2 -.-
2 2 ' dt
j
an-
2

-dx = a -smt+
dt r
. t1cos-t
2 sm--
=a -smt+-.-
IJ
sm t
2 2
j [.
dx acos 2 t dy
. and -d = a cost
dt SIn t t

dy
dy .J!L acost
= tan t
dx dx
--
acos 2 t
dt sin t

t t
X = a( cos t + log tan "2) - a cos t = a log tan "2 ..... ( 1)

1+ yf
Y2

1+ tan 2 t
y = a sin t+
sin t

a
y=- ..... (2)
sin t
t
From (1) tan - = eX/a
2
. a
From (2) SIn t= Y
Curvature and Curve Tracing 251

2
2 tan t 2 a
(i.e.,)
1+ tan 2 t . y
2
x
2e a ([

2X
I+e a Y

y = ~( / a + e - X,,) = (( cosh (%)


y = a cos II (%) is the evolllte of the given curve.
Exercise - 4(C)

c
I. Find the evolllte of the curve x = c t and y = -
t

2. Prove that the equation to the evolute of the parabola x 2 = 4ay is 4(y - 2(/)3 =
27 a x 2

x2
3. Show that the evolute of the hyperbola I is (ax)2/3 - (by)213 =

(a 2 + b2 )2/3

4. Show that the evolute of the cycloid x = a (q - sin q) y = a (I - cos q) is another


equal cycloid.

4.4
4.4.1 Envelopes
Family of curves:
Let us consider the equation of a straight line x cosa + y sina = p. Where a is
a parameter. For different values of a, x cosa + y sina = p gives different straight
lines but all of them are at a constant distance 'p' from the origin. The set of all of
these straight lines is said to form a family of straight lines and 'a' is called the
parameter of the family. For a given p, different values of'a' give different straight
lines which touch the circle x2 + .Y = p2. The circle x2 + 1 = p2 is called the
envelope of the family of straight lines.
252 Engineering Mathematics - I

A curve, which touches each member ofa given family of curves and each point
is the point of contact of some member of the family is called the envelope of the
family of curves.
Letj(x, y, a) = 0 be the given family of curves and a is the parameter.

Now d f'" . h respect to "


. . WIt
I Jerentlatmg . II y
a partla a/(x,y,a) =
0
aa

EI 0 f' a' f romJ'"(x, y, a ) = 0 an daf(x,y,a) = o


Imlllatlon . .III
gIven an equatIOn
aa
terms of x, y which is called the envelope 0/ fhe family of curves.
If the given equation of the family ofcurvesj(x,y, a) = 0 is the quadratic in 'a',
say of the form Aa2 + Ba + C = 0 where 'a' is the parameter and A, B, Care
functions of x, y. Then the envelope of the family of curves is B2 - 4AC = o.

4.4.2 Example

Find the envelope of the family of straight lines y = mx + ~ where 'm' is the
m
parameter.
Solution
a
y=mx+ - ..... ( I)
m

III is the parameter


Diff(l) w.r. to 'm'
a
o=x--2
m

111= ~ ..... (2)

Substituting for' m' in (I)

y = 2.[;;;
Squaring we gety = 4 ax is the envelope of the family of curves
Curvature and Curve Tracing 253

a
Aliter: y = mx + -
III
m2x - 111 Y +a= 0
is quadratic in parameter '/11' (A = x, B = - y, C = a)
8 2 - 4 AC = 0 then gives the required envelop.
.. Envelope of the family of curves is i - 4(a)(x) = 0
i = 4 ax is envelope.

4.4.3 Example
Find the envelope of the family of curves
I J J J
y=mx+ va-nr+b- w here'm'ist heparameter.

Solution
I J J J
y=mx+ vlrm-+b-
(y - I1lx)2 = ([2m 2 + b 2

m 2(x 2 - a 2) + m(- 2 y x) + (V - b2) = 0


is quadratic in parameter '111'
The envelope of the family of curves
(-2 x y)2 - 4(x 2 - a 2)(1- b2) = 0
i
b 2x 2 + a 2 = a 2b 2
2 2
.;. + -=;- = I is the envelope of the family of curves.
a b

4.4.4 Example
Find the envelope of the family of curves x cos 38 + y sin 3 8 = c when 8 is the
parameter.
Solution
Given x cos 3 8 + y sin 3 8 = c (8 is parameter) ..... ( I)
Diff. (I) partially w.r. to '8'
- 3 x cos 2 8 sin 8 + 3y sin 2 8 cos 8 = 0

Hence tan8= y
x
254 Engineering Mathematics - I

Substituting these values in (])

3 [ 13
X
[ ~ X2 Y+ y2 ] +y x
~ X2 + y2
j=c
x 21 = C(X2 + I) is the envelope of the family curves.
4.4.5 Example
2 2
Find the envelope of the ellipse -;- +
a b
4 = I where

a, b are parameters connected by the relation a2 + b 2 = c2


Solution
x2 y2
Given -2+ -
2
=] ..... (1 )
a b
a2 +b2 =c2 ..... (2)
Assume that' a' and' b' are functions of' t'
Diff. (]) and (2) w.r. to '1'
2 (- 2) da + I (- 32)db = 0
x a3 dt b dt

x 2 da y2 db
-3 - + -3 - =0 ..... (3)
a dt b dt
Diff. a2 + b2 = c2 w.r. to '1'
da db
2a- +2b- =0
dt dt
da db
(i.e.,) a- + b- = 0 ..... (4)
dt dt

Eliminating da, db from(3),(4)


dt dt
Curvature and Curve Tracing 255

Using (I) and (2) we get


2
x /

=~=-2
a b c

(i.e.,) [/4 c2

(14 =x2c2
(12= XC

/
-=-

Substituting (12, h2 , values in a 2 + h2 = ("2

x 2c2 + Ic 2 = c2
xl + J,z = I is the envelope of the family of curves.

4.4.6 Example

Find the envelope of the family of curves ~ + Eh = I where (I, b are parameters
{/

connected by the relation ah = ('2.

Solution

Given ~+E = I ..... ( I)


a b
ab = ('2 ..... (2)
Assume that a, b are functions of 't'
Diff. (1) and (2) w.r. to 'I'

- I-) +
x( - da y.-( -I -) =
db O
a 2 dt h 2 dt
x da y db
- - - + 2- - =0 ..... (3)
(/2 II h dl

Diff. ab = ('2 w.r. to 'I'

db da
a-+b-=O ..... (4)
dt dt
256 Engineering Mathematics - I

da db
Eliminating dt ' dt from (3), (4)

x y x y x y
2 2 -+- - -
!L.=lL~-.!L=JL=~
b a ab ab 2ab
Using (I) and (2) we get
x y
=~=-2
b a 2c

x
a = 2c 2

ab

Using (2)
X c2
..... (5)
~= 2c 2
a=2x

y ab
-=--
b 2c 2
Using (2)

b=2y ..... (6)


Substituting a, b values from (5), (6) in (2)
(2 x)(2 y) = c2
4 xy = c2 is the envelope of the family of curves.
Curvature and Curve Tracing 257

Exercise - 4 (0)

1. Find the envelope of the family of curves:

(a) y = mx + a ~I + m 2 when 111 is parameter.

(b) ; = 2a(x - (I) where a is the parameter.


IAns : x 2 - 2; = 01
(c) x cos <p + y sin <p = p where <p is the parameter.

(d) ~ +~ = I when a + b = e (a, b parameters).


a b

(Ans : fx + .JY = -!c ]


(e) ~ + ~ = 1 when a2 + b2 = e2 (a, b parameters).
a b

IAns: fx + .JY = -!cl


(f) ff H + = I when a + b = e (a, b parameters).

2 2 2

(Ans : x 3 + y3 =c 3 ]
In m
(g) Find the envelope of the family of curves ~ + L = I when dnb n = elll + n
ani bill

(h) Find the envelope of the family of straight lines ~+~ = 1 where a & bare

related by the equation d l + bn = en, e being a constant.


n II /J

(Ans : xn+1 + yll+1 = C"+ I ]


258 Engineering Mathematics - I

4.5 Curve Tracing


Generally a curve is drawn by plotting a number of points and joining them by a
smooth line.

If an approximate shape of the curve is sufficient for a given purpose then it is


enough to study certain important characteristics. This purpose is served by curve
tracing methods.

The points to be observed for tracing of plane algebraic curves are given below.

1. Whether the curve is passing through the origin, if so the equations of the
tangents to the curve at the origin.

Suppose F(x,y) = is the algebraic form of the equation of the curve.

F(O,O) = => the curve is passing through origin (i.e.,) If there IS no


constant term in F(x,y) then origin lies on the curve.

The equations of the tangents to the curve are obtained by equating the
lowest degree terms in F(x,y) to zero.

If at 0(0,0) the tangents are:

(i) real and coincident then '0' is called a cusp

(ii) real and different then '0' is called a node

(iii)imaginary then '0' is called a conjugate point

2. Whether the curve is symmetric about an axis or about other any line. If

(i) F(x, y) = F(x-y) => curve is symmetric about x-axis


(ii) F(-x, y) =F(x, y) => curve is symmetric about y-axis
(iii) F(-x,-y) =F(x, y) => curve is symmetric in opposite quadrants.
(iv) F(y, x) = F(x, y) curve is symmetric about y = x

(v) F(-y,-x) =F(x, y) curve is symmetric about y =-x


Curvature and Curve Tracing 259

y
y

----+-----------~~x
--------~~~----------.. x
y' =4ax

(i) (ii)

y
y

----~~~----------------~x ------~~~----~x

~~V
3
x +y'

(iii) (iv)
y

-----..;:::.~"'-------~x

(v)

3. Weather the curve intersects the axes, if so the tangents to the curve at these points.
4. Find the region in which the curve exists (i.e.) the curve is defined. The values of x
for which y is defined gives the extent parallel to x-axis and the values of y for
which x is defined gives the extent in a direction parallel to y-axis.
If y is imaginary for values of x in a certain region then the curve does not exist in
that region.
260 Engineering Mathematics - I

Similarly with respect 10 values ofy .


5. Finding the asymptotes.
An asymptote is a line that is at a finite distance from (0, 0) and is tangential to the
curve at infinity (i.e.) the curve approaches the line at infinity.
(i) Sum of the coefficients of the highest degree terms in x equated to zero gives
the equations of the asymptotes parallel to x - axis.
(ii) Sum of the coefficients of the highest degree terms in y equated to zero gives
the equations of the asymptotes parallel to y-axis.
(iii) To find the asymptotes that are neither parallel to x-axis nor parallel to y-
axis (i.e.) oblique asymptotes, the following method is suggested.
Substitute y =mx + c 111 F(x,y) = 0 and rewrite the equation as a polynomial
equation in 'x'as

$ (m)x
n +$ l(m)x
n-I + .... +$ =0
n n- n
The slopes of the asymptotes are given by $I/(m) = O. Let the slopes be

The values of'c' can be obtained trom $I/-I(m) = 0,$1/_2(m) = 0 (if necessary).

Let the corresponding values of c be c1' c2 ,

Note: if $1/ (m) is a constant then there are no oblique asymptotes to the curve.

6. Obtain dy from F(x,y) = 0 by differentiation.


dx

If in the interval for x, dy > 0 then the curve is increasing


dx

and : < 0, then the curve is decreasing in that region.

.
7. TIle po liltS 0f
maxima an dminima
.
are given by -dy = 0.
dx
Curvature and Curve Tracing 261

~o+-------------------~X

dy
- =
o.gIves XI,X)
dx -
If in an interval for x
2
(i) el
~-
~ > 0 then the curve is concave upwards (X), X 4 ' .... in
-
the figure) and has

a minima in that interval.

~
2
(ii) el < 0 the curve is concave downward (XI' xw .... in the figure) and has a
llx- -
maxima in that interval.

~
2
(iii) If at a point X) (say) in the interval d = 0 then that point is called a point
- dx-
of inflection and at that point the curve changes it concavity to the opposite
direction.

4.5.1 Example
X2 +2x
Trace the curve y = .... ( 1)
x+ 1
Solution:
Substitution of X =0 in (1) gives y = 0:. origin lies on the curve.
F(x,-y):f:- F(x,-y):f:- F( -x, y) etc :. The curve is not symmetric about any line

can be written as y (x + 1) - x
2
1. - 2x = 0 .... (2)
2
(ie) x - xy - (y - 2x) = 0 Lowest degree term is y - 2x = 0

Equation of the tangent at (0,0) is Y = 2x .


Substitutingy = 0 in (1) we have x(x+ 2) = o,x = O,x =-2
262 Engineering Mathematics - I

:. The curve crosses x -axis at (0,0) and (-2,0)


Since x =0 in (I) gives y = 0 only, there is no intercept on y-axis
The curve is not defined x at -I. Hence it IS discontinuous.
:. The curve is defined in the region R - {-I}
Coefficient of highest degree term in x i.e.) of Xl) IS I which is a constant.
Hence there is no asymptote parallel to x -axis.
Coefficient of highest degree term in y is x + I. :. x + 1=0 is the asymptote parallel
to y- axis.
dy
From (2) which is > 0 always except at x = -1
dx
Hence the curve is increasing in the regions.
00 < x < -2, -2 < x < -1, -1 < x < 0 and 0 < x < 00
\
\
\~
\..-
\j.:,
\
\
\
\
\
I
\ /
\ /

1 2

Fig. 4.7
4.5.2 Example
2
x +1
Trace the curve y-
- Xl -1
Solution:
....... ( I)
Curvature and Curve Tracing 263

x= in (I) gives y
y intercept is -I.
= -I :. Hence curve does not pass through (0,0), and

r (-x,y) = r (x,y) curve is symmetric about y-axis

y= gives x
2

+ 1= => x = i curve does not cross x -axis.
Coefficient of highest degree term in x is y-I (from( I
llence y = I is the asymptote parallel to x -axis

Coefticient of highest degree term in y is x2 -] x = ] are asymptotes parallel


to y-axis

dy = Hence y
I
=0 .
gives x =0 II
and Yx=o = -34 < 0 and =I:- 0
dx

:. The maximum point on the curve is (0,-1) and there are no points of inflection.
y ~

l~Y.,
I'J

~
Ix
I
I
I
I
I
--------1---_
I
----r---------
I
I I
I I
I I X
I
I 0 I
I
I
I __ ..JI ________
(0,-1) _
Y=-1
--------1--- I
I I
I I
I I
I I
I I
I

Fig. 4.8

The curve is decreasing in the interval < Ixl < ] and I< x < 00 as dy is -ve there
dx
an d ..
IS IIlcreaslllg 111 -00 < X < 1 as -dy.IS +ve III
. t I"
l1S mterva
i
dx
4.5.3 Example:
3
Trace the Folium x + / == 3axy .... ( I)
264 Engineering Mathematics - I

Solution:
Clearly (0,0) lies on the curve .No intercepts on y-axis.

F (y,x)=/+x'-3ayx=0=F (x,y) :. curve is symmetric about x-axis

Coefficient of highest degree term in y is I. a constant, and hence no asymptote


parallel to y-axis. Lowest degree term equated to zero gives xy = 0, x = 0, y = 0
(i.e) the axes are the tangents to the curve at the origin.

y = x in (I) gives 2x' - 3ax 2 = 0 x = 0, 3;1 (ie) the line y = x meets the curve

at (O,O)and (3~, 3~)

F (-x, - y) = x' - / + 3axy = F (x, y) :. curve is not symmetric about (0,0)


To obtain oblique asymptotes: Substitute y = mx + c in (I) and rewriting in terms
3 2 2
of 'x' we get (I+m )x'+(3m c-3am)x + ..... =0 (ie) 3(m)=I+m'and

2 (111) = 3m 2c - 3am, .....


, (m) = 0 gives m= -1, there is only one oblique asymptote with slope-I

, , , x+y=3a

,
VI

Fig. 4.9
2(m)=0 gives 3m(cm-a)=0 => cm-(l=o (e) c=-a(-:m=-l)

:.Theasymptoteis y=-x-a or y+x+a=O .... (2)


Curvature and Curve Tracing 265
--------------------~----------------------------------------~

2
dy=
From (I) - ay-x
~ - )
:. (dY =-1
dx y -ax dx e~/,32")

Equation of the tangent to the curve at this point is y + x = 31 which is parallel to (2)
4.5.4 Example:

Trace the curve a 2y 2- x 2( a 2- x 2) = 0 ........ ( I )

Solution:
x =0 in (I) gives y = 0,0 (ie) double point on the curve

F (-x, y) = F (x, -y) = F (x, y) curve is symmetric about both the axes.

Lowest degree terms, in (I), equated to zero gives a


2
(/ - x
2
) = 0 ~ y = x
are the tangents
to the curve at (0.0). y = in (I) gives x = a
The curve intersects the x-axis at (a, 0) and (-(1,0) From (I) it can be seen that

for Ixl > a


l is negative (ie) the curve does not exist for Ixl > a

YI = gives x = 0, .i2' points of maxima and III inima.

a
Tangents at the points x = J2 to the curve are parallel to x-axis .Further

ddx2y ) at x a .
= J2 .
IS negative. :. x
a .IS a maxImum
= J2 . . d" .
POlllt an Y IS IIlcreaslIlg
( 2

in 0< x <.i2 while decreasing in .i2 < x < a .Thus after tracing the curve in

the first quadrant, the symmetry about both the areas can be utilized for tracing the
complete curve.

Substituting y = mx + c in (I )gives X4 + a 2 (mx + c r -a 2 2


x =0
266 Engineering Mathematics - I

4 (m) = 1 which is a constant. Hence there are no oblique asymptotes to the curve.
y

~(_-a~,O~)~--------~----------+-~~X

Fig. 4.10

4.5.5 Example:
Trace the curve y2 ( x 2 - a 2) + a 2x 2 = 0 ..... (1)

Solution:
x = Oin (1) gives y = 0 :. (0,0) lies on the curve.
F (x,-y) =F (x,y) =F (x,-y) :. curve is symmetric about both the areas.

Highest degree terms of y equated to zero gives (X2 - a 2) l = 0 => x = a


:. x = a are asymptotes parallel to y-axis. Highest degree terms of x equated to
zero gives x 2(y2 + a 2) = 0 => x = ia ... No asymptotes parallel to x-axis.

Equating lowest degree terms to zero we get a 2(y2 - x 2) = 0 => y x are tangents at
(0, 0) writing y = mx + c in (1) and rearranging in terms of I x I

m2x 4+2mcx3+(c 2 _a 2m2+a 2)x2 -2a 2mcx-c 2a 2 =0

=> c is indeterminate since m = 0


2 (m ) = 0 gives c 2+ a 2 = 0 => c = ai
oblique asymptotes do not exist. Further there are no intercepts on either of the axes.
Curvature and Curve Tracing 267

2
a 2x
( 1) can be written as l = -2 - -
x _a 2
which indicates that curve does not exist

when Ixl > a :. The curve exists only in the region -a < x < a.

Fig. 4.11
dy a4 x
= and y> 0 in first quadrant. Hence curve is increasing in 0 < x < a .
dn y

4.5.6 Example:
Trace the curve l (2 - x) - x2 (1 + x) = 0 ....... ( 1)
Solution:
x=Oin (1) gives y=O :.(0,0) lies on the curve F(x,-y) = F(x,y) :.curve is
symmetric about x-axis . Lowest degree terms equated to zero gives

2l- x 2 = 0 => y = 12 tangents at (0,0) to the curve y = 0 in (\) gives -1,0

curve passes through (-1,0) in addition to (0,0)


Coefficient of highest degree terms of y equated to zero gives 2 - x = 0
(ie) x = 2 is an asymptote parallel to y-axis
2
No asymptote parallel to x-axis as coefficient of x is 1,a constant
268 Engineering Mathematics - I

Substituting Y = mx + c in (I) and rearranging


3 2 2 2 2 2
x ( m + 1) + x ( 1+ 2mc - 2m ) + ( c - 4mc ) x - 2c =0
3 (m) = m 2 + 1 => 3 (m) = 0 gives m = i :. There are no oblique asymptotes.

--~----~~--------~----~x
I (2,0)
... -x
~..., y=- I
'" J2 I
"'",. I
I
I
I
I

Fig. 4.12
2
From (I) Y1 = y2 + (2x + 3x h' h h O 0 < X < 2 were
) . T IS sows t at YI > 111
h t he curve
2y 2-x
is increasing and x = 2 is an asymptote.

5J5j
YI = 0 => x = 0, x = .
4

..
In the secon d quadrant y IS maxImum at x
5-1
=- -- where the tangent IS
4
parallel to x-axis

It can be seen that the curve forms a loop in [-1,0] due to symmetry about x-axis.

4.5.7 Example
Trace the curve / -(x-2)(x-4)2 =0 .. , .. (1)

Solution:
y = O. (I) gives x = 2, 4
Curve intersects the x-axis at (2,0) and (4,0)
Curvature and Curve Tracing 269

F(x, -y) = F (x,y) :. curve is symmetric about x-axis


For x < 2 , / is -ve :. therefore curve does not exist for x < 2
From (I) it can be seen that x = 2 is a tangent at (2,0)

Coefficient of x 3 = l,a constant :. No asymptote oarallel to x-axis


Differentiating (1) w.r.t 'x'
dy _ (x-4)(3x-8)
dx 2y

At (4.0) dy = Lt (x-4)(3x-8) Lt 3x-8 = J2


dx H4 2.J x - 2 ( x - 4) H4 2.Jx-2
Equation of tangents at (4,0) are y = J2(x - 4)

Further dy --}- 00 as x --}- 2 :. x = 2is a tangent at (2,0)


dx
y

--~--~----~~~~~x
(2,0)
,,

Fig. 4.13

dy =
dx
gives x = 8/3. Thus the tangents at x = 8/3 are parallel to x-axis. For the

region.

2 < x < 8/3, dy >


dx
above x-axis and dy <
dx
below x- axis. At x = 8/3, y takes

extreme values. For the region 8/3<x <4 dy <


dx
above x -axis and dy >
dx
below

x -axis.
:. the curve forms a loop in 2 ~ x ~ 4
270 Engineering Mathematics - I

4.5.8 Example
2
2
Trace the curve y ( x + 4a ) - 8a 3 =0 .... (1)

Solution:
y * 0 when x = O. :. origin does not lie on the curve.

Curve is symmetric about x-axis ,since F (-x,y)=F (x,y)


y is +ve for all real x :. Curve completely lies above x-axis.
Substituting y = mx + c in (I)

m3 + cx 2 + 4a 2 mx + 4a 2 c - 8a 3 =0
3 ( m ) = 0 => m = 0 and 2 ( m ) = 0 gives c = 0

dy 16a 3 x
(ie) y = 0 is an asymptote to the curve From (I) -
dx
= - ----
(X2 + 4a2 t
In first quadrant dy < 0 and hence y IS decreasing. dy o at (0, 2a). y s
dx dx
maximum at (0, 2a) .

----------~--------------~x

Fig. 4.14
4.5.9 Example:
Trace the curve y- c cosh x / c = 0 .... (1)
Solution:
x= 0 in (1) gives y = (c/2)[e O +e-D] = r
Curvature and Curve Tracing 271

:. Curve does not pass through (0,0) and intersects y-axis at (0, c).

F (-x,y)= y -(c/2)( e- x / c +e<i<) = F(x,y) curve is symmetric about y-axis

It can be seen from (I) that y>o for all real x .

. h
dy= Sill
- ( x /) e -x -Xlc]
C = -1 [x/c
dx 2
which is >0 for x >0 and < for x <0.
y

Y=c
(D,c)

------~~------------~x
o

Fig. 4.15

Thus the curve is increasing in (0,00), and decreasing in (-00,0).


When x = we have dy = 0. Further y = c for x = 0.
dx
(i.e.) The tangent to the curve at (0, c) is parallel to x-axis.

4.5.10 Example
Trace the curve (x / a )2/3 + (y /a )2/3 -1 = 0 .... ( 1)

Solution:
y :I; 0 when x = :. (0,0) does not lie on the curve.

It can be observed that F (-x,y)= F (x,y)= F (x,-y)


The curve is symmetric about both the axes.
272 Engineering Mathematics-I

--~---------+----------~----~x

(0, - b)

-
Fig. 4.16

x = 0 gives / = b 2 => Y = b y = 0 gives x = a. Thus the curve meets x axis.


at (-a,o),( a,o) and y axis at (o,b ),( o,-b).

For Ixl> a or \YI >b (x/a) 2/3 +(y/b)2/3 -1> 0 Equation is not satisfied.

:. Curve does not exist for Ixl> a or IYI >b.


:. Curve completely lies in the region Ixl ~ a and Iy ~ bl

From (I) dy
dx
-!!..(
a bx
y
a )I/3. In the first quadrant y is decreasing as dy < 0 for
dx
O<x<a.
4.5.11 Example
Trace the curve y2 (a -x) _x 3 = O,a > 0 .... (1)

Solution:
Clearly y = 0, 0 when x = O. Origin is a double point on the curve.

F (x, - y) = F ( x, y). Curve is symmetric about x-axis.


Curvature and Curve Tracing 273

Differentiating (I) w.r.t, 'x' we get -


dy Fx (3a - 2x )
= - 3jJ dy 1(0,0) = o.
dx 2 (a-x) - dx

So x axis is a tangent to the curve at origin.


x = a is an asymptote parallel to y-axis.
3
A" coefficient of x is a constant, there is no asymptote parallel to x -axis.

For x < 0 and x> a curve does not exist (as y2 is negative)

Substituting y = mx + c in (1) and rearranging

m 2 + 1) + x 2 ( 2mc - am 2 ) + x ( c 2 - 2ame ) - ac 2 =0
3
x (

fA (m) = m 2 + 1=> m = i .

I
I x=a
PI
I
I
I
--~~--------.---------~x
(a,O)

Fig. 4.17

Hence there no oblique asymptotes.


3x2 + y2
From (I) YI = ( )
2 a-x y

For 0 < x < a, YI is positive. Hence y is increasing in 0 < x < a .

. 4.5.12 Example
Trace the curve x
2
(a - x) = l (b + x) .. .,(\ )
274 Engineering Mathematics - I

Solution:
Clearly curve passes through origin
F ( x, - y ) = F (x, y) .'. Curve is symmetric about x-axis.
Equating lowest degree terms to zero

by2 _ax 2 = 0 => y = ~.x


There are two real, distinct tangents to the curve at (0,0)

y =0 in (1) gives x2 ( a - x) = 0 => x = 0, a


The curve intersects x-axis at (a, 0)
3
There is no asymptote parallel to x-axis as coefficient of x is a con
y

I
I
I
I
--~I~--------~~--~~~~----~x
I
I
I
X=-b
l
I

Fig. 4.18

Coefficient of highest degree term in y is b + x


:. x + b = 0 is the asymptotes parallel to y-axis
Substituting y = mx + c in (I) and rearranging

x 3(1 + bm 2) + .... --0'I.e., V'3"'(m)-0' - Jb


- gIves m -_+_i

Hence no oblique asymptote to the curve.

t!) can be written as y = xJa - x which shows that y is imaginary in the


b+x
regions x > a or x < -b
Curvature and Curve Tracing 275

Hence the curve completely lies in the region -b < x ~ a


dy ( - 2X2 - 3bx + ax + 2ab)
From (I) - = 3/2 1/) which shows that dy ---) 00 as x ---) a
dx 2(b+x) (a-x)- dx

The tangent x = a to the curve at (a, 0) is parallel to y-axis.


Thus the curve forms a loop between (0,0) and (a, 0) .
4.5.13 Example:

Trace the curve x 2(y2 - a 2) - b2/ =0 ....(\ )

Solution:
x = 0 in (1) gives only y = O. Curve passes through origin.only and
does not intersect either of the axes:
F(-X,y)= F (x,y) = F (x, -y) :. Curve is symmetric about both the axes.

Lowest degree terms equated to zero gives a 2x 2+ b y2


2 =0
showing that there are no tangents to the curve at (0,0)
Coefficient of highest degree terms in y (i.e.) of y2 is x 2- b 2

:. x = b are the asymptotes parallel to y-axis


Coefficient of highest degree terms in x (i.e) of x 2 is y2 _a 2
:. y= a are the asymptotes parallel to x- axis.
Substitution of y = mx + c in (1) gives

x 4 .m 2 + x 3 (2mc) +X2 (c 2 _a 2 _b 2m2)_ 2b 2mcx-b 2c 2 = 0

3 ( m ) = 2mc does not provide value of c since m = 0


276 Engineering Mathematics - I

The asymptotes are y = a which are parallel to x-axis.


Hence there are no oblique asymptotes.

From (I) x ~ ~ by For IYI < a , x is imaginary


y2 -a2

Similarly for Ixl < b, y is imaginary. Hence curve does not exist for Ixl < b,
Iyl<a.
Since the curve is symmetric about both the axes, it is enough to study the curve in
the first quadrant.

J y=a
"-
(0,0) x=b
x =-b
y=-a

(
From (I) Y = /
"
"x
ax
2
-b
2
Fig. 4.19

..... (2)

From (2) it is observed that y ~ 00 as x ~ ba and x ~ 00 as y ~ a


ax
For the first quadrant y = + ~
x2 _b 2
a 2b2
:. Yl = - ~ .Thus Yl < 0 for x> b . So y is decreasing when x> b 111
(x 2 _b 2 )2
first quadrant.

4.5.14 Example
Trace the curve y2 (x - a) - x2 ( X + a) = 0 .... (1)

Solution:
x = 0 gives y = 0 :. (0,0) lies on the curve.
Curvature and Curve Tracing 277

Lowest degree terms equated to zero gIves a x . (2 + y 2) = 0 => x- + y = 0 J J

:. There are no tangents to the curve at (0,0)


J
coefficient of highest degree terms in y (i.e.) of Y IS x - a

:. x = a is the asymptote parallel to y-axis


3
No asymptotes parallel to x - axis since coefficient of x is constant)
Substituting y = mx + c in (I)

x 3 (m 2 -1)+ x 2 (2mc-um 2 - a) + x(c 2 - 2amc)-ac 2 =0


3(m)=m 2 -1=0 gives m=l
a(m2 + 1)
2(m) = 0 gives c = = a when m = I and = - a when m =-1
2m
I
I V'.Sl
/\/y=x+a
/ I (a,2a)
// I
1
I
--~.---~--------~x
(a,O)
I
1
" " " l(a,-2a)

~~+a=o
If""
Fig. 4.20
:. The oblique asymptotes are y =x + a and y + x + a =0
2 2
From (I) 2d = x -ax-a
dx (x -a).J x 2 - a 2

When x ~ -a, dy ~ 00 :. Tangent at (-a, 0) is parallel to y-axis


dx

-dy = o.
gIves x 2 - ax - a 2 1f5
= 0 => x =- -a
dx 2
278 Engineering Mathematics - I

Tangents to the curve at x = 1 + Fs a (A and B in fig) are parallel to x -axis.


2
For 0< x < a y is imaginary
a<x<oo yexists
-a<x<O y is imaginary
- 00 < x <-a yexists
(i.e.) The curve does not exist in the region -a < x < a.

1-Fs
For x = - - a there is no tangent to the curve (since the curve itself is not
2
defined there).

4.5.15 Example
Trace the curve /(a 2 _x 2)_x 2(a 2 +X2) = 0 .... ( 1)

Solution:
x =0 in (1) gives y = 0,0 :. origin is a double pt on the curve.
The curve is symmetric about both the axes sinceF(-x,y) = F(x,y) = F(x,-y).
Lowest degree terms equated to zero gives y = x as the tangents at the origin.
2(2 2)
(i)
can be written as y 2 = X a2 +x2 Th"IS III d'Icates that y +
~ _00 as x ~ a.
a -x
Hence the curve lies only in Ixl < a.
x = a are asymptotes parallel to y-axis.
Substitution of y = mx + c in (l) gives X4 (m 2 +1) + ... = 0
4 (m ) = m 2 + 1 = 0 gives m = i
There are no oblique asymptotes
4 2 2 4
From (1) dy = a +2a x -x
dx (a 2 _x 2)3/2(a 2 +X2yl2

In the first quadrant: dy > 0 for 0< x<a


dx
(i.e.) y is increasing in this region.
Curvature and Curve Tracing 279

Fig. 4.21
4.1.16 Example
Trace the curve l (a 2 + x 2 ) - x2 ( a
2
- x
2
) =0 .... ( I)

Solution:
x = 0 gives y = 0,0 :. origin is a double point on the curve.
F(-x,y) = F(x,y) = F(x,-y) shows that the curve is symmetric about both the
axes.
there are no asymptotes parallel to x-axis, since
coefficient of highest degree terms in x ((ie) of X4 )is constant.

The coefficient of highest degree terms in y (ie) of l 2


is x + a 2
:. there are no asymptotes parallel to y axis.

a 2 -x 2
From (1) y = x 2 2
a +x
For Ixl > a, y IS Imaginary

:. The curve entirely lies in the region -a s x sa.

y = mx + c in (l) gives J. (m) = m 2 + 1 :. m = i


280 Engineering Mathematics - I

There are no oblique asymptotes dy = 0 gives x = a~ -1 J2


dx

:. y values are extreme at x=a~J2 -1 ,where the tangents to the curve are
parallel to x-axes.
4 2 2 4
dy = + a -2a x _x
..... (2)
dx - (2a +x 2)3/2 (2
a -x 2)1/2

(-a,O)

Fig. 4.22

.'. x = a are tangents to the curve at (a ,0). Further (ddxY) (0,0)


= a:
a
= 1 from (2)

Hence y = x are tangents to the curve at (0,0)


The curve thus forms a loop between (0,0) and ( a ,0) and another loop between
(-a,O) and (0,0).

4.1.17 Example
Trace the curve a/ - 4 x2 ( a - x) = 0 ..... ( t)
Solution:
F (x, - y) = F (x, y) :. curve is symmetric about x-axis
F (0,0) = 0 curve passes through origin.
Curvature and Curve Tracing 281

y= in (1) gives x = 0, a (i.e.) the curve cuts the x-axis at (0,0) and (a ,0)
2
Equating the terms of lowest degree to zero we get al- 4ax = O::::? Y = 2x

Thus Y 2x = 0 are the tangents to the curve at (0, 0)

dy = 4x(3a-x2) ::::?(dY ) ----).00

dx 3ay dx (up)
(ie) the line x = a is a tangent to the curve at (a ,0)
substituting y = mx + c in (1) and rearranging the terms

4x 3 +ax 2 (m 2 -4)+ 2amcx-ac 2 = 0

fA (m) = 4 constant. Hence there are no asymptotes to the curve.

For Ixl > a we find y is imaginary.

Curve does not exist for Ixl > a.

________ ~~--------~~~x

Fig. 4.23
4.1.18 Example:
Trace the curve a
21 = x 2( 2a - x) ( x - a)
Solution:
F (+ x, - y) = F curve is symmetric about x-axis.

F (0,0) =0 origin lies on the curve (isolated point)


282 Engineering Mathematics - I

y = 0 in (1) gives x = 0,a,2a.


The curve intersects the x-axis at (a ,0) and (2 a ,0)
Terms of lowest degree are l + 2X2 which shows that there are no tangents to the
curve at (0,0) .

(y2 + 2X2 = 0 gives y = J2ix imaginary)

Proceeding as in 3.1.17 it can be seen that the curve has no asymptotes


y is imaginary for x < a and x> 2a(i.e.) the curve exists only in the region
a:::;x:::;2a.

Further dy ~ 00 as x ~ aor x ~ 2a
dx
:. x =a and x = 2a are tangents to the curve parallel to y-axis.
y

x
(0,0) 0

I I
x=a x = 2a

Fig. 4.24
4.1.19 Example
Trace the curve x( x 2 + y2) = a( x 2 - y2 ),( a> 0) ........... (1) (2001 s)

F (x, - y) = F (x, y) curve is symmetric about x-axis

Solution:
y = 0 in (I) gives x = 0,0, a . The curve meets the x-axis at (0,0) and (a ,0).
Equating lowest degree terms to zero we get
y2 _ x 2 = 0 => y = x which are the tangents at (0,0)
Curvature and Curve Tracing 283

dy ~ 00 as x ~a . Thus x = a is a tangent to the curve at ( a ,0)


dx
For x > a (1) shows that y is imaginary
No part of the curve exists for x> a.
y ~ 00 as x ~ -a :. x = - a is an asymptote to the curve.

Fig. 4.2~

4.1.20 Example
Trace the curve l (a 2+ x 2) = x 2( a 2- x 2) ..... (1)

Solution:
F(O,O) = 0 curve passes through origin

F (-x,y) =F (x,y) =F (x,-y) curve is symmetric about both the axes.

Lowest terms equated to zero gives a 2y2 - x 2a 2 = 0 => y x which are tangents
to the curve at (0,0)

(:) ~ 00 as x ~ a :. x = a are the tangents to the curve at (a, 0)

y is imaginary when x < -a or x > a


:. curve completely lies within the region -a ~ x~a
284 Engineering Mathematics - I

x
(-a,D)

Fig. 4.26
4.1.21 Example
Yrace the curve

Solution:
2
Let F( x,y)= y( a + x 2 ) - a2x =0

F( -x,y) =I:- F(x,y) =I:- F(x,-y), Hence the curve is not


symmetric about both the axes.
The powers ofy are odd. Therefore the curve is symmetric
In opposite quadrants.
F(O,O)=O. So the curve passes through the origin.

The coefficient of highest power of x i.e~ of x 2 is y


So y=O is an asymptote parreJ to x-axis.

The coefficient of highest degree of y is x 2 + a 2 = 0 => x = ai


Hence the extreme points are A(a,aJ2) B(-a,-aJ2)
Thus the tangents at A and B are parallel to x-axis.
The shape ofthe curve is as shown below.
Curvature and Curve Tracing 285

Exercise - 4(E)
Trace the following curves

2 x-2
3. y = 2- - 4. x 3 + / = ax 2 ; x> 0
x -1
5. 9ai = x(x-3a)2
7. y3 =a 2 x-x 3 8. i(a-x)=x 3 ;a>O
8a 3
9. y =2 - - -2 10. i(a+x)=x 2 (b-x)
x +4a
3
11. i = (x-a)(x-b)(x-c) ; a,b,c>O 12. y=x
2
13. i = x (3a-x)
x+a
15. y2 =(X-2)2(X-5)

17. xi =4a 2 (2a-x)

4.2.0 Tracing of curves (polar form)


The equation of the curve in polar coordinates may be in one of the following three forms
F (r,B)=O ,r=F(B), B=F(r)
A study of the following points will simplify the tracing
I. curve is symmetric
(a) About initial line B = 0 if F(r,-B) = F(r,B)
286 Engineering Mathematics - I

(b) About pole '0' if F(-r,B) = F(r,B) or


F(r,J[ + B) = F(r,O)

e = 1[/2

0=0
Pole 0

Fig.4.27(a)

(c) About 8 = J[ if F(r,J[-8) = F(r,8)


2

(d) About 8 = J[ if F(r, J[ -8) = F(r,8)


4 2
3J[ . 3J[
(e) About 8 = - If F(r,--8)=F(r,8)
4 2

2. (a) Values of 8 for which r =0 gives the equation of the tangents to the curve

at the pole

(b) F(r,O) = 0 gives the points of intersection of the curve with the initial line
J[
(c) F(r, -) = 0 gives the points of intersection of the curve with the line
2
8=J[
2
Curvature and Curve Tracing 287

3. (a) If 'i and r2 are the least and greatest values of r then the curve lies in the
annulus between circles of radii Ii and r2

(b) Curve does not exist if r is imaginary for values of () or if B is imaginary


for values of r

4. In t Ile region ' h -e1r > 0 I r I mcreases -dr < 0 I r I decreases .111 that regIOn
. .111 wIlIC .
dB dB
() =rr/2 y

__ ~~-..J...
o_ _ _ _ _ _ _ _ _ ~ . --. X

o 0=0

Fig.4.27(b) Fig.4.27(c)

5. Let p(r,B) be a point on the curve and P T is the tangent to the curve at P, OR
radius vector and is the angle T P R. Then

(i) tan =r dB gives equation ofPT


dr
(ii) = 0 gives tangent parallel to initial line

(iii) do
V' = -Jr .
gives tangent to Il1Itla I I'me
1./(/r. ..
2
6. If the curve meets the initial line at two distinct points and curve is symmetric
about the initial line then part of the curve forms a loop between these two points.
7. If for any value of B (sayB ,) such that It r = 00 then B = B1 is an asymptote to
0-->00
the curve
288 Engineering Mathematics - I

8. Sometimes it may be convenient to change the equation of the curve from polar to
Cartesian form by substituting respectively
x = rcosO, y = rsinO
y

9 = 1t/2

--nf~~Y===~)---7X
! 9=0

Few solved examples are given below

4.2.1 Example
Trace the cardioid
r -a(l +cosO) = 0----(1)
Solution:
F (r,O) = r -a(l +cosO) = F(r,O) curve is symmetric about the initial live.
When 0 = 1[ we get r = 0 :. curve passes through pole and
o= 1[ is the tangent at the pole

Max IcosOI = I :. Max r = 2a

:. The curve is entirely within the circle of radius 'a'.


Hence no asymptote to the curve.

0 0 1[ 21[
1[/4 1[/3 1[/2 21[/3 41[/3 31[/2 51[/2
2a a a 2a
r
a(l + 1/J2) 3a/2 a/2 0 a/2 3a/2

r = 2 a when 0 = 0 and r = 0 when 0 = 1[

.'. The curve intersects the initial Iine at (2a, 0) and (0, 1[ )
Curvature and Curve Tracing 289
------------------~~--------------------------------------

. b
The tangents to the curve are gIven y tan
AI
'I' = rde
-
dr

tan = a(l + cosO) = -cot e = tan (1l2 + ( )...(2)


a(e-sine) 2 2
when e= 0 , (2) gives = ~ (ie)The tangents at (2 a ,O)is perpendicular to the
initial line ..

Fig. 4.28

4.2.2 Example
Trace the curve r = a(l- eose)
Solution:
8 = n/2

8=0
(+2a,n) J--------..... ~---
(0,0)
........
~

Fig. 4.29
290 Engineering Mathematics - I

() 0, J[ 7J[ J[ SJ[ J[ 3J[ 2J[ 4J[ J[


--
2J[ 4' 4 3' 3 2' 2 3 ' 3
a
r 0
a(l- YJ2) ~ 3~ 2a

r = 0 when () = 0,2J[ and r = 2a when () = J[

:. The curve intersects the initial line at (0, 0), (2a, J[)
a(l-cos()) ()
tan = = tan-
asin() 2
:. tan = 0 when () = J[ (ie) the tangent at (2a, J[) is perpendicular to the initial
line.
(work out remain ing points as in 3.2.1)

4.2.3 Example:
Trace the curve r - a (1 + sin ()) = ..... (1)

Solution: r = a when () = 0, J[

r=Owhen ()=-J[/2 ...... (2)


J[
and r = 2a when () = -
2
:. The curve intersects the initial line at (a, 0), (a, J[)
and intersects the line () = J[/2 at (2a"~-i) and (0, - J[/2)
F (r, J[ - ()) = r - a ( 1+ sin ( J[ - ())) = r - a (1 + sin ()) = F (r, ())
:. curve is symmetric about () = ~

The equation of the tangent at the pole is () = -~ (from (2))

Maxlsin ()I =1 :. Max r = 2a Hence curve completely !ie~ ".'!!hin the region of
the circle r = 2a and there are no asymptotes to the curve.
Curvature and Curve Tracing 291

0 0 7r/6 7r/2 57r/6 7r 27r/6 37r/2 117r/6 27r

r a 3(1/2 2a 3(1/2 II
a/2 0 (1/2 a

--------~--~~--~------~.O=O

0= -rr/2

Fig. 4.30
4.2.4 Example:
Trace the curve r - a sin 0 =0
Solution:

0 0 7r/12 7r/4 7r/3 57r/12 7r/2

a 0
r 0
a
- J3
-a
a
-
2 2 2
When 0 = 0 we have r = 0 :. pole lies on the curve

F (r,7r+O) = r-asin(27r+20)=r-asin20= F (r,O)


:. curve is symmetric about pole. ...... (1)

F (r, ; - 0 ) = r - a sin ( 7r - 20) = F (r, 0)


292 Engineering Mathematics - I

and F (r, 3; -8 ) = r -asin(3ff - 28) = F(r,8)

... curve is symmetric about the lines 8 = ff14 and 8 = 3ff14 ...... (2)
from (1) and (2) it follows that the curve is symmetric in all the four quadrants.
r =0 gives 8 = O,ffI2,ff,3ff/2 these four lines are tangents, to the curve, at the
pole.

Max r = Max la sin 281 = a :. The curve completely lies within the region of
the circle r = a .

The line 8 = ffl4 meets the curve at (a,ffI4).


0= rc/2

o =rc ----.-,;:~~rr_Illll:iiiii:::-----__1~ 0 =0

e =3rc/4
Fig. 4.31

Note: Similarly trace the curve r = a cos 28 .


4.2.5 Example:
Trace the curve r2 - a 2cos 28 = 0 ...... ( I)
Solution:

8 0 ffl6 ffl4 ff13 to 2ff13 3ffl4 5ffl6 ff

r a a/J2 0 imaginary 0 a/J2 a


Curvature and Curve Tracing 293

Jr
r =0 gives eos2B = 0 => f) = -
4
Jr
curve passes through the pole and 0 = - are the two tangents at the pole
4
F(r,-fJ) = F(r,fJ) :. curve is symmetric about the initial line.

F(r,Jr -0) = r2 _a 2 eos(2Jr - 20) = r2 _a 2 eos20 = F(r,fJ)

curve is symmetric about 0 =~


F(,., Jr + fJ) = ,.2 - ([2 eos(2Jr + 2fJ) = F(r, 0)
curve is symmetric about the pole

8=0
~ __ ~ ________ ~~ ______ ~~~~x

Fig. 4.32

Since leos 2fJI ~ 1 ,Max r =a


Curve lies completely within the circle r =a
B = 0 gives r2 = a 2 => r = a
curve intersects the initial line at (a,O) and (-a, 0)

r2 is -ve When Jr < 0 < 3Jr (": cos 20 < 0)


4 4
curve does not exist (above the initial line) in this region.
Differentiating (1) w.r.t I fJ I
2
dr 2 2 2B 0 dr _a sin 2B
2 r-+ a SIn - => -=----
dB dB r
294 Engineering Mathematics - I

dO r ff
tan = r - = r. 2 = -cot20 = tan(-+ 20)
dr _a sin 20 2

Thus when O=O} tan = co


O=ff
Hence the tangents at 0 =0 and 0 = ff are .lIar to initial line

When

(i.e.,) the radius vector 0 = %is tangential to the curve at the pole
(': 0 = %gives r = 0)

Similarly the radius vector 0 = 3ff also is tangential to the curve at the pole.
4
4.2.6 Example:
Trace the curve r - a cos 30 = 0 - - - -(1)
Solution:
F (r, -0) = r -acos30 == F (r,O)

Curve is symmetric about initial line


Line 0 =0 intersects the curve at (a, 0 )
Since Icos 301 ~ 1 , from (1), Irl ~ a .Curve completely lies

within the region of the circle r =a


r=O when 30=0 :.30=(2n+l)ff/2, n=0,1,2, .....

(i.e.) curve passes through the pole when 0 = (2n + 1)ff/6( n = O/oS)
These six lines are tangential to the curve at the pole

The part of the curve from 0 = to 0 to ff is AA, 0 .


6
Curvature and Curve Tracing 295

This has a reflection AA20 in the initial line. Thus AAIO A2A is a loop.
F (r,4n/3 -B) = r -acos( 47l' - 3B) = F (r,B)

The curve is symmetric about B = 27l'/3 OBIB is the part of the curve

from B = 27l'/2 to 27l'/3. This has a reflection OB2B in line 0 = 27l'/3

ThusOBI BB2 0 forms a loop. F(r, 8; -B ) = r -acos(87l' - 3B) = F(r,B)

Curve is symmetric about B = 4~

0= 1t/2

(a,O)
......... A2
..... ......... () :::
.... ..... 111t/6
....

(-a,1t/3)

Fig. 4.33

OCIC is the part of the curve from B = 7l'/6 to 7l'/3 and

OC2C is its reflection in B = 4~

:.OCpC20 forms a loop.


It can be seen that OCPC20 is a reflection of OBI B820 in the line B = 7l'
acos3B -1
tan = = -cot3B
-3asin3B 3
~ 00 when B ~ 0
:. The tangent at (a,O) to the curve is ..LIar to the initial line.
296 Engineering Mathematics - I

4.2.7 Example:

Trace the curve r2 - a 2 sec 2 f) =0

Solution:
The given curve can be written as F(r, 0) = r2 cos 2 f) - a 2 = 0 - - - -(1)

f) =0 gives r = a .Curve docs not pass through the pole.


F(-r,f) = F(r,f) :. Curve is symmetric about the pole

F(r,-f) =,.2 cos 2 (-f)-a 2 = F(r,f)


Curve is symmetric about the initial line f) =0
F(r,TC -0) = r2 COS
2
(TC -f)-a 2 = F(r,f):. Symmetric about f) = TC
2
2 2
Since cos 0 S; 1 (i.e.,) IcosOI S; 1 we get r2 2:: a

Curve does not exist in the region ,.2 < a 2

TC TC
l' --) 00 as f) --) - :.0 = - is an asymptote to the curve.
2 2

f) 0 TC TC TC TC
- - - -
6 4 3 2

l' a + 2a J2a 2a 00
-J}

dl' J 1 df) l'


r=- = a- sec- f) tan f) - = -) cos 2 f)cotf)
df) dr a-

tan=rdf) =cotf)=tan(TC -f)) :. =TC/2 when f)=O showing that


dr 2
the tangent ate a, 0) is ~/ar to the initial line .
Curvature and Curve Tracing 297

(-a,O) -\---~-~I-

Fig. 4.34
4.2.8 Example
Trace the curve ,.2 - a 2
cos 2(} = 0
Solution:
() = 0 gives r = a :. curve intersects initial line at (a,O)
,. = 0 gives cos 2fJ = 0 ~ B = 7r/4 pole lies on the curve

and () = 7r /4 are two tangents to the curve at the pole

Icos201 S 1 ~ IrI sa. Curve completely lies within the region of the circle I' =a

Curve is symmetric about the initial line.

F (-1',0) =(rr _a 2 cos20 = F (1',0) symmetric about the pole

F (r,7r-(})=r 2 -a 2 cos(27r-20)= F (1',0) symmetric about 0=7r/2

0 Jr/6 7r/4 7r/3 Jr/2 37r/4


0
a a/J2 0 imaginary 0
I'

Curve does not exist in the regions 7r/4 < B < 3Jr/4 and - 3Jr/4 < 0 < -Jr/4
298 Engineering Mathematics - I

The curve forms a loop bctween (a ,0) and (0,0) and has reflection in B = 1(/2

~ dO ~2
Diffcrentiating (1)2r- = -2a 2 sin20 tan =r- = 2
= -eot20
dO dl' a sip 20
= tan (1(/2 + 20)

0= 0 gives = 1(/2

Thus the tangents at 0 = 0 (ie) (a ,0 ) and ( - ( l ,0) are perpendicular to the initial
line.
y

Fig. 4.35
4.2.9 Example:
Trace the curve r - {a + beosB} = 0
Solution:
F (r, -0) = r - {a + beos ( -B)} = F (r, 0)
Curve is symmetric about the initial line.

Since leosBI ~ I we have IrI ~ a + b


The curve completely lies within the region of the circle. r = a+b
o =0 gives r=a+b :.curvemeets 0=0 in A(a+b,O)
o= 1( gives r = b curve meets B = 1( in B ( a - b, 0)
{l -
..
Curvature and Curve Tracing 299

Thus when a*- b curve meets the initial line at A and B

When a = b we get r = 0 curve passes through the pole.


Curve meets the line B = 1/2 at (a, n/ 2) .

Curve passes through the pole when () = cos--! (-{l/ b) (since r = 0)


Hence the tangent to the curve at the pole is the line B = cos"! ( -a/ b) which

(i) does not exist when a > b

(ii) is B = cos-!(-l)=n when a=band

(iii) exists when a < b

0 0 n/3 1/2 2n/3 n


--

r a+b a+b/2 a a-h/2 a-b


dB
tan = r - = a+hcosB .
showmg that tan = 00 when 0 = 0
dr -hsinB
:. = n /2 (ie) the tangent to the curve at (a + b, 0) is .l/llr to the initiallinc.

Further *- 0 for any value of 0 and hence no tangent to the curve is parallel to

the initial line

When a =h
l+cosO .
tan = . -cotB/2=tan(n/2+0/2).
smO
When 0 = n we get = n (ie) the initial line is a tangent to the curve at (0, n)

Wht1f\ a*- h we have tan ~ 00 as () ~ n (i.e.) The tangent to the curve at


(a - b, n) is also perpendicular to the initial line.
300 ____E_n--:9:::...ineerin9 Mathematics - I

I (a+b,O)
I (2a,0)
I
a>b a=b

Fig. 4.36 (i) Fig. 4.36 (ii)

a<b

Fig. 4.36 (iii)


4.2.10 Example:
Trace the curve r = a ( sin f) + 1/sin f)) ..... ( I)

This example illustrates that changing the coordinate system from polar to cartesian
facilitates the tracing.

(I) can be written as ,.1 = a(r sin 0 + ~


r SIn f)
1 ..... (2)

Substituting r cos f) = x and r sin f) = y (2) reduces to

....(3)

No constant term in (3) :. curve passes through origin

Lowest degree terms equated to zero gives 2/ + x = 0 => x yJii


2

The tangents at (0,0) are imaginary.


Curvature and Curve Tracing 301
--------------------=-----~------------------------------------

x = 0 in (3) gives y = 0,0,2 a


y-axis cuts the curve at (0,2 a) and (0,0) is an isolated point

Equating coefficient of the highest dt'gree terms in y to zero gives y = ()


(i.e.) x-axis is an asymptotes to the curve.

F (-x, y) = F (x, y) :. curve is symmetric about y-axis


Differentiating (3) W.r.t 'x',

dy 2x ( a - y) . (dY )
dx = 3/ +X2 -4ay .. dx (O,la) = 0

The tangent at (0, 2a) is parallel to x - axis

(0,2a) y = 2a

o x

Fig. 4.37

From (3) x = y J2a-


y-a
y

when y < a or when y > 2a ,clearly x is imaginary (i.e.,) The curve exists only
in the region a < y ~ 2a
Exercise 4.2
Trace the following curves

1. r 2 cos20=a
l
2. r=l+J2cosO 3. r=asin 2 0secO
4. r=3+2cosO 5. r=asin30 6. r=a(cosO+secO)
l
7. r=asin40 8. r=a(coso+----) 9. r2 =asin20
cosO
a2
)0. r = a(I-sinO) 11. r2 = - -
cos20
302 Engineering Mathematics - I

4.3.0 Tracing of curves when the equation is given in parametric form:


Suppose the equation of the curve is in the tonn
x = ./; (t) and y = ./; (t) ..... (1)

where t is the parameter. The study of the following points are useful for tracing the
curve
I. If for some value for I , say I) , .I; (t) = 0 and .t; (I) = (l
then the curve passes through the origin.
2. If.1; (I) is an odd function and .t; (I) is an even function

then the curve is symmetric about y-axis.

3. If J; (I) is even and J; (I) is odd

then the curve is symmetric about x - axis


4. Intercepts on X(Y)a.xis are obtained by solving .1;(1)=0 (.t;(t) =0)
respectively.
5. Greatest and least value of .I; (I) and .t; (I) give the region in which the curve
exists.
6. If It (t) or 1; (I) tends to 00 as I -) t) then t = t) is an asymptote to the curve.

7. (ddxY)I~II = 0 indicates that t = t) is a tangent to the curve parallel to the

x-axis.

8. (: ) -) 00 as I -) ') then I = I) is a tangent parallel to the y-axis.

9. If It(t+a)=j;(1) and .t;(t+a)=.t;(t) then the curve is periodic of period


a
10. Sometimes it may be convenient to transform the equation of the curve from
parametric to cartesian form for tracing the curve

4.3.1 Example:
Trace the curve x = at 2 ,y = 2at
Curvature and Curve Tracing 303

Solution:
Eliminating t from x and Y

l = 4a 2t 2 = 4a.at 2 = 4ax
y

~~+- __ ~ ______-+x
(0,0)
l = 4ax
---...;..-
Fig. 4.38
This is a parabola with vertex at origin y aXIs is a tangent to the curve at the
origin.

4.3.2 Example:
Trace the curve x = acos/,Y = bsint

Solution:
Eliminating t from both x and Y

(= J+( ; J~ 1 wh ich is an ellipse (standard fonn)

y (O,b)

---------+----~~----~--------~.x
(-a,O) (a,O)

(O,-b)

Fig. 4.39
4.3.3 Example:
3
1
Trace the curve x = (2 ,Y = t--
3
304 Engineering Mathematics - I
--------------------------------------
Solution:
Eliminating't'

2
y2 =t (I-t/{f =x(l-73'f =i(3-X)2
(i.e.,) 9i = X(3-X)2 -----(1)
y =0 gives x =O,x =3.
The curve passes through origin and again intersects x - axis at (3,0).

F(x,-y) = 9y2 -x(3 _X)2 = F(x,y)


The curve is symmetric about x - Q.'Cis and hence it forms a loop between (0,0)
(3-x)Fx
and (3,0) y = which shows that the curve does not exist for x < 0
3
(i.e.,) to the left of Y-axis
Lowest degree terms equated to zero gives x =0 (i.e.,) y-axis is a tangent to the
curve at origin.
Coefficient of x 3 is I (a constant) and coefficient of i is a constant.
Hence, there are no asymptotes parallel to either x or y-axis
Substituting, y = mx + c in (I) and rearranging
3 2
_x + x 2 (9m +6) + ........ = 0

fA (m) =-1 a constant, fA (m) =0 gives m =J{i


:. No oblique asymptotes exist

Fig. 4.40
Curvature and Curve Tracing 305
--------=------------------------------------------

dy =![l-X]
Ix 2 ~

Y 0:. The curve is extreme at x = 1


(d ) =
dx ,~I

elY)
( dx =+ ___ 1
(3,0) - J3
:. J3Y = (x - 3) are tangents to the curve at (3,0)
4.3.4 Example:
Trace the curve x = a(t + sin I), Y = a(l + cos t)

Solution:
For no value of f both x and Y simultaneously vanish

:. The curvc does not pass through the origin.


x is an odd function while y is even

:. The curve is symmetric about y -llxis

x(t) = 0 gives t =0 .Correspondingly x = 0 and y = 2a

dy =dYidt =_ llsint =-tany,;=tan(TC-t/)


dx dx/dt a(i+cost) .2 12
When 1=0 slope tan = tanTC ~ = TC

:. The tangent at (0,2a) is parallel to x - axis

t -TC _5% -~
0
~ 5% TC

X llTC
-a(~+ 5;) -a(; +1) 0 a(I+;) a(~+ 5:) (lTC

a(l+ ~)
y 0 2a a (/(1+~)

306 Engineering Mathematics-I

y(t) = 0 gives cosl = -1 ~ I = 7l",37l", ..... .


(i.e.,) 1= (2n + 1)7l" n=1,2,3, ....
corresponding x is a7l",3a7l", ....

Further x ~ 00 as I ~ 00 and IYI ~ 2a


:. The curve completely lies within the region 0 ~ Y ~ 2a

and is period of period 27l"

Fig. 4.41

4.3.5 Example:
Trace the curve x = a(t -sinl);y = a(l-cost)

Solution:
X(I) =0 only when 1=0

y(t) = 0 gives 1= 2n7l" n=0,1,2, ....


x(O) = 0 = y(O):. curve passes through origin.
Since Icos'l ~ I we have y 2 0

The curve is completely above x - axis and lies within the region 0 ~ y ~ 2a

(y = 2a when' = (2n + 1)7l",n = 0,1,2, .. )


The curve meets x - axis at x = 2an7l", n = 0,1,2, ....
y is an even function and x is odd
Curvature and Curve Tracing 307

:. curve is symmetric about y-axis

LI x(t)
t~oo
= 00 but LI y(t) is finite
l~cIJ

:. There are no asymptotes to the curve

tan = : = cot ~ = tan ( ; - ~)


Jr I
~=---
2 2
The tangents to the curve at t = 2nJr, n = 0,1,2, ....... are parallel to y-axis

The curve is periodic of period 21f

~------~------~--~--~--------~----~x
-4a1t -2a1t o 2a1t 4a1t

Fig. 4.42

4.3.6 Example:
Trace the curve x = aeos 3 e,y = bsin 3 e
Solution:

The functions cos 3 e and sin 3 e are periodic of period 2Jr .

Hence it is sufficient to trace the curve for one period.


For no value of e both x and y vanish. (0,0) does not lie on the curve

x( e) =0 gives e=~ ,
yeO) =0 gives e = O,Jr
308 Engineering Mathematics - I

:. The curve meets x - axis at x = a and y-axis at y = b

Since Icos
3
01 s:; 1 and Isin '01 s:; 1 we have Ixl s:; a, Iyl s:; b
e 0 Jr/6 Jr/3 Jr/2 2Jr/3 5Jr/6 Jr

3J3 -2J3a
x a --a -a
8 a/8 0 -a/8 8

y
1
-b 3J3 b ~J3 b b
-
0 8 8 h 8 8 0

dy b
tan = - = - - tan e
dx a
tan =0 when e = (2n+ l)Jr
(i.e.,) The tangent to the curve at these points is parallel to x - axis tan ~ 00
when

e ~
2n+l) Jr
(-2-
:. The tangent to the curve at these points is parallel to y-axis

It can be observed that for corresponding to x for some e there is -x for Jr - e


and corresponding to x for some e there is - y for - e .
:. The curve is symmetric about both the axes
(O,b)

o =...nE-_ _-+ __~3!Jii-(a,o)


(-a,O) 0=0

0= -n12
(O,-b)

Fig. 4.43
Curvature and Curve Tracing 309

4.3.7 Example:

Trace the curve x = a[ cose + ~ log tan ~ J,:I = asin e


2

Solution:

yeO) = 0 but x(O):f:- 0 . (0,0) docs not lie on the curve

yeO) = 0 gives e= 0 and lyllll'Lx =a


x is an even function and y is odd. So the curve is symmetric about the x - axis

Corresponding to each x( e) there is -x for 1C - e


:. Curve is also symmetric about y-axis

Since Isin el S; 1 we have IYI S; a.:. The curve entirely lies in the region

-(I S; Y S; a .

x( ~) = a [ cos ; + log tan : ] = a [0 + log 1] = 0

Similarly, x[ -;] = 0
Corresponding y values are y(; ) =a and y( -; ) =-1

Thus the curve intersects the y -axis at (0, a) and (0, -(I)
1 l

tan~ = <!vdx
a -sine+ - .~
=
[
J
sinti = cos' 0
acosO sinOcosO
= cot 0 = tan (1C _
2
e)
:.=1C_ e
2

Hence, =0 when e = ~ and = 1C when e = -~


310 Engineering Mathematics - I

(i .e.,) The tangents to the curve at e =:= 7r/2 are parallel to x - axis

Further e~ 00 or -00 according as e ~ 0 or 7r


.". X - axis is an asymptote to the curve.

(O,-a)

Fig. 4.44

4.3.8 Example:
3at 3al 2
Trace the curve x = - - , ,y = - - , a > 0
l+r' l+t'

Solution:

X
3
= (3a)3
- , .I,y
3 3
= (3a)3
-3 .1 6
l+r 1+1
3a )3
3 3
27a t
.". x 3 + y3 = ( __ 1\1 +/
3
) = ?
1+1
3 3
(l+t t
3at 3at 2
=3a'--3 ' - - 3 = 3axy
l+t l+t
Thus the Cartesian form of the curve is x +
3
i = 3axy

For tracing the curve see example 1.3

4.3.9 Example:
a(l-t 2 ) 2bl
Trace the curve x = y = - -2
1+ t 2 ' 1+ t
Curvature and Curve Tracing 311

Solution:

x
2
(1_12]2 i
a2 = 1+t 2 ' b2
2 2
Thus ~ + L -_ I ( see examp Ie 3.3 .L-'"')
2
a b

Exercise 40

I. x = aCt +sin t),y = a(l + cost)


2. x = a sin 21(l + cos 2t), Y = a cos 2t(l- cos 2t)
3. x = a[ cosO -log(l +cosO) ],y = asin 0
4. x = aU -sint),y = a(l-cost)

Exercise 3.1
I . An asymptote to the curve i (a + x) = x (3a - x) is
2

(i) x-a=O (ii) x+a =0 (iii) x-3a=O (iv) none of these

I ADS: (ii)
2. The curve a 2y2 =x2 ( a2 - Xl) is symmetric about

(i) x-axis (ii) y-axis (iii) both the axes (iv) none of these
I ADS: (iii) I
3. The curve x + 3
i = 3axy is symmetric about the line .............. .

[ADS: y = x)

4. If the tangents to the curve i = (x - a)( x - b)2 ,( b > a,) at the point x = bare

inclined at angle 0, tan 0= .............. .

[ADS: .jb-a )
5. The two tangents to the curve i = x 3 at the origin are real and distinct.
I ADS: false)
312 Engineering Mathematics - I

6. For the curve r = a sec B tan B the origin is a cusp.


[Ans: true)
7. The curve x = a (t + sin t), Y = a (1 - cos t) is symmetric about x-axis.

[Ans: false)
8. x - 2a = 0 is an asymptote to the curve l (2a - x) = x 3
[Ans:true)
5
Application of Integration to
Areas, Lengths, Volumes and Surface areas

5.1 LAGRANGE'S METHOD OF UNDETERMINED MULTIPLIERS


The definite integrals are useful in formulating important physical applications like
(i) lengths (ii) areas and (iii) volumes and surgace areas of sol ids otrevolution.
5.1.1 (a) The area bounded by a curve y = f(x} , the axis of x and the two ordinates x = a
and x = b is given by the definite integral.

S:ydx= J:f(x)dx

y = f(x)

. x'-~----!--- "'---=--x
o
y'
Fig.5.1.1(a)
Similarly the area bounded by a curve x = fry) the axis of y and the two abscissae
y = c and y = d is given by the definite integral.

S:xdy = S:j(y)dy
314 Engineering Mathematics - I

x' ---:,-+----------------,
o x
y'
Fig.5.1.1(b)
(b) If the equation of the curve is in prametric form,
then the area

=
f ll dx
y - dt or
f l
2 dy
x - dt
I, dt II dt
where tl' t2 are the limits of integration depending on the boundary of the curve.
(c) The area enclosed by the polar form of the curve r = f(B) and two radii vectors
(} = a, (} = f3,, is

= fPCd(}=~ fP[r(})Jd(} (area of sector OPQ =


1 2
- r d(})
a 2 2 a 2

o~~----------------
Fig.5.1.1(c)
5.1.2 Example
Find the total area within the curve c?y = c?x2 - x4
,
" Y ,,
Y = - x" " " "",
/ , / 'y =x
...........8
,,
A
x ' ---~---~~~~~~~-~x
,,
,,
,
,,
,
y' ,,

Fig. 5.1.2 --
Application of Integration to Areas, Lengths, Volumes and Surface Areas 315

Solution:
The curve has two equal loops between the lines x = a and x = -a and is symmetric
about x axis. The total area within the curve
= 4 (the area of the half of the loop i.e., 'OABO')
= 4 Laydx
ax 1~2--2
=4
fo -va
a
-x dx

=~x_l fa~2_x2j2(_2x}lx
a -2 0

3
( 2
_ -2\a -x 2 )-2 4( 2~-
- 3 =--\O-a J2
3
a
2 o

4 2
= 3a Sq.units
5.1.3 Example:
Find the complete area of the curve a 2y2 = :x3(2a - x)

y
x = 2a
B
x' x
0
(0, 0)
A(2a,0)

y'

Fig. 5.1.3
Solution:
The curve is symmetrical about x-axis it cuts the x-axis at the points 0(0,0), A(2a,0).
The curve consists of a loop lying between x = 0, x = 2a
The required area
=20ABO
2
= 2 fo ;dx
2a X 3/ 2.J2a - x
= 2f dx
o a
Substituting x = 2a sin2 ()
316 Engineering Mathematics - I

We have dx = 4as in e cose


x=O=>O=O
x=2a=>O=Jr/2

Area =~
a
Jll
0
2 (2a)12 sin 3eEa cose.4asine cose de

= 32a 2 J: 2 sin
4
e cos 2 e de

= 32 a 23.1.1 /2 2
--Jr = 1111 Sq. units
6.4.2
5.1.4 Example:
Find the area of the segment cutoff from the parabolay = 2x, by the straight line
y = 4x -1.
Solution:
The given curves are y = 2x ..... (i)
Y = 4x - 1 ..... (ii)

x'-----Ib-~_1_-=-------- x

y'

Fig. 5.1.4
Solving (i) and (ii) we get the points of intersection as

J.! -.!}
A(.!2'I}'Lll8' 2
The required area = area AOBA (shaded region)
= area CBEAOOC - area BOADOCB
= J ! {y + 1)dy- J
1
-124
1
-12
y2 dy (from equations (i) and (ii) respectively),
2

~ ~[~2 +yL -[Y:L


9
= 32 Sq. units
Application of Integration to Areas, Lengths, Volumes and Surface Areas 317

5.1.5 Example:
Find the area between the curve.xlY = a2(y - x 2) and its asymptotes.

y
B

x' ---j===::;>+-:;::::=::::f-..!..--- x

'y

Fig. 5.1.5
Solution:
The given equation can be written as

y2{a 2 _X2)= a 2x 2 => y2 = ~2X2 2


a- -x
The asymptotes to the curve are given by
a 2 _x 2 = O=> x::;:a
The curve is symmetric about both the axes and passes through the origin.
The shaded portion is the required area.
The complete area = 4(areaOAB in the first quadrant)
= 4 Laydx

=
4J" 12ax
0 2
dX=-2J" -2ax ,~dx
oi2 2)1/-
va -x ~ -x
12

= - 2a X
~2
1
2
- X ) a
10 = -4a[:2
-J a - x
2 Jl0I
2
= --4a[O - a] = 4a2 Sq.units.
5.1.6 Example:
Find the area bounded by the curves y = 9x, x 2 = 9y,
Solution:
Solving y = 9x and.xl = 9y
The points of intersection of the two curves are 0(0,0) A(9,9)
318 Engineering Mathematics - I

x'----+----~=F=-.:=--~-----x

Fig. 5.1.6
The required area = area OBACD
= area ODACO - area OBACO

= S:=oydX - S:=oydx
(parabola y = 9x) (parabola x 2 = 9y)
X2
= f 93 J";dx- f9 dx = 27 sq. units
009
5.1.7 Example:
asin 3 t .
Find the area bound by the cissoid x = asin 2t, y = and Its asymptote.
cost
Solution:
The equation of the curve is
asin 3 t
x = asin 2t, y = ---
cost

y
x=a

x,-----l..~;;;;;;;~~~--x
o

y'

Fig. 5.1.7

a 2 sin 6 t
y = cos 2 t a-x
Application of Integration to Areas, Lengths, Volumes and Surface Areas 319

The equation of the given curve is transformed into cartesian form. The curve is
symmetric about x-axis and x = a is the asymptote.
The required area = 2(shaded area)
1/2
=2Jao ydx =2J: _x_
a-x 0 ( )
' dx

Writing x = asin 2 B, dx=2asinBcosBdB

= 4a
2fl! 2Sin. 4ede = 4a 2.-.-.-
3 I n
042 2

4
5.1.8 Example:
Find the area included between the cycloid x = a(e - sinO), y = a(1 - cosO) and its
base.
Solution:
The equation ofthe curve is
x = a(B- sinO) y = a(1 - cosO)

x ' - - - - - - - - - : : ' I ' -.......~.........'-'-'~.........'---'-:::-...".----- x


e =0 : e = 2n
'y A(2an ,0)
Fig. 5.1.8
The required area = area OABO

= f02;, dx
= J2l! Y dx .dO
o dO
= f ;(1 -
0
2
cose )a(I - cose)de

= 4a 2f 2l! Sin. 4-de


e = 8a- fl!Sin 4-de
?e
o 2 0 2
320 Engineering Mathematics - I

=
?
8a-.2 f
0
",2
sin 4 tdt
()
(taking -=t)
2

= 371a2 Sq.units
5.1.9 Example:
Find the area between the curve x = a(O + sinO), y = a(1 - cosO) and x-axis.
Solution:
The required area
= 2 area OAB

= 2f"ydx .dO =2f"aQ-cosO)aQ+cosO)dO


o dO 0

"/2
=
2
2a .2
f
0 sin 2 0dO

.-------f------. A

x,---=--~~~~~~l--- x
o =-1t 0 0 =0 8
'y

Fig. 5.1.9
2 I 1t
= 4a .-.-
2 2
= mJ Sq. units
5.1.10 Example:
Find the complete area of the curve given by the equations.
x = acos3 fJ, y = bs in 3 ()
Application of Integration to Areas, Lengths, Volumes and Surface Areas 321

Y
8(0, b)

x'---~?-+=~----x
A(a, 0)

Y'

Fig. 5.1.10
Solution:
The required area
= 4 area OABO
=4Jaydx =4Jo y dx de
1t 2 de

= 4 I:'2bsin3B~3Bcos2BsinB)
1t/2 4 2
= 12ab J sin e cos e de

3 1 1 1f
= 12abx-.-.-.-
642 2
3
= 81fQb Sq. units.
5.1.11 Example:
Find the area of loop of the curve -? = a 2 cos2f)

e =1[/4

x'----~~--------~~------~~~~
e =0
8 x
c

Y'

Fig. 5.1.11
322 Engineering Mathematics - I

Solution:
The required area
= 4 areaOABO

= 2f"4~9 = f" 4r2d9


o 2 0

"!4 a2 ,,'4
= fo 2
a cos 29 d9 = -~in29]o
2
a2
2
5.1.12 Example:
Find the area of the curve r = a(l + cosO)
Solution:

B
c
() = 1t
A

Fig. 5.1.12
Required area
=2 . (area OABCO)
r2"
=2
f-d9
o 2
= f: a Q+cos9 )2d9
2

= J: a
2
4(cos %Jd~ 2

= 4a 2 f"o cos ~9
2
4

"/2
= 4a 2 f cos t .2dt
0
4
(where t = 0/2)

2 3 I 1r 3Jlll 2
= 8a ---=-- Sq. umts
422 2
Application of Integration to Areas, Lengths, Volumes and Surface Areas 323

5.1.13 Example:
Find the area of the portion included between the carbo ids , = a (l + cosB') and
, = a(l - cosB').
Solution:
Required area
= 4(OCBDO)

=4 [I 1t
0
2,2 d8
2
1

4-~-~r::---_~ r = aQ + cas8 )
A 8 =0

Fig. 5.1.13
= 2
I
1t / 2
0 a-V -cas8 )2d8
1 {,

= 2a 2 L1t/2a 2 Q- c~s8 )2d8


~!2 +~.~}= a2 (3n -8) Sq.units
2
2
= 2a { [8 - 2sin8]

5.1.14 Example:
Find the area afthe curve r2 = a2sin28.

8=~
4

8 =n/4
x

Fig. 5.1.14
324 Engineering Mathematics - I

Solution:
The total area of the curve = 2 area of one loop of the curve

= 2 . -1 fll .!2r 2de = f1l 2?


1
a- sin 2ede
2 0 0
2 2
= ~ [- cos28] ~ 2 =~ n+ 1]=a2 Sq . units
2 2
Exercise 5 (A)

1.
x
2
Find the whole area of the ellipse ~ + [;2 = 1
i
(Ans: 1t ab]
2. Find the whole area of the curve x 2(x 2 + .I) = a 2(x 2 - .I)
[Ans: cl(Jr - 2)J
3. Find the area of the curve a2x 2 = y(2a - y)
(Ans: Jra2 ]
4. Find the area bounded by the curve xy = 4el(2a - x) and its asymptote.
(Ans: 4Jra 2 )
5. Find the area included between .I = 4ax and y = mx

8a 2
(Ans: - 33 )
m
6. Find the area included between the parabolas .I = 4a(x + a) and
.I = 4b (b - x)

(Ans: ~(a + b}Fab )


3
7. Find the area common to the circle x 2 + y2 = 9 and parabola x2 = 8y

[ADS: "31 {r;:; .


2,,2 + 27 Sin -1(2J2i}
- 3- ) I

2
3JTa
8. Show that the area of the loop of the curve ely = r(2a - x)(x - a) is -8-
9. Find the area of the loop of the curve r = asin2B.
JTa2
[Ans: - ]
8
10. Find the area common to the circles r = aJ2. - 2acosB
[Ans: el(Jr - 1)]
Application of Integration to Areas, Lengths, Volumes and Surface Areas 325

5.2.1 Lengths of plane curves:


(a) The length of the arc of the curve y = fix) included between the points whose
abscissae are 'a' and 'b' is

s=J h (d)2 dx
I+~ or
<J dx

(b) The length of the arc of the curve x = f(y) included between the points whose
ordinates are 'c' and 'd' is

2
d dx
s=J 1+-
( dy )
dy
('

(c) The length of the arc of the curve x = fit), y = g(t) included between two points
whose parametric values are 'a' and '/3' is

(d) The length of the arc of the curve r = j(B) included between two points whose
vectorial angles are 8/ and 82 is
2

s= J- 9??

9,
r- +( r -
dr
de )
dt

(e) The length of the arc of the curve 8 = j(r) from r = r/ to r = r2 is given by

s= J,,-,. , (de )2 dr
1+ r dr

5.2.2 Example:
Find the complete length of the curve
x2(a2 - x 2) = 8a2 y
326 Engineering Mathematics - I

Solution:
y

x'------4--------lIf-------+------ x
A'(a,O) A(a, 0)

'y

Fig. 5.2.2
The curve is symmetrical about both the axis, one loop is formed in between x = 0
and x = a and another loop is formed in between x = 0 and x = - a.
The total length of the curve = 4(1ength OABO)

Total length = s = 4 f: I+( : J dx .... ( I)

The equation of the curve is


:x2(a2 - x 2 ) = 8a2y2
Differentiating with respect to 'x'

16a 2y dy =2xa2 _ 4x 3
dx
dy _ x(a 2 _2X2)
dx - 8a 2 y

dy )2
1+ ( dx =1+
x2(a2 _2X2
- -
r
64a 4/
Substituting for y2 from the equation of the curve
8a2y2 = x 2(a 2 - x 2 )
Application of Integration to Area!, Lengths, Volutnes and Surface Areas 327

9a 4 -12a 2 x 2 +4X4
8a 2{a 2 -x2)

.... (2)

= .J2 2a2 sin- I (I}=2ha. Jr


a . 2
s= .J271
5.2.3 Example:
Find the length of the loop of the curve
3ay2 = x(x - a)2
Solution:

8
x'-------*J.U.jtJ.LL~~+_-- X
o
(0,0)

'y

Fit. S.l.3
328 Engineering Mathematics - I

The curve is symmetrical about x - axis, the loop is formed between lines x = 0
and x = a, differentiating 3ay = x(x - a)2 w.r., to x.

6ay dy = x.2{x-a)+ {x-a)2


dx

dy = (x-aX3x-a)
dx 6ay

dy )2 (x-a)2(3x-a)2
1+ (- = 1 + -'---------''------'::--::-----"-- ... (1)
dx 36a 2y2
Substituting 3ay2 = x(x - a)2 in (I) we get

dy )2 {x-aY{3x-aY
1+ (- = I + -'---------''--'--------'--
dx 12ax{x-a)2

12ax+9x 2 +a 2 -6ax
12ax

1+ (:r = (3~2+;)2 .... (2)

The length of the loop of the curve

=2 r (3x+a) dx
o 2Fa-Fx

s= ~Ja3-Fx+ax-12dx
2,,3a 0

=
1 r:
r::;- L2x
32
+ 2ax
12] a
0
,,3a

= ~ [2a 3/ 2 + 2aFa]= 4~
,,3a ,,3
5.2.4 Example:
Find the perimeter of the loop of the curve 3ay = r(a - x)
Application of Integration to Areas, Lengths, Volumes and Surface Areas 329

Solution:

x'-------;~t..J.J,.t..J.J,."'f__:__-- x

Fig. 5.2.4
The curve is symmetrical about x-axis and the loop of the curve lies between x = 0
and x = a.
Perimeter of the curve = 20ABO

a (4a-3x)
s= 2
f o 2J3a.Ja-x
dx

s = _1- fa a+3{a -X~lx


afj 0 .Ja-x

= -1-[-2a{a-x) 2 -2{a-x)'2] ~
afj

= Ir:;- [0 + 2afa + 2a 3!2]= 4a/ fj


a-v3
5.2.5 Example:
Find the length of the arc of the parabola y2 = 4ax cut off by the line 3y = 8x.
330 Engineering Mathematics - I

Solution:

x'-----~!OII"I'!J ...------x

Y'

Fit. ,.2.6
Solving the equations of the J>araboLa anct the fine gives the points of intersections

0(0,0) and A (~: ' 3; )


y=4ax

2ydy =4a
dx
dy 2a
-=-
dx y

1+ (dy)2
dx
=1+L
40 2

The length of the arc OBA of the

s=

. a

= _1 [y ~y2 +4a2+ 40 log~~~y24a2 }T2


2

2a 2 2.. Jo
= a[~~ + IOg2]
Application of Integration to Areas, Lengths, Volumes and Surface Areas 331

5.2.6 Example:
Find the length of an arc of the cycloid x = a(t - sin t), y = a( I - cos t).
Solution:
y

------~o~--~a-n--~~a~n----A~------x
(2a1t,0 )

Fig. 5.2.6

dx = a(l- cost), dy = asint


dt dt

(~~J +(~J =~2Q-costY+a2sin2t] 1/2


= 2asint/2
The length of an arc of the cycloid

= 2 Io" 2asin 7i dt
= 4a[- 2cost/2] ~
= 8a

5.2.7 Example:
Find the total length of the curve
X2/3 y2/3

a 2/ 3 + b 2/ 3 =I
332 Engineering Mathematics - I

Solution:

y
B(O,b)

A(a,O)
x'-----E:-~-~------x

B'(O,-b)
y'

Fig. 5.2.7
The parametric form of the curve is
x = acos 3 (J, y = bsin3 0
The total length of the curve = 4. length OABO

s=4
f1t2[(-3asinecos2e)+~bsin2ecoseJ
0
2 :\2J12
de

Substituting
clcos 2o + b 2sin 2 0 = z2
and (b 2 - a 2) 2sinO cosO dO = 2z dz

h z 12 Z3/ h
s = 12 fz. ( 2
a ,b -a
2) dz = 2 2
b -a 3 a

~ 4 ~ (b
2 2
= 3 _ a3 )= 4(a + b + ab)
b--a- a+b
5.2.8 Example:
Find the length of the arc of the curve given by x = asin21(J + cos21),
y = acos2t(J-cos2t) measured from the origin to any point.
Application of Integration to Areas, Lengths, Volumes and Surface Areas 333

Solution:
x = asin2t(1 +cos2t)
dx
dt = 2acos2t(1 + cos2t) + asill2t( -2sin2t)

dx
dt = 2a(cos2t + cos4t)
dx
dt = 4acos3tcost .... ( I)
and y = acos2t(1 - cos2t)
dy
dl = - 2asin2t(1 - cos2t) + acos2t(2sin2t)

dy
dl = 2a(sin4t - sin2t) = 4acos3tsint .... (2)
The length of the curve from origin to any point

Using (I) and (2)


L (~; r r+(; dt

s= f~ J(4a cos3/COS/)2 + (4acos 31 sin 1Ydl


s = f'4a cos 3tdt = sin 3/11
lIa
0 3 0

4a . 3
s= -Sin I
3
5.2.9 Example:
Find the perimeter of the cordioid r = a(1 + cosO).
Solution:

e = 1t 0=0
------~~------~-----------x

Fig. 5.2.9
334 Engineering Mathematics - I

dr
r = a( I + cos 0) :=) dB = -a sin 0
The perimeter of the cordiod
=20ABO

= 2 fox 1'2 +(~ J dO

= 2f" Ja 2 Q+ COSO)2 + (- asinO )2dO


o
= 2 L" J2a 2
Q+cosO )dO

= 2 f"o 4a COS2~2 dO
2

x 0
= 2
f 2a cos-dO
o ,..,
L

= 4a.2.sin-
B/
20
= 8a
5.2.10 Example:
Find the perimeter of the curve r = 2acosB.
Solution:
The equation of the curve is r = 2acosB.
It is a circle passing through pole whose centre is on the initial line at a distance
8/2 from pole.

0=0

Fig. 5.2.10
Application of Integration to Areas, Lengths, Volumes and Surface Areas 335

The perimeter of the curve


=20ABO

=2J n'2 r2+(dr)2dB


o dB

= 2 f: ~(211~-;;~OJ ~-(-
12
2asinB)1 dO

J(
= 4a-
2
= 2aJ(
5.2.11 Example
Find the length of the arc of the parabola Ijr = 1 -t cosOcut off by its latus rectum.
Solution:
LSL' = 21 is the latus rectum of the parabola.

Fig. 5.2.11
The length of the arc L' AL = 2AL

n
(dr)2 dB
=2
Io
;2 1
r+ -
(IB
Equations of the curve is Ijr =1 + cosOc
logr = logl + log(l + cosO)
~ dr = 0 _ - sin 0
rdO l+cosO
336 Engineering Mathematics - I

dr
-=rtanB/2
dB
1t/2 ~ 2
The length of the arc = 2
f 0
? ?
r + r- tan-9/2d9
1t/2
= 2
f rsec9j2d9
0

1t2 I
= 2
fo 1+ cos9 .sec9j2d9
1t 2 /
= 2
fo 2cos 9 2 .sec9j2d9 2

1t!29 2
= I
fo 2
sec-.sec 9j2d9

= / L1t/2 ~I + tan 2 9j2 sec 2 9j2d9


writing tan Bj2 = t

I
sec 2 B/2.-dB = dt
2

= 2{~F2 +~log(1 +F2)]

= l[ F2 + log(1 + F2)]
Exercise - 5(8)
1. Find the length of the arc of the parabola y = 4ax cut off by its latus rectum.

[ADS: 2a[F2+log(l+F2)]}

2. Find the perimeter of the loop of the curve 9ay = (x - 2a}(x - 5ai

(ADS: 4.fja J
Application of Integration to Areas, Lengths, Volumes and Surface Areas 337

3. Find the length of the arc of the parabola x2 = 4ay from vertex to one extremity of the
latus rectum.
[Ans: a[ 12 +Iog(l + 12)]J
4. Find the length of an arc of the parabola y = x2 measured from the vertex.

[Ans: ~Jl+4x2 +-.!.-sin- I {2x)]


2 4
5. Find that the length of the arc of the curve y = log tanh (x/2) from x = 1 to x = 2

2
e +
[Ans: log ( -e- ]
I)
6. Find the length of the arc of the curve y = x(2 - x) as x varies from 0 to 2.

[Ans: -.!.-log(2 +
2
J5)+ J5 ]
7. Find the length of the curve x = a(B + sinO) y = a(l- cosO)
[Ans: 8a)
8. Find the length of the arc of the curve x = eOsinB, y = eOcosB from B = 0 to
B = 1[/2

9. Prove that the loop of the curve


(3
X = (2, Y = (- - is of length 4fj
3
10. Find the perimeter of the cardioid r = a(l- cosO)
[Ans: 8a)
11. Show that the arc of the upper half of the curve r = a(l - cosO) is bisected by
e= 21[/3

5.3.1 Volume and Surface of solids of revolution:


(a) Volume of the solid generated by the revolution of the area bounded by the curve

y = f(x), the x-axis, the ordinates x == a and x == b about the x-axis is J:1t)12 dx.

(b) Volume of the solid obtained by the revolution of the curve x = fry), the y-axis the

abscissae y = c and y = d about the y-axis Ld 1tX


2
dx.
338 Engineering Mathematics - I

(c) Volume of the solid obtained by the revolution of the curve x = }(t), y = $(t) about

x-axis is f '2 ny2 dx dl = f '2 n ~ Cr)r dfCr ).dl


'] dt '] dt
(d) Volume of the solid obtained by the revolution of the curve x = j{t), y = i...t) about

y-axis is f'21t\"2 dy dl = f'2n[f'Cr)Y d$Cr).dl


'] dt" dl
(e) Volume ofthe solid obtained by the revolution ofthe cure r = j{ fJ) about the initial line
IS

02 f. . f\)2 d{rcose )d0


=
S n\"sIno
OJ de
.

(f) Volume of the solid obtained by the revolutuon of the curve r = j{fJ) about the line
perpendicular to initial line

=
f rrx- dy = 12 n \" cose )2 -"-.de
h?
a 81
dv
dO
f.

82 f. 0)2 d{rsine),lO
=
f
0]
n \" COSo
de
.U'

(g) The volume of the solid generated by the revolution about the initial line (x-axis) of
the area bounded by the curve r = j{ fJ) with the radii vectors B = a, B = {3 is

I
O=~ 2nr 3 SIlIO
'_0 d{\
= - 0
O=a 3
(h) The volume of the solid generated by the revolution of the area about the line
B = ni2 (y-axis) of the area bounded by the curve r = f(e) with the radii vectors
B=a, B={3is
o=~ 2n
=
I -r3 cos OdO
8=a 3

5.3.2 Example:
Find the volume of the solid formed by the revolution of the loop of the curve
y(a + x) = x 2(a - x) about x-axis.
Application of Integration to Areas, Lengths, Volumes and Surface Areas 339

Solution:
y = 0 gives x = 0, a
:. The loop is formed between x = 0 and x = a x = - a is the asymptote to the curve.

y
x=-a

Fig. 5.3.2
:. The volume formed by the revolution of the loop about x-axis

(Jr x2 (a-x)l
= 1( ex
o a+x

~ 1{- ~ + ax' -2a'x+2a'log(a+x)I

= 21(a 3 [ log 2a - log a -1 ]

3
= 2Jra [IOg2-1] cubic units.

5.3.3 Example:
Find the volume of the solid generated by revolving the curve xy = 4(2 - x) about
y-axis.
340 Engineering Mathematics - I

Solution:
y

x' -------,o..-+----f.,-,,;-,~O)r- x

Fig. 5.3.3
The given curve can be written as

xcY+4)=8

The volume of the solid obtained by revolving the given curve about y-axis

64
J y+4 )2
OC!

= 2n (~ dy
o

Substitutingy = tanO ~ dy = 2sec 2 OdO


dO

1t / 2
= 8n f 0 I +cos2e de

1 ] 1t 2
= 8n [ e +2 sin28 0

= 4n2 cubic units.


5.3.4 Example:
The part of the parabola y2 = 4ax cut off by the latus rectum revolves about the
tangent at the vertex. Find the volume of the reel thus generated.
Application of Integration to Areas, Lengths, Volumes and Surface Areas 341

Solution:
y
L(a,2a)

S
x'------------+-M+--+-------------x

J?--+--4.I L'

y'

Fig. 5.3.4
Volume of the reel generated

=2fo 211

[ '}Y
1[-
y-
4a

Jf Y-1 ]2<l
= 8a 2 [ 5 0

4Jfa 3
= -- cubic units
5
5.3.5 Example:
Find the volume ofa sphere of radius 'a'.
Solution:
Equation of the circle of radius 'a' is x 2 + Y = a2 .
y

x'------------~~~~~~~~---------------x
B A
(-a,O) (-a,D)

y'

Fig 5.3.5
342 Engineering Mathematics - I

By revolving this circle about x-axis we get a sphere of radius 'a'.


a

f
1
Volume of the sphere = ~"rry-dx

= rr
f ll (2
~11 \q - x-' ) dx

= "34 JlU 3 cubic. Ul1lts.


5.3.6 Example:
Find the volume of the solid generated by the revolution of the curve x = (l em;3 (),
y 3
= bsin () about x-axis.

Solution:
y

B(D,b)

x' x
(a,D)

B'(O,-b)
'y

Fig. 5.3.6
The volume of the solid generated by the revolution of the curve x = aeos 3 (),

y = hsin3 () ((i.e.,) x: ',33 + <~: = 1) IS


a- .h-
dx
=2fO 21tJ' de
7t
.dB 2

= 2fO rr(bsin e y~3acos2esine )de


3
11/2

32JlUh 2
- - - cubic units.
105
Application of Integration to Areas, Lengths, Volumes and Surface Areas 343

5.3.7 Example:
Find the volume of the solid fonned by the revolution of x = a( e- Sillf/),
y = a(l - cosf/) about its base.

Solution:
y
o = n/2

Fig. 5.3.7
Volume obtained by the revolution of the area OABO about the base (x-axis).

O=21t dx(10.
= n
f
()=o
y2-
dO
3f8=21t1J )~
= na 8=0 ~ - cosB dB

= na"
3
.8 J
0=0
21t
2sin 6 O/2dO

(e12 = t => dB = 2dl)

= 16na3.2 J1t 2 sin6 tdt = 32u3n(6 -I 6 - 36 - 5 .n I 2)


o 66-26-4
= 5,(2a3 cubic units.
5.3.8 Example:
Find the volume generated by revolving the curve r = a( 1 - cosq) about the initial
line.
344 Engineering Mathematics - I

Solution:

O=~
2

o = It
A

Fig. 5.3.8
The volume generated by the curve

= 2n f1t r3 sin8 d8
3 0

= 2n f1t a Q-cas8)1 sin8 d8


3
3 0

3
2na J1t Q-cas8
1- cosO = t
sin OdfJ = dt
1
= --
3 0
ysin8 18 [
and():O~ 2

8Jfa 3
= - - cubic units.
3

5.3.9 Exam'pre:
Find the volume orthe solid formed by the revolution of the curve r = a + hcosO
(a<b) about the initial line.
Solution:

Volume =2n
- f1t r3 sin8d8
3 0

= ~
')n f1t {,a+hcos8 )3 sin8d8
3 0

(a+bcOS()=t ~ sint1f() = - ~ ,t :a+b ~ a-b)


Application of Integration to Areas, Lengths, Volumes and Surface Areas 345

= 21t
3
J U

u+h
-
h
p(_ dt)
b

= - 21t SU-h t3d!


3b u+b

=- ~~[,;[
= ~[(a+bt -(a-b)4]
6b

41l'Q(2 2)
= -3-\a + b cubic units.

5.3.10 Example:
Find the volume of the solid generated by revolving the lemniscate? = a2 c:os2B
tr
about the line B= 2
Solution:

Fig. 5.3.10
The curve is symmetric about q = tr/2

Volume = 23. 1t I1t 4,3 case de


3 0

4 1t 3
= ~ S4(a 2COS2e)2 cosede
3 0

= i3 1ta 3
I1t4 (cos2e l2 case de
0
346 Engineering Mathematics - I

(taking .J2 sinO = sint => .J2 cos(}dO = cos/dt)

- 2.J27lZJ3 [4--I
-- 7C]
4 --3 -
3 4 4-22

.J27C 2a3
- - - cubic units.
8

Exercise - 5(C)

1. Find the volume of the solid generated by revolving the clips :: x ~: = I


(a) About the major axis

(b) About the minor axis


4 2
[ADS: 37lZJ b J
2. The curve yea + x) = r(3a - x) revolves about the x-axis. Find the volume generated
by the curve.
[ADS: ~(8Iog2 - 3)J
3. Show that the volume of the solid generated by the revolution of the curve (a -x)y

7C 2a3
= a2x, a~out its asymptote is -2-.
4. Find the volume of the solid generated by the revolution ofthe curve y(a2 + x2) = a3
about the asymptote
Application of Integration to Areas, Lengths, Volumes and Surface Areas 347

5. Find the volume of the solid obtained by revolving the loop of the curve
1
a 2 == x 2(2a - x)(2a - x)(x - a) about x-axis.

(Ans:
60
6. Find the vloume generated by the portion of the arc y == JI + x" lying between
x ~ 0 and x = 4 as it revolves about the x-axis.
(Ans: 767[/3)

7. Show that the volume of the solid generated by the revolution of the cycloid

. . . 3 , 1 87['
x == a( 8 + SinO), Y == aU -- cosO) about the y-axIs
IS - 7[" l - -
2 3
8. Find the volume of the solid generated y revolving the cycloid x = a(8 + sinO),
y = aU + cosO) about its base.
IAns: 5~l31
9. Find the volume of the solid generated by revolving r = a2cos28about initial line.

(Ans:
7[(1' ( h + 1)-"\/2h] I
Ii rl3log"\/2
12
10. Find the volume of the solid generated by revolving r = acos38 between 8 == -7[/6
and e == 7[/6 about the initial line.

197[(1'
(Ans: 968 )

5.4.1 Area of the Surface of revolution


(a) Surface area of the solid generated by the revolution about the x-axis of the area
bounded by the curves y = fix), the x-axis the ordinates x = a, x = b is

h
21ty Is ==
Ii> ds
21ty - Ix
I X=Q X=U dx

(b) Surface area of the solid generated by the revolution about y-axis of the area bounded
by the curve y = fix), the y-axis and the abscissae y = c, y == I is

I
Y=J 21tX cis == IJ
X=Q Y={
cis
21tX -
dy
(~V
[ d~==
dy
1+(dX)2]
dy
348 Engineering Mathematics - I

(c) The parametric form is

f
'=/2

/=/1
ds
.21ty-dt
dt [ d~
dt
== (dX)2 + ({~y)2l
dt dt

(d) The polar form is

I - 21trsine (dS-de )de


O?
91

5.4.2 Example:
? '0
Find the surface area of solid generated by revolving the curve x-i-' + y'? '3 == a-I?',
about x-axis.
Solution:
The parametric form of the curve is x = acos3 e, y = asin 3 B.

Y
B(O,a)

x' x
A'(-a,O)

8'(0,-a)
Y'

Fig. 5.4.2
The surface area of the solid due to revolution about x-axis
== 2(surface area of the solid generated by revolving an arc in the first
quadrant of the astroid is)
" 21ty-.de
ds
f de
2
= 2

41t f Y - ) +(2) .de


?
dx -
lt/2 d
=
de' de (

asin 3 e ~(\.3acos 20 sine )2 +( 3asin 2 e cose )2de


= 41t
I0
It/2
Application of Integration to Areas, Lengths, Volumes and Surface Areas 349

"n
= 12na 2
J
0' - sin-le cosO dO

ffn
2sinSe - 12JlU 2
= 12JlU [ - 5 - j0 = - 5 - Sq. units.

5.4.3 Example:
Find the area of the surface formed by the revolution of the ellipse x2 + 4y = 16
about its:
(a) major axis (b) minor axis
Solution:
2 2
Equation of the ellipse is ~ +L = I
16 4
ely x
dx 4y

16y2 +X2

16/
(a) Surface area formed by the revolution about major axis
-l ds
= 2
o J2ny-dx
dx
350 Engineering Mathematics - I

= .Jl[~(~ r -x' + *
~: Sin-{ )I
=
Jl]
8"[1 + 4"9 Sq. units.

(b) Surface area formed by the revolution about minor axis

= 41t L2 J Xl + 16yl dy

= 41t J: J16-4 y 2 +16y dy


2

= 8Jl. s: (1 r + y' dy

o
Application of Integration to Areas, Lengths, Volumes and Surface Areas 351

5.4.4 Example:
Find the surface area formed by revolving cycloid x = a(B+ sinO),y = a = (/ - cosO)
about the tangent at the vertex.

Solution:

e =-1[
y e =-1[

a1[ a1[

0=0
x' - - -.........-.....;~"""'"--..I---- X

y'

Fig. 5.4.4
Surface area required

= 2(surface area generated by revolving the arc in the first quadrant about OX)
x ds
= 2
Io
21ty- dB
de
= 4 IX (dX)2 +(dy)2 de
II 1t)' de de

=161t a-. IX sm-


. -cos
e -e de
ry ?

o 2 2

(%=t => dB = 2dt)


= 321ta- ry I 0
X/2
sin 2 t cost dt

32JlU 2
= -- Sq. units.
3
352 Engineering Mathematics - I

5.4.5 Example:
Find the area of the surface of reel thus generated by revolution of the part of the
y
parabola = rax bounded by the latus rectum about the tangent at the vertex.
Solution:
y

x'------+-~-+_------ x

Fig. 5.4.5
y = 4ax

dy 2a
dx y

a ds
Surface area = 2
fo
21tX-dx
dx

d
=2fa 21tX 1+.1:'. )2 dx
o ( dx

= 4n fo" x.J x 2 + ax dx
a

= 4n J:[(x+~r -(~rl2dx
Application of Integration to Areas, Lengths, Volumes and Surface Areas 353

= 4lT

o
2
= JZa [3J2 -log(J2 + 1)] Sq.units.

5.4.6 Example:
Find the area of the surface of revolution formed by revolving the curve r = 2acosB
about the initialline~
Solution:
Equation of the circle is
r = 2acosB

dr= -2
- ' B
asm
dB
y

x'-------~I-----
o ....--x
y'

Fig. 5.4.6

The surface area = J" 21ty -dO dO


o
2 ds

2
= L lIi2 dr
21trsinO r2 + ( dO )
dO

= J:/ 21trsinOJ4a 2cas 0 + 4a sin 0 dO


2
2 2 2

"/2
= 81ta J sinO cosO dO
2
0

= 4mr Sq. units


354 Engineering Mathematics - I

5.4.7 Example:
Find the surface area of the solid formed by the revolution of the cardioid
r= a(J + cosO) about the initial line.

Solution:
Equation of the curve r = a(J + cosO)

dr . B
-=-asm
dB

0=0

Fig. 5.4.7

The surface area = Io"2ny-de


de
ds

= 2n L" rsine r2 +(* J


de

= 2n I: 2
rSineJa (i +cose y +a 2sin 2e de
= L" a(i + cose)sine .Ja 2(i + cose)2 + a
2n 2
sin 2e de

= 16na I" COS3~ sin ~ cos ~ de


2
o 2 2 2

= 16na I" cos "2


2 e sm"2
0
. e d8 4

= [25 2eJ"
16a 2n - - cos 5 -
0

32mi
= -- Sq. units
5
Application of Integration to Areas, Lengths, Volumes and Surface Areas 355

5.4.8 Example:
Find the surface of the solid generated by revolving the lemniscate,.2 = a 2cos2B
about the initial line.
Solution:

Fig. 5.4.8

Equation of the curve is ,.2 = a 2cos2B


" ds
fo 21ty-dO
4
Surface area = 2
dO
2
41t In 11/4 dr
rsinO r2 + ( dO )
dO

1[/4 ~ a4 ~
= 41t
f 0 rsinO r- +-~ sm-20 dO
r-

= 41ta 2 [_ cosO ] ~ 4
356 Engineering Mathematics - I

Exercise - 5(0)
? ?

I. Find the total area of the surface obtained by revolving the ellipse x~ + y~ = 1 about
a- b-
its major axis.

2. Find the surface ofa sphere of radius 'a'.


(Ans: 4Jlu 2 ]
3. Find the surface of the solid generated by revolving the arc of the parabolay = 4ax
bounded by its latus rectum about x-axis.

(Ans: ~7lU2(2J2 -I)J


3
4. Find the area of the surface generated by the revolution of the cycloid x = a(t - sint),
y = a(l - cost) about x-axis.
64 2
[Ans: 3 7m )

5. Find the area of the surface of the solid formed by the revolution of the cardiod
r = a(l - cosO) about the initial line.
32 2
(Ans: SJl"a )
6. The lemniscate? = a2cos2B revolves about a tangent at the pole. Show that the
surface area generated is 47lU2

5.5.1 Double and triple integrals:


Letj(x,y) be a continuous and single valued function of x and y within a region
R bounded by a closed curve 'c' and upon the boundary c. Let the region R be
subdi vi doo in rrty mrrtne- into n subregi ons of areas 8R\, 8R2 .... 8Rn
Let (x" y,) be any point in the subregion of area 8Ri .
Consider the sum

The limit of this sum as n ~ 00 (i == 1,2, .... ) is defined as the double integral ofj(x,y)
over the region R and is written as
fff(x,y}iA
Application of Integration to Areas, Lengths, Volumes and Surface Areas 357

Hf(x,y}ixdy
II
= Lt
n~oo ,=\
I f(x" yJ) R,
(a) Suppose the region R is described by the inequalities
c ::::; y ::::; d and g(y) ::::; x ::::; heY)

y
y=d
...--_ _.,....::--....f-

x = g(y) X = h(y)

y-c
x'---------+---------
o x

Y'
Fig. 5.5.1 (a)
Then
t ="
ff f(x,y )dydx=Jff(x,y )dxdy = f fX=II(Y)f(r,y)dXdy
x=g(y)
II II Y~

(b) Jfthe region R is described by inequalities


a ::::; x ::::; band g, (x) ::::; y ::::; h,(x)
y

x ' - - - - -.......
oof----I----""----x
y'

Fig.5.5.1(b)
x=h y=lI\ (.)

Then ff r(x,y}ixdy = fff(x,y}iydx = f f y=g\ (.) f(x,y)dx.dy


II II x=a

(c) If the region R is bounded by the lines x = a, x = b, y = c, y = d (rectangle)


h " " h
Then Hf(x,y}dxdy = f ff(x,y}dydx= f ff(x, y}ix dy
II a c ( a

Note: The order of integration is immaterial for constant limits


358 Engineering Mathematics - I

5.5.2 Example:
I 2
2
Evaluate I I{x + y2 }Ix dy
o I

Solution:

I
8 2dy
=
03 I I
-+2y 2 ---y
3

3
Y7 7 I
il
2
Y +- dy=-+-yl
o
3 J
3 3 0

1 7 8
= -+-=-
333
5.5.3 Example:
4 K-~
Evaluate I Ixydydx
o
Solution:
4 J4-x 4 I'=J4-x

I fxydydx = f f [xydy] dx
I 0 x=l 0

4 y=K-~

f fxydy.dx
x=l 0

2
= 4f~(4 -X}tx = 4x _~14 9
2 4 6 I 2
x=l
359
Application of Integration to Areas, Lengths, Volumes and Surface Are."'''

5.5.4 Example:
1 2- r

Evaluate J Jx dXdY
o J~'
2

Solution:

v=1

J1(2- Y )' -l(J.i~)\~v


y~O

= -1
3
J8 - -
)'=1
V1 v2
-12)1 + 6- --')
3
V- l IV
~
\'~O

= 1[S-1-6+2-lJ
67
60
5.5.5 Example:
Find the value of Hxy(x+ y}/xdy taken over the region enclosed by the curves

y = x and y = x2.
Solution: y
y=x

x' _ _ _ _ _ ~~:-:.....----- x

y'

Fig. 5.5.5
360 Engineering Mathematics - I

7
R is the rogion bouuded by the curves y = x and J' = x~

1= ff'"y(x + y')dxcly
II

x5 x5 x7 x8 I
= 10+]5-14- 24 10
I I I 1 3
= -+----- =-
10 15 14 24 56
5.5.6 Example:

Evaluate IfR
A 1- Y
dx{1y where A is the area in the positive quadrant of the circle

x2 + y2 = 1.
Solution:
y

x'----------~r-~~~~~~~------x
8(1,0)

y'

Fig. 5.5.6
Application of Integration to Areas, Lengths, Volumes and Surface Areas 361

_I ~ 1- (2

= - ofx[~ -/'2-~--] () Ix
I I
= -- fX{I[-Q-X 2)]2 -I}dx
o
I

= - f{x x}It
2
-
o

-x 3 X-J ]1
- --+-
- [ 3 2 ()

-I 1 1
= -+-=-
3 2 6
5.5.7 Example:
tlHhY
Evaluate ffe dt dy over the triangle x = 0, y = 0, ax + by = I

Solution:

x:: : Ol----t---"
x'------------~----~~----~~----x
~ AU,a)
Fig. 5.5.7
362 Engineering Mathematics - I

1 = ffea<+hY dx dy

1= I I'"
,=0
lY: '-S-f:/\
y=o
e j
+hI' I
-(Y(X
I

= _
I -;-R~ e ,(I-a<) _e
llX+) -----
h lLt
1dx
+O
bo

= ~[.:.
b a a
-.:. - 0 + ~l
a

ab
5.5.8 Example:

Evaluate ffydxdy where R is the region bounded by the parabolas y = 4x and


R

xl = 4y.
Application of Integration to Areas, Lengths, Volumes and Surface Areas 363

Solution:
y

x'--------~~--~------x

y'

Fig. 5.5.8
Solving the curves i' = 4x, x 2 = -/y

X2 )2
( - 4 =4x
x= O,x= 4
(0,0), (4,4)

1= ffydxdy
11

48
5
364 Engineering Mathematics - I

5.5.9 Example:

Evaluate ff(f; - y2 }iydx where R is a triangle with vertices at (0, 0), (10, )),
II

(I, I).

Solution:

=1
8(1.1) Y
,....-.,------""!:II... A(10.1)
x ____________ ________________---------x
~

0(0.0)

y'

Fig. 5.5.9

The triangle OAB is within the limits y ~ x ~ lOy and 0 ~ y ~)

I x~IOy

ff(J~ - y dydx) =
II
2
J J(FxY -
y=o x=y
y2)dxdy

JelY ~~~y- /y
I [ 3 ]IOY
=
o y Y
Application of Integration to Areas, Lengths, Volumes and Surface Areas 365

Exercise -: 5(E)
I. Evaluate the following double integrals:
2 2

1. f f(x 2 + y2 ~xdy
0 0

32
IAns: - I
3
3 x+2

2. f f~vdx
-I x2

8
IAns: -I
3

3.
} Jdxdy
J 4
J

1 3 (x + y)-
25
IAns: log-I
24

II ~1I2_x2

4. f fidydx
o 0

25(/5
(Ans: -15- 1

II h

5. f f(x 2+ y2 )ixdy
o 0

ah (J
IAns: 3\U- + h- ]
J)

3Jru 4
[Ans: --I
4
366 Engineering Mathematics - I

a 2a-x

8. f fxy{O;dx
o );2

3a 4
[Ans: - ]
8
J J

9. Find the value of ffxydxdy taken over the positive quadrant of the ellipse ~: + ;~ = I

a Ja 2 -y2
10. f f ~a2 _x 2 - 2
y dxdy
o 0
3
7fa
(Ans: - ]
6
x=1 y=x y

II. f f eX dxdy
x=o y=O

e-\
[Ans: - )
2

12.

[Ans: 7f log(.J2 + 1)1


4

13. j fSin(x+ y)dxdy


x 0
o
[Ans: I]
Application of Integration to Areas, Lengths, Volumes and Surface Areas 367

14. Evaluate H-,ydxcZV where R is the quadrant of the circle x 2 + y2 = {/2 when

x~O,y~O.

{/4
[Ans: -I
8

15. Evaluate Hxydx((v whose R is the region bounded by x-axis, ordinatex ~ 2a and the
?
curve x- = .Jay.
a4
[Ans: 3 1

16. Evaluate H(x + )')d'({~}' whose R is the triangular region bounded by y = 2x, y = ~
/I
and y = 3 ~ x.

17. Evaluate Hxydx(~Y where R is the triangular region with vertices A(~6, 2), B(~ 1,3)
and C(9, ~7).

~1025
[Ans: ----:n- 1
18. Evaluate Hxydn'y over the region in the positive quadrant for which x + y ~ 1

I
[Ans: 241
5.5.10 Change of variables:
The variables x, y in Hf(x, y ~/xdy are changed to It, v with the help of the relations
II

x = j(u, v), y = filt, v) then the double integral is transferred into

fIt!; (11, v)f2 (It, v)]!JI}illdv


1/'

dx dx
du elv
where J = dy cZY and 'R' is the region in the uv plane corresponding to the region
du dv
R in the xy plane.
368 Engineering Mathematics - I

5.5.11 Changing from Cartesion to Polar co-ordinates:


x = reosO, y = rsinB
dx (be
J = dr dO =ICOSB -rsinBI =r
dy dy sinO rcosB
---
dr dB

Jff(x,y)dxdy = Jff~cos9,rsinO] /Jld9dr


/I /I'

= Jff[rcosO,rsinOp'dltlr
/I'

Note: In polar form dxdy is replaced by rdBdr


5.5.12 Example:
Evaluate Jf~ a 2 - x 2 - y2 dxdy over the semi circle xl + Y = ax in the positive
quadrant.
Solution:
Substituting x = reose,
y = rsinB, in xl + Y = ax
,.2eos2 0 + ,.2sin2 0 = an'osO r = aeosO
which is the equation ofthe given circle in polar co-ordinates.

x' --------~I'-.-lo._~-_+-_- X
o A(a,O)

Y'

Fig. 5.5.12

From the figure it can be sedn that


r = OP = aeos8
Application of Integration to Areas, Lengths, Volumes and Surface Areas 369

for the upper halfofthe circle, Ovaries form 0 to "27T where as for any intermediatary
value of 0, r varies from 0 to or i.e., () to acmO.
1[

2 acosO,--_ _ _ __

JJ~a2-x2-idx(ZV= f fJa2-r2cos20-r2sin20rdrdO
II I) ~O r~O

where R is the shaded region.

1t

3 2
= ~ f~
3 0
-sin 0)dO
2

5.5.13 Example:
{/ I 2 J
\ a - y-

Evaluate f f{x 2 + y2 }Ix~v by changing into polar coordinates.


o 0

Solution:
From the given limit x = ~ a 2 - i it is clear that one of the boundaries is the circle x 2
+ y = (r, taking x = rcasO, y = rsillO. The given rigion is the first quadrant of the
7T
circle. Here r varies from '0' to '(I' and 0 varies from '0' to "2.
1t

l Ja 2 -y2 2 a

f f~2 + y2 )dxdy = f fr2.rdrdO


o a O~~O

1t
370 Engineering Mathematics - I

(14 1l
=-x-
4 2

8
5.5.14 Example:
00 W (2 2)
Transform the integral I Ie - '+Y dxdy to polar coordinates and evaluate it.
o r=d
Solution:
The limits of x and yare both from 0 and' 00 '. Therefore the region is in the first
1l
quadrant where r varies from '0' to ' 00 ' and B varies from '0' to 2
Substituting x = rcosB, y = rsinB, and dxdy = rdrdB.
1t

1t 1t
1t

2 [2jW
= I --2
e-r
de =-
2
1 ?-
J'
1
rde =-[e]
2 0
2

o 0 0

4
Changing of order of inte8,ration:
The double integration can be integrated with respect to y first and then with respect
to c or it can be integrated with respect to 'x' first and then with respect to y'. In the
former case, the limits of integration are determined for the given region by drawing
strips parallel to y-axis while in the second case by drawing strips parallel to x-axis.
5.5.15 Example:
, a 2a-x
Evaluate by changing the order of integration I Ixydx:v
o "
a
Application of Integration to Areas, Lengths, Volumes and Surface Areas 371

Solution:
2
The given limits are x:O~a,y:~~2a-x
a
2
i.e., the region is bounded by x = 0, x = a and y = ~ y = 2a - x i.e., x + y = 2a.
a

x' _ _ _ _ _ _.-.."""'....._ _ _ _ _---::a...-_ _ X

Y'

Fig. 5.5.15
a 2a-x

J Jxydxdy
o x2/a
=
OA(,O
Hxydxdy + Hxydxdy
('AIl(,

y=a x=[:ry y=2ax=2a-y

= J Jxydxdy + J Jxydxdy

a 1 2(/
= ~ Ji dY+2" JY(2a- yydy
o (/

5.5.16 Example:

Change the order of integration and evaluate


a H
J J(x 2 + y2 }txdy
o x
a
372 Engineering Mathematics - I

Solution:

The given limits are x = 0 x = 0 and y = ~a and y = .[; i.e.,


a
ay = x.
y

x'------------~~-------------x

Y'

Fig. 5.5.16

f 2J~2 y2)1xdy =:tEX2 y~dx}y


+ +
a

= f ~3 i
I (

o
3
- a 3 l + ai - ay4 } y
3

a3 a
=-+-
28 20
5.5.17 Example:
a 2&
Evaluate f fx 2dxdy by changing the order of integration.
o 0
Solution:
The given limits are x = 0, x = a and y = 0, y = 2.[;;; i.e., y = 4ax.
Application of Integration to Areas, Lengths, Volumes and Surface Areas 373

y x=a

x' -----...",..jl--""-::-I------ x

y'

Fig. 5.5.17

=
I
3" f
2a [ 3
a -
[')
Y
4a
]3] dy 3"[a y - 64a
=
I 3 I
3 x
Y7]2(1
7 0

5.5.18 Example:

f yfx
4 4

Change the order of integration and evaluate 2 X 2 dxdy .


o +y
Solution:

y x=4

x'------..,t"f'---~~--- x

y'

Fig. 5.5.18
374 Engineering Mathematics - I

The given limits are y = y = 4 and x = y, x = 4.

4 4
r -I dx= I7r-dx=-7r [x ]4 =7r
= ian 0
o 0 4 4
5.5.19 Example:
a '22
vaL _yL

Evaluate by changing the ordr of integration f fxydxdy


o 0

Solution:

x'------------+---~+_~--~-----------x

y'

Fig. 5.5.19

The given limit are y = 0, y = a and x = 0, x = ~a2 _ / i.e., x 2 + y2 =a2

2l~ dx
xL
X
o 2 0
Application of Integration to Areas, Lengths, Vo'lumes and Surface Areas 375

8
Triple integration:
Let u = j(x,y,z) be a single valued function of the independent variables x, y, z defined
through the region V. Divide the region V into 11 subregions t5V I' t5V 2 ... t5V nand
P(x l , YI' z) be any point inside or on the boundary of the subregion t5V /' then the sum
II

under the limit Lt If(x"ypzJdv, is defined as triple integral.


11 ~oo ,~I

II

Lt If(xpy"z, )dv, = HJf(x,y,z}1v


11 ~ 00 ,~I /'

5.5.20 Example:
\ z x+z

Evaluate JJ J(x + y + z }ixdyd::.


-I 0 x-z

Solution:
\ z x+z

I = Jdz Jdx J(x + y + z }ixdydz


-I 0 x-z

\ z
Jaz fi2z(x + z)+ 4xz}ix
-\ 0
376 Engineering Mathematics - I

JZ 3 + 2z 3 + 2z 3dz
-I

1 [ 4]1
=SJ.::\lz=S 24 =0
-I -I

5.5.21 Example:
2 r x+y

Evaluate J J JeX+Y+ZdXdydz
000

HJe'[J'dZH'LT+[[e'.(e' r
Solution:

t"dy

fe<[ feY{e-HY -\ }IY}b:


o 0

2 4<
e - 3 7< +e-rdx
J---e--
o 2 2
Application of Integration to Areas, Lengths, Volumes and Surface Areas 377

5.5.22 Example:

Evaluate

1 1

I= f f[xz t~dnly
\'=0 r::: 1,2

1 ) 1 \=1

f .~ - X3
\'=-0 t:..::y2
dy

y=()
f (I-6
1 y4
_0_+-
2
dy y6l
3

~ [~< +~;I 3~
5.5.23 Example:

Evaluate

Solution:

4[-~+-S'11 ]2.[; dz
f
x 42 2 ~1 X
{-}
z=O 2 2 J4; <=0

4In::dZ [z22rJo
= 1f

=81f
378 Engineering Mathematics - I

5.5.24 Example:
Evaluate HJryzdzdydx over the volume enclosed by three coordinate planes and the
I'
plane x + y + z = I.
Solution:
The plane x + y + z = I meets the coordinate axes in A( I ,0,0),8(0, I ,0) and C(O,O, I).
z
C(O,O,1)

Fig. 5.5.24

WX;adZdyd!~ ,1, :I('JpIz }!YdX


1(2
I I-x
XY[ ~ r' }n
dy

= J JX;[(I-X- y )2_ 0}/YdX


x=o y=o
I I-x

= ~ J JXy(I+X2+y2_2X-2y+2Xy~ydx
x=O ),=0
Application of Integration to Areas, Lengths, Volumes and Surface Areas 379

5.5.25 Example:

Evaluate ff~X2 + i + z2 }Izdydx: where V is the volume of the cube bounded by the
I'

coordinate planes and the planes x = y = z = {[,

Solution:
z

Fig. 5.5.25

ff~x2 + y2 + z2 )lzdydx = 'f 'f 2


'f(x + y2 + z2 )Iz(zvch
I' x~o \'=0 Z~O

Jra-))
(/ 4 4
a a
+-+-dx
x=o
3 4

= [ ~' a' + ';' x+ ~' x I


=d
380 Engineering Mathematics - I

5.5.26 Example:
Evaluate by tripple integration the volume of a hemisphere of radius o.
Solution:

Fig. 5.5.26
The volume of the hemisphere

a r;;C;'1- ,----
V=4 J J [z]ga -r 2 2
-1'2 dyd"(
X~O y~O

2 3
V = -1rG
3
Application of Integration to Areas, Lengths, Volumes and Surface Areas 381

Exercise - 5(F)
1. Evaluate the following integrals by changing the order of integration:
3 [4-:':;'
I. J
0
J(x+ y}lxdy

241
IAns: 60 I
5 x2

2. J fx{x + i }Ixdy
0 0
2

29 56
IAns: 24 I

"
,2,,1,,2_x 2

3.
0
f 0
fy 2dx((V

({~ lW4
IAns: - + - 1
16 32
I 2-x

4. J J!...dxdy
0
v x .

IAns: 2 log2 - II
I j;
5. f
0 I'
fxy(X + y}lxdy

3
IAns: 1
56
41 2\'at
,.
6. f
o x2/4a
Jdydx

2
16a
IAns: -I
3
2 4

7. Jfx 2 2
+ y dxdy
I 3

IAns: 10)
382 Engineering Mathematics - I

II. Evaluate the following tripple integrals:



u x y

I. f f fxyzdxdydz
000

a6
[Ans: 48]

2. f 'J 1-](> + i + Z1 }lwlydz


o 0 0

I
[Ans: 20]
, ,
3.
f II' J-XIY~ dydz
() 0

I
[Ans: 3"]
122
2
4. f f fx yzdxdydz
(J U I

[Ans: I]

5.
III
f f f(x + y + z)dxdydz
() () ()

3
[Ans: 2]
I JI-x 2 ~~2_y2
6. f f fxyz dtdydz
o 0 0

I
[Ans: 48]
Application of Integration to Areas, Lengths, Volumes and Surface Areas 383

3 I Fv
7.
JJ
I I 0
Jxyz dxdydz
x

13 1 1og3 )
(Ans:] ( 9-6 ]

I I-x x+y

8. JJ
o 0 0
z
Je dxdydz

1
(Ans: -]
2

4 x !BY
9. JJ
o 0
Jz dxdydz
0

(Ans: 16]
o J 02_x 2 niX

10. J J
o _J02_x2 o
2
Jz dxdydz
"This page is Intentionally Left Blank"
6
Sequences of Series
6.0 Sequence
A function fN ~ S, where S is any nonempty set is called a Sequence
i.e., for each nE N, ::l a unique elementj{n) E S. The sequence is written asj{I),j{2),
j(3), ..... j{n).... ,and is denoted by (j(n)}, or <j(n, or (/(n. If j{n) =an ,the sequence is
written as ai' a 2 an and denoted by , {an} or < all > or ( all ). Here j{ n) or all are the
d h terms of the Sequence.
6.1.1 Example: 1 ,4,9, 16, ......... n 2 , (or) < n2 >
s
386 Engineering Mathematics - I

6.1.2 Example: ~3 ,~,-;-, ~


I 2- J n
(or)(~)

n

N s

6.1.3 Example: 1, 1, 1. ..... I.. ... 0r<l>

6.1.4 Example: 1 ,-1, 1, -1, ......... or (( - I r- I


)

Note 1. If S c R then the sequence is called a real sequence.


2. The range of a sequence is almost a countable set.
Sequences of Series 387

6.1.5 Kinds of Sequences


1. Finite Sequence :A sequence < all > in which an = 0 Vn > mEN is said to
be a finite Sequence. i.e., A finite Sequence has a finite number of terms.
2. Infinite Sequence: A sequence ,which is not finite is an infinite sequence.
6.1.6 Bounds of a Sequence and Bounded Sequence
1. If :3 a number 'M' ~ an:::; M, Vn E N, the Sequence < an > is said to be
bounded above or bounded on the right.

Ex: I,.!.), ,....... here all :::; 1Vn EN


2 3
2. If :3 a number 'm' ~ a" ~ m, Vn E N, the sequence < a" > IS said to be
bounded below or bounded on the left.
Ex : 1 , 2 , 3 ,..... here a" ~ 1 Vn EN
3. A sequence which is bounded above and below is said to be bounded.

Ex: Let an = (-1 r (1 +;)


n 2 3 4
3/2 -4/3 5/4

2--

1 __

o 2 3 4 5 n
-1 -

From the above figure (see also table) it can be seen that m = -2 and M = l.
2
:. The sequence is bounded.
388 Engineering Mathematics - I

6.1.7 Limits of a Sequence


A Sequence < (Ill > is said to tend to limit 'I' when, given any + ve number' E "

however small, we can always find an integer 'm' such that Ian -II <E, Vn ;:::: m,
and we write Lt all =I or (an ~ I)
n->co

n2 + 1 1
Ex: If an = 2 then <a >~-.
2n +3 n 2
6.1.8 Convergent, Divergent and Oscillatory Sequences
). Convergent Sequence: A sequence which tends to a finite limit, say' I' is called
a Convergent Sequence. We say that the sequence converges to 'I'
2. Divergent Sequence: A sequence which tends to oo is said to be Divergent
(or is said to diverge).
3. Oscillatory Sequence: A sequence which neither converges nor diverges ,is
called an Oscillatory Sequence.
Examples
1
1. Const'der the sequence 2, -,
3 -4 ,-,.....
5 here an =1 +-
234 n
The sequence < all > is convergent and has the limit 1
1 1 1 1
an -1 = 1+ - -1 = - and - < E whenever n > -
nnE n
1
Suppose we choose E= .001, we have - < .001 when n> 1000.
n
1
2. If all =3+(-lr--<a/l > converges to 3.
'n'
2
3. If an =n +(-lr .n,<an > diverges.
4. If an = ..!. + 2 (-1
n
r '< an > oscillates between -2 and 2.
6.2 Infinite Series
6.2.1
If < UII > is a sequence, then the expression u1+ u2 + u3 + ........ + un + ..... is called an
co
infinite series. It is denoted by I Un or simply I Un
11=1

The sum of the first n terms of the series is denoted by sn


i.e., sn = U, +u2 +u3 + ...... +un;sps2's3' ....sn are called partial sums.
Sequences of Series 389

6.2.2 Convergent, Divergent and Oscillatory Series


Let Iu" n
be an infinite series. As ~ 00, there are three possibilities.
(a) COl1vergent series: As n ~ oo,s" ~ a finite limit, say's' in which case
the series is said to he convergent and's' is called its sum to infinity. Thus
Lt
11----)00
8" =S (or) simply LIs" =8
00

This is also written as III + 1I} + II] + ..... + 11" + .. .1000 = s. (or) I UII = S
11=1

(or) simply IlIlI = s.


(b) Divergent series: If 8 11 ~ 00 or -00, the series said to be divergent.
(c) Oscillatory Series: If 8 11 does not tend to a unique limit either finite or
infinite it is said to be an Oscillatory Series.
Note: Divergent or Oscillatory series are sometimes called non convergent series.
6.2.3 Geometric Series
. 1
TIle senes, +x+x- + ..... x + ...
1 II-I
IS

(i) Convergent when Ixl < 1, and its sum is _I_


I-x
(ii) Divergent when x ~ 1.
(iii) Oscillates finitely when x = -I and oscillates infinitely when x < -I.
Proof:
The given series is a geometric series with common ratio 'x'
I-x"
sn =- - when x *1 [By actual division - verify]
I-x
(i) When Ixl < 1:
LI s
n;->oo n
=
1- x
n->oo
( 1)
Lt - - - Lt (XII]
- - =--
n->oo 1- x 1- x
1
[ since xn ~ 0 as n ~ 00 ]
. 1
:. T he senes converges to - -
I-x
x" -1
(ii) When x ~ 1: sn =- - and sn ~ 00 as n ~ 00
x-I
:. The series is divergent.
(iii) When x = -1 : when n is even, sn ~ 0 and when n is odd, sn ~ 1
:. The series oscillates finitely.
390 Engineering Mathematics - I

(iv) When x < -1, s" ~ 00 or -00 according as n is odd or even.


:. The series oscillates infinitely.
6.2.4 Some Elementary Properties of Infinite Series
1. The convergence or divergence of an infinites series is unaltered by an
addition or deletion of a finite number of terms from it.
2. If some or all the terms of a convergent series of positive terms change their
signs, the series will still be convergent.
3. Let Iu" converge to's'
Let 'k' be a non - zero fixed number. Then Ikll" converges to h.
Also, if I u" diverges or oscillates, so does I ku"
4. Let I II" converge to 'I' and I v" converge to '111 '. Then
(i) I (u" + v,,) converges to ( 1+ 111 ) and
(ii) I(u" -v,,) converges to (/-m)
6.2.5 Series of Positive Terms
Consider the series in which all terms beginning from a particular term are +ve .
Let the first term from which all terms are +ve be ul
Let I Un . be such a convergent series of +ve terms. Then, we observe that the
convergence is unaltered by any rearrangement of the terms of the series.
6.2.6 Theorem
If Iu" is convergent, then It un
n->oo
= o.
Proof:
Sn=u l +U2 ++U"
sn_1 = ul + u2 + ...... + U,,_I,' so that, un = sn - sn_1
Suppose IU n =/ then It sn =/ and Lt sn_1 =/
n---+oo n~oo

It un = It (Sn-Sn_I); It s,,- It S,,_I =/-/=0


n~oo n---+oo n~oo n~oo

Note: The converse of the above theorem need not be always true. This can be
Observed from the following examples.
Sequences of Series 391
,
1 1 1 1
(i) Consider the series, 1+ - + - + ....... + - + .... ; lin = -, It lin =0
2 3 n n n~oo

But from p-series test ( 2.7) it is clear that I!n is divergent.


. . 1 1 1 1
(ii) ConsIder the sertes, + -) + -) + ..... + -) + ..... .
-2
1 2- 3- n-

lIl/ = ~, It lin = 0, by p series test, clearly I ~ converges,


Ir n~OCJ n-
Note: If Lt lin ::t:- 0 the series is divergent;
IJ-too

2n -1
Ex: 1'" = - 2n ,here Lt Il---Ht)
lin =1
I lin is divergent.

Tests for the Convergence of an Infinite Series


In order to study the nature of any given infinite series of +ve terms regarding
convergence or otherwise, a few tests are given below.
6.2.7 P-Series Test

Th e '111f illite
i ' sertes,
. ~1
~- = -1+ 1
- +1
- + ...... , IS
n~1 n" 1" 2" 3"
(i) Convergent when p > I, and (ii) Divergent when p ~ 1. [JNTU Dec 2002, A 2003]
Pro(~r:

Case (i) Let p> I . J) > 1 3" > 2'" => _1 < _1_
, , '3" 2"
1 1 1 1 2
-+-<-+-=-
2" 3" 2" 2" 2"
111111114
Similarly, -+-+-+-<-+-+-+-=-
4" 5" 6" 7" 4" 4" 4" 4" 4"
1 1 1 8
-+-+ .... +-<-, and soon.
8" 9" 16" 8"
Adding we get
1 2 4 8
"-<1+-+-+-+ ....
~ n" 2" 4" 8"
1 1 1 1
I.e., I n" < 1+ 2(,,-1) + 2 2(,,-1) + 2 3(,,-1) + ..... .
392 Engineering Mathematics - I

The RHS of the above inequality is an infinite geometric series with common ratio
1
2,,-1 < 1(since p > 1) The sum of this geometric series is finite.

Hence f ~n
11=1
is also finite.

:. The given series is convergent.


1 1 1 1
Case (ii) : Let p = I; I - = 1+ - + - + - + ......
n" 2 3 4
1 1 1 1 1
We have, -+- >-+- =-
3 4 4 4 2
111111111
-+-+-+->-+-+-+-=-
567888882
11 111 11
-+-+ ....... - >-+-+ ..... - = - and so on
9 10 16 16 16 16 2
I_I = 1+(!+!)+(!+!+!+!)+ .....
n" 2 3 4 5 6 7
1 1 1
~1+-+-+-+ .....
2 2 2

(Since sn = 1 + n ~ = n; 1and /1~'" SII = 00 )


1
The sum of RHS series is 00

:. The slim of the given series is also 00;:. f_l-n


11=1
P
(p = I ) diverges.

1 1
Case(iii):Let p<l, I7=1+ + 1 +.....
2P Y
1 1 1 1
Since p < 1,- > - - > - ...... and so on
21' 2' JP 3'
1 1 1 1
I->I+-+-+-+
P
.......
n 2 3 4
From the Case (ii), it follows that the series on the RHS of above inequality is
divergent.

I Jl
n
is divergent, when
P
P< I

Note: This theorem is often helpful in discussing the nature of a given infinite
series.
Sequences of Series 393

6.2.8 Comparison Tests


1. Let I
Un and I Vn be two series of +ve terms and let I VII be convergent.
Then I Un converges,
1. If un ~ Vn' \:;f n EN
U
2. or -.!!.. ~ k\:;fn EN where k is> 0 and finite.
vn

3. or un ~ a finite limit> 0
vn
Proof:
1. Let I Vn =I (finite)
Then, u l +u2 + .... +lln + ...... ~VI +v2 + ..... VII + ..... ~l >0
Since I is finite it follows that I UII is convergent

2. un ~ k ~ UII ~ kv \:;fn EN, since ll


, I VII is convergent and k (>0) IS
vn

finite, Ikvll is convergent:. IU n is convergent.

3. Since Lt un is finite, we can find a +ve constant k,3 ~ < k\:;fn EN


n-'>oo vn vn

:. from (2), it follows that IU II


is convergent
2. Let I UII and I Vn be two series of +ve terms and let I VII be divergent.

Then I Un diverges,
* 1. If un ~ VII' \:;fn EN

or * 2. If un ~ k, \:;fn EN where k is finite and"* 0


vn

or * 3. If Lt ull is finite and non-zero.


n-'>OO Vn

Proof:
1. Let M be a +ve integer however large it may be . Science I VII is divergent,
a number m can be found such that
VI +V2 + ..... +vn > M, \:;fn > m

u +u2 + .... +un > M, \:;fn > m(un ~ vJ


l
394 Engineering Mathematics - I

I 1I" is divergent
2 11, 2:: kv,,\:;In
I v" is divergent ~ I kv" is divergent

I u" is divergent

3. Since Lt ~ is finite, a + ve constant k can be found such that un > k, \:;In

(probably except for a finite number oftenns )


:. From (2), it follows that I Un is divergent.
Note:
I. In (I) and (2), it is sufficient that the conditions with * hold \:;In> mEN
Alternate form of comparison tests : The above two types of comparison tests
2.8.( I) and 2.8.(2) can be culbed together and stated as follows:

If Iu" and I Vn are two series of + ve terms such that Lt


"->00
un =
vn
k,

where k is non- zero and finite, then I u" and I Vn both converge or both
diverge.

Note:
I. The above fonn of comparison tests is mostly used in solving problems.
2. In order to apply the test in problems, we require a certain series Vn whose I
nature is already known i.e., we must know whether I Vn is convergent are
divergent. For this reason, we call I Vn as an 'auxiliary series'.
3. In problems, the geometric series (2.3.) and the p-series (2.7) can be
conveniently used as 'auxiliary series'.

Solved Examples
6.2.9 Example
Test the convergence of the following series:
3456 4567
(a) -+-+-+-+..... (b) -+-+-+-+ .....
1 8 27 64 1 4 9 16

~ (n +1) 1/4 -n ]
00 [ 4
(c)
Sequences of Series 395

Solution
(a) Step 1:
To find "ul/" the d" term of the given series. The numerators 3, 4, 5,
6 ...... ofthe terms, are in AP.
n'" term tl/ = 3 + (11- 1).1 = n+2
n+2
are 1 ,-,
3 2' 33 43
Denomlllators , ..... n ,,, term = n \ ; 111/ = --3-
n
Step 2:
To choose the auxiliary series I VI/ . In 1I11 , the highest degree or 11 in the
numerator is I and that of denominator is 3.

:. we take, VII = ----:l=I = 2


n 11
Step 3 :

It ~= It n+2xn 2
= Lt n+2 = It (1+~}=1
1/-->00 vl/ I/~OO3
n I/~OO 11 I/~OO 11 '

which is non- zero and finite.


Step 4:
Conclusioll:
It ~=1
n-----}oo VII

:. IUI/ and I v" both converge or diverge (by comparison test). But

I VI/ ,,-?1
= ~n- is convergent by p-series test (p = 2 > 1); :. Ill" is

convergent.
4 5 6 7
(b) -+-+-+-+ .....
1 4'-9 16
n+3
Step 1 : 4 , 5, 6, 7, ..... in AP , tn = 4 + (n -1) 1 = n + 3

Step 2 : Let "~ v" = -1 be tIle auxi'1'lary series


.
11

Step 3: Lt
I/~'"
~
v"
= Lt (11~3)xn = It (1
n- ,,~oo I/~OO
+l) = 1,
n
which is non-zero and

finite.
396 Engineering Mathematics - I

Step 4 : :. By comparison test, both I utI and I v" converge are diverge
together.

But Iv" =I-n1 is divergent, by p-series test (p = 1); :. Iu" is

divergent.

=n 1+-'4n-.+ ~G-l)~+
2! n
. . _1 =n[-'-.-~+ . . .J
4n 32n

= 4~3 - 3;n 7 + .... = ~3 [~- 3;n 4 + .... .J


Here it will be convenient if we take v" =~
n
Lt utI = Lt
n~oovn n~oo 4 32n
(! -
-1-4 + ..... ) =
4
!,
which is non-zero and finite

:. By comparison test, I Un and I VII both converge or both diverge. But


1
by p-series test IV n =-3
n
is convergent. (p = 3 > 1); :. Iu" is

convergent.
6.2.10 Example
{j'--3n-2-+-1
If u
n
=
~hn3 +3n+5
show that I Un- is divergent

Solution
As n increases, utI approximates to
r:::-? II II
~ 3n 2
l 2/
Jl3 n73 3/ 3 1
~2n 3 = 2Y.; x nYt = 2}~ . nX2
Sequences of Series 397

Ii
U )13
If we take vn = - 1-/ , Lt -.-!!.. =-I which is finite.
nl12 n-->""VII 274

1
[(or) Hint: Take vn = ~ , where II and 12 are indices of 'n' of the
n' ,
largest terms in denominator and nominator respectively of ull Here
1 1
VII =-3-2 =-1 ]
n4 3 nl2

By comparison test, I VII and I U II converge or diverge together. But

"L..,. VII =, , 1 .IS d'Ivergent by p -


L..,.-I,-' . test (smce
senes . p = -1 < 1)
Wl2 12
L un is divergent.

6.2.11 Example

Test for the convergence of the series. Jf ~ J! ~


+ + + + ......
Solution

Here, un =~ n: 1;

Take
1 1
vn =-1-I =-0=1, Lt u.
---- n n-->"" V
~ Lt )
11-->""
11 = 1 (finite)
n2 2 n 1+-
n
I VII is divergent by p - series test. (p = 0 < 1)
:. By comparison test, I Un is divergent, (Students are advised to follow the
procedure given in ex. 1.2.9(a) and (b) to find" un" of the given series.}
6.2.12 Example
1 1 1
Show that 1 + IT + l2 + ....... + l!:! +..... is convergent.

Solution

un = l!:!1 (neglecting 1sl term)

1 1 1
- - - - < -----==,--- =
1.2.3 ......n 1.2.2.2 .....n -ltimes (2n-l)
398 Engineering Mathematics - I

1 1 1
"1I" < 1+-+-? +-, + ..... .
~ 2 2- 2'
which is an infinite geometric series with common ratio ~ <1
2
" 2,,-1
:. ~ 1 IS. convergent. (1.2.3(a)). Hence I "/I is convergent.

6.2.13 Example
Test for the convergence of the series, _1_ + _1_ + _1_ + ...... .
1.2.3 2.3.4 3.4.5
Solution
1
u = .
" n(n+1)(n+2)'
1
Take V"=-3
n
3

Lt
,,~ao V
U"
n
= n~ao
Lt 3
( 1
n 1+--
n)(1+-2 )= 1 (finite)
n n
By comparison test, I U" ' and I v" converge or diverge together. But by p-

series test, I v" = I ~ is convergent (p = 3> 1 ); :. Iu" is convergent.


n
6.2.14 Example
If u" = -J n4 + 1 - -Jn4 -1, show that I Un is convergent. [JNTU, 200s]
Solution
1 I

u" = n2 ( 1+ n1)2
4 - n 2 ( 1- n 4 1)2
~n2[(1+ 2~4 - 8:' + 16~12 - ....)+- 2:4 - 8:' -16~12 - .....)]
=n
2
[-;-
n
+ ~
8n
+ .... J= ~ [1 + ~ + .... J
n 8n
1 u'
Take vn = -2 ' hence Lt -E... =1
n ,,~ao v"
Sequences of Series 399

:. By comparison test, I lin and I v" converge or diverge together. But

I Vn =~ is convergent by p -series test (p = 2 > 1):. I Un is convergent.


n
6. 2.15 Example
1 1 1 I-'
Test the senes - - + - - + - - + ..... lor convergence.
l+x 2+x 3+x
Solution
1
un =--;
n+x
Take VII =-,
n
UII _ n
then =
n+x 1+~
n

n~""
Lt (_1_] 1; I X
= VII = I!n is divergent by p-series test (p =1 )
1+-
n
:. By comparison test, I Un is divergent.
6.2.16 Example

Show that fSin(!)n


n=1
is divergent.

Solution
Take

U n
. (1) sint
SIn -
Yn
Lt --E...
n~"" V
= n~""
Lt ( )
1
= Lt -
I~O t
(where t = 1n) = 1
n _

n
Iu I n
, VII .both converge or diverge. But I VII = I ~ is divergent

(p -series test, p = 1); :. I Un is divergent.


400 Engineering Mathematics - I

6.2.17 Example

Test the series L sin -I ( : ) for convergence.

Solution
_I 1
u =sm _.
n
n'
1
Take vn =-
n

u sin -{~)
( ()) ( . . _ 1
Lt 2..= Lt ( ) ;= Lt - . - =1 Takmgsm '-=()
)
v
/1---*00tHoo
/I
1 8---*0 sm ()
_
n
n
But L v" is divergent .Hence L u" is divergent.

6.2.18 Example
1 22 33
Show that the series 1 + - 2 + - 3 + - 3 + ..... is divergent.
234
Solution
1 22 33
Neglecting the first term, the series is 1 + - 2 + - 3 + - 4 + ..... .Therefore
234
n" nn nn
U - - n-
"~(n+l)"" ~ (n+l)(n+l) ~ n(l+~).n"(I+~J
1 1
Take vn =-
n

1
e

which is finite and L vn = L!n is divergent by p -series test (p = 1)

L un is divergent.
Sequences of Series 401

6.2.19 Example
. 1 3 5 .
SIlOW t Ilat t Ile series - - + - - + - - + .......00 IS convergent.
1.2.3 2.3.4 3.4.5
Solution
1 3 5
--+--+--+ .......00
1.2.3 2.3.4 3.4.5
1
Iii 2n-1 ( 2- n )
n term~ u" ~ n(n+l)(n+2) ~ n' (I+~)(I+~)
1
Take Vn = - 2
n

it un = it _I (2-~ ) (_1 ) +
n~oo VII HOO n
2
(1 + ~'~)(1+ 2n) n
2

0
it Un =( 2)( ) =2 which is finite and non-zero
HOO Vn 1 +0 1+0
:. By comparison test Llln and LVII converge or diverge together

But LVII = L n1 -2 is convergent. :. LUll is also convergent.

6.2.20 Example
1
LII~I J;;n + ,J;;+l
00

Test whether the series is convergent


n+ I
Solution
1
The given series is L-==---==
00

J;; +.j;;+1
11=1

1
402 Engineering Mathematics - I

Take

which is finite and non-zero


Using comparison test I Un and I Vn converge or diverge together.

But I Vn = I l is divergent (since p = Ii)


I Un is also divergent.

6.2.21 Example

Test fo~ convergence f[ ~n + 1 - nJ [JNTU 1996,2003, 2003s]


n=1
3

nih term Un = n[(1 +~))~


n -1] = n[1 +~+
3n Jj U~
1.2
-1) .~+
n ..... -1]
= 3:2- 9:5 + ...... = :2 (1- 9:3 + ......) ; Let
1
Vn =-2
n
Then f~L X=n~Jlj - X n 3 + .... )= lj *0
:. By comparison test, I Un and I Vn both converge or diverge.
But I Vn is convergent by p -series test (since p = 2 > 1)
convergent.
6.2.22 Example
Show that the series, ~ + ~P + ~P + ....... is convergent for p > 2 and divergent
JP 2 3
for p 5, 2
Sequences of Series 403

Solution

III n +1 n ( 1+ In) ( 1+ 1~)


n term of the given senes = U = --~ = =
I n" n" 11
,,-1

Let
I = _1_.
liS take v Lt u
n,,-I '11-"') /v
= l:;t: O n/
lI
'

... L 1111 and L VII both converge or diverge by comparison test.


BlIt LVII = L }~>-I converges when p -I> 1 ; i.e., p >2 and diverges when
p -1 ~ 1 i.e p ~ 2; Hence the result.
6.2.23 Example

L (?II + 2
7-3J
OCJ 1
Test for convergence
11=13 +1
Solution

"" J 2"(1+ 3~,,)r'


l3 '~3 1 ) J
11
1
( +

Take V -
"
Jf. VII
_.
3"'
U
--'!...-
1+ 3
- [ 1 1,'
2
I )1,-
n -
+ n 3
Lt !!A = 1 :;t: 0; :. By comparison test,
v
1l-7r:iJ
L un and L VII behave the same way.
ll

BlIt L... vn" = ~(2)"2 = ~- + -2+ (2)3


L... - -
2
I. I. a geometflc1
+ ..... , W lIC 1 IS sefles Wit 1
11=1 3 3 3 3
common ratio )34, I) :. LVII is convergent. Hence LUll is convergent.
6.2.24 Example
. 1 4 9
Test for convergence of the sefles, + + + ..... .
4.7.10 7.10.13 10.13.16
Solution
4,7, 10, .............. is an A. P; til = 4 + (n -1) 3 = 3n + 1
7, 10, 13, ............ is an A. P; 'II = 7 +(n-l)3 = 3n+4
and 10, 13, 16 , ............. is an A. P; 'II = 10 + ( n - 1) 3 = 311 + 7
404 Engineering Mathematics - I

n2 n
ll= =---:-----:------:-----;----;----;-
II (3n+1)(3n+4)(3n+7) 3n(1+ V ).3n(1+4 ).3n(1+ 7/ )
" 3n 3n 3n
1
27n(1+ 1, )(1+ 413n
/3n
;' )(1+ 7/
/3n
)
Taking vn =! ,we get
n
Lt ~ = _1_:;t: 0; :. By comparison test, both LUn and L vn behave in the
IHOO vII
27
same manner. But by p -series test, L VII is divergent, since p = I. :. Lilli is
divergent.
6.2.25 Example
",hn2 -Sn+1
Test for convergence ~ 3 2
4n -7n +2
Solution
Iii .J2n2 - Sn + 1
n term of the gIVen senes = un = 3 2
4n -7n +2
Let vn =-2
n

~/ ~/?
L lin

nloo ~ = IH~
L

l n 2 - / n + / n- n 2
n3 (4-/~ +~<3) x-I

~/ V J
J

:. By comparison test,
L
nloo

convergent. [p series test - p


l 2 - / n + / n2
(4 _ 7/ +
/n
L un
=
2/)
'n
and
3

L vn
2 > I] :.
r;;
'112
= 4 :;t:
0

both converge or diverge. But


L un is convergent.
L VII is

6.2.26 Example
"
Test tI1e senes ~ un ,w hose n Iii term (
IS I .)
4n 2 - I
Sequences of Series 405

Solution
1
Ull = (4n 2 -/.);

=Lt lrn
2
1
Let VII =-\,
n-
Lt
11->00
_llil
VII Il->ao
2
(
n /'n
4- "
o
)1=-4 *0
:. I Ull and I Vn both converge or diverge by comparison test. But I VII IS

convergent by p -series test (p = 2 > 1) ; :. I UII is convergent.

6.2.27 Example

If un = (; ).Sin (;), show that IU II is convergent.

Solution
1
Let VII = -? ,so that '" VII is convergent by lJ -series test.
n- ~

ll
Lt _" - LI sin J (1)n - It (-sin I )
11->00 ( V
n
- n->ao (1 n ) - 1l~<X' I
where t = lin, Thus Lt (~J =1 * 0 Jl~<t) VII

:. By comparison test, I is convergent. Un

6.2.28 Example

Test for convergence I 1tan(l;>


Take vn = II ~/; Lt [ulIl ] = 1 * 0 (~s in above example)
In 2 n--->oo I VII
Hence by comparison test, IIIIl converges as I VII converges.

6.2.29 Example

Show that fsin


2(~) is convergent.
11=1 n
406 Engineering Mathematics-I

Solution
. ?(I) ; Take vn =-?1
(
Let un =sm- -
n n-

l
sin 1/ .
(~)= Lt ')12 =Lt(smt) 2

Lt
I/~Cf) V n~oo 1,
<n t I~O
n ,'n

where t = II ; Lt (lin/v/ ) = 12 = 1 0
In n~oo n
:;t:

:. By comparison test, I and I behave the same way.


UI/ Vn

But I Vnis convergent by p- series test, since p 2 > 1; :. I = UI/ is convergent.

6.2.30 Example:

Show that f
n=2/
Vlog (n1/) is divergent.
Solution
U
II
= Inlogn'
II . log 2 < 1 => 2log 2 < 2 => 1/ > II .
12log2 72'
Similarly }jIOg3> X,.Xlogn> Yn,nE N
L In
1/ Iogn > L In
II ; But L II
In
is divergent by p-series test.

By comparison test, given series is divergent. [If I Vn is divergent and


u/I ~ vn"i/n then IU n is divergent.]
(Note: This problem can also be done using Cauchy's integral Test. See ex 1.6.2)
6.2.31 Example

Test the convergence of the series I


00

n=1
(c + n rr (d + nr' , where c, d, r, s are all

+ve.
Solution

Th e n Ih term 0 ft h
e senes = un = r
1 ,
(c+n) (d+n)
1 U/' _ 1
Let vII = -r+.\- Then --------=------
n Vn
n
r
( 1+ ~ J +~- J (1+~J (1+ ~J
.n" ( 1
Sequences of Series 407

_
Lf 1111 -_ 1 ---
-;-- 0 :.
," L)III and "
~ v" both converge are diverge, by comparison
11-)00 vJl
test.
But by p-series test, L VII converges if (r + .\) > 1 and diverges if (r + s) ~
:. L II" converges if ( r + s ) > I and diverges if (/"1 s) ~ I.

6.2.32 Example
~ -(1+ 1 )
Show that ~n "is divergent.
1
Solution
-(1+ 1 ) 1
11 =n "=--
"
n.n 1n
1 1I 1
Take VII =- ; Lt -" = Lf - = 17:- 0
n 11---)00 V 11-+00 In
" n
1 1 1
For let Lt -v = y say; log y = Lt - - .log n = - Lt -.-!1 = 0
n
Il-)ct) II ,,-too n 1 11-)00

y = eO = 1 (:) using L Hospitals rule)

By comparison test both L 1I" and LV" converge or diverge. But p-series

test, L VII diverges (since p = I); Hence L lIlI diverges.

6.2.33 Example
~ (n+ar
Test for convergence the series ~ , a, h, C ,p, q, r, being +ve.
II~I (n + b)" ( n+ c )"
Solution
408 Engineering Mathematics - I

1
Take v
II
=- - . It u = 1"* 0 .'
I1P+lJ-r' II~OO V
_II

II

Applying comparison test both I lin and I VII converge or diverge.


Rut by p-series test, I VII converges if (p+ q -r) > I and diverges if (p + q -r)
~ I.
Hence I lIlI converges if (p + q - r) > 1 and diverges if (p + q - r) ~ I.

6.2.34 Example
Test the convergence of the following series whose nl" terms are:
(311+4) 1
(a) (b) tan-;
( 211 + 1)( 211 + 3)( 211 + 5) 11

(c) (d) 11 11
(3 +5 )'

(e)
11. 311
Solution

(a) Hillt:Take VII =~;Iv"


11-
is convergent; It
,,~oo
(u,,)=~"*o
v 8
(Verify)
"
Apply comparison test:
I lIlI is convergent [the student is advised to work out this problem fully]
(b) Proceed as in 1. 2.16; Iu" is convergent.

(c) Hint: Take v" =-2


1
; It -"
(u) = It ((l+J~lr = 3e = 1 "* 0)11 -?
11 ,,~oo VII II~OO 1 + 3/~ e e-

v" = -\- is convergent (work out completely for yourself)


11

(d)
U" ~ 3" +5" ~ 5"
1 1
II 1
+(~J] ;Take v" ~~ ; "If.
1 (Un)
~ ~l ,,0
Sequences of Series 409

LUll and L VII behave the same way. But L v" is convergent since it is

a geometric series with common ratio ~<1


5
:. LUll is convergent by comparison test.
1 1
(e) - - ~ - \;fn EN, since n.3";:::: 3n ;
n.3" 3" '
1 1
L -3;;-
n. <L -:;-; _1
.....( I)

The series on the R. H .S of (1) is convergent since it is geometric series with


1
r=-<1.
3
:. By comparison test . I -I-. IS convergent.
n. 3"
6.2.35 Example
Test the convergence of the following series.
(a) 1+ ~+2J
21+~+3 J + J 1+~+32+4 2 +.............. .
+
1 +2-
1 +2-+3- 1-+2 +3 +4
12+22+3 2 12+22+3 2 +42
12+22
(b) 1+ 3 + 3 3 3
+ +............. .
1 +2 3 1 +2 +3 C+2 3 +3 3 +4 3
Solution
(n+ 1)
n--
U _ 1+2+3+ .... +n _ 2 _ 3
(a)
n - 12 + 22 + 3 2 + .....wJ - n ( n + 1) _on
(2 -t-_
1) - (2 n + 1)
6

Take Vn =- U = Lt ( -
Lt _" 3n- ) =-*0
3
n n ..... '" 2n + 1
n .....'" VII 2
I Un and I VII behave alike by comparison test.
But I Vn is diverges by p-series test. Hence I Un is divergent.
(2n+ 1)
12+22+ .... +n2 n(n+1)~6-- 2(2n+1)
(b)
un = 13 + 23- + .....n 3 = (
2 n +1
)2 = 3n (n + 1)
n ----
4
410 Engineering Mathematics - I

Hillt: Take vn =!
n
and proceed as in (a) and show that IU II
is divergent.

Exercise 6(a)

1. Test for convergence the infinite series whose nih term are:
1
(a) [Ans: divergent)
n-Fn
.Jn~I-Fn
(b) [Ans : convergent)
n
(c) .Jn2 + I-n [Ans : divergent)
Fn
(d) [Ans : convergent)
n 2 -1

(e) .Jn3 +1-N [Ans : divergent]


1
(f) [Ans : divergent]
~n(~+I)
Fn
(g) [Ans : convergent]
n 2 +1
2n3 +5
(h) [Ans : convergent]
4n 5 +1
2. Determine whether the following series are convergent or divergent.
1 2 3
(a) 1 + 3- 1 + 1 + T2 + 1 + T3 + ............ [Ans : divergent]
12 22 32 2+ IOn
(b) r+23+]T+ ........ + n3 +..... ....... [Ans: convergent]

1 1 1
(c) J1 +.J2 + .J2 +J3 + J3 + J4 +...... ........ [Ans : divergent]

234
(d) 2+-2 +2+ ............... [Ans : divergent]
345
1 1 1
(e) 2+-3 +4+ .......... [Ans : convergent]
123
Seq uences of Series 411

00 ~n2 + 1
~ ~4n2 +2n+3
(f) (Ans : divergent]

(g) f(8 Yn -1) .................... (Ans : divergent]


J
3
3n +8
(h) L 5n
00

J +9
5 ...................... .. (Ans : convergent]

123
(i) - + - + - + .......... . (Ans : divergent]
1.3 3.5 5.7
6.3
6.3.1 0' Alembert's Ratio Test
Let (i) LU n beaseriesof+vetermsand(ii) Lt un +J
II~OO un
=k(~O)

Then the series L un is (i) convergent if k < 1 and (ii) divergent if k > I.

Proof:Case(i): Lt un + J =kI)
n~oo un
From the definition of a limit, it follows that
U
3m > 0 and 1(0 < I < 1) 3 ~ < IV n ~ m
un .
i.e., U m+ J <I , U m+ 2 < I ,..........
Um U m+ J

Um + Um+J + Um+ 2 + .............. = Um [1 + uu m


+
m
J
+ um+ 2 + ..... ]
um

< um(1 +1 +/2 + ...) = um.-I-(I < 1)


I-I
1
But Um . - -
I-I
is a finite quantity:. L un
00

is convergent

By adding a finite number of terms u J + u2 + ...... + um- J ' the convergence of the
00

series is unaltered. L un is convergent.


n=m
412 Engineering Mathematics - I

U
C(l!t'e (ii) Lt ---.!!.l. = k > 1
n-->oo 11n

There may be some finite number of terms in the beginning which do not satisfy

the condition ull +' 21. In such a case we can find a number 'm' 3 u +' 21, 'lin 2 m
ll

Ull UII

Omitting the first 'm' terms, if we write the series as u, + 112 + llj + ......... , we have
!!2. 2 1, u3 21, u4 2 1 .......... and so on
u, 112 u3

U, + 112 + ...... + till = u, (1 + u + 2


ll,
113
tl2
.!!2. + ..... J
u,
(to 11 terms)

2 ull (1 + I + 1.1 + ....... to n terms)

II

Lt
n~oo
Iu" 2 Lt n.u, which ~ 00; :. IU
11=1 Il~CO
Il is divergent

6.3.2

Note: 1

The ratio test fails when k = 1. As an example, consider the series, I ~n


n='
p
u n P 1
Here Lt ---.!!.l. = Lt - - = Lt -- =1
n-->oo U n-->oo ( n + 1) n-->oo ( 1+ 1/ )
n ,In

i.e., k = I for all val ues of p,


But the series is convergent if p > 1 and divergent if p::; 1, which shows that when

k = 1, the series may converge or diverge and hence the test fails.
Note: 2 Ratio lest can also be stated as follows:

If I Ull is series of +ve terms and if Lt ~ = k ,then I Un is convergent


11-->00 u +'
ll

If k > 1 and divergent if k < 1 (the test tilib '.I. hen k = I).
Sequences of Series 413
-~---~ ~----- ----~---- -----

Solved Examples

Tests for convergence of Series


6.3.3 Example
\
X X X
(a) -+--+---+ ................ .
1.2 2.3 3.4
Solution
x" Xl/II

= n (11 + 1) ;111/+1 = (11 + 1)( n + 2) ,

r.
111/

~/!.!..'.l= Xl/+I n(n+1) 1


1/" (n+ 1)(n + 2 r x" ~ (I + ~
II
Therefore LI _1/_,1 =X
11----)00 1111

:. By ratio test IUI/ is convergent When Ixl < and divergent when
Ixl> I;
1, 1 II
When x = I, ul/ = ) ; fake vn = - 2 ;
J ( Lt _1/ = 1
n- 1+ lin
. n v1/ 11->00

:. By comparison test IUI/ is convergent.


Hence I Un is convergent when Ixl ~ 1 and divergent when Ixl > 1
3
(b) 1+ 3x + 5x 2 + 7 x + ...... .
Solution
Un = (2n-l)xn-l;ul/+ 1
=(2n+l)x"

L1-= -+
ul/+ I L t (2n -1) x=x
1/-->00 lin 1/-->00 2n-l
:. By ratio test I Un is convergent when Ixl < 1 and divergent when

Ixl > 1
When x = 1: ul/ = 211 -1; Lt un = 00; 11----)00
:. I UI/ is divergent.

Hence IlIl/ is convergent when Ixl < 1 and divergent when Ixl ~ 1
00 1/

(c)
I-+-
n- + 1
1/;1
414 Engineering Mathematics - I

Solution
I
x" X"+
U =2
-- . U =----
n n +1' n+1 (n+l)2 +1
2
Hence un +1 ( n +1 )
-;;: = n 2+ 2n + 2 x

2
Lt un +1 = Lt n (
1
+ >;:2) (x)=x
n~'" Un II~'" n2(1+~+2)
2
n n
:. By ratio test, L un is convergent when Ixl < 1 and divergent when Ixl > 1
1 1
When x=I'u
n
= - - . Take
n2 + 1 '
v =-
n n2
:. By comparison test, LU n is convergent when Ixl ~ 1 and divergent when

Ixl >1
6.3.4 Example

Test the series f (n:n -1}n,


n~'" +1
x> 0 for convergence.

Solution

U" =(:::: )X";U"" = [~:: :i: :: ]X'HI


2 2
Lt un+1 = Lt [( 2n + 2n ) ( n2 + 1)] x
n~'" un n~'" n + 2n + 2 (n - 1) .

4
n (1+2/n)(1 +l/n) 1
=nIf", [ n4(l+2/n+2/n2)(1-l/n) =x
:. By ratio test, LU n is convergent when x< 1 and divergent when x> 1 when
x = 1,
n2 -1 1
U = 2- - Take vn =-0
n n +1 n
Sequences of Series 415

Applying p-series and comparison test, it can be seen that L un is divergent when
x= 1.
.'. L un is convergent when x < 1 and divergent x ~ 1

6.3.5 Example
2P 3P 4P
Show that the series 1+ ~ + l] + l1 + ..... , is convergent for all values ofp.
Solution

Lt .un +1 = Lt [(n+IY x l.fl]= Lt { 1 (n+I)P}


n~oo un n~oo In + 1 nP n~oo (n + I) n

= Lt (
n~oo
1
) x Lt 1+ - )P
n + 1 n~oo n
(I = 0 < 1;
L lll/ is convergent for all 'p'.

6.3.6 Example
Test the convergence ofthe following series
1 1 1 1
-+-+-+-+
P P P ........... .
F 3 5 7
Solution
1 1
u = u =---
1/ (2n-IY' (2n+ly1/+1

ul/+ 1 = (2n-Iy _ 2P.n P(I-l/2nY. Lt un+1 =1


un (2n+ly - 2PnP(I+l/2ny , HOO un

.'. Ratio test fails.


1 nP 1
Take v = _. U
-E... = = U
. Lt -E... = -1P
n nP' vn (2n -I)P ( I)P , I/~OO V 2 ,
2 P 1-- n
2n
which is non - zero and finite 7
.'. By comparison test, LU n ar.d L vn both converge or both diverge.
H6 EngineerinG M~th~ni1!!l'S~

But by p - series test, I VII -= I -I 11"


'~onverges when p > I and di\ c, :'t_ '

when p ~ I
:. I iI" is convergent if /' I and divergent if p ~ I .

6.3.7 Example
" ' (n -1- l).\""
Test the com crgence of till' "cric~ L.. - - -,--.X >0
1/, I 11
Solution
( n + I) x" ( n + 2) X"-l I
lin = 1 ;1I1Itl - - - - , - , -
n (11+1)

;
I ; ~
1I 1

II
illi rl
LI -- = ..:
L,
'
! ,,- _.. , : , "" - - - ,
i
.x = X
11->''0 111/ " ,.w + 11 ( 1+ 1 )'
n, \.. n
By i'".1i j() test, I UII converges when x < 1 and diverges when x > 1 .
n+ I
When ..x= I ' IIII = -
n3-

Take v" = -~ ; By comparison test I UII .is convergent ( give proof)


n
.'. I UII is convergent if x ~ 1 and divergent if x > I.

6.3.8 Example
Test the convergence of the series
. ~(n2
(I) L.. -+-2
1) (.. 1 2.5.8 2.5.8.11
II) + - - +
... ) 1 1.2 1.2.3
+ ... (III - + - + - - +
II_I 2" n 1.5.9 1.5.9.13 3 3.5 3.5.7
Solution
(n2 1) n 2 1
(i) I ---;+-2 I-II + I-2
00 00 00

= Let
2
II-I n 2 n 11=1 11=1
Sequences of Series 417

U = (n+1)2 .1l11+1 = (n+1)2 ~


11+1 2"rl' 2"tl 2
U" 11

Lt1111+1
-- _ L
t-. 1(1 +-1)2 --<
_1 1
I1~OO lin 2 11-+00 n 2

By ratio test I Un is convergent .By p -series test, I v" IS

convergent.
:. The given series (I Un + I v,,) is convergent.
(ii) Neglecting the first term, the series can be taken as,
2.5.8 2.5.8.11
--+ +
1.5.9 1.5.9.13
Here, 1sl term has 3 fractions ,2 nd term has 4 fractions and so on .
:. nih term contains (11 + 2) fractions
2.5. 8....... are in A. P.
(n + 2 t term = 2 + ( 11 + 1 ) 3 = 3n + 5 ;
1. 5. 9, ....... are in A. P.
( n + 2t term = 1 + ( n + I ) 4 = 4n + 5
2.5.8 ..... (3n+5)
U =----'--~

n 1.5.9 ..... ( 4n + 5)
2.5.8 ..... (3n+ 5)(3n+ 8)
un+1 = 1.5.9 ..... (4n+5 )( 4n+9 )
un +1 _ (3n+8) .
-;,:- - (4n + 9) ,

n(3+~)
Lt u +1 = Lt =l
n
!) <1
l1

HOO un HOO n ( 4 + 4

By ratio test, I Un is convergent.


(iii) 1,2,3, ........ are in A. P nih tenn = n ; 3.5. 7 .......... are in A.P. d" term =
2n+ 1
418 Engineering Mathematics - I

[
1.2.3 .....n 1
un = 3.5.7 ..... (2n+l)

n 1
U +
1.2.3 .....n(n+l)
= [ 3.5.7 ..... (2n+l)(2n+3)
1
Il
U +
I
=(~)
U l 2n+3

n.(l+!)
Lt UIl +I = Lt n = ! <1
n~oo Un /HOO n(2+~) 2

:. By ratio test, L un is convergent.

6.3.9 Exercise
Test for the convergence of the series L 1.2.3.(....n )
3.5.7 ...... 2n+ 1
Solution
1.2.3 ...... n 1.2.3 ...... (n+l)
un = 3.5.7 ...... (
2n+l) ; U n+1 =
3.5.7 .....: ( );
2n+3

U n +1 = n+l . Lt = Lt n(l+
U,,+I /~) =!<l .
un 2n+3' /HOO un HOO n( 2+ %) 2 '
:. By ratio test, LUll is convergent.

6.3.10 Example
~ 1.3.5 .... (2n-l) n I( )
Test for convergence L.. .x - x> 0
11=1 2.4.6 .... 2n
Solution
1.3.5 .... (2n-l) nl
The given series of +ve terms has u" = .x -
2.4.6 .... 2n
1.3.5 .... (2n+l) n
and U,,+I = 2.4.6 .... (2n+2 ) x
Sequences of Series 419

2 1 x 2n(1 + 1 )
Lt 1111+1 = Lt ( ~ = Lt 2n .x=x
IHOO U// //-->00 2n + 2 ) IHOO 2n( 1+ 22n)

By ratio test, I u// is converges when x < 1 and diverges when x > I when
x = ), the test fails.
1.3.5 .... (2n -1)
Then U = <1 and Lt u// ;f::. 0
II 2.4.6 ..... 2n 11-700

:. I U/I is divergent. Hence I UII is convergent when x < ), and divergent


when x ~ 1
6.3.11 Example

Test for the convergence of 1+ -2 x + -6 x 2 + ........ + ( -


2// --2 ) x /I-I + ..... ( x > 0)
5 9 2/1+1
Solution

Om itting 1" tcnn, II, =( ~: :~ ) x'-' ,(n " 2) and 'II,' arc all eve.

U = (2"+1 - 2) x/l. Lt (~) = Lt (211+1 - 2) x ( 2// + 1) X


IHI ( 2//+1 + 1) '/1-->00 u ll
11-->00 2"+1 + 1 2/1 - 2 .

2n+1 (1- 12/1) 2// ( 1+ 12/1) 1


= //~~ [ 2//+1 (I + l''2/1+1 )2/1 (1- 2, 2/1 ).x = x ;
Hence, by ratio test, I Un converges if x < 1 and diverges if x> 1.
2/1-2
When x = ), the test fails. Then u/I = -n - ; Lt
2 +1 IHOO
un = 1 ;f::. 0; :. I Un diverges

Hence I Un is convergent when x < ) and divergent x > 1

6.3.12 Example
(3 -4i)"
Using ratio test show that the series I 00

converges
/1=0 n!
Solution
_ (3-4i)" I . _(3 -4i),,+1 / .
un - In! ,un+1 - /(n+l)! '
420 Engmeerlng MC3tht::rnatl~~
---------------------------------=--

1II1tl)_ 1 I (3-4i)_0
1 t ( --- ---- <1
II ',,,,llil - ":'h n + 1 - II

Hence, by ratio test, l: 1111 l:onverges.

6.3.13 Example
.
DlsclIss t Ile nature 0 f t he series,
. - 2 x + - 3 x 2 + - 4 x 3 + ........00 (x0
>)
3.4 4.5 5.6
Solution
Since x > 0 , the series is of +ve terms;
(n + 1) II (n + 2) X/HI
llil = (n+2)(n+3)x > Zln+1 = (n+3)(n+4)

Lt IIn+ I - [(n + 2)2.X ]


II~OO - - ; ; : - ( n + 1)( n + 4)

= Lt r 2 (1 + 7;;)2
n
2
.x ] = X.
HOO l n 1+ ( Yn + j~2 ) ,
Therefore by ratio test, I Un converges if x < 1 and diverges if x> 1
. (n+l)
When x = 1, the t(.st falls; Then un =( )( ) ;
n+2 n+3

Taking v =!. Lt = un = 1 :;t: 0


n n ' n~oo vn

:. By comparison test IU n and I Vn behave same way. But I Vn is divergent

by p-series test. (p = 1);


.. I Un is diverges when x =1
:. I Un is convergent when x < 1 and divergent when x ~ 1

6.3.14 Example
. . ~ 3.6.9 .... .3n.5n
DIscuss the nature of the senes ~ ( )( )
4.7.10 ..... 3n+l 3n+2
Sequences of Series 421

Solution
3.6.9 ..... 3n 5"
Here, U = .
n 4.7.10 ..... (3n+l)(3n+2)'
3.6.9 .... .3n(3n + 3)5"+ 1
U II + I = 4.7.1O ..... ( )(
3n+l ) (3n+5
3n+4 );

Lt
11 I
~ = Lt (3n+2)(3n+3).5
--'-----_-'--c-'---,-----_'---:-

11->00 llil 11->00 (3n+4)(3n+5)

= Lt
5.9n2 (1 + 2 3" )(1 + 3/3'
Inn
)1 = 5 > 1
11->00
f
2
(1
9n + 4 3n )( + 5, 3n) 1
:. By ratio test, I lin is divergent.

6.3.15 Example
00

Test for convergence the series I n l - II

Solution
1I11 = n ; u +1 = (1)-11
n+
I-II
; lI

':;,:' = = (n;I,!" n(n:l)" =~C:J


Lt un+1 = Lt !.[_l_,]" =O}=O<1
n~'" U 11->00 n 1+ 1,' e
II ,'n
:. By ratio test I Un , is convergent

6.3.16 Example
2 3
I ~ ,for convergence.
00

Test the series


11=1 ~
Solution
2n 3 2(n+ 1)3
Un = ~ ; U II +I = In + 1
422 Engineering Mathematics-I

lln+1 = 2(n+l)3 x ~ = (n+l)2 = (1+;~r


un In + 1 2n 3
n3 n
u
Lt --.!!..l = 0 < 1;
n-+oo Un

:. By ratio test, L u/l is convergent.

6.3.17 Example
2/1 ,
Test convergence of the series L~
nn
Solution

n 1
+ (n + I)! /1 ( ~ )11
un +1 = 2 ~=2
1111 (n + 1)"+1 . 211 n! n+ 1
U 1 2
Lt --.!!..l = 2 Lt =- < 1 (since 2 < e < 3)
IHOO ull Hoo(l+ )-~r e

:. By ratio test, LUll is convergent.

6.3.20 Example
Test the convergence of the series L un where u/I is
n2 + 1 x n- I
(a) 3/1 +1 (b) --,(a>O)
(2n+lf
(c)
1.2.3 ....n )2 (d)
~
( 4.7.10 .... .3n+3
~1+3n
(e) 3n + 7n
3 2
)xn
( 5n 9 + 11
Solution

(a) Lt
n-+oo (~
ull n-+oo
I)
u = Lt [(n+l)2 +1 x3/1-+1]
3n+1 + 1
-
n2 + 1
Seq uences of Series 423

1
= -<1
3
:. By ratio test, LUll is convergent.

Lt (u )= Lt [ (2n + 3r x~(2n+lf]
n+ I

l
(b) xn
n I
11-->00 un x
n-->oo -

aa
= Lt
2 n (I+!' )a
. 2n .x = X
IHOO
[ 2a na (l+ }Snf
By ratio test, LUll convergence if x < 1 and diverges if x> 1.

When x = 1, the test fails; Then, 1I = 1 ; Taking v = _1a awe have,


/I (2n+ If II n

Lt
n-->OO
(Un) =
Vn
Lt
n-->oo
(_n_)a =
2n + 1
Lt
n-->oo (
1 = _1
2 + ;!~r 2
a
* 0 and finite (since a > 0).

:. By comparison test, L un and L VII have same property

But p -series test, we have


(i) LVII convergent when a> 1

and (ii) divergent when a ~ 1


:. To sum up, (i) x < 1, LU n is convergent Va.

(ii) x > 1, L un is divergent Va.

(iii) x = L
1, a> 1, un is convergent, and

(iv) x = 1, a ~ 1, L un is divergent.
424 Engineering Mathematics - I

U I [ I.2.3 ....n(n+I) 4.7.1O .... (3n+3)]2


(c) Lt~= Lt x-----'--------<-
IHoc> ltll IHoc> 4.7.IO .... (311+3)(3n+6) 1.2.3 ....n

= Lt [ (n + I) X]2 =! < I
IHoo 3 ( n + 2) 9
:. By ratio test, IlIlI is convergent

(d)

= LI

:. By ratio test, I U" is convergent.

U I lr 3 ( n + 1)3 + 7 ( n + I r Sn 9 + II ]
(e) ~=
LI
HOC> lIlI
LI
Hoc> t
S( n + 1 + II
x 3
3n + 7
xx

= Lt
U
--"...!.. = Lt
f3n3(1+ ~ )3 +7n2(1+ .1.1)2
.n n x
Sn9(I+l!'s/ 9) ]
n xX 1

n->oc> U" n->oc>


9
Sn (I + V)9 + II
.n
3n + 7;1
3
/ 3n
(I 3)
3n
3
{(I + .1/)3
n
+2_(1 + 1/)2}
3n n i
Sn 9 + (1 I/VS 9
)
= Lt x n xx =X
n->oc>
Sn
9
{( I+ }~r + SI~_} 3n
3
(1 + .%n 3)

:. By ratio test, I Un converges when x < 1 and diverges when x > 1.


When x = 1, the test fails,

Then U = 3n3(1+ jjn) =~ (1+ l~n)


6
" Sn
9
( 1+ IYsn9) Sn (I + IYsn 9)

Taking Vn = ~, we observe that Lt U" = l;f: 0


n "->OC>~ S
Sequences of Series 425

:. By comparison test and p series test, we conclude that L un is convergent.


:. L u" is convergent when x::::; 1 and divergent when x > 1.

Exercise - 1(b)

1. Test the convergency or divergency of the series whose general term is :

x"
(a)
n
I Ans: Ixl < legl, Ix ~Ildgt ]
(b) nx n-I .......................... . I Ans : Ixl < legt, Ix ~Ildgt I

(c) (~:. :n X
"' I Ans : Ixl < legl, Ix ~Ildgt I

(d) (:::: )X" ......... . [ Ans : Ixl < legl, Ix ~Ildgt I

(e) lfl [ Ans: cgt.]

(f) 4"lfl [ Ans: dgt.]


nil

( n 3 +1 )"
(g) [ Ans: cgt.]
(3 + 1)
n

2. Determine whether the following series are convergent or divergent :


x x2 x3
(a) - + - + - + ..............
1.2 3.4 5.6
(Ans: Ixl::::; legl,lxl > ldgt ]

x2 x3
(b)
X
1+-2+-2 +-2 + ............. .
234
I Ans : Ixl : : ; legt, Ixl > Idgt ]

1 x2
(c)
1.2.3 4.5.6 7.8.9
X
- - + - - + - - + ..... . ( Ans : Ixl::::; legt, Ixl > Idgl ]
2 3 n
x
(d)
X

2
X
1+-+-+-+ ..... - 2- + ....
5
X

10 n +1
(Ans: Ixl::::; legt,lxl > ldgt J

1.2 2.3 3.4


(e) -+-2 +-3 + .............. . [Ans: Ixl > legt,lxl::::; Idgt]
x x x
426 Engineering Mathematics - I

6.4 Raabe's Test

Let I u" be series of +ve terms and let E!. {n(:.:, - I)} ~ k
Then
(i) If k > 1, ~>n is convergent.
(ii) If k < 1, L un is divergent. (The test fails if k = 1)
Proof:
Consider the series

Case (i):
In this case,

Lt n{~-l} =k
n-+oo U +
n 1
>1

We choose a number 'p' 3 k > p > 1 ; Comparing the series L U" with L Vn

which is convergent, we get that L un will converge if after some fixed number
of terms
un Vn (n+ l)P
U +
n 1
> Vn+1 = -;;-
i.e. If.,
Un)
n ( - - 1 > p+
p(p-l) 1
.-+ ......... from (I)
Un+1 L2 n

i.e., If Lt n(~-I P
n-+oo U +
n 1
Sequences of Series 427

i.e., If k > p, which is true. Hence Llln is convergent .The second case also
can be proved similarly.

Solved Examples

6.4.1 Example
Test fe>r convergence the series
1 x3
x+-.-+-.-+--.-+ .....
1.3 x 1.3.5 x 7

2 3 2.4 5 2.4.6 7
Solution
Neglecting the first tern ,the series can be taken as ,
1 x 3
1.3
-.-+-.-+--.-+ .....
x5
1.3.5 x7

2 3 2.4 5 2.4.6 7
1.3.5 .... are in A.P. nih term = 1+(n-1)2=2n-1
2.4.6 ... are in A.p. n'h term = 2 + (n -1) 2 = 2n
3.5.7 ..... are in A.P nih term = 3+(n-1)2 = 2n+ I

uti ( nih term of the series) =


21l 1
1.3.5 .... (2n-1) X +
=----'----;------::--"--
2.4.6 .... (2n) 2n+1
1.3.5 .... (2n-1)(2n+ 1) X
2n
+
3

1l1l+1 = 2.4.6 .... (


2n)2n+2
() . -
2n+3
1.3.5 .... (2n+1) X 2n +3
lIn+1 _ 2.4.6 ... .2n (2n+1)
-;;:- 2.4.6 .... (2n+2)"(2n+3)"1.3.5 .... (2n-1)" X 2n + 1
(2n + 1)2 x 2
( 2n + 2) ( 2n + 3)

4n-1 ( 1+-- 1)2


II 2n 1
LI ~ = LI x
2
= X-
IHOO Un n~oo 4n 2
( 1+ 22~ )(1+ ~~ )

:. By ratio test, LU" converges if lxl < 1 and diverges if Ixl > 1
If Ixl = 1 the test fails.
428 Engineering Mathematics - I

x2 =1 and ~ = (2n + 2)( 2n + 3 )


Then
1111+1 (2n+ 1r
~ -1 = (211 + 2)( 2n + 3) _ 1 = 6n + 5
111/+1 (2n+1)2 (2n+l)2

LI
tHOO
{ (
n -
ul/ -
ul/+
1
1J} = l I ( 6n
2
+ 5n
I/~OO 4n 2 + 4n + 1
1
n
2
( 6+ ~) 3
= Lt =- > 1
11-->00 n 2 (4 + ~n + _n-\ ) 2
By Raabe's test, I UI/ converges. Hence the given series is convergent when
Ixl ~ 1 an divergent when Ixl > 1 .
6.4.2 Example
Test for the convergence of the series
3 3.6 2 3.6.9 3
1+-x+--x + x + ..... ;x> 0
7 7.10 7.10.13
Solution
Neglecting the first term,
3.6.9 .... 3n 1/
ul/ = .x
7.10.13 .... 3n+4
3.6.9 .... 3n(3n+3) 1/+1
ul/+ 1 = 7.10.13 .... ( ) ( 3n+7 ).x
3n+4
lIl/+1 = 3n + 3.x ; Lt lIll+1 =X
lIl/ 3n + 7 1/-->00 lIlI

:. By ratio test, I UII is convergent when x < I and divergent when x > I.
When x = I The ratio test fails. Then
u 3n + 7. lin 1 _
ll _ 4
1I1/+1 - 3n+3' lIn+1 - - 3n+3

LI {n(~-l)}= Lt (~)=i>l
11-'>00 lln+1 11-->00 3n+3 3
Sequences of Series 429
--~-----------------------------------------------------------

:. By Raabe's test, LUll is convergent .Hence the given series converges if x ~ 1


and diverges if x> 1.
6.4.3 Example
Examine the convergence of the series
f 12.5 2.92.... (4n-3)2
11=1 42.g2.122 .... (4n)2
Solution
12.5 2.92 ( 4n - 3)2
U = ---------'-------'-
II ~ ~
4-.8-.12-.... 4n )2 7 (

12.5 2.9 2.... ( 4n -


3)2 (4n + 1)2
litH 1= -~--~---2--'---(-----')2'---(--'--------'-)2-
4-.8-.12 .... 4n 4n+4
( 4n + 1)2
Lt u lI 1
= Lt ~ = 1 (verify)
n->oo un
+
11->00 ( 4n + 4 t
:. The ratio test fails.
Hence by Raabe's test, LU n is convergent.

6.4.4 Example

Find the nature of the series L (l!:!)2 xn


, (x > 0)
~
Solution
(l!:!)2 II (l~l)2 11+1
ull = ~ .x ; un + 1 = 12n+2 .x

ulI +1 _ (n+l)2 X'

ull (2n+l)(2n+2) '

n2 ( 1+ 1/ ')2
Lt u n +1 = Lt ;' n .x = ~
11->00 U
n
n->004n2(1+}/
, 2n
)(1+2/)
2n
4
:. By ratio test, L un converges when:! < 1, i. e ; x < 4; and diverges when x >4;
When x = 4, the test fails.
430 Engineering Mathematics-I

x = 4 ~ = (2n + 1)( 2n + 2 )
When
- '1l11+1 4(n+l)2
U -2n-2 -1
_" -1=
lln+1 4 (n + 1 r = .
2 ( n + 1) ,

LI
II-->if.>
[n(~-11]= -12 <1
li
ntl

:. By ratio test, I lin is divergent


Hence I Un is convergent when x < 4 and divergent when x> 4

6.4.5 Example
,,4.7 .... (3n+ 1) n
Test for convergence of the series ~ x-
1.2.3 .... n
Solution
_ 4.7 .... (3n+1) n. _ 4.7 .... (3n+l)(3n+4) n+1
lin - X 'U n + 1 - () X
1.2.3 ....n 1.2.3 ....n n + 1 .

II
Lt ~ = Lt [(3n+4) .x] = 3x
n-->oo Un
n-->oo ( n + 1)

:. By ratio test I Un converges if 3x <1 Le., X <! and diverges if x >! ;


3 3
If x= !, the test fails
3
1 [ [(n+ 1)3 ] [-1 ] 1
When
x ~:3' n u.:, -1
II ]
~n 3n + 1 -1 ~ n 3n + 4 ~- (3+ ~ 1
Lt
n--><Xl
n[~-I] = -!3 < 1
II
11+1

:. By Raabe's test, IU n is divergent.

:. " lin
~ . convergent wl
IS len I - and 'Ivergent w Ilen x
x <d 1
~-
3 3
Sequences of Series 431

6.4.6 Example
1
, 3x 4x 2 5x
1 est for convergence 2 + - +- + ~ + ........... (x > 0 )
2 3 4
Solution

The nih term II =(n+l)x tl - I


H '
11

_ (n+2) .tl. II'HI _11(11+2)


Hel
II - (11+1) X '--;;,:- (11+1)2 .x

11 1(1+ 2)
lr
Lt I = LI 11 .x = X

11 + )-
(1 1
_H_' 1

H-N, 11 H~-'UJ 2
tl
11

:. By ratio test, IU II
is.convergent if x < I and divergent if x > I
Ifx= I, the test fails.

Then Lt 11
[
II
1 = Lt
[ 1]
-111
(11 + 1)-
II--'>UJ
_tl -

IIl/el
1 n(n+2)
,I/---W'

=LtI1
[11 ( 111]
+
11->00
=0<1
2)
:. By Raabe's test III// is divergent
:. I Zlil is convergent when x < I and divergent when x 21
6.4.7 Example
. . 3 3.6 3.6.9
FlI1d the nature of the series - + - + + ......00
4 4.7 4.7.10
Solution
3.6.9 ..... 3n 3.6.9 ..... 311(311 + 3)
1I 11 = 4.7.10 ..... ();1I1/+
3n+l = 4.7.10 ..... ( )(
311+1 )
311+4
1

3 3 3n(I+3 )
1111+1 =~. LI llll+1 = LI 3n = 1
ZlII 3n + 4' /1--'>00 llll 11->00 3n( 1+ 4 )
3n
Ratio test fails.

LI 11 { - -
[ ,lll/+1
tl--'>oo
Zltl I}] - LI [ n (3n+4
/1--'>00
---
3n + 3
I)]
432 Engineering Mathematics - I

11 n 1
= Lt = Lt =- <1
,,->003(n+1) 11->003n(1+,]n) 3

:. By Raabe's test L U" is divergent.

6.4.8 Example
If p, q > () and the series
1 p 1.3.p(p+1) 1.3.5 p(p+1)(p+2)
1+"2 q + 2.4.q(q + 1) + 2.4.6 q(q + l)(q +2) + ....
is convergent, find the relation to be satisfied by p and q.
Solution
1.3.5 ..... (2n-1) p(p+l) ..... (p+n-1) . I
un = [negle"tmg l' term 1
2.4.6 .... .2n q(q + 1) ..... (q + n-1)
1.3.5 ..... (2n-1)(2n + 1) p(p+ l) ..... (p + n -l)(p+ n)
lI"tl = 2.4.6 .....2n(2n+ 2) q( q + 1) ..... ( q + n-1)( q + n)
U,,+l _ (2n+1) (p+n).
--;z- - (2n + 2) (q + n) ,

Lt ~ = Lt [2n (1+ 12n ) n (1+ ~'~ ) J= 1


11->00 lI" 11->00 2n( 1+ 1"2n)" n( 1+ ~~)
:. ratio test fails.
Let us apply Raabe's test

,~.H::':, ~1)]~ ,~.H ~::~)~~::~~ ~1}]


Lt [n {2q ( n + 1) - ~ (2n + ) + nIl
1
11->00 n 2 ( 1+ ~/~ ) ( 2 + },~)

r
Lt 2q(1+ )~)-p(2+ J~)+11=2q-2p+l
II->ool 2 2
Sequences of Series 433

~ . 2q-2p+l
Since L..,.llil IS convergent, by Raabe's test, >1
2
=> q - p > Yi, is the required relation.

Exercise 1 (c)

00

1. Test whether the series I Un is convergent or divergent where


I

2 2.4 2.6 2..... ( 2n - 2 )2 2n


II
II
=
3.4.5 ..... (2n-I)(2n)
X [ADS: Ixl:::; legl, Ixl > Idgl ]
2. Test for the convergence the series
f 4 .7 . 10 ..... (3n+l) x" [ADS:
1
Ixl < -egl,lxl I
~ -dg/]
I lfl 3 3

[ Ans : divergent]

(ii) 3.4
- x +4.5 5.6 3 + ..... ( x> 0)
- x 2 +-x
1.2 2.3 3.4
[ ADS: cgt if x :::; 1 dgt if, x > 1 ]
~ 1.3.5 ..... (2n -1) xn ( )
(iii) L.. . ( ) x>O
2.4.6 ..... 2n 2n + 2
[ ADS: cgt if x :::; 1 dgt if, x > 1
2 3
(iv) 1+ (U)2
II
X + (1l)2 x
l1
+ (lJ)2 x
l2
+
..... .
(x > 0)

[ ADS: cgt if x < 4 and dgt if, x ~ 4

6.5 Cauchy's Root Test


I'
Let L un be a series of +ve terms and let LI u/n = I . Then
n-->co
L un is convergent

when I < 1 and divergent when I> 1


434 Engineering Mathematics - I

Proof:
I,' II
(i) Lt u/" =1<1=>3a +venumber 'A'(!<A<1):HI,/" <A,Vn>rn
I1~OO

(or) u ll <A",Vn>rn
Since A < 1, I A" is a geometric series with common ratio < I and
therefore convergent.
Hence I UII is convergent.
II
(ii) Lt
11-->00
UII /11 = I> 1
:. By the definition of a limit we can find a number r 3 u/ > 1Vn > r
'II

i.e., ull > Vn > r


i.e., after the 1sl 'r ' terms, each term is> 1.
Lt
11-->00
2>11 = 00 :. LUll is divergent.
Note:
When Lt{ In) Un = 1, the root test can't decide the nature of LUll' The fact
n-->oo

of this statement can be observed by the following two examples.

1 ) J~ )3 =1
1. Consider the series I Yn3n :-LfU'/" =Lt
11-->00
1/

11-->00
(
-3
n
= Lt ( V1
11-->00 nl II

2. Consider the series L Yn, in which LfUlly" = Lt ~ = 1


11-->00 11-->00 n11l
1/
In both the examples 'given above, LfU/" = 1. But series (1) is convergent
(p-series test)
And series (2) is divergent. Hence when the limit= 1, the test fails.

Solved Examples

6.5.1 Example
Test for convergence the infinite series whose nIh terms are:
(i) 1 (ii) 1 (iii) 1
n'" Oogn)" [1 +~r
Sequences of Series 435

Solution

, 1
. Lt u 1n
Lt -n2 = 0 < 1
= lI~oo
, n~co II '

By root test I Un is convergent.

1 ~. 1 ,. 1
(ii) u = u n = - - . Lt u 'n = Lt --=0<1
n (log n)'" n log n ' n->oo n n->oo log n '
By root test, I Un is convergent.

(iii)

6.5.2 Example
Find whether the following series are convergent or ,) i vergent.
'" 1
(i) I-n-
-1
n=13

Solution
1/
In

1/
7n

1 1
Lt un Yn = n----+oo
Lt =-<1
n~ct) 3 '

By root test, I Un is convergent.


111
(ii) 1+-2 +3+-4 + .....
234
436 Engineering Mathematics - I

Solution

Lt ( - 1 )}~ = 0 < 1
1 ' Lt U in 1/
1I
11
=-
nil 'Il~OO n
= Il~OO nil

By root test, I lin is convergent.

(iii)

Solution

U =
[(n+1)x r
-'=--'----':--=----
n n"+1

r
II

Lt u};, = Lt [{(n+1)x ],'n


n---j.c:/J n Il~OO n"+ 1

Lt [{ ( n + ] ) X}" .-
1]), _
- ~ n + 1)
Lt ( - - x '1I-'
n-->oo n n n-->oo n nl n

(l+!)X.+, = Lt x.+,=x
Lt
n n'II n-->oo n/ n
n-->oo

:. I Un is convergent if Ixl < 1 and divergent if Ixl > 1 and when Ixi = 1 the test
fails.
1
Then Take vn =-
n

vn n"+ 1 nn
r
Un = (n + 1 .n = (n + 1 = (1 + !)n ;
n
r U
Lt-"=e>l
11-->00 VII

:. By comparison test, I Uti is divergent.


(I Vn diverges by p -series test)
Hence I Un is convergent if Ixl < 1 and divergent Ixl ~ 1
6.5.3 Example
,/
If un = n ,,2' show that I Ull is convergent
(n+l)
Sequences of Series 437

Ill

l
2
I n n II n
Lt u" " = Lt /1
2 ; = Lt = " = Lt (-)-"
"~a) II~OO n + 1)" ,,~oo
( n + 1)
J ,,~oo n + 1 (

=
,,~OO l I e
+-
L.
~
Lt [_1_]" = < 1 ;:. "u" converges by root test.
n
6.5.4 Example

Establish the convergence of the series r


~ + ( % + ( % + .' ...... J
Solution

U
11
= (_n_)" . . . . .
2n+l
(verify);

Lt u };, = Lt (_n_)=!<1
,,~oo n II~OO 2n + 1 2
By root test, I UII is convergt:nt.

6.5.5 Example

Test for the convergence of Ioog


--.x
/I

11=1 n+ 1
Solution

U/I = [_11 ]~ .x"; Lt u"v,,


,,~oo
= Lt
Il---+oo
[_11 JI x =x
1+- 1+-
n n
IU is convergent if Ixl < 1 and divergent if Ixl > 1.
:. By root test, n

When Ixl = 1 : un = J n ,taking vn = ~ and applying comparison test, it can


n+I n
be seen that is divergent
I U/I is convergent if Ixl < 1 and divergent if Ixl ~ 1 .
438 Engineering Mathematics - I

6.5.6 Example

Show that ~ ( n~ _ I) n converges.


Solution

Un =( nYn -If
Lt
n~oo
unYn = Lt
n . . . . . ct)
(n~-I)=I-I=O<I(since Lt
1l~Cf)
n'~:' =1);
:. L Un is convergent by root test.

6.5.7 Example

. the convergence 0 f the series whose nI h


Examine termIS ( n + 2 )n .Xn
--
n+3
Solution

Un = ( n+ 2 )n .xn; Lt unYn = Lt (nn+3


2
+ ) X = X
n+3 n~ n~<XJ

:. By root test, L un converges when Ixl < 1 and diverges when Ixl > 1
2)n
n
(1+-
When Ixl = 1 : un = (n +
n+3
2) ; Lt Un n~<XJ
= Lt
n~<XJ ( 1+-3 )
nn
n
2
e I
- 3 = - '" 0 and the terms are all +ve .
e e
:. L un is divergent. Hence L un is convergent if Ixl < 1 and divergent if
Ixl~l.
6.5.8 Example
Show that the series,

[~: -HI +[~: -H' +[:: -H +. . . 3

isconvergent

u" ~ [(n :":t -n: r~ (n: T[( n: r r 1 1


-1
Sequences of Series 439

~ 1 1 1
Lt un II =-.--=--<1
IHOC) 1 e-l e-l
:. By root test, LUll is convergent.

6.5.9 Example
00 -Ill
e
Test Llllll for convergence when u m = 2

111=1
(1+2 mfll
Solution

Hence Cauchy's root tells us that LU m is divergent.

6.5.11 Example:
Test the convergence of the series '" ~
~ 112
e
Solution
II
II n/n
Lt
11-)-00
U /n
n
= n--.r:1J
Lt - = 0 < 1
en
:. By root test, LU n is convergent.

6.5.12 Example
2 32 2 .
Test the convergence of the senes, -2 x + - 3 X + ...
( n + 1 .x
I
r l
+ ...... , x > 0
1 2 n'H
440 Engineering Mathematics - I

Solution

n+I ) 11 .xn ]);, n+I 1


Lt unYn
I
= Lt [(
= Lt [ (- - ) .-/.x]
n~oo II~OO n"+ 1 II~OO n n)'n

= Lt
n~oo
[( + -1). 1]
1
n
I/.x
n l II
= [.
I.I.x = x SInce Lt n/Vn = 1]
II~OO

:. By root test, L un converges if x < 1 and diverges when x> 1 .


When x = 1 , the test fails.
I)n _.
= 1 +- 1 1
Then II11 . , Take vn =-
( n n n

Lt un
n~oo Vn
= Lt
II~OO
(1 + !)n
n
=e 0 :;t:

By comparison test and p-series test, L un is divergent.


Hence L un is convergent when x < 1 and divergent when x ~ 1.

Exercise 1 (d)

1. Test for convergence the infinite series whose d h terms are:


1
(a) .............................. [ Ans : convergent]
2" -1
1
(b) ---'2::--n ( n :;t: 1) [ Ans : convergent]
(log)
3n+ 1 )n 4 4
(c) ( 4n+3 x ................................ . [Ans: Ixl < -cgt,lxl
3
~ -dgt ]
3

(d) [ Ans: cgt for all x ~ 0)

(e) [ Ans: convergent)

(t) [ Ans : convergent]


Sequences of Series 441

(g)
(2n2 -I r [ Ans: convergent]
(2n )211

(h) (n}~ -I rn [ Ans: convergent]

-n) . . . . . . . . . . . . . . . . . .
_112

n-I
(i) (
[ Ans : divergent]

(j) (n:l)"'(x>o) ........................ . [ Ans : x < 1 cgt, x ~ 1 dgt ]

2. Examine the following series for convergence :


x x2 x3
(a) 1+2+)2+43 + ..... ,x>O ................. . [ Ans: x ~ 1cgt,x > Idgt ]

(b) ~+(H +UoJ +. . . . . . . . . . . . . .. [ Ans : convergent]

6.6
6.6.1 Integral Test
+ve term series,
(1)+(2)+ ..... +(n )+ .....
where (n) decreases as n increases is convergent or divergent according as the
integral
Proof:
r (x) dx is finite or infinite.

Let Sn = (l) +(2)+ ..... +(n)


From the above figure, it can be seen that the area under the curve y = (x)
between any two ordinates lies between the set of exterior and interior rectangles
formed by the ordinates at
n = I, 2, 3, .....n, n + I , ..... .
Hence the total area under the curve lies between the sum of areas of all interior
rectangles and sum of the areas of all the exterior rectangles.
Hence
{(l) + (2) + ..... + ( n)} ~ r+ (x)dx ~
1
{(2)+ (3) + ..... + ( n+ I)}

.. Sn ~ r (x)dx~Sn+I-(l)
442 Engineering Mathematics - I

v= O(n)

P,

P2
0
1

Q Cn +1

3 Bntl
11~1
0
Il
1 2 3 n n +1
AI A2 A) An ~

As n ~ 00, Lt SI1 is finite or infinite according as [ (x)dx is finite or infinite.


Hence the theorem.

Solved Examples

6.6.2 Example
1
Test for convergence the series I ---
00

n logn
n=2

Solution

r
- xlogx
1 It [ r' 1 dx]
dx - n~oo 1 xlogx
= It [log log
11~00
x]~ = 00

By integral test, the given series is divergent.


6.6.3 Example

Test for convergence of the series

Solution

dx - It [f-
p 1

[ _1
xP x
1
dx]
11~00 P
It [ x- + ]11
n~oo - p + 1 I

= _1_ Lt [ n l - p -1 ]
1- P IHOO
Sequences of Series 443

ClIse (i)
,,1 .
If P > I, this limit is finite; L..J - IS convergent.
n"
ClIse (ii)
,,1 . .
If P < 1, the limit is in finite; L..J- IS dIvergent.
n"
Ca.\e (iii)

Ifp= 1, the limit Lt logxl;' = Lt (logn)=oo;:. I_l- IS divergent.


11...--.)00 11-)00 n"
Hence I_l- is convergent if p > I and divergent if p :S
n"
6.6.4 Example
n
Test the series "-2
~e"
00

for convergence.

Solution
n
1111 = -2 = (n){say);
e"
(n) is +ve and decreases as n increases. So let liS apply the integral test.
1(x){lx= x
I
I e-,2
I
dx =! le- dt{t=x
2I
1 2
,dt=2xdx}

= -"21e 00
-I 1
I = -"21(0 --;;1) = 2e
1' W IlIC. hIS fiillIte.
.

By integral test, IlIl1 is convergent.


6.6.5 Example
1
Apply integral test to test the convergence of the series "-sin - 00 (7f)
~n2 11

Solution

Let (n) = ~2 sin ( : ); (n) decreases as 11 increases and is +ve .

J(x)dx =
00 00
J-21 sin
(}x
7f
2 2 X X
444 Engineering Mathematics - I

Letix =t

TC
J
-! sintdt = !costl~/=!
~ TC /2 TC
finite, _TC/
/ x
2dx = dt

Yx2dx=- ~dt
:. By integral test, I Un converges x = 2 => t = ~ x = 00 => t = 0
6.6.6 Example
Apply integral test and determine the convergence ofthe following series.

(a) (b) (c)

Solution
(a) (n) - 3n is +ve and decreases as n increases
- 4n 2 + 1

~x dx (4X +1=t=>Xdx= Ysdt )


2
j(x)dx= j
I 14x +1 x=l=>t=5,x=00=>t=00

I ( x ) dx = Lt [3-8 f-dt]
00

I
t
n~oo
= Lt [3-log t -log 5]=
8
5
I
n-->oo
00

:. By integral test, I Un diverges.

(b) ( n) = ~n3 decreases as n increases and is +ve


3n +2
j(x)dx =
I
1~X3
13x +2
dx

1 OOfdt 1
= (; t
= (;[logt]5 = 00
00 4
[where t= 3x + 2]
5

By integral test, I Un is divergent.

(c) (n) = _1_ is +ve, and decreases as n increases.


3n+l
00 1 1 dt
00 1 00

f(x)dx= f-dx= f--[t=3x+l]=-logtl~=00


I I 3x+ 1 4 3 t 3
Sequences of Series 445

:. By integral test, I Un is divergent.


6.7
6.7.1 Alternating Series
A series, u l - u2 + u3 - u4 + - + ..... + ( -1 ),,-1 un + ..... , where un are all +ve,
is an alternating series.
6.7.2 Leibneitz Test

If in an alternating series I (-1 ),,-1 .un ' where un are alI +ve,

(i) un > un+I ' Vn, and


(ii) Lt ull = 0, then the series is convergent.
n ........ oo
Proof:
Let u l - u2 + u3 - u4 + ...... be an alternating series (' un 'are all +ve)
Let ul > u2 > u3 > u4 , Then the series may be written in each of the following
two forms:
(UI -U 2)+(U3-U4)+(U5-u6)+ ......... (A)

ul -(u 2 -U3 )-(U4 -u5 )- ............ (B)


(A) Shows that the sum of any number of terms is +ve and
(B) Shows that the sum of any number of terms is < ul .
Hence the sum of the series is finite. :. The series is convergent.
Note:
If Lt ull -:t:- 0, then the series is oscillatory.
n--.<XJ

Solved Examples

6.7.3 Example
Cons!derthe series
. 1--+---
1 1 1 .....
234
In this series, each term is numerically less than its preceding term and nih term
~O as n~oo.
:. By Leibneitz's test, the series is convergent.
(Note the sum of the above series is Loge2 )
6.7.4 Example

Test for convergence L (-I ),,-1


2n-1
446 Engineering Mathematics - I

Solution

The given series is an alternating series I (-1 ),,-1 un ,where U


n
= _I_
2n-I
We observe that (i) un > 0, Vn (ii) un > Un+ I ' Vn (iii) Lt
/1---700
Un =0
:. By Leibneitz's test, the given series is convergent.
6.7.5 Example
111
Show that the series S - 1- - + - - - + ...... converges.
3 9 27
Solution
( ),,-1
The given series is ~
~
-1 1
3n-
= "'(_1)/1-1
L.J
u"
I
where u, = _1_1 is an alternating
I 3/1-
series in which 1. un > 0, Vn 2. un > U/I+1' Vn and 3. Lt un = 0;
ll---7oo

Hence by Leibneitz's test, it is convergent.


6.7.6 Example
X 2 x3
Test for convergence of the series, - - - ~ + - - 3 - +..... , 0 < x < 1
1+x 1+x- 1+x
Solution
n-1
( -1 ) .xn n-1
The given series is of the form'" = "'(-1) u,
L.J 1+ XII L.J n

x"
where UII=--II Since O<x< 1, ulI>O,Vn;
. I+x'
x" X"+1
Further, U -u =--
n ----
II n+1 l+x I+XIl+1
x" _X"+1 x" (I-x)
=--:-----:--:------;-

( 1+ x" ) ( I + x"+ 1 )
0< x < 1 ~ all tenns in numerator and denominator of the above expression are
+ve.

Again, x" ~ 0 as x" ~ 00 since 0 < x < 1; .'. Lt U = --.


o =0
II~OO II 1+0
:. By Leibneitz's test, the given series is convergent.
Sequences of Series 447

6.7.7 Example
(-1 ),,-1
L ----r========
00

Test for convergence


1I~2 ~11(11+1)(n+2)
Solution
The given series is an alternating series "(-1)"-1
L..J ull
1
where
1111 = ~11 ( 11 + 1)( 11 + 2) ; u > 0, \1'11 ;
ll

Again, ~(11+1)(n+2)(n+3) >~n(I1+1)(n+2)


1 1
~(11+1)(n+2)(11+3) < JI1(I1+1)(11+2) ,\1'11
I.e.,
1
Further, Lt u = Lt =0
IHOO II 11-->00 J 11 ( 11 + 1)( 11 + 2 )

By Leibnitz's test, f( -1)"-1 un


2
is convergent

6.7.8 Example
Test for the convergence of the following series,
1 2 3 4 5
---+---+--+
6 11 16 21 26 ....
Solution
11
Given series, L (-1)
00

n=1
n-I
- - = L (-1)
511 + 1
II-I
Un is an alternating series

11
u =-->0\1'11 .
n 511 + 1 '
11 11+1 -1
----- ( )( ) ~ Un < Un+ I' \1'11
511+1 511+6 511+1 511+6
11 1
Again, Lt u = Lt --=-:;t:0
n-->oo n IHOO 511 + 1 5
Thus conditions (ii) or (iii) of Leibnitz's test are not satisfied. The given series is
not convergent. It is oscillatory.
448 Engineering Mathematics - I

6.7.9 Example
Test the nature of the following series.

(a) I
00 ( -1 r- I

(b) I --'--::--'r---
( -1
2
I

(c)
1 f;z +J;+i n +1
Solution
1
(a) u,,= I ~>OVn
vn+vn+l
1 1
U" - U,,+I = f;z + J;+i - J;+i + J n + 2

:. By Leibnitz's test the series converges.


1 1 1
(b) u" =-2->0,Vn; - 2 - > 2 =>U" >un+I,Vn;
n +1 n +1 (n+l) +1
Lt
n-->oo
UI/ =0 :. By Leibnitz's test, given series converges.
1
(c) un = I ~.l 1 > 0, Vn ;
~
1 1
In + 2 > In + 1 => I ~ < I ~.1 1 => UI/ > lln+P V n
I ')

~~
By Leibnitz's test, given series converges.
6.7.10 Example
111
Test the convergence of the series sJ2 - sJ3 + sJ4 - +..... .
Solution
(_1)"-1 1
I
00

The series can be written as -'-;===== U =---;==


n=1 sJ;+i /I SJn+l
(i)

(ii)

(iii) Lt un
n-->oo
=0 .
'

By Leibnitz's test, the given series converges.


Sequences of Series 449

6.7.11 Example
Test for convergence the series, 1- h + ~ - ~ + .....
Solution

The given series can be written as


(-If (omitting \'1 term)
I--
2n
1 1 1 1
->OVn; ->--=>u >lll/+I'Vn; Lt -=0 lI
2n 2n 2n + 2 2n IHOO

(-If
:. By Leibnitz's test, I--
2n
is convergent.

6.7.12 Example
1 1 1
Test for convergence the series, 1- - + - - - + ..... .
3! 5! 7!
Solution

General tenn of the series is /-1)"-;


2n-I !

The series is an alternating series; ( 1 ) > OV n


2n-I !
1 1 1
>( ) => Un> Un+ I' Vn EN; Lt ( ) =0
( 2n - I!
) 2n - 1 ! IHOO 2n - 1 !

By Leibnitz's test, given series is convergent.

6.8 Absolute convergence


A series IUII is said to be absolutely convergent if the series I IU I IS
II

convergent
6.8.1
Consider the series
1 1 1
IU =1--3 +3"--3 +-+ ......
II
234
1 1 1 1 00

Ilun l=1+-3 +3"+-3 + ...... = I-3


2 3 4 I n

By p - series test, I I
IUn is convergent (p = 3 > 1 )
450 Engineering Mathematics - I

Hence I Un is absolutely convergent.


Note:
1. If I Un is a series of +ve terms, then I lIlI = I IUI/ I
For such a series, there is no difference between convergence and ahsolute
convergence. Thus a series of +ve terms is convergent as well as absolutely
convergent.
2. An absolutely convergent series is convergent. But the converse need not he
true.

Consider I (_1)"-1.-!-n =1-1..+1..


1 2 3 4
-1..+ ..... .

This series is convergent (1.7.3)

ButI I(-1 ),,-1 )~I =1+ ~ + ~ + ~ + ....... is divergent (p-series test).


Thus I is convergent need not imply that II I is convergent (i.e.,
Un UII

I is not absolutely convergent).


Uti

6.9
6.9.1 Conditional Convergence
[fthe series Ilu,,1
is divergent and I Un is convergent, then I Un is said
to be conditionally convergent.
6.9.2 Consider the Series
1-1..+.!-1.. .......
234
IU II
is convergent by Leibnitz's test. (Ex. 1.7.3)

But I IUn I= 1+ ~ + ~ + ~ ..... is divergent by p - series test.

:. I Uti is conditionally convergent.

6.10
6.10.1 Power Series and Interval of Convergence
A series, ao + a1x + a2 x 2 + ..... + anx n + ..... where 'an' are all constants is a
power series in x.
It may converge for some values of x .
. Sequences of Series 451

Lt UII +I = Lt a/HI x (I st term is omitted.)


n~<Xl till n~<Xl all

=kx (say) where Lt a ll +1 = k


n~'" all

Then, by ratio test, the series converges when Ikxl < 1 .


i.e., it converges '\Ix E ( ~1 , ~ )(k *- 0)
The interval ,)
(~1 ~ is known as the interval of convergence of the given

power series.

Solved Examples

6.10.2 Example

Find the interval of convergence of the series I \


'" II

11;1 n
Solution

"Lf.( U~:, )="Lf.(n: l)'x =H.[ ~.~


By ratio test, the given series converges when
1

Ixl < 1, i.e., x


I
J: =

E
x
(-1,1)
When x = 1, L un = L in 3' which, is convergent by p series test.

:. L un is convergent when x =1
Hence, the interval of convergence of the given series is (-1, I]
6.10.3 Example
Test for the convergence of the following series.

(a) 1- YJ2+ YJ3- YJ4+


(b) 1+ X2- }i2- ~2+ Ys2+ ~2- (;2- h2+ . . . .
452 Engineering Mathematics - I

(c) I-X~+X~-~+ ......... .


n
(d) I(-I)"(n + l)x , with x <-.!..
o 2
Solution
(a) The series is of the form L (-I),,-I un where un = y.;;;
It is an alternating series where (i) un > OV n (ii) Un > 1l,l+IVn and
(iii) Lt
n~oo
un = 0;:. By Leibnitz test, the series is convergent.

Again the series 1+ YJ2 YJ3 YJ4


+ + + ...... is divergent, by

p - series test.
Hence the given series is conditionally convergent.
00

(b) L lu,,1 = L;;:2


p=1
which is convergent by p - series test.

The given series is absolutely convergent.

It is convergent.

(c) The given series is


2n-2 2n-2
"(_I)n-l. x
~
. ="(-1 ),,-I U . Iu
(2n-2)! ~ n' n
I=_x__
(2n-2)!

-U n +1 = 1 1 21 ; Lt -lln+1
.x =0 < 1
u" (2n -1 )(2n) tHoo ll"

By ratio test, the series Llu,,1 converges Vx; i.e., L u" is absolutely
convergent Vx;
(d) Here, Iunl =(n + I)xn;1 un+1 1= (n + 2)x n+1 (neglect 1 sl term)

lu I
Lt ~ = Lt
(n + 2)
Ixl = Lt
(1 + 7n)~ Ixl =Ixl < 1 }i
(": x < 12 )
IHOO Iunl IHOO (n + 1) IHOO (1 + Yn)

:. Llunl is convergent Vx, i.e., given series is absolutely convergent

and hence convergent.


Sequences of Series 453

6.10.4
Show that the series 1 + x + x/6 + x/6 + ..... converges absolutely \;Ix

Solution

Lt IU'HII =13 =0 < 1 when x*-O [since Ix"-II Iu 1= Ix"l ]


111" 1= (n-I)!'
'1->00 lUl I n . n!
IItl

:. By ratio test, Ilunl is convergent \;Ix *- O.


When x = 0, the series is (1 + 0 + 0 + ...... ) and is convergent
:. Ilu,,1 converges ~ Ill" is absolutely convergent \;Ix.
6.10.5 Example
Show that the series, 1 - ~ + ~ - ~4 + ........ is absolutely convergent.
3 3- 3
Solution

, which is a geometric series with common ratio ~ <1


3
:. It is convergent. Hence given series is absolutely convergent.
6.10.6 Example
Test for convergence, absolute convergence and conditional convergence of the
series,
1 ] 1
1--+---+ ......... .
5 9 13
Solution
The given alternating series is of the form I (-1 r- I
Un' where, un = _1_ .
4n-3
1 1
Hence, u" > O\;ln EN; U,,+I = 4 (n+ 1) -3 = 4n+ 1
1 1
u -u =-----
n ,,+1 4n - 3 4n + 1
4n+I-4n+3 4
= - - - - - - > 0, \;In E N
(4n - 3)( 4n + 1)
(4n - 3)( 411 + 1)
1
I.e., un > un+I ' \;In EN Lt II = Lt - - = 0;
n-+oo 11 fl~a) 4n - 3
All conditions of Leibnitz's test are satisfied.
Hence (-1 I I
r-
un is convergent.
454 Engineering Mathematics - I

lu 1=-1-.
/l
4n - 3 '
Take v =! .
11'
n
Lt
//--+00
lunl = Lt
v" ,,--+00
n I
n( 4 - 3/)
=! *- 0
4
and finite.
n
By comparison test, I 111// I and I v" behave alike.
But by p - series test, I v" is divergent (since p = 1) .
Ilu// I is divergent and :. The given series is conditionally convergent.

6.10.7 Example

Test the series I (_1)"-1.


//=1
I, ,for absolute / conditional convergence.
}\1 n
Solution
The given series is an alternating series of the form I (-1 r-Iu// .
Here
1
(i) un = 3vn" Vn EN

(ii) 3(n+l3n=>3-Fn+l >3,Jn,Vn.


1 1. \-I N
:. 3.Jn+l <3,Jn ,I.e., ll'/+I <u//,vnE

1
And Lt
n~oo
un = Lt
n---)-oo
, =0
3" n
:. By Leibnitz's test, the given series is convergent.

But I (_1)//-1.__ =
3,Jn
1
I_I-
3,Jn
is divergent by p- series test (since

1
p= -<1)
2
:. The given series is conditionally convergent.

6.10.8 Example
Test the following series for absolute / conditional convergence.

(a) I(-If-I , sin(~a) (b) I(-I)"-I.~


//=1 n n=1 n +1

(c)
I (-1)" (d) " (_1)"-1.
L..J
nJr
e3n+1
n

,,=0 (2n)!
Sequences of Series 455

Solution

(a) lu,,1 = Isinnal


112
1 [
< n 2 since Isin nal < 1
]
considering VII
II
= 1112 and using

comparison and p - series tests, we get that I lUl I is convergent I lin is


absolutely convergent.
(b) By Leibnitz's test, the series converges.
.
Tak1l1g v" =-
1
n
.
,by companson and p- series tests, . "n. 2
~ - ,- , IS
n- +1
seen to

be divergent.
Hence given series is conditionally convergent.

(c) Take lu,,1 = _1_;


2n!
Lt
,,->0')
II I I= 0 < 1; By ratio test, I lUl I is convergent;
III +
lIlI

Hence given series is absolutely convergent.


Jt'''
(d) lu,,1 = e~)"+I ; By root test, is convergent, given senes is absolutely

convergent.
[In problems (a) to (d) above, hints only are given. Students are advised to
do the complete problem themselves]
6.10.9 Example
Find the interval of convergence of the following series.

(a) :t (_1)"-1 xX
,,~I
3
(b):t (_1)"-1 n(x:3 2)"
II~I
(c) log(l+x)
Solution
\ "\ \"+1\
(a) Let the given series be I lIlI ; Then Iunl = ~ ; lun+,1 = X 3
n (n+ 1)

/3 [_11J' lxHx:
00

1+--
n
:. By ratio test, Ilu,,1 is convergent if Ixl < 1
I Un is absolutely convergent if Ixl < 1 ;
i.e.,
:. I u" is convergent if Ixl < 1
456 Engineering Mathematics - I

1 1 1
Ifx= I,thegivenseriesbecomes 1-23"+ 33 - 4 3 + ....... .

W hIC I1 IS
. ."
convergent, sIllce 1 IS
~ -3 . convergent.
n
Similarly, if x = -1, the series becomes I -~ = -I ~ which is also
n n
convergent.
Hence the interval of convergence of I UII is (-1 ~ x ~ 1)
(b) Proceeding as in (a),
lulI+11
Lt-=--
Ix+21
IHOO Iu'" 3
:. I U is convergent if Ix + 21 < 3, i.e., if -3 < x+ 2 < 3, i.e., if
II

-5 < x < 1.
If x = -5, I lIlI =I (-llll-I .n ,and is divergent ( in both these cases

Ifx= I, IU = I( -1)"-1 .n, and is divergent


II
Lt
ll--i.rJJ
UII:I; 0 )

Hence the interval of convergence of the series is (-5 < x < I)


x2 x3 X4
(c) log (I +x)= x--+---+
234
...... .
I( -1)"-1 ~ = IU
00 II

= II
(say)
n=1 'n

Ixnl 111+11
IUnl=-;-; lun+1= :+1 1

lun+11
Lt - - = Lt 1 I x I= I I X
Iunl
/1-+00 (1 +~) /1-+00

By ratio test, I lUl I is convergent when Ixl < 1


i.e., I Un is absolutely convergent and hence convergent when -1 < < 1. x
When x = -I, "~ u = "(
~ -1 )11-1 . -1 = 1- -1 + -1 - -1 + ....... ,
ll
n 2 3 4
which is convergent by Leibnitz's test. (give the proof)
Sequences of Series 457

Whenx= 1, :L>n = -(1+~+!+~+


234
...... ) which is divergent,

since L ~n is divergent by p -series test (prove).

Hence I Un is convergent when -1 < x ::; 1


Interval of convergence is ( -1 < x ::; 1 ).

Exercise -1 (e)

1. Use integral test and determine the convergence or divergence of the following
series:
1
Ln 2
[ Ans : convergent]

00 1
2. ~ n{logn )2 [ Ans : convergent]

2. Test for convergence of the following series:


1 1 1
l-~ +l:!: -l2 + ...................................... [ Ans : convergent]

'" {-If-l
2. ~(2n-I)(2n) [ Ans : convergent]

[ Ans : convergent]
I
3. Classify the following series into absolutely convergent and conditionally
convergentseries:

[ Ans : abs.cgt ]

[ Ans : abs.cgt ]

{-If
[ Ans : abs.cgt ]
3. I n{logn)2
4. Find the interval of convergence of the following series
2n x n
1. L ~ [ Ans : -00 < x < 00 ]
458 Engineering Mathematics - I

[ Ans : - 1 ::;; x ::;; 1 ]

r Ans : -1 < x ::;; 1 ]

111
5. (a) Show that 1- - 2 + - ? - - 2 + .... is absolutely convergent.
2 3- 4
111
(b) Show that 1 - J2 + j j - J4 + .... is conditionally convergent.

Tests of Convergence - A Summary


00

I. The geometric series Lx n I


- converges if 1x 1< 1 , diverges if x ~ 1, and oscilates
n=1
when x::;;-1
2. If L u" is convergent, Lt ull
n->oo
=0 [The convergent need not be necessary ]

3. P - series test:- f ~n
11=1
is convergent if p > 1 and divergent if p::;; 1

4.-C-omparison lest :- The series LUll and L vn are both convergent or both

divergent if Lt ~ is finite and non - zero.


n->oo VII

5. D 'A/ember! 'sRatio test:- L un converges or diverges according as


U
Lt 2:':l. < 1 or > 1
n->oo u ll

(
Alternately, if Lt
n->oo
~
u
>1
+
or < 1) . If the limit = 1, the test fails
ll 1

6. Raabe's test: L un converges or diverges according as

.~HU::1 -I}]> lor <I .


7. Cauchy's root test: LU n converges or diverges according as Lt
n->oo
(U}) < lor> 1

(Iflimit = 1, the test fails.)


Sequences of Series 459

8. Integral test: A series L (n) of +ve terms where (n) decreases as n increases
co

is convergent or divergent according as the integral J (x) dx is finite or infinite.


1
co

9. Alternating series - Leibnitz 's test: An alternating series L (-1 ),,-1 un convergent
n=1

if(i) un = ull +1' \In and (ii) Lt un


Il---+CO
=0
10. Absolute / cO':lditional convergence:
(a) L
un is absolutely convergent if L Iun I is convergent.
(b) LUll is conditionally convergent if LUn is convergent and Llun I is
divergent.
(c) An absolutely convergent series is convergent, but converse need not be true
. i.e., a convergent series need not be convergent.

Miscellaneous Exercise - 1 (e)

1. Examine the convergence of the following series:


111
1. - + - + - + ........................................... . ( cgt.]
1.2 3.4 5.6
e 22 32
2. -3-+-3-+-3-+ ..... . ( dgt.]
1 +1 2 +1 3 +1
2 22 23
3. -+-+-+ ...... . ( dgt.]
1 2 3
1 2 3
4. 1+2 + 1+22 + 1+23 + ..... ( cgt.]
3
X x2 x
5. --+--2 +--3 + ..... (x>O) ........ ( cgt. if x~l dgt. ifx> I]
l+x l+x l+x
2 3
3x 4x
6. 2x+-+-+ ..... (x > 0) .......................... ( cgt. if x ~ 1 dgt. if x > 1)
8 27
7. 1+!+ 1.3 71.3.5 +.... ................................. (dgt.]
2 2.4.62~4-
32 32.5 2 32.5 2.7 2
8. 2" + -2-2 + 2 2 2 + ... ... ............................... [ cgt ]
6 6 .8 6 .8 .10
9. 3.4 + 4.5 + 5.6 +.... ................................ (dig.]
1.2 2.3 3.4
460 Engineering Mathematics - I

(l!)2 (tJ)2 2 (lJr ~


10. tJ .X+ l1 x + 12 x + .... (X>O) ............. [ege ifx<4,dgt. if x24 ]

X x2 x3
II. 1+ 22 +j2+4T+ ..... (x>O) ........................ . [ egt.if x::::; 1, dgt. if x > I]

12. 4+ (4 5
3x 5)2 x 2+(6)3 +x3+ ..... (x>o) ............ . [ egt if x < 1, dgt. if x 2 1 ]

13. L(l+~r [ dgt. ]

[ egt. ]

a"
15. 2:--2,a<1 [egt.]
l+n
111
16. 1 - - + - - - + - + -..... [ Abs. egt.]
2.2 3.3 4.4

2. Examine for absolute and conditional convergence of the following series:


n 3

"(_I)n 33 n
I. L.,; 2
.......... ......... ............. .... [ Abs. egt.]

(-l .n
2. 2: 2f n
[ Abs. egt. ]

[Cond. cgt]

[ Cond.egt.]

3. Determine the interval of convergence of the following series:


x 2 x 3 X4
x+-+-+-+ ..... .............................. .
234
" (x+lf [-3<x<1 ]
2. L.,;
n.2"
Sequences of Series 461

Exercise - 1 (g) (Objective type questions)

1. The infinite series

(i) convergent (ii) divergent


(iii)oscillatory (iv) ndne of these I ADS :(i) ]
. 1+n
2. T he senes - - 2 IS
1+n
(i) convergent (ii) divergent
(iii) oscillatory (iv) none of these I ADS :(ii) ]
. 1 1 1 1
3. The senes 1.J}- 2J2 + 3J3 - 414 + .....
(i) oscillatory (ii) absolutely convergent
(iii) conditionally convergent (iv) none of these [ADs :(ii)]

4. The series 1-.!. + ..!. -.!. ...... IS


234
(i) oscillatory (ii) divergent
(iii) convergent (iv) none of these I ADS :(iii) ]
3 4
X X X .
5. The interval of convergence of the series X--+---+ ...... ,IS
234
(i) -oo<x<oo (ii) -1 < x < 2
(iii) -1 < x :-::; 1 (iv) none of these I ADS :(iii) ]
6. The series _1_+~+~+ .........00 IS
1.2 3.4 5.6
(i) convergent (ii) divergent
(iii) oscillatory (iv) none of these I ADS :(ii) ]
123 .
7. T he senes - + - + - + .........00 , IS
1.3 3.5 5.7
(i) conditionally convergent (ii) convergent
(iii) divergent (iv) none of these I ADs :(iii) ]
.2345
8. The senes - + - P + - + - P + ........00 is convergent if
V 2 JP 4
(i) p < 2 (ii) p= 2
(iii) p> 2 (iv) none of these I ADs :(iii) I
9. The series 6 - 10 + 4 + 6 - 10 + 4 + 6 - 10 + 4 + 6............. 00 is
(i) convergent (ii) oscillatory
(iii) divergent (iv) none of these [ADS :(ii)]
462 Engineering Mathematics - I

111
10. The series - + - + - + ......... IS
2.4 4.6 6.8
(i) convergent (ii) divergent
(iii) oscillatory (iv) none of these [Ans :(i) ]

2. Indicate whether the following statements are true or false:


1
I. The series " is convergent. [ False]
LJ 1+ Til
2
n +5
2. The series I 2n2 + 7 is not convergent. .. ................... . [ True]

1 1 1 . d'
3. The series U+ 12 + lJ + ........ IS Ivergent... ................ . [ False]

3
x x
4. The series x - - + - - + - ....... , converges when -1:s x :S 1.. [True]
3 5
(_1)"-1
5. The series " is absolutely convergent. ........................ [ True]
LJ n.5/l
6. The series x + 2X2 + 3x 3 + 4X4 + ....00 is convergent if x> I........ [ False]

x2 x3
+ - + ........00 is divergent if x ~ 1 ...............
7. The series x + - [True]
3 2
x 2! 3 3! 3 4! 4
8. The series 1+-+2"x +-3 x +-4 x + ...... +00 is convergent
2 3 4 5
if x> e [ True]
-"J;;
9. The series I ( 1 + .);; ) is divergent ............................ . [ False]

10. The series "21 +"32 x + ( 4"3)2 x 2+"5


( 4 )3 x 3+ ....... IS. convergent
ifx< I . [ True]
II. The series 1- 2x + 3x 2 - 4x 3 + .....00 ( x < 1) is divergent... ........ . [ False]
2 3 4
. x
12. The senes - - - --2
x +-- x 3 - --
x . T]
4 + ......00 IS convergent ...... [ rue
l+x l+x l+x l+x
. sin x sin 2x sin 3x
13. The senes -3- - --3- + --3- + ......00 converges absolutely ...... [True]
1 2 3
Sequences of Series 463

14. The series I (~1-1 is conditionally convergent. [ True]

2
Ih. 3n + 5
15. The sertes whose n term IS is convergent. [ False]
(n+2)"
3. Fill in the Blanks :

L ar l -
00
I
1. The geometric series converges if_ _ _ __ [ Ans: 1r 1< 1]
11=1

2. If a series of +ve terms ""


L.. un Lt
is convergent, n->oo un =- - - [Ans: 0 ]

3. I {{!n +I-n}
11=1
3
IS _ _ __ [ ADs: convergent. ]

4. If L 3n3-4
00

is divergent ,value ofp is _ _ __ [ Ans: ::;; 4]


1/=1 (n+5Y

5. The interval of convergence of I u" where u" ~ ( :: :~ r x, is _ __

[ Ans: -1 < x < 1]


6. L un is convergent series of +ve series. Then Lt
11-00
(un 11
1( ) is
----

( ADs: < 1]

7. The series 8 - 12 + 4 + 8 - 12 - 4 + ....... is ( ADs: Oscillatory]

8. If un > 0, V nand LUll is convergent, then Lt


11->00
[n {~-I}]
u
is _ __
+
n 1

[ADS: > 1]

9. If the series I(-Ifan,(all >OVn) is convergent, then for all values of n,!!JL
~ aMI

is- - - ( ADS: > 1]

10. If un =(I+!)-n2 , Lt u l/ n= _____ [ ADS: 1/e]


n n->oo
lI
464 Engineering Mathematics - I

Tests of Convergence - A Summary


00

1. The geometric series I X"-I converges if Ixl< I, diverges if x~ 1 and oscilates


n~1

when x~ 1.
2. IfIll" is convergent, Lt u"
Il-~OO
=0 [The converse need not be necessary ]

3. p - series test:- f ~n
,,~I
is convergent if p > I and divergent if p ~1
4. Comparison test: The series L u" and I Vn are both convergent or both

divergent if LI u" is finite and non - zero.


n~oo v"
5. D'ALEMBERT'S Ratio test : IU n converges or diverges according as

< l o r > 1 (Alternately, L t1l,,+1


- <lor >1). If the limit

= I, the test fai Is.


6. Raabe's test: I U" converges or diverges according as Lt
n-)oo
[n{~
Unt-I
-I}] I I
> or <

7. Cauchy's root test: I U" converges or diverges according as Lt


n~oo
(u~ J< 1 or> 1 ;
(If the Limit = I, The test fails).
8. Integral test: A series L rj> ( n) of +ve terms where rj> ( n) decreases as n increases
frj> ( x ) dx is finite or infinite.
00

is convergent or divergent according as integral


1
00

9. Alternating series - Leibnitz's test: An alternating series I (-1 ),,-1 un (where


n~1

Un> OVn ) is convergent if(i) un > lin+1 ' Vn (ii) Lt


n~oo
Un =0
10. Absolute/ conditional convergence:
(a) LUn is absolutely convergent if II Un I is convergent.
(b) IU n is conditionally convergent if Iu" is convergent and II U" I is
divergent.
(c) An absolutely convergent series is convergent but converse need not be true.
i.e., a convergent series need not be convergent.
Sequences of Series 465

Solved University Questions

1. Test the convergence of the series:


123
- - + - - + - - + ........ .
1.2.3 2.3.4 3.4.5
Solution
Let ull be the d h term of the series;
n 1
Then, U - -
II - n(n+1)(n+2) - (n+1)(n+2)
1 U n2
Let VII = -2 ; then, Lt --'!.. = Lt ( )( )
n 11->00 V
II
tHoo n +1 n +2
1
= Lt =1
11->00 ( 1) ( 2) ,
1+;; 1+;;

Which is non-zero and finite.


:. By comparison test, both LUll and L VII converge or diverge together.
But L VII is convergent by p-series test (p > 1):. LUll is convergent.

2. Show the every convergent sequence is bounded


Solution
Let (all) be a sequence which converges to a limit '/ 'say.
Lt
1l~C()
all = I => given any +ve number E, however small ,

we can always find an integer' III ' , ) , Ian -II <E, 'ifn ~ m
Taking E = jaIl -II < 1;
1, we have,
I.e., (I - 1) < all < (i + 1), 'if n ~ m
Let A = min {a"a 2 , am _,,(1 -1)} ,and fl = max {a"a 2 , am-,,(I + 1)}
Then obviously, A ~ an ~ fl, 'ifn EN;
Hence (an) is bounded.

3. Show that the series,


1 1 1
S =1--+---+ .......... converges.
3! 5! 7!
466 Engineering Mathematics - I

Solution
The given series is an alternating series 2)-1 ),,-1 u" '
1
where u" =( ) We observe that,
211-1 !
(i) ll" > 0 and u" > 1l,,+1' '1111 and
1
(ii) Lt II = Lt =0
,,--+00" IHoo ( 211 - 1) !

:. By Leibneitz's test [7 .2 ] the given series converges.


00

4. Show that the geometric series I qlll = 1+ q + q2 + ....... converges to the slim
111=0

1
- - when /q/ < 1 and diverges when /q/ ~ 1
l-q
Solution
See theorm 2.3 (replace' x ' by 'q') .
5. Define the convergence of a series. Explain the absolute convergence and
conditional convergence of a series. Test the convergence of series

I[l+ J,;r'
Solution
For theory part, refer 2.1,2.2, 8.1 ,9.1, and 9.2
,
Problem: Let u" = (1 + _l_)-n
J;;
; Lt (u~{,) = Lt (1 + _1_)-n
11--+00 J;; ,,--+00

1 1
~ H-[l+ J,;J =7<1

By Cauchy's root test, I Un is convergent.


1 1.3 2 1.3.5 3
6. Test the convergence of the series, 1+-x+-x +--x + .....
2 2.4 2.4.6
Given that x> O.
Solution
1.3.5 ..... (211-1) n
Omitting the first term of the series, we have, ------'--------'- x and
2.4.6 .... .211
Sequences of Series 467

1.3.5.(2n-l) 1/ 1.3.5 ..... (2n+l) n+1


UI/ = 2.4.6 .... 2n
X ; UI/+ I = (
2.4.6 ..... 2n + 2
).x;

L u n+ 1 L (2n + 1 )
1/100 ---;;: = 1/100 2n + 2 .x = x
By ratio test, LUI/ is convergent when x < 1, and divergent when x > 1

The ratio test fails when x = 1


Whenx= 1, ~-1 = 2n+2 -1 =_1_
U n +1 2n + 1 2n + 1

Lt [n(~-I)]= Lt (_n
n~oo 2n +
U Il +1 IHOO 1
)=!<1
2
:. By Raabe's test, LUI/ diverges.

:. The given series converges when x < 1 and diverges when x 21 .

7. Test the convergence of the series, 1 + ( 2 ) x 2" ( 3 )2 x 2 +"5


"3 +"4 (4 )) x 3 +.........x > 0
Solution
Neglecting the 1st term,

u" =[(=:~H'
u 1n
1/
II
= (n+l)
- - X= _ _
n+2
n x
1+2/
[1+ 1/]
n
Lt uny.n =
ll----too
x; By Cauchy's root test, "~u// is cgt. when x < I and dgt. when x >

1; when x = 1 , the test fails.

(1 + l/~r
When x = 1, un =( , )n;
1+ 3/~
Lt Un = ee = -e1* 0
IHOO
-2

:. LU n is divergent.

:. is cgt. when x < 1 and dgt. when x 2 1 .


468 Engineering Mathematics - I

8. Test the series whose nih term is (3n -1) / 2n for convergence.

Solution
(3n-l) {3(n+l)-I}
U
n
= 2"
.
'
U
IHI
=-'--------'-
2"+1
U,,+I _ (3n + 2) - - -- 1
<1
L (Un+1
un 2(3n-l) n->oo Un 2

:. By ratio test, L un is convergent.


1
L <X)

9. Show by Cauchy's integral test that the series


n;2 n(logn r converges if p > 1

and diverges if 0 < p ~ 1


Solution
1
Let ( x ) = ; x ~ 2 ; Then ( x) decreases as x increases in [2,00]
x(logxy
<X) 00 ax <X) d I-p

f(x)dx=f x(logx)
2 2
p = I --;;;
log2
U
=~ II
P
1
00

log2 ;

. I
[Takmg log x = u, - ax = du x = 2 => u = log 2 and x = 00 => U = 00 ]
x
Case (i) : p > 1 => 1- P < 0 => Integral is finite, and
Case (ii) : 0 < p ~ I => Integral is infinite.
Hence, by integral test, the given series converges if p > I and diverges when
O<p~1 .
JI

10. Test the convergence ofthe series I (I + J,;)"""


Solution
Sequences of Series 469

1 1
L1 U;,II = - <1 [2 <e <3I .
n~oo e
By Cauchy's root test, I u" is convergent.

11. Test the convergence of the se'ries, I,,~2 -(1)"


00 (

n-l
11
"
.x)' 0 < x < I

Solution
xn
The given series is of the form I (-1)" 11" ,where u" = (
n n-l
)

This is an alternating series in which (i) lin > 0 and 1I" > 1l1l+IVn EN.
further L1 ll" = o. Hence, by Leibnitz test, the series is convergent.
11-00

. . 1 x2 X4 x6
12. DISCLISS the convergence of the series, J. + r;; + r:; + t. + ......... .
2'1'1 3'1'2 4'1'3 5'1'4
Solution
2n
n Iii
term 0 f th
e series = u = x r--:-:; ( . . I sl term )
omlttmg
ll
(n+2)'I'n+l
X
21
1+2 "1I+1,J;;+2 rn+i 2

U
llt
1= (n + 3).J n + 2 ; ---;;: = (n + 3) .x

/HOO
LI ~= I/~OO lrVn.M.
It" (+ 3.~) 2l=x
LI
1
X
2
;

By ratio test, I converges if x "II


2
< 1 , i.e., if Ixl < 1, and diverges if
x
2
> 1 , i.e., if Ixl > 1 ;

When x
2
= 1, u = ll
1rn+i ;taking vn = -3-' ,
(n+2) n+l n i2
]I
U n/ 2
Lt = Lt =1
(1 + ,'n
2/) \fIT?n
hl
_II

n~oo vn n~oo n%

By comparison test, I Un and I VII both converge or diverge together;


470 Engineering Mathematics - I

But L vn is convergent by p-series test.


:. L un is convergent if Ixl ~ 1 and divergent if Ixl > 1
00 211
13. Test the convergence
<::1 of the series " " x
L..--~=
n~1 (n+ 1)J;;
Solution
X 211 x2n+2
U = . U =----==
n (n+l)J;;' 11+1 (n+2)rn+i
,r-:-; ~
u n+1 = vnvn+l. x2 = \jIT /~ .x 2 ; Lt u n +1 =x2;

un n+2 (l+;ln) 11-+00 Un

By ratio test, L un converges when Ixl < 1 and diverges for Ixl > 1 .

When Ixl = 1 , un = 3/ ( 1 .) taking VII = ~ and applying the comparision


n 2 1 + 1/ n/ 2
.n
test, we observe that LUll is convergent.
Hence L lin converges when Ixl ~ 1 and diverges when Ixl > 1 .
2 3 4
14. Find the interval of convergence of the series, ~+~+~+ .........oo
234
Solution
11+1
.
For tIle gIven serIes, un
x
= --; un +1
n+1 n+2

Lt un +1 = Lt ~
n~'" U
1+
n~oo ( 1+ 2/
II) X=X

n /n

By ratio test, L un converges when Ixl < 1 i.e., -1 < x < 1


1
When x = I U =--
, n n+1

Taking
un
;-=--.,
Vn 1+;11
n
Sequences of Series 471

and, Lt un = 1 *- 0 and finite.


II~'" VII

:. Both IU II
and I VII converge or diverge together.
But I Vn diverges:. IU n also diverges when x ~ I.

When x = -1, the given series is

-1 --I + -1 --1 + wIh I . .


lIC IS a ternatll1g senes \\ 1111
2 3 4 5
Ut > un+IVn and un ~ 0 as n ~ 00

By Leibneitz's test IU II
converges when x =-1

.. Interval of convergence is [(-1, 1) i.e., -1 ~ x <1


~ 1.3.5 .... (211+1)
15. Test the convergence of the series L.
11=1 2.5.8 .... (311+2)

Solution
1. 3.5.... ( 2n + 1) 1.3.5 .... {2n+3)
un = 2.5.8 .... (3n+2 ); un +1 = 2.5.8 .... (3n+5 )

~ = Lt r2 + ( ~
3
Un +1 _ 2n+3. Lt 11 )] = <1
Un 3n+5' n~'" Un n~'" l3 + ( 5 11
) 3
By ratio test, I UII is convergent.

(-If
16. Prove that the series I---
n(logn)3
converges absolutely.

Solution

Iu 1- n(1ognr
n -
I ; "'f
2 X(logxf
dx = "'f d!
log2 r'

(where t = log x) -11


= - : -2 = - - 1 ,which is finite.
t g log 2
By integral test LlulIl is convergent.
" L un converg..:s absolutely.
472 Engineering Mathematics - I

17. Test the convergence of the series L.J


'" (2n3 + I) X ,x> 0 II

n +1

Solution

n/I! term 0 ft l . .
le given series, U = -2n+
3-X
I II

n n +1
_[2(11+1)+1] n+1 _ 2n+3 n+1
un + 1 - 1 X - ,x
(n+1) +1 (n+1} +1
II 1 (2n+3).x"H (113+1)
=
Lt
IH"O
~I/~t
1111
1,1
11->00 { ( r
n + 1 + I}
x --'-----'---
xn ( 2n + I)

x=.x

l1y ratio test, IlIlI converges if x < I and diverges if x> I. If x = I the test fails.
2n+ I . 1
Whenx = I, un = - , - ; Takmg VII = 2 '
n +1 n
LI _"
It
= LI -2n3-+X
I
n- = 2"* 0
?
and filllte

II~'OO VII 11->00 n +1

I lin and I Vn converge or diverge together.

But I Vn converges:. I Un also converges.

Thus, I lIlI converges when x ~ 1 and diverges when x > 1.

(-I)" (log n )
18. Test the series I00
2 ' for absolute/conditional convergence
II~I n

Solution

Un = (-1)" (;ogn) ; IUIII=(lo~n)


n n
Sequences of Series 473

"'flO~, x dx -- ' 'f le-'dl I/


[taking logx=l, x=e ' , /xdx=log/]
2 x- log2

~ -Ie -, + e-, I:.2 ~ 0 - [I-log 2].e -'"" ~ i(!Og 2 -I),


which is finite.
:. By integral test ~]un is convergent => I I 1I11 converges absolutely.
(Note that I UII is cgt. by Leibnitz's test).

19. Test the convergence of the series I I


(log log n )"

Solution
Given that u, = I
I (log log n)"

LI u'1/n
n-->oo n
= LI
n-->oo
[I] =
log log n
0<1

By Cauchy's root test, I Un is convergcnt.

20. Find the interval of convergence of the series,


7
I x 3 1 3 x 5 1.3.5 x
x+-.-+-.-.-+--.-+ ..... .
2 3 2 4 5 2.4.6 7
Solution
1.3.5 ..... (2n-l) Xlll'l
Term of the series, ull = .( ) (neglecting I st term)
2.4.6 ..... 2n 2n + 1
1.3.5 ..... (2n-I)(2n+l) X211 +1
u lI
+1 = 2.4.6 .... .2n (
2n +)
2 . (2n + 3)

~/n
2
4
Lt u
11-->00 un
lI +1 = Lt [
n-->oo
(2n + 1)2
(2n + 2)( 2n + 3
X2]
r
Lt [ n ( + + I n2) X2] = x2
I/-'>OO 19
n 2( 4 + ~ + 6 n2 r
By ratio test, I Un converges when x < 1 , i.e.,
2
Ixl < 1 => -1 < x < 1
2
When x = 1 ,the test fails;
474 Engineering Mathematics - I

Then ~_1=(4112?+1011+6 -1)= ~11+5


ull
+1 411- + 211 + 1 411- + 211 + 1

Lt [11(~-1)]= Lt n2(8+1~) =2>1


IHOO lIn+1 IHOO n ( 4 + 7~ + /~~2 )
2

By Raabe's test, LUll converges when x


2
= 1, i.e., x = 1.
:. Interval of convergence of L lin is (-1:::; X:::; 1)
7
Vector Differentiation

7.1.1 Vector Point function and vector field


Let P be any point in a region 'D' of space. Let r be the position vector ofP. Ifthere
exists a vector function F corresponding to each P, then such a function F is called
a vector point function and the region D is called a vector field.
Note: In what follows i,j, k are unit vectors along X, Y, Z axes respectively
For example, consider the vector function
F = (x - y) i + xyj + yzk ..... (1 )

Let P be a point whose position vector is


r = 2i + j + 3k in the region D of space.
At P, the value of F is obtained by putting x = 2, Y = I, z = 3 in F.
I.e. At P, F = i + 2j + 3 k
Thus, to each point P of the region D, there corresponds a vector F given by the
vector function (I).
Hence F is a vecotr point function (of scalar variables x, y, z) and the region D
is a vector field.
476 Engineering Mathematics - I

Scalar point junctioll and scalar fielit


Ifthere exists a scalar 'f' given by a scalar function 'f' corresponding to each point P (with
position vector r) in a region 0 of space, of' is called a scalar point function and 0 is called
a scalar field.
As an example, let P be a point whose position vector is r = 2i + j + 3k.
Consider f= xyz + xy + z
Then the value of f at P is obtained by putting x = 2, y = I, z = 3
i.e., At P, f= 2.1.3 + 2.1 + 3 = II
Hence the scalar' II ' is attached to the point P.
The function 'f' is a scalar point function (of scalar variables x, y, z), and 0 is a
scalar field.
Note : There can be vector and scalar function of one or more scalar variables.
7.1.2 Differentiation of a vector
Ifr (u) = r,(u)i + r2(u}j + riu)k, (where rl' r2, r3 , are scalar functions of 'u') be a
vector function of' u' , then,

dr Lt Or Lt r{ u + ou) - r{ u)
du ou ~Oou ou ~O ou

~ dr,. dlj. dr2 . dr3 k


= L.,-l=-l+-}+-
du du du du
Example

If r(u) = (3u 2 + 5u + 6) i + 3U7 - 4uk, Find dr , when u = 1


du

dr = {~(3U2 +5U+6)li+{~(3U2)}j+{~(-4U)}k
du du 1 du du

Note: We can apply the above rule of derivative to the case of partial derivatives also
ex : If A = (ryz)i + (xyz}j - (3;Jyz2)k

a
2
A
find - - at the point (I, -I, 2)
axOv
Vector Differentiation 477

aA
-
ay
= { - a (2
ay
x yz )t. a {xy 2Z
1+-
Oy
}. j -a- {3xyz-
Oy
3 ') ')} k

= (x 2z)i + (2xyz)j - (6x1yz2) k

= 2xz i + 2yzj - 18x2yz2 k

At the point (I, -1,2)

a2 A
ax~v = 4i - 4j + 72k

7.1.3 Application to space curves


Let ret) = x(t)i + y(t)j + z(t) k represent the position vector of a point (x. y. z) on a
space curve whose equations are given by x = x(t), y = yet), z = z(t), where 't' is
time.

dr dx. dy. dz (K + M, Y + ~Y, z + 6Z)


Then -=-I+-J+-k
dt dt dt dt

and
x

(i) dr represents the velocity vector v (or tangent vector) of the point (x, y, z)
dt

(ii) d 2 r represents the acceleration vector a at the point (x, y, z)


dt 2

Ex: If a particle moves along a curve x = e-t, y = 2 cos 2t, z = 2 sin 2t, where 't' is time.
I) find velocity and acceleration at time t = 0, and
2) find also their magnitudes
Sol: r = xi + yj + zk
= (e-t)i + (2 cos 2t)j + (2sin 2t)k
478 Engineering Mathematics - I

dr d d d
v= - = - (e-~i + - (2cos 2t}j + - (2sin 2t)k
dt dt dt dt
= (-e-t)i -(4sin 2t}j + (4 cos 2t)k

d 2 r dv d d. d
a= -=-=-(-e-t)i-
2
-(4sm2t}j+ - (4cos2t)k
dt dt dt dt dt

= (e-t ) i-(8cos 2t)j-(8sin 2t) k .... (2)


Putting t = 0 in (I), velocity as t = 0 is v = -i + 4k
Magnitude = 10
putting t = 0 in (2) acceleration at t = 0 is a = i-8j
Magnitude = J65
7.2 Gradient of Scalar Function
7.2.1 The Vector differential operator 'DEL' or 'NABLA', denoted as 'V' ' is
defined by

=i
a .a
-+ }-+ -
k a
V' ax ay az (i,j, k are unit vectors in x, y, z directions)

This operator' V' ' is used in defining the gradient, divergence and curl.

Properties of' V' ' are similar to those of vectors. The operator is appled to both
vector and scalar functions.
7.2.2 Gradient

If ~ (x, y, z) is a scalar function, defined at each point (x, y, z) in a certain region of


space and is differentiable, the gradent of ~ (shortly written as grad ~) is defined
as,

.a .a
grad ~ = V' ~ = 1-+ }-+k-- ~
a)
( ax ay az

(which is a vector function)


Vector Differentiation 479

:. If ~ defines a scalar field, 'grad' ~ or V ~ defines a vector field.

7.2.3 Physical significance of 'grad ~' :

If ~ (x, y, z) = c (c being a constant) represents a surface, then 'grad ~'represents


the normal vector to the surface at the point (x, y, z)
For, if r = xi + yj + zk, is the position vector of the point (x, y, z) on the surface, we
have, dr = (dx) i + (dy) j + (dz) k which is in the tangent plane to the surface of
(x, y, z)

(.,' ~ =c)

:. The vector' V ~' which is 1.- r to the tangent plane is the normal vector to
~ = c at(x, y, z)

7.2.4 Directional Derivative

If a be any vector, V~.a which represents the component of V ~ in the direction of


lal .'~
a is known as the directional derivative of' ~ , in the direction of a,
(1) Physically the directional derivative is the rate of change of' ~ , in the direction
ofa.
(2) The directional derivative will be maximum in the direction of V ~ (i.e.,

a= V~) and the maximum value of the directional derivate = It~~ = IV~I
7.2.5 Some basic properties of the gradient
If ~ and \jJ are two scalar functions,
'J) grad (~+\jJ) = grad ~ + grad \jJ(or) V(~+\jJ)= V~+V\I'

Proof: grad (~+ \1') = ~


V( + \jJ) = {! (~+ \jJ)} i + { ; (~ + \jJ)} j + {! (~+ \I')} k
480 Engineering Mathematics - I

o~. ~. ~ ) + (Oljl.
(
= -I+-j+-k -I+-j+-k 8ljI. 8ljI )
ox oy OZ ox ay OZ

= Y'~+Y'ljI

(2) grad (~ljI)= ~ (grad\fl) + ljI(grad~) (or) Y'(~ljI) = ~ (Y'ljI)+(Y'~)\jJ

Proof: grad (~ljI) = Y'(~ljI)

= {! (~ljI)}i +{; (~ljI)} j + {! (~ljI)}k


<.

oljl a~).1+
= ( ~-+ljI- (8ljI
~-+- O~).j+ (8ljI ~)k
~-+ljI-
ox ox ay oy OZ OZ

= ~(Oljl i + oljl j + oljl k) + ljI (o~; + o~ j + o~ k) = ~(Y'ljI) + ljI(Y'~)


ox ay oz ox oy OZ

= ~(grad \fI) + ljI(grad ~)

~) ljI (grad~ ) - ~(gradljl )


(3) If ljI :;t: 0, grad ( ljI = (ljI)2

(Proof is left to the reader.)

Solved Examples

Ex. 7.2.6 Iff= x2yz, find gradfat the point (I, -2, 1).
o~. o~. o~
Sol: f= x2yz; ": grad ~ = Y'~ = 0/ + ay } + OZ k,

o 0 0
gradf= -8 (x2yz); + - (x2yz}j + -8 (x2yz)k
x ay 'Z

= (2xyz)i +(x2z}j + (x 2y)k

:. At the point (1, -2, ]), grad ~ = - 4; + j - 2k

Ex. 7.2.7 Find the unit normal to the surface xy +yz + zx = 3 at the point (I, 1, I).

Sol: If ~ = c is a surface, Y'~ is the normal to it.


Vector Differentiation 481

Here f= xy +yz + zx

= (y + z)i + (x + z)j + (y + x)k


:. normal at(l, 1,1) = 2i + 2j + 2k

. 2;+ 2j + 2k ;+ j+k
:. Umt normal = ~
22 + 22 + 22 J3
Ex.7.2.8 (a) Find the directional derivative ofl= 2e2x-ytz at (1,3, I) in a direction
towards the point (2, I, 3).

Sol : 1 = 2e2x-ytz; Vf = 2e 2x- ytz (2i - j + k )

Vll(1,3,1) = 2.e 2- 3+1(2i - j + k) = 4i - 2j + 2k


Let A = (1,3, I) and B = (2, 1,3),
AB = (2-1)i + (l-3)j + (3-1)k = i - 2j + 2k = a (say)

. .
DlrectlOna I denvatlve
. . JI1. the d'IrectlOn
. 0 f a = Vf. a
(4i-2j+2k).(i-2j+2k) (4+4+4)
= = =4
,Jl +4+4 3

(b) In the 'Problem (a)' find the maximum value of the directional derivative

Ans: Maximum value of directional derivative = IV11


= I 4i - 2j + 2k I = ,J16 + 4 + 4 = 2 J6

Ex.7.2.9 Find the acute angle between the surface xyz = 2 and:x2 + y + z3 = 6 at the
point (2, 1, 1).
Sol : Let 1 = xy2z = 4 be the surface (1)
-
482 Engineering Mathematics - I

Normal vector to (1) at (2, 1, I) = Vfl(2 1,1) = 1(Vz); + (2xyz}j + (xy)k 1(2, I, I)

= ; + 4j + 2k = a (say).
Let g = (.xl y + .?) = 6 be the surface (2)

Normal vector to (2) at (2,1,1) = Vg 1(2,1,1)= (2x; + 2yj + 2zk)i(Z,I,I)

= 4; + 2j + 2k = b (say)
:. Angle between the surfaces
= Angle between the normals to them
= Angle between a and b

- cos _1(a,b)1
- =cos -II 4+8+4 I
- I ab .Jl+16+4.J16+4+4

= cos
-1 (
J2t16)
J24 = cos -I ( 16)
6.J14 = cos
-I ( 8)
3.J14

7.2.10 Find the constants p and q such that the surfaces px2 - qyz = (p + 2)x and
4x2y + z3 = 4 are orthogonal at the point (1, -1, 2)
Sol: Letf= px2 - qyz - (p + 2)x = 0 be surface (1), and
Let g = 4.xly + z3 = 4 be surface (2)

Normal to (1) at (1, -1, 2) = Vf 2)=(81 ; + 81 j + af k 11(1_12)


(1-1
" ax ay az y"
= [(2px- P - 2); -{qz)j - (qy}k]I(I,-1,2)= (p - 2); - (2q)j -(q)k = a (say)

Normal to (2) at (1, - 1, 2) = Vg 1(1,-1,2)

= [(8xy); + (4x 2)j + (3z 2)k]1 (1,-1,2) = -8; + 4i + 12k = b (say)

Since the surfaces (1) and (2) are orthogonal, a.b = 0


:. -8(p - 2) + 4(-2q) + 12(q) = 0
=> -8p + 16 - 8q + 12q = 0
Vector Differentiation 483

=> -8p + 4q + 16 = 0
=> 2p - q = 4 ..... (i)
Since the point (1, -1,2) lies on (1), we have,
p + 2q - p - 2 = 0 => q = I
from (i) we get,p = 5/2 :. p = 512, q = 1

7.2.11 If r = xi + yj + zk and r = Irl = ~ x 2 + y2 + Z2 show that grad (r3) = 3r r

Sol: Let ~ = r3 = (x 2 + T + z3)3/2


a~
Then, ax = 2"3 (x 2 + T + z2)3/2-1 . 2x = 3x r

a~ a~
similarly 8y = 3yr; and az 3zr

a~. ~. ~
grad ~= - l + - ) +- k
ax 8y az

= (3xr) i + (3yr) j + (3zr) k


= 3r (xi + yj + zk) = 3r r

Aliter: Ifr2 = xl + T + z2, we have, 2r ar = 2x => ar _~


ax ax r

.. ar yarz
Slmtiarly =- .- =-
8y r'az r

grad ~ = Vr 3

a 3. a 3 . a 3 ..1 ar 2 ar. 2 ar
= -(r )l+-(r )}+-(r )k =3r-i+3r -}+3r - k
ax 8y az ax 8y az

..1
=3r y.[x.
-l+-)+- zk] =3rr
r r r
7.2.12 Evaluate grad r"
Sol: Let ~= rn = (xl + T + z2)nl2
~ =!!{x 2 + y2 + Z2
ax 2
r/ 2
-
t
.2x =nx{x 2 + y2 z2 r/z- t
484 Engineering Mathematics - I

..
sImilarly -o~ = ny (X 2 + y2 + Z2 )'?z-/,I and -o~ (222)";-1
= nz x + y + z
ox oz

o~. ~. ~k
grad rn = grad ~ = - I + - j +-
ox 0' oz
= n, {x 2 + y2 + Z2 )'fi- I (xi + yj + zk) = n rn-2.r

Aliter: g
rad ~
(-o~).
ox
1+ (o~l.
- j+ (~)
oy OZ
o~ or. o~ or. o~ or
- k=-.-I+-.-j+-.-k
orox or0' oroz

= nrn- 1 . !.. i + nxn- I . y j + nxn- I . !... k = nrn-2.r


r r r
Ex.7.2.13 If A is a constant vector prove that grad (r . A) = A
Sol: Let A = A I i + Aj + A3k (A I' A 2 , A3 being constant functions)
r = xi + yj + zk
r. A = A1x + A2Y + A3z = f(say)
of 81 81
ox = AI' 0' = A2, OZ = A3

:. grad (r.A) = gradf= 81


ox
i+ 81 j+ 81 k
0' oz
=A i+Aj+A k=A
I 3

~I(r).r
Ex.7.2.14 Prove V (~(r=--
r
Sol: Let f= ~ (r)
81 ar I'X or x
- = ",I (r) - = ~ (r).-
ox 'Y ox r ox r

81 =~I(r)ar =~I(r).y or y
0' 0' r 0' r

81 = ~I(r) or = ~I(r).!... or z
-az =-r (see Aliter of ex. 7.2.11)
OZ OZ r

81. of. 81
.. V (~ (r = V f= 0/ + 0' ) + oz'''
<j>1(r) <j>1(r)r
= W (xi+yj+zk) = W
Vector Differentiation 485

Ex.7.2.15 Find the equations for the tangent plane and normal line to the surface
.r
z = x 2 + at the point (2, I, 5).
Sol: Let r = xi + y) + zk be the position vector of any point P(x, y, z) on the surface.
Let r l = xli + yJ + zl k be the position vector offixed point A(xI'YI' z,) on the
surface.
Then AP = (x - xI) i + (y - YI) + (z - z\) k = r - r,
Let n be the normal to the surface at A.
Then, since AP is perpendicular to n, we have,
(r-r\)n=O ..... (I)
which is the equation to the tangent plane at A.
Here, in the given problem
r-r, =(x-2)i+(y-I)+(z-5)k

and 11 = V (xl + .r - z) at A (2, I, 5)

= (2xi + 2y) - k)I(2.).5) = 4i + 2) - k


:. The tangent plane at (2, I, 5) is, (from (I,
4(x - 2) + 2(y - I) - I(z - 5) =0 ~4x + 2y - z = 5 .... (2)
From (2), the direction ratios of the normal Iine at A are 4, 2, -I
:. Equation 10 tpt( pormal line at A are,
x-x)y- Y) z-z) he \
-- = w re
~,=--

abc
(xl'YI' zl) = (2,_1,5) and (a, b, c) = (4; 2, -I)
:. The equations of normal line are x - 2 = Y -I = z - 5
4, 2 -I
From the above example. we have to remember the following :

Let + = c be any given surface and (xl' YI' zl) be a point on it; then

(I) Equation to the tangent plane to +=cat(xI'Y"z,) is


486 Engineering Mathematics - I

(2) Equations to Normal line at (xl,y\, zl) are


x-x, = Y- y, = z-z,
8<1>/8x 8<1>/ay 8<1>/8z

Exercise 7(a)

I. If <jl = 2xz3 - 3xlyz, find V <I> and IV<I>I at the point (2, 2, -I)
[Ans: (i) 22i+ 12}-12k(ii)2M3]
2. If V = 2x i - 3y) + z 3k, and <jl = 2xyz - 3z2, find V. V<I> and Vx V<I> at the point
(1,2,3)
[Ans . (I) - 426 (2) 6i + 352} t 156k]
3. Iff=2xyzandg=xly+z,fing V (f+g) and V (fg) at the point(I,-I,0)
[Ans : -2i + ) - k; 2k ]
7
4.,Jr + 1 ~) [AI:1S : (6 - 2~/2 - 2r -3) r ]
2
4. Evaluate V (3r -
~r
5. If <I> = r2 e- , show that grad <I> = (2-r) e-r r
r

6. Find a unit normal vector to the surface z = xl + Y at the point (I, -2, 5)
I
[Ans: 51 (2i - 4j - k)

7. Finq the equations to the tangent plane and the normal line to the surface
xz2 + xly = z - I at the point (I, -3,2)
.

.. x-I y+3 z-2
[Ans: (I) 2x- y- 3z + 1 = (n) -2 =-1- =-3-
"
8. Find ~uations to the Tangent plane and normal line to the surface y = xl + z2
at the point (1, 5, -2)
x-I y-5 z+2
[Ans: (i) 2x-y-4z= 5 (ii) -2-=-=1=--=-4

9. Find the directional derivative ofU = 4xz3 - 3x2yz at (2, 1, -2) in the direction of
(3i - 2) + 6k).
384
[Ans: 7]
10. Find the directional derivative of <I> = 4e2x- y+z at the point (1, I, -1) in a direction
towards the point (2, 3, 1)
[Ans : 8/3]
Vector Differentiation 487

II. Find the values of the constants a, b, c so that the directional derivative off= axl
+ byz + cz2x 3 at (1,2, -I) has a maximum magnitude 64 in a direction paraIlel to
z-axis.
[Hint: /0:./ at (I, 2, -I) is IlleI to z-axis.
:. Equate coefficients of i and j to zero and IV ~ = 64. Thus get 3 equations in a,
b, c and solve them]
[Ans: a =6, b = 24, c = - 8]
12. Find the acute angle between the surfaces xlz = 3x + z2 and 3x2 - l + 2z = I at
thepoint(I,-2, I)

J3
[Ans : cos- I - - ]
7J2
13. Find grad ~ if r = xi + yj + zk, r = Irl and

I
(i) ~ = Log r (ii) ~ = - (iii) ~ =r
r

[Ans: (i) r/~ (ii) -r/~ (iii) r/r]


14. Find the directional derivative of g = x 2l + lz2 + z 2x 2 at the point (I, I, -2) in the
direction of the tangent to the curve x = e-t, y = 2 sint + I, z = t - cos t at t = o.
[Hint: Tangent vector to the given curve is

dx. dy. dz k
-l+-}+-
dt dt dt

2
[Ans: J6]
15. Find the acute angle between the normals to the surface xy = z2 at the points
(1,9,3) and (3, 3, -3)

16. If r = xi + yj + zk and 4> = x 3 + Y + z3 - 3xyz, show that r, grad 4> = 34>.


488 Engineering Mathematics - I

7.3 The Divergence of a Vector Function


7.3.1 If A = A)i + A2k is a vector function, defined and differentiable at each point
(x, y, z) in a certain region of space [i.e., A defines a vector field], then the divergence
of A (abbreviated as 'Div A') is defined as,
DivA= V.A

. a }-+
= (1-+ . a k a)
ax ay -az' (A )1'+A21'+A3k)
= (aAax i + aAay + aA3)
I 2
az
(since i.i = j j = k.k = 1)
Note: (1) Div A is a scalar field
(2) V.A 7:- A.V
7.3.2 Physical significance of the divergence
If A represents the velocity of fluid in a fluid flow, Div A represents the rate offluid
flow through unit volume. (or) Div A gives the rate at which fluid is originating at a
point per unit volume.
Similarly if A represents the Electric flux or heat flux, Div A represents the amount
of electric flux or heat flux that diverges per unit volume in unit time.
7.3.3 Some properties of Divergence
If A, B , are vector functions and '/' is a scalar function, then, prove that
(1) Div (A + B) = Div A + Div B (i.e) V. (A + B) = -V . A + V .8
Proof Let A = A)i + Aj + A3k
B = 8 1i + B~ + B3k
A + B = (A) + 8) i + (A2 + 8 2 )j + (A3 + 8 3) k
a a
Div A = -(AI +B l )+-(A 2 + 8 2 )+-(A 3 + 8 3 )
a
ax ay az
= (aA1 + aA2 + aA3)+(aBI + aB2 + aB3)
ax ay az ax ay az
~

= Div A + Div B.
Vector Differentiation 489

(2) Prove that, Div (fA) = (grad.f) .A + .f{Div A) i.e. V .(fA) = (V .f). A + .f{ V A)

Proof: Let A = Ali + A'll + A3k then fA = fAli + fA'll + fA3k


a a a
V .(fA) = ax (fA.)+ ay (fA2)+ az (fA3)

= fQ~+A. af + f aA2 +A2 af + f aA3 +A3 8j


-Ox ax ay ay az az

..... (1)

..... (2)

..... (3)

(1), (2), (3) => V .(fA) = (v.f).A + f(v .A)


7.3.4 Solenoidal vectors: A vector A is said to be solenoidal if Div A = 0

Solved Examples

Ex. 7.3.5 If A = (.x2y) i + (xy2z)j + (xyz)k, find div A at the point (1, -1,2).
Sol: A = (.x2y) i + (xyz)j + (xyz)k
. aA. aA 2 aA 3
DIVA=-+-+-
ax ay az
a 2 a 2 a
= -(x y) + -(xy z) + -(xyz)
ax ay az
= 2xy + 2xyz + xy
= xy (2z ,+ 3)
:. At (1, -1, 2), Div A = (1) (-1) [4 + 3] =-7
490 Engineering Mathematics - I

Ex. 7.3.6 If V = 2xyi + 3x2yj - 3pyz k is solenoidal at (1, 1, 1), find 'p'.
. a a 2 a
Sol: Dlv V = -(2xy)+-(3x y)+-(-3pyz)
ax oy oz

= 2y + 3xl - 3py
At (1. 1, 1). Div V = 5 - 3p
5
Since V is solenoidal, Div V = 0 :. p = 3"
Ex.7.3.7 Ifr = xi + yz + zk, and r = Irl, show that Div (r3 r) = 6 r3

Sol: r= ~X2+y2+Z2 :. r3 r = (xl+y +z2)3/2 (xi+yj+zk)

= Ali + Aj + A3k (say)


then, A I = x(xl + y + z2)312
A2 = y(x2 + Y + z2)3/2
A3 = z(xl + y + z2)312

oA2 .1. .. l oA 3 2 .. l
similarly ~ =3y r+r~ - =3z r+r
OZ

. oA I -
DlvA= - +
oA2 oA3
-+-
ax oy oz

= 3r (xl + Y + z2) + 3r3 = 3r3 + 3r3 = 6r3


Aliter: r3 r = r3 xi + r3 yi + r3zk.
a Or x
-8 (r3 x) = r3.1 + x . 3r2 -8 = r3 + x . 3r2 . -
x' x r

= r3 + 3xlr
Vector Differentiation 491

o 0
Similarly ay (~y) = ~ + 3yr and oz (r3z) = ~ + 3z2r

DivA=6~

l
Ex. 7.3.8 Evaluate Div [ r grad (r-3) ] or V. {rvC 3 )}

_
--3r4 or . ar . --4 -or k--3r
-1-3r4 -j-3r _ --4 -i+-j+-
y. Z (x. k)
ox c:y oz r r r
= -3r-5 (xi + yj + zk)

r grad (~) = -3r--4 (xi + yj + zk)

= Ali + Aj + A3k (say)


where AI = -3r--4x, A2 = -3r--4y , A3 = -3r--4z

aA I = ~ [-3r--4 x]
ax ax
= -3r--4 . 1 + x. -3. -4 r- 5 . :: = -3r--4 + 12 x r- 5 . ~
= -3r--4 + 12 x 2 r-6

oA
Similarly, c:y2 =-3r-4 + 12yr-6

:. Div (r grad r-3) =


oA
_I + __
oA
2 +_3
oA
ax Oy az
= -9r-4 + 12 r-6 (x 2 + y + z2) = -9r-4 + 12 r--6 . r2 = 3r-4
(Alternate method is left to the reader).
492 Engineering Mathematics - I

Ex.7.3.9 Show that V .(x" r) = (n + 3) r". Hence show that r/f3 is solenoidal.
Sol: r" r = r" (xi + yj + zk)
a a a
V .(r" r) = -a (xr")+ ~, (yr") +- (zr")
x v)' az

y + z.-
= 3r" + n ['1-1 ( x.-X + y.- z)
r r r

= 3r" + n r"-2 . r2 = (n + 3) r"


Ifn = -3, (r-3 r) = (-3 + 3) r-3 =0

r
:. 3 is solenoidal
r

Ex.7.3.10 Prove that Div (CIA + C 2 B) = C I Div A + C 2 Div B, where CI' C 2 are
constants.

Sol: Let A = A I i + Aj + A3k


B = Bli+ Bj+ Bl
CIA + C2B = (CIA I + C 2B I) i + (C IA2 + C 2B2U + (C IA3 + C 2 B 3 )k
a a
Div (CIA + C2B) = ax (CIA I + C2B I) + Oy (C IA2 + C 2B2)

=C (MI+M2+M3)+C(aBI+aB2+
I ax Oy az 2ax ay
aB
az
3)
Vector Differentiation 493

Ex.7.3.11 If A = 2xi + 3yj + 5zk and f= 2xyz, find div (fA) at (1,2,3).
Sol: fA =2xyz (2xi + 3yj + 5zk)
= (4x 2yz)i + (6xyz)j + (10xyz2)k

a a a
Div (fA) = ax (4x2yz) + ay (6xyz) + az (10xyz2)

= 40xyz
:. At (\,2,3), div (fA) = 240
Aliter: div (fA) = D.(fA) = (V f). A + f( V A)

Vf = 2yzi + 2xzj + 2xyk


V fA = 4xyz + 6xyz + \ Oxyz = 20xyz
f(v .A) = 2xyz (2 + 3 + 5) = 20xyz

Hence V .(fA) = 40xyz and at (\,2,3) V .(fA) = 240

Ex.7.3.12 Iff, g are scalar fields show that V fx V g is solenoidal

j k
vfx V g = a.fjax aljay atjoz
agjax agjay agjaz

(Suffixes denote partial derivatives)


a
Div (V fx V g) = L ax Vygz - fzgy)

= L (!y gxz + gzfxy - fz gxy - ~fxz)


=0
:. V fx V g is solenoidal
494 Engineering Mathematics - I

Exercise 6(b)

I. If V = (xlz)i - (2y3z1)j + (vz)k, find div A at the point (I, -I, I)


[Ans. - 3]
2. Ifr = xi + yj + zk, find div r [Ans.3]

3. If F = (3,xyz2)i + (2xy3)j - (xlyz)k, and = 3xl - yz,


find (i) Div F (ii) Div (F) and (iii) Div (grad); at the point (I, -I, I)
[Ans. (i) 4 (ii) I (iii) 6 ]
4. If V = (3xly - z)i + (xz 3 + y4)j - 2x3z 2k, find grad (Div V) at the point (2, -I, 0)
[Ans. - 6i + 24j - 32k]
5. Evaluate: (I) Div (r2r) (2) Div (r r) (3) grad Div (r/r) (4) div (r/r3)
[ Ans. (1) 5r2 (2) 4r (3) -2r/r3 (4) 0 ]
6. Show that V = 3y4z2i + 4x3z7 + 6x2y3k is solenoidal
7. Show that the vector F = (2xl + 8xyz)i + (3~y - 3xy)j - (4yz2 + 2x3z)k is not
solenoidal, but G = xyz2 F is solenoidal.
8. If a is a constant vector and V = a x r, where r = xi + yj + zk, show that V is a
solenoidal vector.
9. Determine the constant 'b' such that the vector, V = (2x + 3y)i + (by - 3z)j +
(6x- 12z)kis solenoidal
10. Ifr, and r2 are vectors joining fixed points A (x"YI' z,) and B(x2'Y2' z2) to a variable
point P(x, y, z) prove that r, x r2 is solenoidal.
11. If r = xi + yj + zk and r = Irl show that, Div (grad rn) = n( n+ I )rn- 2 .
12. If g = r-2n , find div (grad g) amd find 'n' such that 'g' is solenoidal.
2n(2n -I) I
[A ns. ?n+2 n--
,- 2 ]
r-

7.4 Curl of a vector function


7.4.1 If A is a differential vector function, then curl A is defined as, curl A = V' x A

j k

If A = Ali + Aj + A3 k , then Curl A = a/ax a/ay a/az


AI A2 A3
Vector Differentiation 495

= (aAJ _ aA 2 )i + (aA I _ aA3)j + (aA 2 _ aA I )k


0' az az ax ax 0'

Note: The curl of a vector is also a vector


7.4.2 Physical significance of curl
Let r = xi + yj + zk be the position vector cf a point P(x, y, z) of a rigid body rotating
about a fixed axis about the origin 0 with an angular velocity ffi = ffili + ffi.j + ffi)k.
Then the velocity V of the particle P is given by,

i j k
V = ffi x r = ffil ffi2 ffi3

X Y z

j k
Curl V= a/ax 8/ay a/8z

= i (ffi 1+ ffi I) + j (ffi2 + ffi 2) + k ( ffi 3 + ffi3) = 2ffi


Thus the curl of velocity vector is twice the angular velocity of rotation.
7.4.3 Irrotational Vector: A vector V whose curl is zero is said to be an irrotational
vector.
7.4.4 Properties :(1) V x (A + B) = V x A + V x B (or) curl (A + B) = curl A + curl B
Proof: Let A = Ali + A.j + A)k and
B = Bli + B.j + B)k so that
496 Engineering Mathematics - I

i j k

Curl (A+B) =
ajax ajay ajaz
AJ+BJ A2 +B2 A3 +B3

j k j k
= ajax ajay ajaz + ajax ajay ajoz

= curl A + B
(2) If is a scalar function and A is a vector function
Curl (A) = ((curl A) + (grad) x A
(or)
V x (A) = ((V x A) + (V {P) x A
Proof: If A = Ali + Aj + A3k, then cj>A = cj>Ali + cj>Ai + cj> A3k
j k
L\ x (cj>A) = ajax ajay ajaz
cj>AI cj>A2 cj> A 3

= i[~(q,A3)-~(cj>A2)]+ j[~(cj>AJ)-~(cj>A3)J+ k[~(cj>A2)-~(cj>AJ)]


ay az az ax ax ay

= "i[.I."'ayaA3
~
+ A aq, _.I. OA2 - A aq,]
3ay "'az 2az

= .I. L( aA 3 _ aA2)i + L(A aq, _ A acji


'" ay az 3ay 2az

i j k i j k
= cj>ajax ajay ajaz -?> aq,jax aq,jay acj>jaz
AI A2 A3
=cj>( V x A) + (V cj x A
7.4.5 Conservative vector field:
A vector field F, which can be derived from a scalar field q, such that F = Vcj>, is
called a conservative vector field and q, is called the scalar potential ofF.
Vector Differentiation 497

Solved Examples

Ex.7.4.6 If A = (xy)i + (yz}j + (zx) Ii; find (a) curl A and (b) curl curl A at (1,2, -3)
Sol : A = xyi + yzj + zxk
j k
(a) curl A = V x A = a/ax a/ay a/az
xy yz zx

= I.[-(zx)--(yz)
a a].[ a a ] +k[-(yz)--(xy)
+ } -(xy)--(zx) a a]
ay & & & & ay
= i(O - y) + j(O - z) + k (0 - x)
= -yi - zj - xk
:. curl A at (1, 2, -3) = -2i + 3j - k
(b) curl A= V x (V xA)

=V x (-yi-zj-xk)

i j k
= a/ax a/ay a/az
-y -z -x

[aya a].[
= i -(-x)--(-z)
&
a a ] +k[-(-z)--(-y)
+ } -(-y)--(-x)
fu fu
a
fu
a ]
ay
= i(O - (-I) + j(O - (-1 + k(0 - (-I
=i+j+k
:. curIAat(I,2,-3)=i+j+k
Ex.7.4.7 Show that V = xi + Ij + z3k is irrotational -
Sol: Curl V = V x V

a 3)---(y
= I.[-(z a 3]) +k[-(y
a 2]) + }.[-(x)--(z
a a 2)--(x)
a ]
ay az oz & fu ay
=0 :. V is irrotational
498 Engineering Mathematics - I

Ex.7.4.8 If F = (4x + 3y + az)i + (bx - y + z)j + (2x + cy + z)k is irrotational, find the
constants a, b, c
i j k
Sol: Curl F = ajax ajay ajaz
(4x+3y+az) (bx- y+z) (2x+cy+z)

= i[~(2X+CY+Z)-~(bX-
ay az
y+Z)]+ j[~(4x+3y+aZ)-~(2X+CY+Z)J
az ax

+ k[~(bX-
ax
y+Z)-~(4x+3y+az)]
ay
= (c - l)i + (a - 2)j + (b - 3)k
Since F is irrotational, curl F = 0
:. c - I = 0, a - 2 = 0, b - 3 = 0
i.e., a = 2, b = 3, c = 1
Ex.7.4.9 If r = xi + yj + zk, and r = Irl, find curl (rn r)

Sol : r = xi + yj + zk, r = Irl = ~ x 2 + y2 + Z2


rn r = (.xl + Y + :?)n/2 (xi + yj + zk)

j k
curl (rn r) = ajax ajay ajaz
1/ 1
III
x(x 2 + y2 + Z2Y2 Y(X2 + l + Z2Y2 Z(X2 + l + Z2 )"2

= I.[ z."2
n x- x + Y 2 +z 2) 2z J
(? + y-?+z-?hJLY- Y."2n (2

+ ).[ x."2(X n 2 + y 2 +z 2 ).2x ]


n 2 + y 2 +z 2 ).2z-z."2(x

= i(O) + j(O) + k(O)


=0
Vector Differentiation 499

Aliter: rn r = rn (xi + yj + zk)

j k
curl (rn r) = ajax ajay ajaz
xrn yr" zr"

= "'.[
L..,l - a ( zr n) - -
a ( yr n)] = "'.[ ar - y.nr n-I -ar]
L..,l, z.nr n-I -
ay az ay az

_
-
'" .[
L..,l
Y - y.n.r II_I -;
f/zr n-I -; z]
= i( 0) + j( 0) + k (0) = 0
Ex. 7.4.10 Prove that, ifF = (x + Y + I)i + j - (x + y) k, F. curl F = 0

j k
Sol: Curl F = ajax a/ay ajaz
x+ y+1 -x-y

= i [-I -0] + j[O + I] + k[O -1]


= -; +j - k
:. F curl F = -I (x + y + 1) + 1.1 + 1(x + y) =0
Ex. 7.4.11 P(x, y, z) is a variable point and Q(xl' YJ, zJ)' R(x2, Y2' z2) are fixed points.
If U = QP and V = RP; Prove that curl (U x V) is equal to 2(U - V).

Sol: P(x, y, z), Q = (xl' Yl' zJ)' R = (x 2, Y2' z2)


:. QP = (x -xJ); + (y - YJN + (z - zJ)k = U
RP = (x - x 2); + (y - Y2 N + (z - z2)k =V
j k
UxV= (X-XI) (Y-YI) (Z-Zl)
(x-x 2) (Y-Y2) (Z-Z2)

= ~{(y - YJ) (z - z2) - (z - z\) (y - Y2)} ;


=~ {y(z\ - z2) - z(yJ - Y2) + (yJ z2 - Y2 z J)}i
500 Engineering Mathematics - I

Curl (U x V) = I[ ~{x(y, - Y2)- Y{X, -X2)+{X'Y2 -x2 y,)}

-! {Z{X, -X2)-x{Z, -Z2)+{Z,X2 -Z2XJ}}

= L(-(XI - X2) - (xI - X2);


= L -2(x l - X2 ); = 2L(X2 - xI);
= 2(U - V)
Ex. 7.4.12 If F is a conservative vector field show that curl F = 0
Sol. F is a conservative vecor field.

V'" 8$. 8$. 8$ k


:. There exists a scalar field '~' such that F = 'I' = ax 1 + ~ J + az

; j k
:. Curl F = ajax ajay ajaz
a~jax a~j~ 8$jaz

i( a ~ a ~
2 2
== L _ ) == 0
~az az~

Ex. 7.4.13 Show that F == (6xy + iJ)i + (3r -z)j + (3xz 2 - y)k is irrotational. Find ~ such
that F == V ~

Sol: Curl F = ajax


i
j
aj8y k
ajaz J
6XY+Z3 3X2 -z 3xz2-

== i (-1 + 1) + j (3z2 - 3z2) + k(6x - 6x)


==0
:. F is irrotational

8$. 8$. a~k


L tF== V~== -I+-J+-
e ax ay az

:. : == 6xy + iJ ..... (I)


Vector Differentiation 501

a~
- = 3x2 -z ..... (2)
Oy

a~ ?
- = 3xz~ - y ..... (3)
az
Integrating equation (1) with respect to x we get
~ = 3.x2y + xz 3 + fly, z) ... :. (4)
Differentiating (4) partially w.r.t 'y'
a~ af(y,z)
ay = 3.x2 + ay ..... (5)

From (2) and (5) we have


af(y,z)
ay =-z ..... (6)

Integrating (6) w.r.t 'y' we get


fey, z) = -yz + h(z)

Hence ~ = 3x2 + zx 3 - yz + h(z) ..... (7)


I
difffrentiating (7) w.r.t 'z' we get

..... (8)

Comparing (3) and (8) we have


h l(z) = 0, :. h(z) = constant, 'c' say
Hence ~ = 3.x2y + xz 3 - yz + c (from 7)

Aliter: -~ =6xy+z3 .... (I)


ax
a~
ay = 3.x2 - z .... (2)

..... (3)
502 Engineering Mathematics - I

Integrating the above equations respectively w.r.t x, y, and z, we get


= 3x2y + xz3 + f(y, z)
~ .... (4)
~ = 3x2y - yz + g(z, x) .... (5)
~= xz3 - yz + h(x. y) ..... (6)
~ should satisfy all the above three equations simultaneously
(i.e.) (4), (5) & (6)
3x2y + xz3 - yz + c, where c is a numerical constant.
:. ~ =
Note: [Herej= -yz, g = xz3, h = 3x2y will satisfy (4), (5) & (6)
Ex.6.4.14 Iff(r) is differentiable, show that f(r)r is irrotational
Sol: fir) r = fir) (xi + yj + zk)

Curl (f(r) r) = %x
j
o/Oy %z
k j
xj{r) y j{r) zj{r

= I{; {zj{r)}- ! {y j{r)})


= I{z./{r)~ - y./{r):)
= I{z./{r)~ -y./{r);) =0
.. f{r)r is irrotational
Ex. 7.4.1SlfU and V are irrotational, prove that U x V is solenoidal
Sol: Let U = Uti + U2i + Ui
V=V t i+V2i+ Vi
U is irrotational :. curl U = 0

=> Li[OU 3 _ OU 2 ] = 0 ..... (1)


oy oz
..
sImilarly, '" .[OV
L-l - 3 oV 2] = 0
Oy oz
- - ..... (2)
Vector Differentiation 503

} k
VxV= VI V2 V3 =1:;(U 2V 3 -U 3 V 2)
VI V2 V3

..... (3)

au 3 _ au 2 au I _ au 3 . au 2 _ au I
From (I) ,we have, - - - - - , - - - , - - - - ..... (4)
0' az az ax ax 0'
av3 _ av2 aV _ av3 . av2 aV
an d from (2) we have - - - -I- - - - -I
_
..... (5)
, '0' az' az ax' ax 0'
Substituting the six equations of(4) & (5) in (3), we observe that all the 12 terms of
(3) will get cancelled. Hence Div (U x V) = 0
~ U x V is solenoidal

Exercise - 7(c)

I. If V = (2xz2);_ (yz)j + (3xz3)k, andf= xlyz, find the following at the point (I, I, I)
(a) curl V (b) curl (jV) (c) curl (curl V)
[Ans. (a) ; + } (b) 5; - 3} - 4k (c) 5; + 3k]
2. If 'g' is a scalar function, show that curl (g grad g) = 0
3. Find the value of the constant 'p' for which the vector V = (pxy - z3) + (p - 2) xl}
+ (I - p) xz2k is irrotational.
[Ans: 4]
4. Find the constants a, b, c so that the curl of the vector A = (x + 2y + az);
+ (bx - 3y - z)j + (4x + cy + 2z)k is identically equal to zero
[Ans. 4, 2, -1]

5. If r = xi + y) + zk, r = Irl, show that curl (;Z ) = 0 and find a scalar function of' such
r
that 2"= -v J,.f(a) = 0 where a> o.
r

[Ans :f= log (~)]


504 Engineering Mathematics - I

6. If r = xi + yj + zk, and p, q are constant vectors, show that (I) curl [(r x p) x q]
= ( p x q) and (2) curl [(p.q)r)] = o.

7.5 Laplacian Operator: V2

7.5.1 V2 = V.V = (i~+


ax j~+k~).(i~+ j~+k~)
ry az ax ry az
a2 + -a2' + -a22 ) .IS called the Laplacian
. Operator
=
( ax 2 ay2 az
-

'V2' can be applied to both scalar and vector functions as shown below.
2 a2~ a2~ a2~
V~=-+-+-
ax 2 ry2 az2
where '~' is scalar function
If A = Ali + Aj + A3k, is a vector function, then

a2 a2 a2 )
V2 A = ( ax2+ ay2 + az2 (Ali + Aj + A3k)
= (V2AJ+(V2A2)j+(V2A3~
7.5.2 Vector Identities: we shall give below some vector identities with proofs.
\. If '' is a scalar function curl grad ~ = 0, (or) V x V ~ = 0
acjl acjl. acjl )
Proof: Curl grad cjI= V x (-i+-}+-k
ax ry az
j k
ajax ajry ajaz
acjljax acjljay acjljaz

= L>[~(acjl)-~(a~)l
ay az az ry

= 0, assuming that '~' possesses continuous second order


partial derivatives.
Vector Differentiation 505

(2) IfY is a vector function, Div (Curl A) = 0 (or) V.(Vx A) = 0


Proof: Let A = Ali + A-j + A3k

CurIA= " .(-


~I aA-3 -aA-, )
ay az

Div (curl A) = "~- a {aA3 aA)}


ax -ay- - -az-
a~A3 a2 A2 a2A4 a~A3 a~A~ a~AI
= -----+-----+-----
axay axaz ayaz ayax azax azay
=0
(3) If A is a vector function, curl (curl A) grad (Div A) - V2 A (or)

Proof: curlA= "~l.(-


aA-3 aA, )
ay -az-

j k
Curl (curIA)= a/ax a/ay a/az
aA3_ aA2) aA I _ aA3) aA2_ aAI)
( ay az ( az ax ( ax ay

= I[~(aA2 _ aA I)_~(aAI _ aA 3)]i


ay ax ay az az ax
506 Engineering Mathematics - I

= -'1 2A + '1('1.A)
(4) '1.(AxB)=B.('1xA)-A.('1xB), (A, B are vector functions) (or) Div(AxB)=B.
curl A-A. curl B.
Proof: Let A = A) i + A?J + A3k and B = B) i + B?J + B3k

i j k
A x B = AI A2 A)
BI B2 B)

= I i(A2B) - A)B2)

Div (A x B) = "-(A2B) -A3B 2)


a
LJ ox
= " (A OB3 + B OA2 + A OB2 + B OA3)
L:. 2 AX 3 AX 3 AX 2ax ..... (1)

'1xA = a/ax
AI

..... (2)
Vector Differentiation 507

Similarly,

A. (V' x
" (BB3
0' - BB?)
B) = L.J AI Bz- ..... (3)

Expanding the summations of (I), (2) & (3), we observe that,


Div (A x B) = B. (V' x A) - A. (V' x B)

7.5.3 Operation of V' on product of two functions


Suppose ~ and 'P, are two scalar or vector point functions. When V' is operated on
the product of ~ and \j1, the following rule is useful.
V' (~'P) = V' (~o \{J) + V' (~'P 0) wherein the suffix '0' indicates that the function
is not to be varied, that is, V' is not to be operated on that function. After the
completion of operation of V' the suffixes are dropped.
While proving the identities the following are useful.
(a) a.b = b.a
(b) a x b = -b x a
(c) axa=O
(d) a.(bxc)=(axb).c
(e) a x (b x c) = (a. c)b - (a. b) c
(f) (a. c) b = a x (b x c) + (a . .b}C
(g) a. a = a2
(h) x operation is always between vectors only
vector identities using V' operation:

(i) V' .(FG) = V' . (FG o) + V' . (FoG)

= V' F.G o + Fo V' .G


:. div (fG) = G. grad F + F div G

(ii) V' x (FG) = V' x (FG o) + V' x (FoG)

V' x (FG o) = V' F x Go due to (h)

V' x (FoG) = (V' x G)Fo=Fo(V' xG)

:. V' x (FG) = V' F x G + F( V' x G)


(i.e.) curl (FG) = (grad F) x G + F curl G
508 Engineering Mathematics - I

(iii) V'. (F x G) = V' .(F x Go) + V'. (Fo x G)


V' . (F x Go) = (V' x F) .G o due to (d)
= Go .( V' x F) due to (a)

V' . (Fox G) = - V' . (G x F0) due to (b)


= - V' x (G. F0) due to (d)
=-(V' x G). Fo
=-Fo(V' x G) due to (a)
:. V'. (F x G) = G. (V' x F) -F. (V' x G)
Thus div (F x G) = G. curl F - F. curl G
(iv) V' x (F x G) = V' x (F x Go) + V' x (Fo x G)
V' x (F x Go) = (V' . Go) F - (V' . F) Go due to (e)

= (Go V') F - Go (V' . F) due to (a)


V' x(FoxG)=(V'.G)Fo-(V'Fo)G due to (e)

= Fo (V' . 0) - (F o V') G due to (a)


:. V' x (F x G) = (G . V') F - G( V' .F) + F (V' .G) - (F. V' ) G
Thus curl (F x G) = (G. V' ) F - (F. V' ) G + F (V' .G) - G (V' .F)
"I-
(v) V' (F . G) = V' (Fo . G) + V' (F .G o) {
V'(Fo.G)=Fox(V' xG)+(Fo. V')G due to (t)
V' (F . Go) = V' (Go. F) = Go x (V' x F) + (Go. V') F due to (t)
:. V' (F . G) = F x (V' x G) + G x (V' x F) + (F . V') G + (G . V') F
Thus grad (F . G) = F x curl G + G x curl F + (F . V') G + (G . V') F
(vi) curl (grad F) = V' x (V' F) = (V' x V') F = 0 due to (c)
(vii) div (curl F) = V' .( V' x F) = V' x (V' . F) due to (d)
= ( V' x V' ). F = 0 due to (c)
(viii) curl (curl F) = V' x (V' x F)
= (V' . F) V' - (V' . V' ) F due to (e)
= V' ( V' . F) - V' 2 F (since V'. must not appear at the end)
= grad (div F) - V' 2 F
Vector Differentiation 509

Solved examples

Ex.7.5.4 If/= x 21z2, find V 2/ at (1,2, 1)


2 2 2
0 / 0 / 0 /
Sol: V Y = --J +--+-J-
ox- 0/ oz-
= 21z2 + 6x2yz2 + 2x21

:. at (1, 2, 1), v2j= 16+ 12+ 16=44.


Ex.7.5.5 Show that, if r = xi + yj + zk, r = \rI, then V 2r ll = n(n + 1) rll 2

Sol: ~ = (x2 + Y + z2)n12, (:r = ~x2 + y2 +Z2 )

... (1)

2
Similarly -02 (r n) = n ( n- 2)y 2r n-4 + nr n-2 ... (2)
, oy
2
and -a2 I!
( n)
=n ( n- 2)z-r
J n-4
+nr n- 2 ... (3)
oz

= 3n,.n-2 + n(n-2) ~-4 (x 2 +y + z2) adding (I), (2) & (3)


= 3n~-2 + l1(n - 2) r"-2 (": x 2 + y + z2 = r2)
= n(n + 1) ~-2
510 Engineering Mathematics - I

Aliter: -a (r") =nr 11-1 .-=nr


ar 11-1 .-
x
ax ax r

= n,.rr2 . x

~~II)=
2 n(r ll
-
2.1 + x.{n _ 2)r"-3. ar)
az ax

~ n(,"-' + (n-2}r"-'. x:)


= nr"-2 + n(n-2) r"--4. xl ... (I)
2
Similarly -a2 (r ") =nr 11-2 +n (n- 2)r n-4 .y 2 ... (2)
, ay

and ~2 ~n ) = nr n- 2 + n{n - 2 )r n-4.Z2 ... (3)


az
Adding (I), (2) & (3) we get,

=> V2rn = 3nr"- 2 + n{n - 2)r n- 4.r2


= ,.rr2[3n + n2 - 2n]
= n(n + I) r"-2

Note: Ifn =-1, we have vt~ )= 0, which means that (.; ) satisfies the Laplace's equation

av au
Ex.7.S.6 If V .U =0, v .v =0, v xU=-at' v xV = at' show that Uand V satisfy
a2u
the wave equation V 2U = at 2

Sol: vx(vxU)= v x (_av) = -a{vxv)


at at
2
-a(au)_ -a u
= at at - ----ai2 ... (1 )
Vector Differentiation 511

But V x (V x U) = V (V .U) - V 2U

= - V 2U (": V U = 0) ... (2)

from ()) & (2), a,2 = V2U , which shows that U satisties the wave equation.
i'iu

Similarly;

Vx (Vx
au a
v)= Vx- =-(Vx u)=--
-a~v
... (3)
at at2
at
and vx(vxY)=V(V.Y)-V 2 Y=-V 1 Y(": V.Y=O)
... (4)
(3) and (4) => Y satisfies the wave equation.
Ex.7.5.7 If V.Y = 0, show that V x[V x {V x (V x V)}] = V 4y (or)
curl [curl {curl (curl Y)} ] = V 4y
Sol: We know that
V x (V x Y) = V (V .Y) - V 2y [from 7.5.2 (3)]

= - V 2y (": V.Y = 0)
= - U (say)
Then the given expression
= - V x( V x U) = V 2U - V CV .U) [from 7.5.2 (3)]

= V 2( V 2y) - V (V . U) (": V 2y = U)

= V 4y - V (V . U)

V . U = V .( V 2y)

= {~) ~}{V2V}
= I {i ~ }.(V2(~i + V2i + V k)) 3

= V2(a~ + aV2 + aV3)


ax 0' az
= V2(DivV)= V2(0) = 0 (": Div Y = V . Y = 0)
Hence the problem
512 Engineering Mathematics - 1

Exercise 7(d)

(I)

(2)

(3) Show that V-


ry( v. r"r) =-;-:t
2

Show that V-<j> r


ry () = ~ry
d"<j> 2 df
+--
(4)
dr- r dr
1 ~'ry
show also that, if V-<j> = 0, then <j> = C] + -- where CI' C 2 are constants
r
I I
(5) If r = xi + )1 + zk, prove that, curl (k x grad -) + grad (k. grad -) = 0
r r
7.6 VECTOR INTEGRATION
7.6.1 Ordinary integration of vectors
(I) If A(u) = A](u)i + A2(u}j + Aiu)k be a vector function of a scalar variable 'u'
(A,(u), Aiu), A 3(u) assumed to be continuous in any given interval), the indefinite
integral of A(u) is given by.

fA(u)du = i fAl(ll)du+} fA" (u)du +k fA3(1l)du


d
(2) If there exists a vector B(u) such that A(u) = du (B(u)), we can write

fA(U )du = f:U (B(ll ))dll = B(u) + c


where c is a constant of integration independent of u.
(3) The definite integral of A(u) between the limits 'a' and 'b' in the above, is written as,
h b d b
fA(u)du = f- (B(u))du = B(u)+cl = B(b) - B(a)
a a
du a

7.6.2 Line Integrals:


Let A(x,y,z) be a continuous vector function defined in the entire region of space.
Let c be any curve in the region. Divide c into n intervals by taking points
A = Bo, BI' B2 ... Bn(= B).
Let Pi be any point in the interval B i_, Bi
0" ' .;.;.
Vector Differentiation 513

A= 8 0

Let ro' rJ ...... rn be the position vectors of points Bo' B)' B2 ...... Bn respectively. Let
us consider the sum,

The limit of this sum as n ~ 00 and 1&,:1 ~ 0 is defined as the line integral of A
along the curve c and is denoted symbolically by

f A.dr or dr
f A.-dt;
dt
which is a scalar.

If c is a closed curve, the integral is written as fA.dr


Cartesian form of line integral:
If A=AJi+A-j+Ai
dr = (dx)i + ({Mi + (dz)k,

f A .dr = fA1dx + A2 dy + A3 dz
c

Note: f<l>dr, and fAX dr are also examples of line integrals.

7.6.3 Physical appliations:


(1) Work done by a force
(I) If A represents a force and dr is an element of the path of the particle along a
curve c, then the line integral
{J

fA .dr
J'
(P, Q are 2 points on c)

represents the work done by force A in moving the particle from P to Q.


514 Engineering Mathematics - I

(2) flow or circulation:

(2) If A is the electric field strength, the line integral given above i.e., JA.dr, is

called the flow of A along c. If c is a closed curve it is often referred to as circulation


of A around c.
{J

In general, the line integral JA.dr, will depend on the path from P to Q
/'

7.6.4 T"eorem: Prove that the necessary and sufficient condition for the integral fA.dr,
c

to be independent of the path c joining any two points is that A is a conservative


vector field, (or) there exists a scalar field I/> such that A = VI/> (or) curl A = 0,
[i.e., the work done by the force A in moving the particle from one point to another
is independent of the path if A = VI/>]
Proof Let P = (x I ,y I ,Z I)' Q = (x2'Y2,z2) be any two given points on the curve c. Let A = VI/>
where I/> is single-valued and has continuous derivatives.
(J

(1) Work done = JA.dr


I'

Q
fVI/>.dr=
G~ i-+j-+k-
01/> 01/> 01/
(dxi+dyj+dzk)
p p ox ry oz

Qol/> 01/> 01/> (J


J-dx+-dy+-dz= fdl/> = I/>(Q)-I/>(p)
/' ox ry oz I'

= ~(x2'Y2,z2) -1/>(xI'Yl'zl)
i.e., the integral depends upon the two points only but not on the path joining them.
(This is true only if I/> is single-valued at all points P and Q).
(2) The integral is independent of the path. Then,
(x,y,z) (x,y,z) dr
I/>(x,y,z) = f A.dr = f A. ds ds
(Xl ,YI ,Zl) (Xl ,)'1 ,zl )

Differentiation with respect to's' gives

dl/> =A. dr ...... (I)


ds ds
Vector Differentiation 515

But
d<j> =V<j>dr ...... (2)
ds ds
dr dr
(I) - (2) => (A - V<j. -d = 0, which is true irrespective of-
s ds
:. A= V<j>
Solved Examples
4

Ex. 7.6.5: F(t) = (3t 2-t)j + (2-6t}j - 4tk, find (a) JF{t}dt (b) JF{t}dt
2

Sol: (a) JF{t}dI=; J{3/


2
-/~t+ j J{2-61}dI-k J41dl

~ (I' - I~} +(21-31'); -2t'k+c


(b) fF{t}dt =(/3 -~lj + (21 -3t 2)j - 2t 2k + cj = 50; - 32j - 24k
2 2 2

x/2

Ex.7.6.6 Evaluate: n{3sin e)i + {2cose)j}ie


o
Sol: Given integral
xl2 x/2 xl2 ri/2
=; J(3sine}de+ j J{2cose}de=-3cosel; +2sinejJ =3;+ 2j
0 0 0 o
Ex.7.6.7 The acceleration a ofa particle at any time 't' ~ 0 is given by,
a = e-tj - 6(t + 1}j + (3sint)k
Find the velocity V and displacement r at any time 't' given that V = 0
when t = 0 and r = 0 when t = O.
d 2r
Sol: a = e-t ; - 6(t + l}j + (3sint)k = -2
dt

:. V ~ : ~ Jadl ~ - e-t i - 6(1~ +I lj - (3cost)k + C, ..... (1 )

(C 1 is constant of integration)
But V = 0 when t = 0
:.-i-3k+C 1 =0=> C 1 =;+3k
Substituting in (I), we get
Velocity V = (I - e-t)i - (3t2 + 6t}j + (3 - 3cost)k
516 Engineering Mathematics - I

Integrating,
r = (I + e-t)i - (t3 + 3t2)j + (3t - 3sint)k + C 2 ..... (2)
(e 2 being constant of integration)
But r = 0 when t = O.
:. (2) => + i + C 2 = 0 :. C 2 = - i
From (2), r = (t - 1 + e-t)i - (t3 + 3t2)j + (3t - 3sint)k
Ex.7.6.8 Show that
d2F dF
f
Fx--=Fx-+c
dt 2
dt
Sol: We know that

~(FxdF)=FX~(dF)+dFxdF
dt dt dt dt dl dl

Hence the result.


Ex.7.6.9 If A = 2ti + 3t7 - (4t + 1)k, and B = ti + 2j + t 2k, find
2 2

(i) f(A.B}dt f(A x B}dt .


(ii)
o o
Sol: (i) A.B = (2t)t + (3t2) (2) - t2 (4t + 1) = 2t2 + 6t2 - 4t3 - t2 = 7t2 - 4t3

i j
2
(ii) A x B = 21 3/
t 2

= (3t4 + 8t +2)i + (--4t2 - t - 2t3)j + (4t - 3t3 )k

196. 62.
= -1--j-4k
5 3
Vector Differentiation 517

Ex.7.6.10 If F(2) = ; + 2j - 2k and F(3) = 6; - 2j + 3k,


3 dF
evaluate IF.-du
2 du
Sol: From vector differentiation,

d dF dF
We get, -(F.F)=F.-+-.F=2(dF)
F.- so that
du du du du '

F. dF =~~(F.F)=~~IFI2
'du 2 du 2 du

But F(2) = ; + 2j - 2k ~ IFI2 = I + 4 + 4 = 9, when u = 2


and F(3) = 6; +2j - 3k ~ IFI2 = 36 + 4 + 9 = 49, when u = 3

3 dF I
. IF.-du=-[49-9] =20
., 2 du 2

Ex. 7.6.11 If F = (.xl - 2y); - 6yzj + 8xz2k, evaluate IF.dr from the point (0,0,0) to the
c

point (I, I, I) along the following paths


(1) x = t, Y = t2 , z = t3 .
(2) the straight line from (0,0,0) to (I ,0,0), then to (1, I ,0) and then to (1,1, I) and
(3) the straight line joining (0,0,0) to (1, I, I).
Sol. (1) x = t; Y = t 2; z = t 3,.
dx = dt; dy = 2t dt; dz = 3t2 dt,
when t = 0, the point is (0,0,0), when t = 1, the point is (1,1, I)

IF.dr= I(x 2 -2y}tx-6yzdy+8xz 2 dz


c c

/;1 1
J~2 -2t 2 }it -6.t 2 .t 3(2t}dt + 8t~3 J{3t 2 }it =
2 6 9
J-t dt -12t dt + 24t dt
/;0 o
518 Engineering Mathematics - I

Irf 9 6 2 \, 24t lO 12t 7 (3 I


= J~24t -12t -t p t = - - - - - - !
o 10 7 30

12 12 1 252-180-35 37
-----= =
5 7 3 105 105
Aliter: Along C, F = (t2 - 2t2 )j - 6. t 2 t 3j + 8.t.t6 k = - t 2 j - 6tJ + 8t7k
dr = (dx)i + (dy}j + (dz)k = dt i + (2t dt}j + (3t2 dt)k
I

J(- t -12t + 24t ~t


2 6 9
JF.dr = (Taking dot product of F and dr)
c 1=0

37
105
(2) Let 0 = (0,0,0), P = (1,0,0), Q = (I, 1,0), R = (I, 1,1 ).
Then along OP, y = 0, z = 0, dy = 0, dz = 0 and x varies from 0 to 1.
I I
2 2
:. JF.dr= J(x -2.0}tx-6(OXOXO)+8x(0)2(0)= Jx dx= .... (I)
~ x~ 0
Along PQ, x = I, z = 0, dx = 0, dz = 0 and y varies from 0 to 1.
I

:. JF.dr= J(12-2y~-6y(0)dy+8.1.(0)2(0)= Jo=O .... (2)


I'Q y=O

Along QR, x = I, Y = 1, dx = 0, dy = 0 and z varies from 0 to I.


I I
2
:. JF.dr= J(12-2.IXO)-6.1.z(0)+8.1.z2.dz= J8z dz=% .... (3)
QR z=O 0

Adding (I), (2), (3), fF.dr =~ + 0 +! = 3


3 3

(3) The equation of the straight line joining (0,0,0) to (1,1, I) is f = f


~ = = t (say),
so that x = t,y = t, z = t, dx = dy = dz = dt 't' takes values from 0 to I.
I I

.. JF.dr = n~2 2t)- 611 + 8tJ }it = J(8t 'Vt


2 3 2
- 5t - 21
c o o
Vector Differentiation 519

Ex. 7.6.12 Find the total work done by a force F = 2xyi - 4zj + 5xk along the curve x = t2,
Y = 2t + 1, z = t3, from the points t = 1 to t = 2.
Sol: x = t2 , dx = 2t dt; y = 2t + 1, dy = 2dt; z = t\ dz = 3t2 dt

Total work done = fF.dr


c

= f(2xyi-4zj + 5xk}.{(dx}i + (dy}j + (dz}k} = f2xydx-4zdy+ 5xdz


c

2 2

= fl2J 2(21 + I}J2Idl _{4./ 3 )2dt + {5/2 )3/2 dl = f{8/ 4 + 4/ 3 -81 3 + 15/ 4)dl
'~I 1

2
f{23/ 4 -4/ }11= [1
23--/4 ]2 =23
3
5
713 638
- (32-1)-(16-1)= - - 1 5 = -
1
5 5 1
5 5

Ex.7.6.13 If c is the curve y = 3x2 in the xy - plane and F = (x + 2y)i - xyj,


evaluate fF.dr, from the point (0,0) to (I ,3).
S'ol: Since c is a curve in xy - plane, we take r = xi + yj, so that
F.dr= {(x+2y)i-xyj}. {(dx)i+(dy}j} =(x+2y)dx-xydy.
1st Method: By taking the curve in parametric coordinates as,
x = t, Y = 3t2, dx = dt, dy = 6t dt, so that t varies from to 1 to get the points
(0,0) & (1,3): We have
(1,3) 1 1

fF.dr = fv + 6t 2) dt -/{3t 2) (6tdt) = fv + 6t 2 -181 4)dt


(0,0) '~O 0

t2 6/ 3 18t 5 1 1 18 5 18 11
= -+---1 = -+2--=---=--
2 3 5 0 2 5 2 5 10
2nd Method: >: =3x2, dy = 6x dx and x varies from
to 1.
1
.. fF.dr = f(x + 6x 2}Ix - x.3x 2.6x.dx
c 0

1
~( 2 11
= Jx+6x -18x 4 \px
,
=--
o 10
520 Engineering Mathematics - I

Ex. 7.S.14 Find the work done by the force F in moving a particle once around the circle
'C' in xy - plane, ifthe centre of the circle is origin and radius is '2' and
F = (x +y + z)i + (2x + y}j + (2x - y +z)k.
Sol: xy - plane is z = 0,
:. F = (x + y)i + (2x +y}j + (2x - y)k and r = xi +yj ~ dr = (dr)i + (dy}j
F. dr = (x + y)dx + (2x + y)dy.
y
The equation of the circle is x = 2cosS, Y = 2sinS
:. dx = - 2sinS dS, dy = 2cosS dS
S varies from 0 to 2n

Work done = fF.dr

2lt
= J(2COSS + 2sin sX- 2sinS)de + (2.2cosS + 2sin SX2cosS)dS
o
2lt 2lt
-~ J{- 2sin2S -4sin S + Scos S + 2sin 2S}ie = J{scos S - 4sin S ~S
2 2 2 2
o 0

2lt 2lt
= n4(1 + cos2S)- 2(1- cos2S)}iS = J(2 + cos2S)dS = [2S + 3sin 2S~lt
o 0

= 4n
Ex.7.S.1S Show that the necessary and sufficient condition for a vector field V to be
conservative is curl V = 0
Sol: a) Necessary condition: If V is conservative, :3 a 'cp' 3 V = V cpo
curl V= curl (V cp) = 0 (see 7.5.2 (1))
b) Sufficient condition: Let V = V I i + V'li + V 3k
i j k
Curl V = V x V = 0 ~ 8/ Ox 8/ By 8/ 8z = 0

~ I(8VBy 8V8z )i = 0
3 _ 2

.... (1)
Vector Differentiation 521

The work done by the force field V in moving a particle from (x\,y\,z\) to (x,y,z)

is IV.dr
c

= Iv; (x,y,z)dx + V2 (x,y,z)ay + V3 (x,y,z)dz


c

where V is a path joining (x\,y\,z\) to (x,y,z)


Let us choose a particular path consisting straight line segments from (x\,y\,z\)
to (x,y\,z\) to (x,y,z\) to (x,y,z) and denote the work done along this path by a
scalar function ~(x,y,z);
x y z
:. ~(x,y,z) = IV;(X'Yl,ZI)dx+ IV2 (x,y,zJly+ IV3 (x,y,z)dz ..... (2)
XI Yl
From (2), it can be seen that,
8cI>
8z = Vix,y,z) .... (3)

.z
= V2 (X,y,ZI)+ V2(X,y,Z~ = V2(X,y,ZI)- V2 (x,y,z)- V2(X,y,ZI)
.... (4)

from (1)

y z
= V\(x,y\,z\) + V;(X,y,ZI~ + V;(x,y,z~
Yl Zl

= V \(x,y\,z\) + V \(x,y,z\) - V \(x,y\,z\) + V \(x,y,z) - V \(x,y\,z\)


= V \(x,y,z) .... (5)

(3),(4),(5) => :i+ : j + : k=V;(x,y,z)i+V2 (x,y,Z)j+V3 (x,y,z)k=V

=> V = V~
Hence the proof.
522 Engineering Mathematics - I

Ex. 7.6.16a) Show that F = yi + (2xy +z2)j +2yzk is a conservative force field.
b) Find its scalar potential.
c) Find the work done in moving an object in this field from (1,2,1) to (3, 1,4)

j k
a) V x F= Max a/fJy a/az
Sol:
i 2xy+Z2 2yz
= i(2z - 2z) + j(O - 0) + k(2y -2y) = 0
:. F is a conservative force field.
b) Let 'cjl' be the scalar potential of F.
1st method:

2
:. acjl
ax = y2 .... (1) :=2Xy +z .... (2) acjl_2 z
az - y .... (3)

Integrating (1) w.r.t x, (2) w.r.t. y, & (3) w.r.t. z, respectively, we get,
cjl = xY + j(y,z), cjl = xY +yz2 + g(z,x), and cjl = yz2 + h(x,y).
These equations will be consistent iff, g, h are taken as
j(y,z) = yz2, g(z,x) = 0, h(x,y) = xy.
Hence cjl = xy + yz2 + constant
2nd Method: Since F is conservative, JF.dr is independent of path joining
c

(Xl,yl,zl) to (x,y,z)
using method of problem 7.6.15(b),
x Y Z

cjl = J&12}ix + J(2Xy + z/ }ry + J2yzdz = xy~ ( +(xy 2 + Z12 y) r+yz2 f


XI YI zl Xl .Y] zl

2
= xYI - XlYI + xy +z 12y - ry I - ZI 2y I + yz2 - yz I
= xy2 + yr - x1yl2 - Zl2Yl = xy2 + yz2 + constant.

3rt/ Method: Since F. dr = Vcjl.dr = acjl dx + acjl dy + acjl dz =dcjl ,


ax fJy az
Vector Differentiation 523

d~ == cY)dx + (2xy + z2)dy + (2yz)dz == cYdx + 2xydy) + (z 2dy +2yz)dz


== d(xy) +d(yz2) == d(xy2 + yz2)

=> ~ == xy + yz2 +constant


(0,1)8
1'2=(3,1,4)

c) work done == f F.dr == fd~


c IHI,2,1)

(3,1,4)
'=:-:::----i----"'I\( 1,0)
== ~(P2) - ~(PI) = xy + xz2 I
(1,2,1)

=(3.1 2 +3.4 2)-(1j2+ 1.1 2)=51-5=46.

Ex. 7.6.17 Evaluate f(x 3dy + y 2dx) where c is the boundary of the triangle whose vertices
c

are (0,0), (1,0), (0, I).

Sol: Let I == f(x 3dy + y2 dx)


c

I == II +12 +1 3, where II == f, 12 == f, 1 f
OA AH
3=
HO

(i) Along OA, y == 0, => dy ==


.
11 == Jx 3 0 + 02 dx == 0

(ii) AlongAB, x+y=1 ,y=l-x=> dy=-dx,xvaries from I toO.


o
:.1 2 = fx 3 {-dx)+{I-x)2 dx
I

_~o =0_(_~_~)= __1


3
= J(x 2_x 3 +1_2X)dx=_x
o 3 4 I 3 4 12
(iii) Along BO, x ==> dx = :. 13 =fo.dy+ i.o =
1
:. 1 = II + 12 + 13 = - -
12
Ex.7.6.18: If 1= xy2z2 ,evaluate fldr where the curve 'c' is given by

x = (, Y = (2, Z =(3 from t = to I .


524 Engineering Mathematics - I

Sol: f=xy2 Z2 =/(/2y .(/3 Y=1 11

dr=dx i+dyj+dz k
2 2
dr =(dt)i + (2tdl )j + (3/ dl )k =(i + 2tj + 3/ k )dl
1 1 1 1
Jfdr
c
= Jill
1=0
(i + 2tj + 3/ 2k )dl =i Jill + dl+ j J2/
0 0
12
dl+ k J3/ 13 dl
0

./12 I 2/ 1 3/ 14 I 1. 2. 3
13
=1- J+l-J+k- J=-I+-l+- k
12 0 13 0 14 0 12 13 14
Ex. 7.6.18: If A = 3zi - 2xj + yk , and c is the curve given by
x = cos I, Y = sin 1 , Z = 2cos I,
evaluate JA x dr from I =0to I = 2
c
1t .

i j k
Sol: A x dr = 3z - 2x y
dx dy dz
= (-2x dz- y dY)i+(y dx-3z dZ)j +(3z dy+2x dx)k
x = cos I, Y =sin I, z = 2 cos I
~ dx = (-sin/)dl,dy = (cos I) dl, dz = (-2sin/)dl
:. (1) ~ Ax dr = i[(-2cos/)(-2sin/) - sinl . cost]dt +j[(sin/)(-sint) - 3(2cost)
(-2sin/)]dl + k[3(2cos/)( cost) + (2cos/)(-sin/)]d/.
=i(3sint cost) dt + j[(12 sinl COS/) - sin2/)] dl + k(6cos2/- 2sint cost) d/.
1C/23 ft/2[ 1
J J
!
Axdr=i J-sin2Idt+ j
.. c o 2 0
6sin2t--(1-cos2/) dl
2
1C12
+ k ~3(1 + cos2/)-sin 2/}it
o

.3 (_COS2/)1C12.[ 1 sin 21 Jft/2 [ 3. COS2/]ft/2


= 1- + J -3cos2/--+-- +k 3/+-sI02/+--
220 24 0 2 20

= {
3
-4 )(-1-1)+ j[-3(-1-1)- : + 0]+k[3; +O+~(-l- ])]

= %;+(6- :}+e; -1)k


Vector Differentiation 525

Exercise - 7(e)
4

I) IfU(t)=(2t2 -I)i+3tj+(2-t)k. Find (a) JU(t)dt,(b) Ju(t)dt


2

2/
[Ans:(a) ( ~-I
3
12
2
i+ 3t j + ( 2t-
t2
\06 l
2 k,(b) 3i+18j-2k]
11/2
2) Evaluate: J(6sillu)i -(3cosu)j +uk
o
2
1t
[Ans: 6i-3j+-k]
8
2 2

3) If A(s) = si - s2j + (s + l)k, B(s) = 2si + 6sk find (a) JA.Bds, (b) JA x Bds .
o 0

100
[Ans: (a) 3' (b) - 48i - 12j + 16k]

4) If A = ti - j + 2tk, B = t2 ; - tj +2k, C = ; - 2j + 2k, evaluate


1 1

(a) J{A.(BxC)}dt,(b) J{Ax(BxC)}dt


o 0

I
5. 37. 8 k
[Ans' (a) -- (b)
+-} +- ] --I
. 2 6 3' 3
5) The acceleration of a particle a at any time t ~ 0 is given by a = eli + (2cos2t)j +
(2sin2t)k If the velocity V and displacement r are both zero at t = 0, find V and r at
any time t.
[Ans: V = (el - I)i + (sin2t)j + (1 - cos2t)k,

r =(i -t-I)+~(l-COS2t)j +(t -~Sin2t}]


3
6) Evaluate fA. dA du, given that, A(2) = 4i - 2j + 3k and A(3) = 2i + j + 2k
2 du
[Ans: - 10]
526 Engineering Mathematics - I

Exercise 7(f)

I. If ~ = xyz, valuate I~dr, where c is the curve x = t3, y = t2, z = t, from t = 0 to I


c

I. I k
[Ans: JI i +4J+ 7 1
2. If F = xi - yzj + z2k, and c is the curve given by x = t, Y = t3, z = t 2, evaluate
(i) IF.dl" and (ii) fF.dr, from t = 0 to t = I.
e C

.) 5. 7. 1\ k .. 23]
[A ns: ( I -71-I5J +1"2 ,(II) 24

3. If A = (2x + 3)i + xyj + (zx - y)k, evaluate fF.dr along c where c is

(a) the curve x = t3,y = 2t2, z = t from t = 0 to I.


(b) The straight lines from (0,0,0) to (1,0,0), then to (1,0, I) and then to (1,2, I)
(c) The straight Iinejoining (0,0,0) and (I ,2, I).
491 13 14
[Ans: (a) 105' (b) (c)2' "3]
4. If A = (3xy - 2r)i + (x - y)j, evaluate fA.dr along the curve c in xy - plane given
c

by y = x 3 from the point (0,0) to (2,8).


1308
[Ans: 3.5]
5. IfF = (2x- y)i + (x- 2y)j, evaluate fA.dl" where c is the closed curve shown in the
c

figure below.
y

x
Vector Differentiation 527

6. If A = (3x + 2y)i + (x + y)j, and c is the boundary of the tringle whose vertices are

(0,0), (1,0), (0, I), evaluate fA.dr.


c

3
[Ans: 2" J
7. If A = (2x + y)i + (3x - 2y)j, compute the circulation of A about the circle C:
xl + 1=4, traversed in the positive direction.
[Ans: 81t]
8. Find the work done in moving a particle in the force field.
F = 2x2i + (2yz - x)j + yk, along (a) the straight line from (0,0,0) to (3, I ,2)
(b) the space curve x = 3t2, Y = t, z = 3t2 - t from t = 0 to t = I
113 58
[Ans: (a) 6' (b) 3]
9. (a) Prove that V = (2x siny - 3)i + (x 2 cosy + z2)j + 2(yz + I)k is a conservative
force field. (b) Find the scalar potential ofY. (c) Find the work done in moving an

object in this field from (1,0,-1) to (2, 2"1t ,I).


1t
[Ans: (b) x 2 siny + yz2 - 3x + 2z + constant, (c) 2" + 5]

10. If A = (9xly - 2xz3 )i + 3x3j - 3x2z2k, (a) prove that fA.dr is independent of the
c

curve 'c' joining two given points. (b) show that there is a differentiable function <jl
such that A = \l<jl and find it.

7.7 SURFACE INTEGRALS


7.7.1 Let S be a two-sided surface. Let one side be taken as the positive side. If S is a
closed surface, the outer side is considered as the positive side. Let A be a vector
function. Consider an element of area 'ds' in the surface. Let n be the unit normal
vector to ds in the positive direction. It can be seen that A.n = A cose. (where '8' is
the angle between A alld n and A = IAI) is the normal component of A. Let ds be a
vector whose magnitude is ds and whose direction is that ofn.
:. ds = n ds.
528 Engineering Mathematics - I

Then the integral,

ffA.ds = ffA.Il .ds ..... (1)


s

is an example of a surface integral which is also called as flux of A over s.


If'f' is a scalar function,

ff4>ds ..... (2)

..... (3)

ffAx ds = ffA x nds ..... (4)

are some other examples of surface integrals.


Note (1) The surface integrals can also be defined in terms oflimits of sums. (see 7.7.2)

(2) The notation If is also used to denote a surface integral over the closed
surface s.

(3) Sometimes the notation f may also be used for surface integrals.
(4) Surface integrals can be conveniently evaluated by expressing them as double
integrals over the projected area of s on one of the coordinate planes
{see 7.7.3)
Vector Differentiation 529
/

7.7.2 Definition of surface integral as the limit of a sum:

.,J------...... y

The area S is divided into 'L' elements of area Mill' m = 1,2, .... L,
let P'II = (xm'YIII,zm) be any point in &\)111. Let A(xm,Ym,zl/.)= AHI
Let nlll be the positive unit normal to I'1S111 at Pm. Then (Am.nm) is the normal
component of Am at Pm. Consider the sum,
/,

I
1/1=1
Am.nIllMm ..... (I)

The limit of the sum (l)as L ~oo such thatthe largestdimens~on of each I'1S", ~ 0
(if the limit exists) is known as the surface integral of the normal component of A

over S an d denote d by , IIA.n.d'}


7.7.3 Evaluation of a surface integral
To evaluate surface integrals, it is convenient to express them as double integrals
taken over the projected area of the surface S on one of the coordinate planes
(xy,yz, or zx planes).
If R is the projection of S on the xy - plane, it can be shown that

ffA.nds = ffA.n d\XdY,


, /I n.k
From 7.7.2, the surface integral is the limit ofdle sum
/,

I
111=1
Am .nmMIII ..... (1)
530 Engineering Mathematics - I

;---~----~~~--.Y

The projection of ASmon thexy- plane is InmA.S'II/.kl (or) Illm.kIMi'm which is equal

:. The sum (I) becomes

By the fundamental theorem of integral calculus the limit of this sum as L ~ 00 in


such a manner that the largest Llx m and AYm approach zero is

dxdy
IfA.n In.kl
R which is the required result.

Note: Similarly ifR is the projection ofS on yz and zx planes respectively, it can be seen as

ff A .nds = ffA.n dl'Y~ZI


S R nJ

and ffA.nds = ffA.n dz~xl


S R ln.]
7.7.4 Physical interprettion of surface integrals:
Let A denote the velocity of a moving fluid. Let S be a fixed surface in the fluid. Let
ds be an element of surface. Then A.n ds = A.dS represents the amount offluid that
passes normally through dS in unit time at any point. If the direction ofn is outward
or positive, the amount offluid flow is positive. Similarly if dS' is another element for
which n is in the negative direction, the fluid flow is negative at that point.
Vector Differentiation 531

to fA.nds and it is known as the total flux of A through the entire surface S.
s
A can be a vector denoting physical quantities such as electric force, magnetic
A

n....---...---~C"

force, flux of heat or gravitational force etc. In all these cases, fA.nds
s
denotes

total flux of A through S.

Sioved Examples

Ex.7.7.5 Evaluate fJA.nd\" where A = {x + y2} - 2xj + 2yzk and S is the surface of the
plane 2x + y + 2z = 6 in the first octant.
Sol: A = (x + y)i - 2.>.] + 2yzk
Let ~ = 2x + Y + 2z - 6 z

V ~ = 8q>i + aqy + 8q>k = 2i + j + 2k


ax ~ az
V~2i+j+2k
Unit normal n to S = IV~I = ~22 + 12 + 22

2i+ j + 2k
3

x
532 Engineering Mathematics - I

= .![2x+ 2y2 - 2x + 2y(6 - 2x - y)]


3
[Substituting 2z = 6 - 2x - y ( ... 2x + y + 2z = 6)]

= .!(12y-4xy)
3
If R be the projection of S on the xy - plane.

2 dxdy 3
In.kl = -3 => ds = -l/l.kl- = -dxdy
2

.. IfA.nds = If(A.n) dXdY


/I /I ln.k I

If .!(12y - 4);y )~dxdy


/I 3 2

= If(6 y - 2Xy}dXlry ..... (I)


/I

To evaluate this double integral over R,


(i) Keep x fixed and integrate w.r.t. y from y = 0 to (6 - 2x), ii) and then integrate
w.r.t. x from x = 0 to x = 3.
:. Given integral
3 6-2x

J J(6y-2xy)1ydx
x=O y=O

3 3 3
TI3i _xy2 r:~x dx =
2 3
= J(3 -xX6- 2xY dx = J(J08-108x+36x -4x }Ix

= [\08x-54x2+12x3-x4]~ =324-486+324-81 =81

Ex.7.7.6 If F = 4xzi - yJ + yzk, evaluate IfF.nds where S is the surface of the cube
s

bounded by x = O~ x = I, Y = 0, Y = 1, z = 0 and z = 1.
Vector Differentiation 533

Sol. The surface S can be divided into 6 faces (see figu --, z
o
(i) S\: Face EPFA
(ii) S2: Face OBOC E f---+----(
(iii) S3: Face PFBO
y
(iv) S4: Face OCEA
(v) Ss: Face POCE
x
(vi) S6: Face OBFA

IfF.nd5 = IfF.nds + IfF.nds + IfF.nds + IfF.nds + IfF.nds + IfF.nd.,.


S ~ ~ ~ ~ ~ %
On S \: n = i x = 1
1 1 1 1 1 1 1

IfF.nds = JJ(4Zi - y2 j + yzk }idydz JJ4zdydz= = J2Z21~ = J2dY = 2


SI 0 0 0 0 0 0 0

On S2: n = - i, x = 0
1 1 1 1

HF.n.d.,. = JJ(- y2 j + yzk X- i)dydz JJ(O}lydz= = 0


o 0 o 0

On S3: n = j, y =I
1 1 1 1 1 1 1

HF.n.ds=J J(4xzi-j+ zk).jdxdz =


.'13 0 0
J J(-I)1xdz= J-zi
0 0 0 0
dx= J-Idx=-l
0

OnS 4 :n=-j,y=O
1 1 1 1

:. HF.n.ds = J J(4xzi).(- j)dxdz J J(O)1ydz


o 0
=
o 0
= 0

On Ss: n = k, z = 1
1 1 1 1 1 21

:. HF.n.ds=J J(4Xi-/j+yk}kdxdy=
8S 0 0
J Jydxdy= J~ I
0 0 0 0

On S6: n = - k, Z =0
1 1

HF.n.ds
.'16
=
0
J J{- y2 j }(- k )dxdy =
0
0

. JJF.n.ds =2 + 0 - 1 + 0 + ~ + 0 = ~
.. 2 2
s
534 Engineering Mathematics - I

Ex.7.7.7 If A = z 2i + xlj - yZk, and S is the surface ofthe cylinder xl + Y = 16 included

in the first octant between z = 0 and z = - 5, evaluate jfA.n.ds


s
Sol: z

f d~- ~~'
,. 5 R Q
,....

x
r=-----'JL..-- y
4 '<.
- ' z=Q
n

Project s on xz plane and let the projection be R. (See figure)

HA.n.ds = HA.n dx,d~, ..... (I)


S R n.)
The norma I to x 2 + Y = 16 is
V(X2y2 )=~(X2 + y2) +~(X2 + y2)j
ax ay
= 2xi+ 2yj
. 2xi + 2yj 2(xi + yj) xi + yj
Ul1Itnormaln= ~(2xY+(2yy = 2JX2+y2 =-4-
(.: xl + Y 16)

Z
2X+X 2 y
A.n = ------'--
4
. y
n.J= "4
:. From (I) J JA.nds
S
= J V2X+X2y dxdz
R 4 y/4

[.: J x
o ~16 _x 2
dx=4,

3
= 4z + 64zf = 820
3 3 0 3
Vector Differentiation 535

Ex.7.7.8 Evaluate I I<I>nd~ where S is the surface of problem 7.7.7 above and <I> = x~z
s
dxdz
Sol: We have Ifs <l>nds = If<l>n-
I
II
1
n.)

xi+ yj y
using n = - 4 - ' n.j = "4' the integral on R.H.S. becomes,

R
-
1
80
64 Z.
-l+-}
3
64Z.
3 f Z

I 64(z2.
-.- -1+-)
Z2 .]1
5
8 25(.
=-.--;;-1+) =-1+.1
.) 100(. .)
83 2 2 0 3 ~ 3

Ex.7.7.9 If A = yi -+ (x - 2xz)j - xyk, evaluate I I{curIA).nd~ where S is the surface of


s
the sphere x2 +.0 + z2 = 4 above the xy plane
Sol:
Z n

I--~-+--Y

j k
CurlA= a/ax a/fJy a/az
y x-2xz -xy
536 Engineering Mathematics - I

= i{~(-xy)-~(x-2xz)} + j{~(y)-~(-xy)} +k{~(x-2xz)-~(y)}


~ & & fu fu ~
= xi + yj - 2zk
The normal to the surface is V (xl + .0 + z2) = 2xi + 2yj +2zk
2xi + 2yj + 2zk
Unit normal n = I
,,(2x)2 + (2y)2(2z)2

xi+ yj+zk
2
x 2y2 _2z2
(Curl A). n = 2

The projection of S on xy plane is the circle xl +.0 = 4, Z = 0 (see figure)


.. f.f(curl A.n}ds = f [(CUr! A.n) I:'~f

= fpX2 +i - 2Z2 ) dxdy


/I 2 z 12

2
2 4x ( 2 )
3 x + y2 - 8dxd
f f I 2 2
x=-2Y=_~4_x2 ,,4-x y
OJ

Changing into polar coordinates by taking x = r cose, y = r sine, dx dy =r dr de, the


integral becomes

de= f(8-8}ie=o
8=0
Vector Differentiation 537

Ex.7.7.10 Evaluate J JA.nds where A = yzi + zx} + xyk and S is the part of the sphere
s
x 2 + y2 + z2 = 9 which lies in the first octant.
. V x 2 + y2 + Z2
Sol: Untt normal n to S = IV x 2
+ y +z-
' ,

2xi+2y}+2zk xi+ y}+zk


(.: x2 +y2 +z2 = 9)
~4X2 + 4y2 + 4Z2 3

z
A.n = 3xyz; nk=
. -3
If R is the projection of S on xy - plane, we have,

Jsf A.nds = f fA.n d1xdY


/I n.k I

= f f 3xyz dxdy =9 f fxy dxdy


/I z/3 II

The region R is bounded by x-axis, y-axis and the circle x 2 + y2 = 9; z = 0 chang,iog


to polar coordinates, the last integral becomes
",/2 3 rr/23
9 J f(rcos9Xrsin9)rdrd9=9 J[Jr 3 cos9sin9dr}19
8;0 r;O 0 0

= 9 fo ~4)31 cos9sin9d9=9.-811[/2fcos9sined9
1[/2(
4 0 4 0

(
1[/2
': !cos9Sin9d9=2"
I)
Exercise - 7(g)

I. If F = 18zi - 12x} + 3yk, evaluate J f F.nds where S is that part of the plane
s
2x + 3y + 6z = 12 which is located in the first octant
dxdy
(Hint: Take projection ofS on the xy plane and ds = -1-1
n.k
)
[Ans: 24]
538 Engineering Mathematics - I

2. Evaluate f f V.nd~ , where V = yi + 2xj - zk and s is the surface of the plane


s
2x + Y = 6 in the first octant cut off by the plane z = 4
[Ans: 108]

3. If r = xi + yj + zk find the value of the integral f fr.nds over


s
a) the surface S of the unit cube bounded by the coordinate planes and the planes
x = y = z = I and
b) the surface of the sphere of radius 'a' with centre at the origin
[Ans: a) 3, b) 41ta3]

4. Evaluate f fF.d~ over the entire surface ~fthe region above the xy plane bounded
s
by the cone :l- = x2 + Y and the plane z = 4, if F = 4xzi + xyzj + 3zk
[Ans: 3201t]
5. IfS is the surface of the parabolic cylinder y = 8x in the first octant bounded by the

planes y = 4 and z = 6, evaluate f fF.nds, where F = 2yi - zj + x 2k


s

[Hint: f f F .nds = f fF.n dIY~IZ , where R is the projection ofS on the yz plane]
S R nJ

[Ans: 132]

6. Evaluate f fV.nds over the entire surface S of region bounded by the cylinder
S

x2 + :l- = 9, x = 0, y = 0, z = 0 and y = 8 if V = 6zi + (2x + y}j - xk


[Ans: I81t]

7. Evaluate f f A .nds where A = zi + xj - 3yzk and S is the surface of the cylinder


s
x2 + Y = 16 included in the first octant between z = 0 and z = 5
[Ans: 90]
8. If V = yzi + zxj + xyk and S is that part of the sphere x2 + Y +:l- = 1 which lies in
the first octant, find the value of f fV.nds .
s

3
[Ans: "8]
Vector Differentiation 539

9. Evaluate JJ{(x 3
- yz2)i _(2x2y)j + 2k}.ds over a cube with edges of length 'r'
s
parallel to the coordinate axes
I
[Ans: 2"]
10. If V = xi + yj + zk, and S is the 'triangle with vertices at (1,0,0), (0, 1,0) and (0,0, I),

find the value of I IV.ds .


s

I
[Ans: 2"]
II. If A = xi - yj + (z2 - I )k, find the value of I IA.nds, where S is the closed surface
s
bounded by the planes z = 0, z = I and the cylinder x 2 + y=I
[Ans: 1t]
7.8 VOLUME INTEGRALS
7.8.1 Consider a closed surface in space enclosing a volume V. Then, integrals of the

form I I IAdv and I I I~dv, [A is a vector function, f is a scalar function] are


v "
examples of volume integrals.
7.8.2 Expression of volume integral as the limit of a sum:
Let A be a continuous vector function. Let S be a surface enclosing the region D.
Divide this region D into a finite number of subregions D, ... ,= 1,2, ..... n.
Let L1v, be the volume of the subregion D, enclusing any point whose position
vector is rio
Consider the sum
II

V = LA~JL1v,
,;1

The limit of this sum as n ~oo such that L1V, ~ 0, is called the volume integral of

A over D and denoted by I I I Adv


J)

If A = A I (x,y,z)i + Aix,y,z}j + A 3(x,y,z)k,


540 Engineering Mathematics - I

so that dv = dx dy dz

f f fAdv = iff fA,(x,y,z}lxdydz + j f f f A2(X,y,z}!xaydz +


D D D

k f f f A3(X,y,z}lxdydz
f)

Solved Examples

Ex.7.8.3 If F = (2x 2 - 3z)i - 2xyj - 4xk, evaluate f f fV.Fdv where V IS the closed
v
region bounded by the planes x = 0, y = 0, z = 0 and 2x +2y + z = 4.

Sol: VF = ~(2X2 -3z)-~(2xy)-~(4x)=4x-2x=2x


. ax ry az

2 2-x4-2x-2y 2 2-x 4-2x-2y

.. f f fV.Fdv= f f f 2xdzdydx= f f [ f 2xdz ]dydx


v x=Oy=o z=O x=o y=o z=O
2 2-A 4-2x-2y 2 2-x 2 2-x
2
f f 2xz l dydx = f f 2x(4-2x-2y}!ytlx= f[ f(8x-4x -4xy)t(y]dx
A=O y=O Z=O x=O y=O x=O y=O
2 2-x 2
= f8xy-4x2y-2xy2fdx f[8x(2 - x)- 4x 2(2 - x)- 2x(2 - X)2}1x
x=O y=O x=O

64 8
=16--+8=-
3 3

Ex.7.8.4 Evaluate ff j(V.A}tv over the region bounded by x + Y = 4, z = 0 and z = 3,


2

v
where A = 4xi - 2yj + z2k.

a(4x ) --(2y
V.A=- a 2 a 2 )=4-4y+2z
)+-(z
Sol: ax ry az

x=2 y=J4-x 2 3
.. fff(v.A}!v= fff(4-4y+2z}lv=
v v x=-2 y=-v4-x~
f
f [J(4-4y+2z)dzf/ytlx
12 z=O
Vector Differentiation 541

2 )'~f4-x2
dydx==
x~-2
J[ JC 2l - 12 y)dy]dx
;--- ?
y=-Y4-r-

x r.-? x 2 n
==84[--v4-x 2 +2sin- ' C-)] ==84[O+2C-)-O]==84n
220 2
Ex.7.8.5 Evaluate JJJ<I0v taken over the rectangular parallelopiped 0 s:; x < a, 0 s:; y < b,
J'

os:; z < c and ~ == 2{x + Y + z)

Sol: fJfv ~ ff f2{x+ Y+ z}Jv ~ ,U[}{x + y + z) }YdY

a b a a
== J2exy+ cy2 +e 2YI dx == JC2bex+eb2 +e 2b)dx==bex2 +(b 2e+be 2)xl
x=O 0 x=o 0

== a 2 be + (J(b 2 e + be 2) == abe {a +b + e)

Ex. 7.8.6 If ~ == 4y + 2xz, evaluate JJJ~dv over the region in the first octant bounded by
J'
x2 +1 = 9, z = 0, z = 2.
Sol: JJJ~dv== JvJJ{4y+2xz}lv
I'

J.=3y=J9-x 2 z=2 ~2 3 ?

J J [J(2xz+4y}lz]dydx== J2
JC4yz+xz )f dydx
x=O y=o ==0 0 0 0

3 ~x2 3 ~2
= J[ J(8y + 4xX/y]dx = JC4y2 +4xy)1 dx
o 0 0 0

3
= J[4(9-x2)+4x~9-x2]dx==108
o
542 Engineering Mathematics - I

Ex.7.8.7 Evaluate f f fFdv where F = xzi - 2xj + 2y.k and V is the region bounded by
v
the surfaces x = 0, y = 0, y = 6, z =.x2, and z = 4
Sol:

z Q s
\:
y=
.Q...~
z=x
2
z=4 ,I ~y =6
\ '\
L _________ J// y

-i <
The region V is covered by (see the figure) (a) keeping x andy fixed and integrating
from z = .x2 to z = 4 (base top of column PQ) (b) then by keeping x fixed and
integrating from y = 0 to y = 6 (R to S in the slab) and (c) finally integrating from x

= to x = 2 (where z =.x2 meets z = 4).
:. The required integral is

2 6 4
f f[ f(xzi-2xj+2y 2k)dz}1ydx
x=Oy=O z=x2

264 264 264


=i f f[ fxzdz]dydx+j f f[ f- 2xdz ]dydx+k f f[ f2y dz}1ydx
2

x=O y=O z=x2 x=O y=O z=x2 x=O y=O z=x 2

26 24 26 4 26 4
= i f f x; J dydx+ j f f- 2xz J dydx+k f f2 y 2 z J dydx
x=O y=O z=x2 x=O y=O z=x2 x=O y=O z=x2
Vector Differentiation 543

26 5 26 26

=i J[ J(8X-~)dy}lx+j J[ J(2x 3
-8x)dy]dx+k J[ J(8i- 2x2 y)dy]dx
x=() y=O x=o y=O ~=o y~o

X5 2 8 y3 2X2 3
= i
2
J8xy-~
2
6
I dx+ j J2x'y-8xyl
)'~()
6

)'~O
dx+k J-'--_Y-
3 3
6
I
y=O
Ix
x=o x=()

2 2 2

=i J( 48x - 3x 5
)dx + j J(l2x 3 - 48x)dx + k J(576 - 144x2 )dx
o 0 0

1 2 2 2
= i(24x 2 -:;- x 6 ) I + j(3X4 - 24x2) I + k(576x - 48x 3 ) I
~ x=o ~=O x=o

= 64i - 4~i + 768k


Ex.7.8.8 Find the volume of the region common to the intersecting cylinders x 2 + r =b2

and x2 +z2 = b2 .
Sol:
z 2 2 2
x +z ::: b
/
/ I /'"
",

I I-
II
V - 2 2
_''x +y ::: b
2

V
~o y

Required volume is equal to 8 times the volume of the region shown in the figure
(as the axes cut the volume into 8 equal parts one in each octant)
544 Engineering Mathematics - I

Exercise - 7(h)

1. Evaluate Iff( 2x + y) dv where V is the closed region bounded by the cylinder


80
z = 4 - x 2 and the planes x = 0, y = 2 and z = 0 . I ADS: - I
3
2. If A = (2X2 - 3z)i - 2xyj - 4xk , and V is the closed region bounded by the
planes
x=O,y=O,z=Oand 2x + 2y + z = 4,findthevalueof Iff( curlA )dv
[ADS: 1u -k) )
3. Evaluate Ifff dv where f = 45x 2 y and v is the closed region bounded by the
planes
4x+2y+z = 8,x = O,y = 0 apd z = 0 [ADS: 128)
4. If A = 2xzi - xj + y2 k and v is the region bounded by the surfaces
x = O,y = O,y= 6,
z= x 2
and z = 4, find the value of Iff Adv ADS: 128i-24j+384k

5. Evaluate ffI(Div A)dv taken over the rectangular parallelepiped,

o~ x ~ 1, 0~y ~ 2,
o~ z ~ 3 , where A = (X2 - yz) i + (i - zx) j + ( Z2 - xY) k [ADS:3 6 1

6. Evaluate Iff(Div F)dv for the volume of a cube with edges of length unity
parallel to the coordinate axes where F = (x 3 - yz2 ) i - ( 2X2Y ) j + 2k
[ADS: 1/3)
Vector Differentiation 545

7.9 CURVILINEAR COORDINATES


7.9.0 The students are already familiar with the coordinate systems such as i) cartesian
ii) Polar and iii) parametric.
A study of yet another coordinate system called "curvilinear coordinate system"
will be now taken up.
7.9.1 Transformation of coordinates
Suppose the rectangular coordinates (x,y,z) of any point be expressed as functions
of variables vI' v2' v3 as
...... ( I)
Let the above system of equations (i) be solved for vi' v2' v3 and another system of
equations
..... (2)
Ifthe functions in (I) and (2) are assumed to be single valued and to have continuous
derivatives, the correspondence between the variablesx,y, z and vI' v 2' v3 is unique.
Thus, to a given point P with cartesian coordinates (x,y,z), we can associate from
(2) a unIque set of coordinates (vl'v 2,v3 ) which are called curvilinear coordinates.
The systems of equations (I) and (2) give the "transformation of coordinates".
7.9.2 Orthogonal Curvilinear coordinates
z V3 curve

........ ;
>-----------~-------------y
"

x
The surfaces vI = ci' v2 = c 2 ' v3 = c 3. (cl'c2, c 3 beIng constants) are called
'coordinate surfaces'. (see figure).
Each pair of the above surfaces intersect in curves called 'coordinate curves or
lines'. i.e., vI = c I and v 2 = c2 intersect in v3 curve, v 2 = c2' v3 = c 3' vI = c I in v 2
curve.
546 Engineering Mathematics - I

If the coordinate surfaces taken in pairs intersect at right angles the coordinate
system is called 'an orthogonal curvilinear coordinate system'.
Note: The coordinate surfaces described above are similiarto the coordinate planes of the
rectangular system and the coordinate curves are similar to the coordinate axes.
Infact, the rectangular coordinate system is also an example of curvilinear system
with VI = x, v2 = y, v3 = z; c I = c2 = c 3 = 0
7.9.3 Unit vectors of a curvilinear system
Consider the position vector r = xi + yj + zk, of a point P. using (I), it can be written
as r = r(v l ,v2,v3)

(i) A tangent vector to the VI curve at P(for which v2' v3 are constants) is ~. If e l is
av,
.
the Ul1lt tangent vector we have e I = I I'
ar/av
ar

av,

lilly if ar , ar are tangent vectors to the v2 and v3 curves at P respectively and e2,
av aV3
2

e3 are unit vectors in these directions,


ar ar
we can write , -av = A2 e2 , -
av3 = A3e3
2

where

The unit vectors e I' e2 , e3 are in the directions of increasing v\' v2' v3 respectively.
The quantities A\, A2 , A3 are called 'scale factors'.
(ii) 'Vv, ' is a vector at P normal to the surface VI = c 1. If EI is a unit vector in this
direction,

VV 2 Vv
lilly the unit vectors, E2 = IVv 1' E3 = IVv: I at P are respectively normal the surfaces
2

v2 = c 2' and v3 = cy
Vector Differentiation 547

V3 curve

Thus, in general, at a given point P there exist two sets of unit vectors,
(i) el' e2, e 3 tangential to the coordinate curves and
(ii) E], E2, E3 are normal to the coordinate surfaces (see figure)
7.9.4 Representations of a vector
(a) A vector A can be represented in terms of the unit base vectors e], e2 , e3 or
EI' E2, E3 in the following forms:
(i) A = ale] + a 2e2 +a3e3
(or) (ii) A = A]E] + A2E2 + A3E3'
Here a\, a2 , a3 or AI' A 2, A3 are the components of A in the two systems
. 8r 8r 8r
respectively (b) The vectors - , - , - and Vv" Vv J , Vv3 , are called
av, av2 av3 -

'Unitary base vectors (need not be unit vectors in general). We can also represent
A in one of the following forms; as

Here
CI' c2' c3 are called the 'contravariant components of A and CI' C 2, C 3 are
called the 'Covariant components of A
548 Engineering Mathematics - I

7.9.5 Arc length, area (surface) and volume elements


(i) Let r = r(vl'v2,v3)

:. dr= ~dvJ + ar dV 2 + ar dV3 =AJ(dvJe J+A 2 (dv 2 )e2 +A 3(dv 3)e3 ..... (3)
8v J aV2 aV3
Since ds 2 = dr. dr, the differential ds of arc length is given by,

ds 2 = AI2dvl2 +A}dv} +A/dv/ for an orthogonal coordinate system


(.,' e l .e2 = e2e3 = e3.e l = 0)
(ii) Let dvl' dv 2, be vectors specifying arc-elements along VI and v2 curves
respectively at point P.
i.e., dV I = dr along VI curve and dV 2 = dr along v2 curve.

Since v2 ' v3 are constants along VI curve and vI' v3 are constants along v2
curve, we have, from (3),
..... (4)
The area of the parallelogram formed with the vectors of( 4) as adjacent sides is
the area element or surface element 'ds 3 ' on the surface v3 = c3 (see fig) and it
is given by
V2 curve
ds 3 = Idv i x dv21 = IAIA2dvldv2e31 = AIA2dv ldv2
Oily ds I = A2A3dv2dv3

dS 2 = A3Aldv3dvi
are surface elements on surfaces
VI = c i and v2 = c2 respectively.
(iii) The volume element 'dv' for an orthogonal curvilinear coordinate system is
equal to the volume of the parallelopiped whose coterminous edges are dv I' dv 2 ,
dv 3

i.e., Aldvle l, ~dv2e2' A3dv3e3


(see figure)

:. dv = I(Aldv1e(). (A2dv 2e2) x (A3dv3e3~


= AIA2A3 dv(dv2dv3 le(.e 2 x e31
= A(A2A3 dv 1dv2dv3 (.,' le 1.e2 x e31'; 1)
Vector Differentiation 549

7.9.6 EXPRESSIONS FOR GRADIENT, DIVERGENCE AND CURL IN AN


ORTHOGONAL CURVILINEAR COORDINATE SYSTEM:
7.9.6(1) Gradient of a scalar function ~:

Prove that V'~ (i.e. grad~) = [ Ia~c7v


~
I~ I~
el +~ c7v e2 +~ c7v e3
1
I I 2 2 3 3

Proof: Let V'~ = gle l + g2e2 +g3e3' where gl' g2' g3 are to be determined

ar
dr= -dvI +-dv2 +-dv3
ar ar
c7v1 c7v 2 c7v3
= (A1el)dv 1 + (A 2e2)dv2 + (A 3e3)dv3
d~ = V'~.dr,

d~ = A1g1dv 1 + A2g 2dv2 + A3~3dv3 ..... (A)

Again, d~ = ~ dVI + ~ dv 2 + ~ d,V3 ..... (8)


av l av 2 av 3

Note: The above expression indicates that V'


el
= [ ~a;
~ e2 ~
+ ~ c7v + ~ c7v
e3 ~ 1
I I 2 2 3 3

7.9.6(2) Ifvl' v2' v3 are orthogon~1 coordinates, prove that


(a) lV'vII = A,I,(I= 1,2,3)
(b) e l = E1(I= 1,2,3)
We know that, (from 7.9.6 (I))

V~=~ a~ +!2~ +~ a~
Al c7v1 A2 c7v2 A3 c7v3
550 Engineering Mathematics - I

Hence

lilly I V I= -:;:
n
v 2
e
= ~ IVV3 I= -:;:
2

2
-] e
1\,2, = ~-]
3

3
1\,3 , which proves (a)

(by definition, see 7.9.3(ii))

/= 1,2,3. [from (a)]

y,
7.9.S. (3) Prove that, in orthogonal curvilinear coordinates,

V.(A,eJ=
1 a (A,AzA3) and hence show that,
A]A2A3 Ov]

VV 2 xVV3 = -1- (e2 xe3 ) = -


e, - ..... (C)
A2 A3 AzA3
e] = A2A3 (VV2 X Vv 3)
lilly e2 = A3A, (VV3 x Vv,) and e3 = A,A 2(Vv, X r
VV 2
V.(A]e,)= VAA 2 A3 (Vv 2 x Vv 3)
= V(A,A 2A3}(Vv2 x Vv 3)+ A,A zA3 {V.(Vvz x Vv 3 )}

= V(A,A2A3).~
A2 A3
+ A'A2A3{V.~}
A2 A3
..... (D)

(Using (C))

Now,
Vector Differentiation 551

(Writing the expression for V )

= _I ~(_I_)+O+O
AI Ovl A2 A3

=0

=
1 a (AIA2 A3)
..... (E)
AIA2A3 Ovl
(Operating V)

lilly, we get, V.A 2 e2 ==


1 a (A2A3AI) ..... (F)
AIA2 A3 Ov2

and ..... (G)

Hence, if A == AIel + A2e2 + A3e3,


Div A = V.(Ale l + A2e2 + A3e3)
= V.(Ale l )+ V.(A 2e2 )+ V.(AJ e3)

7.9.6 (4) Prove that in orthogonal curvilinear coordinates,

Vx(AleJ==~~(AIAJ-~~(AIAI) and hence,


A3 AI Ov3 AIA2 Ov2
552 Engineering Mathematics - I

e
.: VVI = -.L, we have,.
AI
V x (Ale l )= V x (A,A,Vv,)
=V(A,AI)xVv, +(A,A,)(VxVv,)

(from 7.5.2 (i) curl grad = vI = 0)

= [!l~(AIAI)+
AI oV
e ~(AIAJ+!2.~(AIAJlx!l
z
A Ov 1..,3 oV AI
I z 2 3

(substituting for V)

..... (i)
lilly, it can be shown that
V x (A 2 e2)= ~~(A2AJ--e'-~(A2AJ ..... (ii)
1..,11..,2 Ov l AzA3 OV3
and

Vx (A3e3 ) = - el - -0- (A3A3 ) - -ez- -0- (A3A3 )


..... (iii)
1..,21..,3 Ov z 1..,31..,1 Ov l

:. curl A = V x A = V x (Aiel + A2 e2 + A3e3)


= Vx (Ale,)+ V(A 2e2)+ V(A 3 e3)

= _e_1
{~(A3A3)-~(A2AJ1+~{~(A'AJ-~(A3A3)1
A2 A3 oV2 Ov3 f 1..,31..,1 Ov3 Ovl f
(Adding R.H.S. of (i), (ii) and (iii))

Ale, A2e2 A3e3


--olov, olOv2 olOv3
1..,11..,21..,3 "I
A,/\', A2A2 A3 A3
Vector Differentiation 553

Ex. 7.9.6. (5) Derive an expression for V2~ in orthogonal curvilinear coordinates.

Sol: (from 7.9.6(1

A=_I~
I AI Ov I '

.. V2~ = V.V~ = V.A

Note: In the expressions for V , grad~, Div A, curl A, and V2~ derived in 7.9.6(1) to

7.9.6(5), if we put vI = X, v 2 = y, v3 = z; AI =A2 = A3 = I, and e l = i, e2 = j, e3 = k;


we obtain the corresponding expressions in (rectangular) Cartesian coordinate
system. [The reader is advised to verify]
7.9.7 Spherical Polar coordinates (Spherical Coordinates)
7.9.7(1} If we take vI = r, v2 = e, v3 =~, the curvilinear coordinate system becomes a
"spherical polar coordinate system".
z

x
554 Engineering Mathematics - I

The equations of transformation are,

x = r sin ecos~,}
y = rsinesin~
..... (i)
z =rcose

where r ~ 0, O:s ~ < 2n, and O:s e:s n .

7.9.7(2) Coordinate surfaces and coordinate curves.


(i) The coordinate surfaces (level surfaces) in a spherical coordinate system are
r = c] which are spheres with centre at.origin [or origin itself if c] = 0]
e = c2 which are cones with verex at origin
(lines if c2 = 0 or nand xy-plane if c2 = n12)
~ = c3 which are planes through the z-axis
(ii) The coordinates curves are:
(a) Intersection ofe = c2 and ~ = c3 is the r--curve and it is a line
(b) Intersection of r = c] and ~ = c3 is the e--curve. It is semi circle if c] :j:. 0
(c) Intersection of r ",,-c] and e = c2 is the ~urve. It is a circle (or point).
7.9.7(3) Scale factors:
r = xi + yj +zk
= (rsinecos~)i + (rsinesin~)j + (rcose)k

~ = Or = (sinecos~)i + (sinesin~)j + (cose)k


Ovl Or

A =
] 8r
18rl=~(sinecos~)2+(sinesin~?+(cose?
=1

8r = 8r =(rcosecos~)i+(rcosesin~)j-(rsine)k
Ov2 as

=r
Vector Differentiation 555

- or = -or = (-rsm. 0 Sill",


. .t.).1+ (rSIl1
. 0 cos",.t.) }.
Ov 3 0<1>

A =
3
lorl=~(-"sinOsin<l2
0<1>
+ (rsinOcos<l2

= r sinO
: .. The scale factors of spherical coordinate system are
A( = I, A2 = r, A3 = r sinO.
7.9.7(4) Base vectors: The base vectors e (' e 2, e 3 are taken for the sake of convenience
as er , eo' etjl respectively. They are,

e = e = _I ~=_I or = (sin Ocos<li + (sin Osin<lj + (cosO)k


r ( AI Ov l AI or
eo = e2 =_1 or = ~[(rcosOcos<li + (rcosOsin<lj -(rsinO)k]
AI GO r

= (cosOcos<li+(cosOsin<jj-(sinO)k

and eel> =e3 =_1 or =-~-(-rsinOsin<li+(rsinOcos<lj


A3 0<1> rsmO

= (- sin <Ii + (cos <I> )j


From the above expressions, it can be observed that
e,..e s = es.eq. = er.eq. = 0

and er xes =eq.; efJ xeq. =e,.; eq. xe r =eo;


[students are advised to verify the above equations]
which show that "the spherical polar coordinate system is a right-handed orthogonal
coordinate system".
7.9.7(5) Arc length, Area element, and volume element in spherical
coordinates:
A( = I, A2 = r, A3 = r sinO
~ 2d 2
.) dS 2 = 11.1 ~2d 2 ~ 2d 2 (from 7.9.5(1))
I VI + 11.2 V2 + 11.3 V3

= 12(dr)2 +r2(dO)2 + (rsinO)2(d<l>f


gives the arc length 'ds'.
556 Engineering Mathematics - I

and

ds r , ds s ' dStjl are the area elements on the respective surfaces


r = cl' e = c2' and cj> = c3.
iii) From 7.9.5(3), dv = 1..)1..21..3 dv)dv2dv3

= I.r.{rsin e)dr.d9.dcj>

i.e., dv = r2 sin edr.d9.dcj> is the volume element.

7.9.8 Cylindrical polar coordinates (cylindrical coordinates)


7.9.8(1) In this case, we take,

Then the curvilinear coordinate system becomes a 'cylindrical coordinate system'.

The equations of trans,formation are

x=pcose, y=psine, z=z.

where

7.9.8(2) Scale factors:


r = xi + yj +zk
= (pcose)i + (psin e)j + zk
Vector Differentiation 557

-8r = -8r = (cos 8) I.+ (.


sm 8)}
av l 8p

8r I= -J cos 8 2 + sm
AI = 8p
82 = I
I
-8r= -8r= (-psm8
. ) i+ (pcos8 ) j
8v2 as

A2 =1:1=~(-psin8)2 + (pcos8)2 =p

8r = 8r = k; A1 = I8r I= 1
8v) 8z 8z

:. scale factors are I, p, 1.


7.9.8(3) Coordinate surfaces and coordinate curves:
(i) The coordinate surfaces (level surfaces) in a cylindrical coordinate system are.

p = cl' which are cylinders coaxal with z-axis (or z-axis itself if c 1 = 0)
8 = c2' which are planes through z-axis

and z = c3' which are planes ~r to z-axis


(ii) Coordinate curves are:
(a) Intersection of 8 = c2 and z = c3 is the p-curve which is a straight line.

(b) Intersection of p = c 1 and z = c3 is the 8-curve which is a circle (or Point)


(c) Intersection of p = c 1 and 8 = c2 is the z-curve which is a straight line.
7.9.8(4) Base vectors: (ep,eo,e z )

ep = _I 8r = (cos 8 )i + (sin 8)j


AI 8p

eo =_1 8r =~[(-psine)i+(pcose)j]
1<.2 as p

= (-sin8)i+(cose))
558 Engineering Mathematics - I

1 8r
e =--=k
~ A3 8z '

which shows that cylindrical coordinate system is a righthanded orthogonal coordinate


system.
7.9.8(5) Arc length,Area element and volume element in cylindrical coordinate system:

(i)

= 12 (dpy + p2(de)2 + 12 (dzY

= (dpy + p2(d8)2 + (dZ)2

gives arc length ods'.

ds p' ds s' ds z represent the area elements on the surfaces p = c l ' 8 = c2 ' and z = c 3
respectively.
(iii) Volume element 'dv':

= l.p.dp.dedz = pdp.de.dz.

7.9.9: EXPRESSIONS FOR GRADIENT, DIVERGENCE, CURL, LAPLACIAN


AND JACOBIAN IN SPHERICAL AND CYLINDRICAL COORDINATES:
7.9.9(1) Gradient of a scalar function 'f

18/ 18/
Grad f= V j=---e l + - - e2 +---e3
18/
(from 7.9.6(1))
AI av l A2 av 2 A3 av
3
Vector Differentiation 559

a) spherical coordinates:
Al = 1, A2 = r, A3 = rsin 0

el =C r =(sin8cos~)i+(sin8sin~)j+(cos8)k
e2 = Co = (cos8cos~)i + (cos8sin ~)j ~(sin 8)k

e, =e<l> =(~sin~)i+(cos~)j

v f= of e +! Wc +_I_ofe
or r as r 0 rsine as <I>

b) cylindrical coordinates:
Al = 1, A2 = p, A) = 1

el =ep =(cose)i+(sine)j

e2 =eo =(~sine)i+(cose)j

7.9.9(2) Divergence of a vector function


If A = AIel + A2e2 + A3 e3, the expression for Div A is given by

Div A= 1
AIA2 A3 8v1
[~(AIA2A3)+~(A2A3AI)+~(A3AIA2)l(from
8v 8v
7.9.6(3
2 3

(a) In spherical coordinates

:. Div A = 2 1.
r sll1e or
[~~2 sin eA )+ ~(rsineA2)+
I
as
~(rA3)l
o~ J
(b) In cylindrical coordinates
Al =1, A2 =p,A 3 =1, VI =p,v 2 =e, V3 =Z

:. Div A = - 1[0-(pAI)+-(A2)+-(pAJ
p op
0 0 J
as oz
560 Engineering Mathematics - I

7.9.9(3) Laplacian (V 2 ): Let f be a scalar function. Then the expression for V2 j m


curvilinear coordinates is given by

[from 7.9.6(5)]
a) In spherical coordinates:
Ai =1, 1.,2 =r, 1.,3 =rsiIl9, Vi =r, v2 =9, v)<l>

1 [ -a ( r2sm9-
a ) +-
a (.sm9- j
a ( -1- aj
V2j = . OJ ) +- - )]
r2 sin 9 ar ar 09 09 a<l> sin 9 O<J>

a2j 2 aj 1 a2j cot 9 aj 1 a2f


= -- +- - +- - -
2 + - - - + ---::---:----
ar2 r ar r2 ae 2
r2 ae r2 sin 9 a<j>2
(after ditferntiation and simplification)
b) In cylindrical coordinates:
Al = I, 1.,2 = p, 1.,3 = 1, VI = p, V2 = 9, V3 =Z

..
V2j = ![~(p
pap
Of)+~(!
op
Of)+~(p
OOpOO OZ
Of)]
GZ

a2f 1 af 1 a2j a2f


= - - + - - + - - -2+ - -
2
ap2 p ap p2 09 az
(after differentiation and simplification)

7.9.9(4) Curl of a vector function A.


If A = Aiel + A2 e2 + A3 e3 '

(from 7.9.6(4

a) In spherical coordinates:
Ai =1, 1.,2 =r, A) =r~';n9, VI =r, v 2 =9, v) =<j>
Vector Differentiation 561

er rea {rsin8}eq,
1
ajar ajae aj~
.. curl A = r2 sin 8
AI rA2 (r sin 8 )A3

b) In cylindrical coordinates:
AI =1, A2 = p, A3 = 1, VI =P, v2 = 8, V3 = Z

7.9.9(5) Jacobian:
(i) Orthogonal curvilinear coordinates:
Let x = x{V"V 2 ,V3); Y = Y{V"V 2 ,V3); Z = z{V"V 2 ,V3) represent an orthogonal
curvilinear coordinate system

axjavl ayjav l azjavl


Jacobian = J( x,y,z ) = axjav2 ayjav2 azjav2
v" v2 ' V3 axjav3 ayjav3 azjav3

since r = xi +yj + zk

[Scalar Triple Product]


562 Engineering Mathematics - I

(ii) Spherical coordinates:


x=rsinOcos<\>
y=rsinOsin<\>
z = rcosO, are the equations of transformation of spherical coordinate system.

. =.,(x,y,z)
Jacobian --
r,O,<\>

ax/ar ay/ar az/iJr


ax/a8 ay/ao az/80
ax/a<\> ay/a<\> az/a<\>

sin Ocos<\> sinOsin<\> cosO


rcosOcos<\> rcosOsin <\> - rsin 0
-rsinOsin<\> rsinOcos<\> 0

= r2 sinO (or) [J = AIA2A3 = I.r.r sinO = r2 sinO]


(iii) Cylindrical coordinates:
x = pcosO,y = psin 0, z = z, are the equations of transformation.

ax/ap ay/ap az/ap


Jacobian = J[x,y,z) = ax/88 ay/88 az/88
p,O,z. ax/az ay/az az/az

eosO sinO 0
-psinO peosO 0 = p
o 0

[(or) J = AIA2A3 = Lp.l = p]

The summary of important results of this chapter is given in a table in the next
page for the convenience of the student.
Cartesian Spherical Cylindrical
Coordinates Coordinates Coordinates
(x, y, z) (r, 0, ) (p ,0, z)

1) Equations of x=x x =r sinO coS x =pcosO


transformation y=y y =r sinO sin y =psinO
of coordinates z=z z = r cosO z=z

2) Scale factors AI = 1, A2= I,A. 3= 1

3) Base vectors i, j, k e,= (sinO coS)i + (sinO sin)j + (cosO)k ep = (cosO)i + (sinO)j
ee= (cosO coS)i + (cosO sin)j-(sinO)k ee= HinO)i + (cosO)j
e~= (-sin)i + (coS)j ez=k

8(x, y, z) 8(x, y, z) _ 2 Ll
4) Jacobian (J) a(r. 0, ) - r smu
a(x,y, z)

5) (Arc Length)2 (ds)2 =(dx)2 + (dy)2 + (dz)2 (dS)2 =(dr)2 + r2(d~)2 + r1 sin 2O(d)2
6) Area elements ds, =dy dz ds , = r2 sinO dO d dsp =pdO d- ~

on the coordinate ds 2 =dz dx ds e= r si nO d dr ds Q=d'"-, dp


surfaces ds 3 =dx dy dSIj>= r dr dO =
ds z pdpdO

7) Volume dv =dx dy dz dv = r1 sinO dr dO d dv = pdpdOdz


element (dv,\
Cartesian Spherical Cylindrical
Coordinates Coordinates Coordinates
(x, y, z) (r, 0, ) (p ,0, z)

8) Grauf

a~ a~
~ O[ ~
()A I
9) Div A -+-+- Div A = (r2sinO AI)
Ox ay clZ ....2
rsm (Jr

+ go (r sinO A2) + ~~ (r A3)]

!!i.!J:i.iEJ. ~ 2!Ji. I 2:J. cotO!Ji. f2:[ + I .!Ji. + I Sf +iEJ. m


or + iJy2 + iJ z tJr2 -r ar +r2- ()02 + - - ao
10) Laplacian ::J
2 r2 Up2 Ii up 7Y a(J2 at co
::J
(v2j) +
2
!Ei
U2
(l)
(l)
r2sin O ~.
::J
co
s::
j k er reo (r sinO)e
I
eI' peo e:
-
Q)
=r
(l)
3Q)
11) CuriA
a/{)x OliJy alaz r2sinO
alar
AI
alfJo
rA2
()/af/J
(r sin{})A3
-p cl/(Jp
AI
(J/r)O
pA2
i.JI (Jz
A3
-
oen
AI A2 A3
Vector Differentiation 565

Solved examples
Ex. 7.9.10 Obtain the equations of transformation from cylindrical to cartesian coordinates.
Sol: Equations of transformation from cartesian to cylindrical coordinates are
x=pcos8 ..... (1), y=psin8 ..... (2) z=z ..... (3)
from (I) and (2), we get, p2 = x 2 + y2

~p=~X2 + y2 ,p> 0

y
from (I) and (2), we also get - = tan 8
x

~8=tan-l(~)
:. The required equations are

p=~X2+y2, 8=tan-'(~} z=z

Note: If a point lies on z-axis, x = 0, y = 0 => 8 is indeterminate. These points on z-axis


are known as singular points of the transformation.
Ex. 7.9.11 Represent the vector A = 2xi - yj +z2k in cylindrical polar coordinates.
Sol: The base vectors in cylindrical coordinates are
ep =(cos8)i+(sin8)j ...... (\)
eo = (-sin8)i+(cos8)j ..... (2)
ez = k ..... (3)
Solving (\) and (2),. we get
i=(cos8)ep -(sin8h

j = (sin8)ep + (cosS)eo
Then A = 2[(cosS)ep -(sin8)eo~-[(sinS)ep +(cos8~o}y+z2e~.
A = (2xcosS - ysinS)ep -[2xsin8+ ycosSh + z2e~

= (2pcos 2S - psin 28 ~p - [2psin8cos8 + psin8cos8to + Z2 ez


(substitutingx= pcos8, y= psin8)

= p(2cos2S-sin28~p -(3psinScos8)eo +z 2e:


566 Engineering Mathematics - I

Aliter: Since cylindrical coordinates form an orthogonal coordinate system, we can write A
as A = a,ep + a2ee + a3ez ... (I) whereat'~, a 3 are to be determined.
From (1) we find,
A.ep =a" A.eo =a2 ; A.ez =a3 ..... (2)
For cylindrical coordinates, we have,
x=pcos8, y=psin8, Z=Z
given A becomes,
A= (2pcos8)i-(psin8)j+z 2 k ..... (3)

ep = (cos8)i+(sin8)j }
Also, eo = (-sin8)i+ (cos8)j ..... (4)
ez =k
using (3) and (4) in (2), we get
a, = 2pcos 2 8 - psin 2 8;
a 2 =-2psin8cos8-psin8cos8=-3psin8cos8; a 3 =Z2

A = a,ep + a2 ee + a3 ez
= (2 cos 28 - sin 2 8) pep - (3p sin8 cos8) eo + z2 ez

Ex. 7.9.12 Represent the vector A = xyi - zj + xzk in the spherical coordinate system.
Sol: Since the base vectors er, eo' ecj> are mutually orthogonal, we can write
A = a,er + a2 eO + a3eq, ..... (J)
where aI' a 2, a 3 are to be found.
From (I) we get, A.er =a,. A.ee =a2 A.eq, =a3 (2)
For spherical coordinates, we have,
x = r sin8 cos<j>, y = r sin8 sin<j>, z = r cos8 ..... (3)
2 2 2
A = {r sin 8sin <j> cos <ji - {rcos8)j + {r sin 8cos8cos<jk ..... (4)
Also we have,
er = {sin 8cos<ji + {sin 8sin <jj + {cos 8 )k
ee ={cos8cos<ji + (cos 8cos 4> )j - {sin 8)k
eq, = (-sin4i+ {cosiJj ..... (5)
Using (4) and (5) in (2), we get,
a, = 1'2 sin 3 8sin 4>cos 2 4> - rsin 8cos8sin<j> + 1'2 sin 8cos 2 8cos4> .
a2 = r2 sin 2 8cos8sin <j>cos 2 <j> - rcos 2 8sin cp - r2 sin 2 8cos8coscp
a3 = _1'2 sin 2 8sin 2 cpcos<j> - r cos 8 cos cp
Vector Differentiation 567

Hence
A = aler + a 2ee + 01 eq,

= {r 2
sin} 8sin $cos 1 $ - rsin 8cos8sin $ + r2 sin Ocos 2 8cosc/>k
+ {r 2 sin 2 8cos8sin $cos 2 $ - rcos 2 8sin $ - r2 sin 2 8cos8cos$ ~o
1 2 2
+ (-r sin 8sin $cos$-rcos8cos$h
Ex. 7.9.13 Prove the following:

(i) e = Seo+ {sin 8 )~eq,


r

(ii) eo = -Ser + {cos8)~eq,


(iii) eq, = (- sin 8 )~er - (cos8 )~eo
where' ' denotes differentiation w.r.t. time 't'.
Sol: (i) We know that
er = (sin 8cos$)i + (sin 8sin $)j + (cosS)k

" cos8cos$8"} i + {
er ={sin 8(- sin$)$+ " sinOcosc/>.$"} j - (sin8)8k
cos8.sin$.8+ "

= e{(cos 8 cos $)i + {cos8sin$)j - (sin 8 )k} + <i>{{- sin8sinc/i + {sin8cos$)j }


" "
= 8e e + sin 8.$eq,
(ii) ee ={cos8cos$}i + (cos8sin C/>}j - {sin 8}k
ee =( - sin8.8cos$ -cos8sin$~ } + ( - sin8.8s;". + coseco.~++ )j -(cOS8.e)k
= - e{(sin 8cos$)i + (sin 8sin$}j + (cos 8 )k} + cos8.+{(- sinc/i + {cos$)j}
" "
= - 8e r + cos8$eq,
(iii) eo!> =(-sin$)i+{cos$}j

eo!> =-cos$~i-sin$~j ..... (i)

(- sin 8 }~er - cos8~ee

= ~[- sin 8{sin 8cos$i + sin 8sin $ j + cos8k}


- cos 8 {cos 8cos c/> i + cos8sin c/> j - sin 8 k}]
568 Engineering Mathematics - I

o 0

= - coscj).cj)i - sin cj).cj) j


= e~ from (i)

Ex. 7.9.14 If ep , eo are base vectors in cylindrical coordinates, show that

(i)(ep)=ee o and (ii)(eo)=-eep,


where'.' denotes differentiation w.r.t. 't'
Sol: ep = (cosS)i+ (sinS)j

= e{(-sinS)i + (cosS)j}

eo = -sin Si + cosS j, eo = - S(cosS)i -e(sin S)j = -Sep


Ex. 7.9.15 Express the velocity V and acceleration a ofa particle in cylindrical coordinates.
Sol: In cartesian coordiantes, the position .vector r = xi + yj + zk .
. dr dx. dy. dz
VelOCIty V = -=-I+-j+-k
dt dt dl dt
d 2r d 2 x. d 2y d 2z
Acceleration a= -=-1+-,
2 2 j+-k
2
dt dt dr dt
In cylindrical coordinates, x = pcosS, y = psin S , z = z and
ep=(cosS)i+(sinS)j, eo =(-sinS)i+(cosS)j, ez =k
~'. i = cosSep - sin Seo ; j :::: (sin S)ep + (cos e)eo (Solving above equations for i andj)
:. r = (pcosS XcosSe p - sin See)+ psin e(sin Sep + cosSee)+ zez = pep + ze z

dr dp de dz
. V = -=-e +P-P +-e [
.: dez =~(k)=O]
.. dt dt p dt dt Z dt dt

o
= pep (0) 0
+p Sea +zez (from Ex. 7.9.14)
Vector Differentiation 569 .

dV d. . . .... .. .. ....
a = dt = dt [pep + pOeo + zeJ = pep + pep + pOeo + pOeo + pOeo + ze z

(since ez = 0)
= pBen + pep + pBeo + pBeo + pB(-Bep ) + ze z ' (using results of (ex 7.9.14

[p- pe 2 ]e p +[pO + 2pB]eo + zez

Ex. 7.9.16 If t = xyz, find 'gradf' in (a) Cylindrical coordinates (b) Spherical
coordinates

Solution:
In cylindrical coordinates (p,O,z), we have, x = pcosO,y = psinO,z = Z

and ..... ( I)

2
f = xyz = p2 Z sin 0 cos 0 = p Z sin 20
2

af = pz sin 20 af = p2 Z cos 20. at = p2 sin 20


ap 'ao 'az 2
2
:. (I) gives, gradf = Vf = (pz sin 20)e p + (pz cos 20)eo + (~ sin 20)ez

(b) In spherical coordinates (r,O,tjJ), we have,

x = rsinOcostjJ,y = rsinOsintjJ,z = rcosO, and

Vf= af e +! af e +_1_ af e ..... (2)


ar r r ao 0 r sin 0 atjJ ;
Here f = xyz = 3r 2sin 2 OcosOcostjJsintjJ

:. af ='csin2tjJ{-sin 3 0+cos02sinOcosO}
ao 2
579 Engineering Mathematics - I

aj =rJsin20cosO.cos2~
o~
:. from (2)
)

~l=(3r2sin20cosOsin+cos~~,. + '; sin2~{-sin30+2sinOcos20}ee

+ (r2 sinOcosOcos2~~~.
E.7.9.17 Itf= pzcosO (in cylindrical coordinates) find Vf.
Sol: f= pz cosO.

-aj = zcoso of = -pzsin 0 aj = pcosO


op , 00 ' oz

aj I aj of
Hence Vf = op ep +p ae eo + oz ez
= (zcosO)ep -(zsinO)eo + (pcosO}ez
Ex.7.9.18 Iff= ,.'2 sin2Gsin+ in spherical coordinates, find Vf.

Sol: aj = 2rsin20sin4>
or
-8f = 2r2 cos 20sin 4>
ae
aj =r2 sin20cos~
o~

8j Jo/ 1 Of
.. ~l = or e,. +-; ae ce + rsinO o~ e.p
= (2rsin20sinq,)e,. +(2rcos20sin~)eij + (2rcosOcos4e.p

Ex. 7.9.19 Show that the vectorfieldA = z{(sin O)ep + (cosoh }-(pcosO}ez, in cylindrical
polar coord inates is solenoidal.
Sol: IfA= Aiel' +A2 co +AJe z then, AI =zsinO; A2 =zcosO; A3 =-pcosO
Vector Differentiation 571

= ~[~(pz sinO)+~(zcOsO)+~(- p2 coso)l


p ap as aZ J
= ~[zsinO-zsinO+O] = 0
p
:. A is solenoidal

1 I I )
Ex.7.9.20 IfA= ( r-cosOer +-eo +-.-Oe+ ,in spherical coordinates, find Div A.
r rS1l1

I I
Sol: Here A =r 2 cosO, A2 =-, A3 = - . -
I r rS1l10

= 1.I
,.- S1l1 0
3
[4r
-sin20+cosO+O
2
1

cotO
= 4rcosO+-2-
r

Ex.7.9.21 Ifj= p2Z2cos20, show that V2j=2p2cos20


Sol: In cylindrical coordinates,
2 af 1 af
V j = - + - - + - -2- + -
1 a2 f a2 f
..... (I)
ap2 2
p ap p2 ao az
j = p2 Z 2 cos20

2
Qf = 2 pz 2 cos2 0 . - j = 2 z 2 cos 20 .
a - at ')
-
ap , ap2 , as = -2p- Z 2 si1120'
_'J
572 Engineering Mathematics - I

:. From (1) we get,


yo2 f = 2Z2 cos 26 + 2Z2 cos 29 - 4Z2 cos 29 + 2p2 cos 29 = 2p2 cos 29 .
Ex.7.9.22 Using spherical coordinates, show that yo2r" = n{n + l)r,,-2 when n is a
constant and r = 0 if n<2
Sol: In spherical coordinates,
2 2
yo2f=a2f+~Of +_1 a f + cot 9 Of + 1 d f
..... (I)
ar2 r ar r2 00 2 r2 a9 r2 sin 20 d~2
Here, f= rn
2
Of = nrfl-I. a f = n{n- 1)r"- 2
.. 8r ' Br2

.. (I) ~ yo2f =V2r" = n{n-l)r"- 2 +~.nr"-I


r
= n{n -1 )r"-2 + 211r"- = n{n + I )r"- 2 .
2

Ex.7.t.23 Iff= r 2 sin29+cos 2 p, find yo2f.

~ = 2sin 29
2

Sol: Of = 2rsin29. a
or ' ar~

Of
-=-sin2+
a2f
-=-2cos2~
2
0+ ' 0+
a 2 f 20f 1 a 2f cot90f a 2f
yo2f - --+--+---+----+-::---::--
2
.. - ar2 r ar r2 00 ,.2 a9 r2 sin 9 a~2
2

+ [CO/9x2r2cos29+
r2 r
2 ~
sm~9
? X{-2COS2P)]

= .2sin 29 +~(2rsin29)+ ~(- 41'2 sin 29)+ co~9 x -2cos2p


r r r

= 2sin 29+ 4sin 29 -4sin 29 + 2cot9.cos20 -4cosec 29.cos2p


r

= 2sin 29 + 2 cot 9.cos 29 -4cosec 2 9.cos2P


r
Vector Differentiation 573

Ex.7.9.24 If A is a vector field, find curl A where,

(\) A= {r:~sino~,+(~cosO}e+ r l2eq,


(2) A = {zcosO)ep -{ZSiIlO)eo

Sol: If A = Ale, + A2eO + A3eq., We have, in spherical coordinates,

reo e, {rsinO)eq,
1
CurlA= ~.
r-smO
alar a/ae a/a~
Al rA2 (rsinO}A3

Here, AI =r sinO, A2
2
=~cosO, A =-
r 3 r2

e, {rsin O)eq,
\
:. Curl A = 2 ajar a/ae a/a$
r sin 0 r2 sin 0 cosO (I/r}sine

== ~. \ [- a {( -smO a (cosO )}e,+ {-a (r 2smO


1. ) - - . a (sinO)}
)-- - - rea
r smO ae r a$ a$ ar r

= 7
r- smO r
1. [~coso.e, +-;-sinO.re
r e _r2 coso.rsinoeq,]

= 2.
1 [\-cosO.e, +--ee
sin 0 -r 3 smO.cosOeq,
]
r smO r r
cotO I
= -3-e, +]ee -rcosOeq,
r r

ep pea ez
\
we have, Curl A = - a/ Bp alae a/az
p AI A2 A3
574 Engineering Mathematics - I

Here, A I = Z cosO, A2 = -z sine, A3 = o.

e peo ez
I P
:. Curl A = - a/ap a/ae a/az
p zcose -pzsinO 0

a a .}e + {-(zcose)--(o)
= -I [{ -(o)--(-pzsme) a a}
poO az Paz ap pee

= ~[psineep +pcosOeo +(-zsinO+zsine)ezl


p

= (sinO)ep + (cose)eo

Ex.7.9.25 Show that the vectorfieldA= (pzsin2e)ep + (pz cos 2e)ee +(~p2sin2e)e= is
irrotational.

Sol: Here, AI =pzsin2e, A) =pzcos2e, A)


-
=~p2sin2e
2

ep peo ez
a/ap alae a/az
p pzsin2e p2zcos2e ~p2sin2e
2
Vector Differentiation 575

:. A is irrotational.

Ex.7.9.26 Show that div (curl A) = 0, where A is a vector function, in orthogonal curvilinear
coord inates.
Sol: Let A = Aiel + A2e 2 + Aj e3
Then

Ale] A 3 e3
%v2 Ojov)
A2A2 A)A,

. . '"
Dlv j = ~ A A A
I [~(
Ov VA A ) I
I 2 3
I 2 3 I

=0
576 Engineering Mathematics -

Exercise - 7(i)
l. Show that the spherical coordinate system is an orthogonal system.
2. Show that the cylindrical coordinate system is an orthogonal system.

[Hint for I and 2: Show that e,.e 2 = e2 .e3 = e3 .e, = 0 etc 1


3. Express the vector, F = 2xi - 3y.j +z,Yk, in cylindrical polar coordinates.
2 2
[Ans: F= (2pcos S-3p 2 sin S}p -~sjn2S+3p2sin2Sh +(pzcosskJ
4. Represent the vector, A = xyi + yzj +zxk, ill spherical polar coordinates.

[Ans: A = r2 sinS{sin 2 Scos 2 ~sin~+sinScosSsin2 ~+COS2 Scos~k

+ (sin Scos8cos
2
~sin~ + cos 2 8sin 2 ~ - sin ScosScos~~o

+ (cosSsin~cos~-sinSsin2 ~cos~}~ 1

5. Express the vectors a) A = yi + xj and b) F = zi - 2xj + yk, in cyl indrical coordinates.


[Ans: a) A = peo b) F = (zcosS - p sin2S)e p - (zsinS + 2pcos2S)eo + (psinS)ezJ
6. Express the vectors a) A = - yi + xj, b) F = xi + 2y.i + yzk, in spherical polar
coord inates.
[Ans: a) A = r sinS eq, b) rsinS [{sinS(I + sin2~) + rcos2~ sin~}er
+ {cosS(l + sin2~)- rsinScosS sin~}e() + sin~cos~ eq,l
7. Find the gradients of the fol/owing function
sinS
i)f= pz sinS ii)f= -2-
r

[Ans: i) Vf = (zsin S)ep + (z cos S)eo + (psinO)ez

ii) Vf=~[(-2sinS)er + (cosO)eoJ


r

iii) Vf =(2rsin
2
~}r - (rsin 2S)eo -(~cosecscos2 ~sin~ )e~1
8. Find the divergence ofth fol/owing vector functions.
i) A = (cosS + sinS) ep + (cosS - sinS)e o + ez

ii) F = ~2 sinS}r + (~COSO )eo + rl2 e~ , iii) V = (pcosO)er + (psinO)eq,


Vector Differentiation 577

.. 4 . 0 1 cos 20
IAns: (i) 0 (II) rsm +2-.-- (iii) 3cosO )
r Sill 0
9. Show that the following vector fields are solenoidal.

(i) A= (zcosO)c" -( z - sin O)co(ii) F = (~cot O)c r -~c() + (r cosO)c


r r
10. Find the curl of the following vector fields:

(i) A= (zsin O)C" + (zcosO)ctJ -(pcosO)c z

(ii) F =(rsinO)c r +(~COSB )c +(~ )e


r

... 1 0
(III)V = -tan
,.
-e
2

IAns:(i) (sinO-cosO)c p +(sin+cos)Cf) (ii) (,.\ cot 0 jer -(cosB)c (iii) 1~2 c)
11. If ' f' is a scalar function in orthogonal curvil inear coordinates v" v2 , V3 prove
that 'Vf' is irrotational.

7.10 GREEN'S THEOREM IN THE PLANE


7.10.1 Green's Theorem:
Let
(i) R be a closed region of the xy plane bounded by a simple closed curve C
(ii) P(x,y,) and Q(x,y) be continuolls functions of x and y having continolls

derivatives in R. Then r Pdx + Qdy =


.l
JI
aQ
c - ap )d'Cdy
11 ax ay
where C is in the positive direction.
Proof: Y
E

a ___ .lI __ ~ _ _7
I A
I
I
I
o~----~c------------~--x
578 Engineering Mathematics - I

Let the equations of the curves DAB and DEB (see the figure) be respectively
y = fl (x) and y = f2 (x). Let R be the region bounded by the curve C. Then,

oP " [ 12(X) oP }
f f=-dxdy = f f -dy ix
II CY x=c y=fI(x) CY

i.e., fp(x,y)ix = - f f~xdy ..... (1)


c II CY
lilly if the equations of the curves ADE be and ABE be taken respectively as
x = gl(Y) and x = g2(Y)' we have,

h a h

= fiQ(g2,y)-Q(g\,y)}iy= fQ(g\,y)iy+ fQ(g2,y)iy


a h a

Thus fQ(x,y)iy = I~Qdxdy


c
f II ~
..... (2)

Adding (1) and (2),

J pdx +Qdy = f' oQ _ oP xdy


'! }lox oy rt
Vector Differentiation 579

7.10.2 Vector notation of Green's theorem


Green's Theorem in the plane can be put in vector notation in the following way.
Let F = P(x, y)i + Q(x, y)j
and r = xi + yj, so that
dr = (dx)i + (dy)j
F. dr = P dx + Q dy

j k
Again,VxF=%x o/ay %z
P Q
OQ OP)
= ( ox oy k, so that

(v x F).k = oQ _ oP
ox oy
Taking dR = dxdy, Green's theorem in the plane can be stated in the vector form as,

fF .dr = JJ(V x F)JaIR


II

..... (A)
7.10.3 Physical interpretation of Green's theorem
I. Let F denote the force field acting on a particle.

Then fF .dr represents the work done in movi.g the particle around a closed
curve C.
:. From (A) it follows that the work done is determined by curl F = V x F.
2. In particular, if V x F = 0 i.e., ifF is conservative (or F = V t)

Then fF.dr = O. i.e., the work done is independent of the path.

3. Conversely, if the integral is independent of the path, i.e., if

fF .dr =0, then vx F=O


In the plane, V x F = 0 is equivalent to saying that : = ~~ where F = Pi +Qj.
580 Engineering Mathematics - I

7.10.4 Application of Green's theorem to the evaluation of area of a simple


closed curve.

The area bounded by a simple closed curve C = lfxu:y - ydx


c
Proof By Green's theorem, we have,

.fPdx + Qdy fI!~{auax - apay rtXl~Y


I"
==

if we put P = - y, and Q = x,
.I.e., -ap =-1, aQ_ == I, au _ap =2,
ay ax ax ay
:. We get,

fXdY - ydx = ff2dxdy = 2A , where A is the required area.


I" II

i.e., A = ~fxdy - ydx .


2 ('

Solved Examples

~ ~

Ex. 7.10.5 Find the area of the ellipse ~+L = I.


a 2 h2
Sol: Parametric equations of the ellipse are,
x = a cosO, y = b sinO
dx = - a sine de, dy = b cose dO.
:. By Green's theorem,

Area of the ellipse = lfXdY - ydx


L

I 2n
=- f(acosehcos 0 + bsinea sine}le
2 0

I 2n
== - fahdO = rrah
2 0
Vector Differentiation 581

Ex. 7.10.6 Evaluate f(y - sinx}dx + (cosx}1y, a) directly and b) lIsing Green's theorem,

where c is the boundary of the triangle in xy-plane whose vertices are (0,0),

(%,0) and (%,1) traversed in the positive direction.

Sol: Let 1= f{y-sinx}d\" + {cosx)dy


1!
X =-
a)alongOA: y=o, dy=O 2

1['2 n<2
f(y - sin x}1x + {cos x }dy = f- sin xdx = cosxi =-1
OA 0 0

alongAB: d\"=O

.. f(y -sinx}lx + (cosx~/y = fOdy


Ali
=
2
along BO: Equation of 013 is y = 2x , dy = -dx ,
1t
7t

f(y-sinx}dx+cosxdy=
/JO
1[(2:
11 ~
-situ )+;cosx}x

x
2
= -+cosx+-stnxi0 2. =(0+ 1 +0)- (1t 1t
-+0+- =1----. 2) 2
1t 1t 11/2 4 1t 4 1t

:. I = Sum of the integrals along OA, AB and 130

7t 2 7t 2
=-1 +0+1- - - - =- - - -
4 7t 4 7t
582 Engineering Mathematics - I

(b) By Green's theorem,

J Pdx + Qdy = Ii( 8Q - ~~ldX dy, Here P = y-sinx, Q = cosx


'jc I
I
dx dy

8Q = _ sin x ap = 1
ax 'ay

[ = !,f( ~sinx~ l)dr try <i[[ (~sinx~ l)dy Jd< = ~i ~ysinx~ y'l d<
2x f
= fo --smx--
7r 2x.
2(

1C
x = -- f xsmx d'(-- f xd'(
2
1C
2 7r

1C X~O
2.

1C ()
7r 2

Ex.6.10.7 Evaluate f(3x+4y)dx+(2x-3y)dy where C is the circle in xy plane with

centre at origin and radius 2 units.

Sol: By Green's theorem,

fPdx + Qdy = 8Q 8Pl dxdy,


Ii(---
II dx dy

Here P=3x+4y, Q=2x-3y ; 8~=4 aQ =2


Oy 'ax
The given integral = ff(2-4)dxdy

= -2 ffdxdy = -2A,
II

where A is the area of the circle C.

= -2 x 1C x 22 = -81C
Vector Differentiation 583

7.10.8 Verity Green's theorem in the plane for the integral 1(3x 2 - 8y2 )dx + 4( 4y - 6xy)dy

where C is the boundary of the region given by

(I) (2) x = 0, y = 0, x + Y= I
(I) The given region is shown in the figure below.

._~ ______ X

The points of intersection of y = x 2 and y = fx are (0, 0) and (I, I).


We have to integrate [ along
(I) y = x 2 from 0 to A.
(2) along y = fx from A to 0, and add the two values.
Alongy=x2, dy=2xdx
x~1 I .
:.1= f (3x 2 - 8X4)dx + (4x 2 - 6x')2xdx = f(3x 2 - 20x 4 + 8x 3 )dx = x 3 - 4x 5 + 2X4 I =-1
AU X~O 0

Alongy = fx ; x = y, dt = 2ydy
yeO II 0 5
:.1= J(3 y 4_8 y 2jdy+(4y-6x 3 )dy= J(6yS-22y3+4Y)(2Y~/y=y6_~y4+2y21=-
AO )' I
2 I 2

5 3
:. 1=-1+-=-
2 2
By Green's Theorem,

1Pdt + Qdy =
c
Ii OQ. OP)dXd
)l ox ~v Y
Here, P = 3x2 - 8y ; Q = 4y - 6xy

oQ _ oP =-6v-(-16v)=IOy
ox oy . .
584 Engineering Mathematics - I

:.I=f flOydxdy= f
I[E 1 I
flOydydx= fSy
2 J~l dx =
x2
I 4 3
fCSx-Sx )dx=-
2
R x=O y=x 2 x=O x=o

Green's theorem is verified.


2. The given region is shown in the figure below.

(0,1)8

x=O
o~--~~----------x
y=0 A(1,0)

AlongOA, y = 0 ::) dy = 0
I I
2 3
:. Given integral = f 3x dx = x 1= I
x=o 0

AlongAB, y = I - x ::) dy = - dx
:. Given integral
o 0
= f~X2 - 8(1- x 2 )}dx + {4(1- x) - 6x(1- x)C-dx)} = fC-Ilx 2 + 26x -12) dx '
x=1 x=1

II 3 2 0 II 8
=--x +13x -12xl =--13+12=-
3 I 3 3
AlongBO, x = 0 ::) dx = 0
y=O 0
Given integral = f 4ydy = 2y21 =-2
y=1 I

8 S
:. The given integral = 1 + "3 - 2 ="3
By green's theorem,

1Pdx + Qdy = f JlnaQ.


C
ap)dXdY
ax ay
R
Vector Differentiation 585

Here, P = 3x2 - 8y2, Q = 4y - 6xy


aQ ap
ax ay = -6y - (-16y) = lOy

= 5(1-x)3i =-~(O-I)=~
3x-1 0 3 3
Hence the theorem is verified.

(2,1)
4
Ex. 7.10.9 Evaluate J(10x - 2x/ dx - 3xl.v dy along the path x4 - 6xl - 4.0 = 0
(0,0)

Sol: P = IOx4 - 2xl, Q = 3x2.0


ar
-=-6xy-
~
=-
8Q
ay ax
The integral is independent of the path. Hence we can use any path. For example,
if we use the path from points (0, 0) to (2, 0) and then from (2, 0) to (2, 1); we can
evaluate the integral.
(i) From (0, 0) to (2, 0) ; y= 0, dy = 0

. 2 512
:. The tntegral = J
o
4
lOx dx - 2x 0 = 64

(ii) From (2, 0) to (2, I); x= 2, dx = 0


2 2 2 2
4 5
:. The integral = JIOx dx = 2x J= 64
o 0
J
0
12y 2 dy = - 4/1 =-4

:. The value of the integral = 64 - 4 = 60

Aliter: Since ar = aQ, we know that {I Ox4 - 2x1)dx - 3xl.0 dy} is an exact
ay ax
differential of (2x 5 - xlI).
(2.1) (2,1) 2 3
2 5 2
The given integral = J d(2x 5 - x /)= 2x - x y3 J= 2.25 - 2.1 = 60
(0,0) (0,0)
586 Engineering Mathematics - I

X
Ex.7.10.10 VerifY Green's theorem for f(e- siny)dx + (e-X cosy) dy where C is the
c

boundary of the rectangle whose vertices are (0, 0) (n, 0) (n, ~ ) and (0, ~ )
2 2
traversed in the +ve direction.
Sol:
y

O(~O~,O~)----~'~----~A~(n~,O~)-x
By Green's theorm,

fPdx+ Qdy =
c
In)l ax 8Q - 8P)dXdY
ay
Here, P = e-x siny, Q = e-x cosy
aQ ap
- - - = - e-x cosy - e-x cosy = -2e-x cosy
8x ay
fPdx+Qdy= If -2e x cosydxdy
c R

x x/2 x x/2
=-2 f [ f e-xcosdyJdx=-2 f eX siny fdx
x=o y=O x=() 0

x
=-2 fe-xdx=2e xl: =2(e- X
-1) .....(i)
c

Again,fPdx+Qdy=f f+ f+ f+ f
c c OA AB BD DO

Along OA ; y = 0, dy = 0

.. fPdx+Qdy=O
OA

Alo~gAB: x = n, dx= 0
Vector Differentiation 587

1tI2

f Pdx+ Qdy = Je 11 cosydy = e- 1I sin YI~/2 = e 11

AB 0

AlongBD;

o t 0
p dx+Qdy= fe-xdx=~ =-1 + e
J -- 1 11
11

Along DO ;
BD

x = 0, dx =
11

DO
JP dx + Qdy = feos ydy =
11/2
sinYI~/2 = - 1

..... (ii)
c
:. From (i) and (ii) it is proved that

J
c
p dx + Qdy = H( 8Q
R 8x
- 8P) dxdy
8y
Hence the theorem is verified.
Ex. 7.10.11 Apply Green's theorem to obtain the area bounded by the curve
x2/3 + y2/ 3 = a 2/3 , a> .
Sol: The parametric equations of the curve are x = a cos3e, y = sin 3e. A rough sketch of
the curve is given below:

A'
x~'--~~~~~~-----x
(6 =rc) A(6 =0)

,
y

x = acos3e, y = asin 3e
dx = -3 acos 2e sine de, dy = 3a sin 2ecose de
By Green's theorem, the area bounded by a simple closed curve C is given by

!J(xdy- ydx)
2c
588 Engineering Mathematics - I

:. The area bounded by the given curve C


21<
= ! J{acos 30.3asin 2 0cosO+asin 30.3acos 2 0sinO}dO
2 {)=o

3 3 3
ar
3 2 21t
= - JCos Osm OdO=-
22 af}2 br
(lr n:a
sm-20dO=-lJ-cos40)dO=--
2 2x 2

2 0 8 0 16 () 8

Ex.7.10.12 Verify Green's theorem for J(2xy-x 2)dx+(x+y2)dy, where C is the


c
closed curve in xy-plane bounded by the curves y = x 2 and = x. y
Sol: By Green's theorem, y
aQ A(1,1)
JPdx+Qdy= Jfc - ap)dxdy
C R ax ay

2 aQ ap
Here, P = 2xy - x , Q = x +,1 - - -ry
Y, ax
= 1 - 2x
o-=:;;~-----x

aQ of y=~
Jjr---)dxdy=
R ax ry
J[ f (1-2x)dyJdx= JY-2xyl~:;; dx I

x=() y=x2
I

()

I 3 5 3 4 I
= jr-v'x -2x-v'x _x 2 +2X3 )dx = '!:x 2 -2.'l:x 2 -~+2.~
o 3 5 3 40

1 4 1 1 1
=-----+-=- ..... (i)
2 5 3 2 30
Again fPdx+Qdy= fCPdx+Qdy)+ J(Pdx+Qdy)
r ~ ~

Along C I' Y = x2, dy = 2xdx, "x" varies from 0 to 1


1 I
:. fpdx+Qdy= f C2x 3 -x2)dX+CX+X4)2xdx= fC2x 3 +X2 +2X5)dx
(') x=o 0
Vector Differentiation 589

AlongC 2, x=y, dx=2ydy, 'y' varies from I to 0


o 0
:. JPdx+Qdy= J(2/_y4)2ydy+(y2+ y 2)dy= J(4 y 4_2 y 5+2 y 2)dy
(', y=1 I

..... (ii)

From (i) and (ii), the theorem is verified.

Exercise -7(j)

1. Evaluate 1(x 2 + y2)dx+3xy 2dy, (a) directly (b) by Green's theorem, where cis

the circle x 2 + y = 4, traversed in the +ve direction.


(Ans: 121t)

2. Evaluate 1(x 2 + 2xy)dx + (x 2 y + 3)dy around the boundary C of the region given

by y = 8x and x = 2, (a) directly and (b) by Green's theorem.


(Ans: 128)

3. VerifY Green's theorem for the integral 1(3x 2 + 2y)dx - (x + 3cos y)dy where Cis

the boundary of the parallelogram with vertices at (0,0), (2, 0), (3, I) and (1, I)
(Ans: -6)

(n,2)
2
4. Evaluate J(6xy - /)dx+ (3x - 2xy)dy along the cycloid x = 8 -sin8,
(0,0)

y = l-cos8. VerifY the result by using Green's theorem.


(Ans : 61t 2 - 41t)
590 Engineering Mathematics - I

5. Using Green's theorem find the area bounded by one arch of the cycloid
x=a(B-sinB), y=a(l-cosB),a>O, and x-axis.

[Ans: 3J[a 2 ]

6. Evaluate f{2x 2 - y2 )dx + (x 2 + i) by Green' theorem where C is the boundary of


c

the surface in the xy plane enclosed by x axis and the semi-circle y = ..Jl- x 2
[Ans: 4/3]

7. Evaluate f(cos x sin y - xy)dx + sin x cos y,using Green's theorem where c is the
c

[Ans: 0]

8. VerifY Green's Theorem in the plane for f{x 2_xy 3 )dx+ &2 -2xy)where C is the
c

square with vertices at ( 0,0) , ( 2,0) and (0 ,2)

[Ans: 8]
(2,1)
9. Evaluate f (12 x3- 2x/ ) dx - 3x2y2 dy along the path x3- y3 + Y - 4xy = 0
(0,0)

[ Hint: Proceed as in aliter of 7.1 0.9]

4 2 31(2,1)
[Ans: 3x - x y
(0,0)
= 44 ]
Vector Differentiation 591

7.11 Gauss Divergence Theorem


7.11.1 Gauss' Divergence Theorem
Let (I) V be the volume bounded by a closed surface S
(2) A be a vector function of position with continuous derivatives.

Then, fffV .A .dv ffA .nds 1A .ds


= =

where n is positive unit normal (outside drawn normal) to S.


Proof:

t------y

R
..... ,
x

S is a closed surface
Let any line II lel to the coordinate axes cut S in at most two points.
Let z = gl(x, y) and z = gix, y) be the equations of the upper portion SI and lower
portion S2 respectively.
Let R be the projection of S on the xy-plane.
If A = Al i + A::J + A3 k
592 Engineering Mathematics - I

Then,

oA OA
fff OZ 3 dv = fff OZ 3 dzdydx
v v

Iq(x,y) oA ff z~g,
ff[ S 0/ dz] dydt= AJX,y,Z)1 dydx
II z~g2( x,y) II z~g2

= f f[A 3 (x,y,g,)- A 3 (x,y,g2)]dydx


R

For the upper portion SI' dy dx = (cosyl)dS I = k. n l dS I


since the normal n l to SI makes an acute angle YI with k.
For the lower portion S2' dydx = (-cOSY2) dS 2 = --k.n 2 dS 2,
since the normal n 2 to S2 makes an obtuse angle Y2 with k.

:. f f A3(X,y,g, )lydx =:f f A 3k .n, dS, and


R "

So that, f f[A 3 (x,y,g,) -A3 (x, y, g2) ]dydx = f f A 3k .n,dS, + f fA3k.n2dS2


II 8, S2

s
f
= fA3k.ndS ..... (I)

Similarly, by projecting S on the other coordinate planes, we get,

f f fO~, dv= f fAl nds ..... (2)


v

oA2
and f f f dv = f f A 2j.n ds ..... (3)
v cry s

(I) + (2) + (3) =>

fff(o~, + O~2 + o~3)dV= ff(A ,i+A 2j+A3k).ndS


v s

i.e. f f fV.A dv = f fA.n dS which proves the theorem.


S
Vector Differentiation 593

7.11.2 (a) Express Gauss' Theorem in words and (b) obtain its Cartesian form.
(a) Gauss' Theorem states that "The surface integral of the normal component
of vector A taken over a closed surface is equal to the integral of the
divergence of A taken over the volume enclosed by the surface.
(b) Let A=A 1i+A 2 j+A 3k,

. aA aA? aA 3
Then dlv A = V.A = ~-I + -"'- + ---
ax ay az
Let the unit normal n to S make angles a,P,r with the +ve coordinate axes so
that, cosa=n. i, cosp =n. j, cosr=n.kand cosa, cosp, cosr are the
direction cosines of n

n = ( cos a )i +( cos P)+( cos r ) [ n. i = "I = cos a etc.]

:. A.II =Alcosa+A 2 cosP+A 3 cosr


Hence the divergence theorem can be written in Cartesian form as,

JJr{ aA + aA + aA )dXdYdZ = J J(A I cosa + A2 cos p + A3 cosr)dr;


v Jl ax ay az s

= JIA,dydz + A 2dzdx + A dzdy


s
3

Solved Examples

Ex. 7.11.3: Evaluate using the divergence theorem = J J F .n d~ where


s
F = 2xyi + yz2 j + xzk and S is the surface of the parallelepiped
bounded by x = 0, y = 0, z = 0, x = 2, Y = 1 and z =3 .

Sol: By the divergence theorem; HF.n ds = JHV.Fdv


s

Here, V.F =~(2Xy)+~(yz2)+~(XZ)


ax ay ay
=(2Y+Z2 +x)

I IF.n ds = HI(2Y+Z2 +x)dv


s
594 Engineering Mathematics - I

2 I 3 2 I 3

I I Ie 2 y + Z2 + x)dxdydz = I I [ I(2y + Z2 + x)dz] dxdy


X;O y;O z;O X;O y;O =;0

2 I 3 3 2 I

= II2Yz+~+xz dxdy== Ifc 6 y+9+3x)dxdy


00 3 0 00

2 I 2 I 3 22 2
= I[ I (6y+9+3x)try]d\" = I 3i +9y+3xyl dx== I(12+3x)d\"=12x+ ;
X;O y;O 0 0 0 0

=24+6= 30
Ex. 7.11.4 Verify the divergence theorem for F = (4xy)i - (j2)j + (xz )k, over the cub(
bounded by x = 0, x = I, Y = 0, Y = 1, z = 0 and z = 1.
Sol: By the divergence theorem

II F.n dS == I I IV.F dv
\ V

Here F = (4xy)i - (j2)j + (xz)k


a a 2 a
V.F==-(4xy)--(y )+-(xz) =4y-2y+x=x+2y
ax ay az
I I I I I I

I I IV.Fdv == I I Ie x + 2y)dzdydx == I I [ Ie x + 2y)dz ]a:vdx


v x;Oy;Oz;O x;Oy;O =;0

I I

== I fi xz + 2 y z1 dydx
x;Oy;O

I I I I I 2 3
== If I(x+2y)dy]dx=
X;O y;O x;O
Ixy+y
2
1

0
dx=
x;o
IeX+I)d~==~ +xl~==2

To evaluate IIF.ndS z

The surface S contains 6 faces (see figure)


S) - Face EPFA, S2 - Face OBDC
S3 - Face PFBD, S4 - Face OCEA
Ss - Face PDCE, S6 - Face OBFA J---+--+---y

x
Vector Differentiation 595

The surface integral IIF.ndS is equal to the sum of the surface integrals on the
above 6 faces.
on S" n = i, x = I, F.n = 4y; dS = dy dz
J J J J J J

:. JJF.ndS= J J 4ydydz= J[ J4ydz]dy= J 4yzl~dy= J 4ydY=2/1~ =2


y;O :;0 F-O :~O y~O 0

on S2' n = -i, x = 0, F.n =


:. IIF.n~S=O

on S3' n = i, y = I, F.n = -I, dS = dx dz.


I I J I

:. I IF.ndS= I[ I-Idz]dx= I-z dx= I-dx = --xl~ =-1

on S4' n = -i, y = 0, F.n =


:. I IF.ndS=o
84

on S5' 11 = k, z = I, F.n = x ; dS = dx ely


J J J J I J 1 2 1

:. I I F.lldS = I I xdxdy = I [ Ix dy 1 dx = I xy dx = I xdx = ~ = ~


85 X~O y=o X;O y;O x;(} 0 x=O 2 0

on S6' n = k, z = 0, F.n =
:. I IF.ndS =
Hence :. II F.ndS = 2 - 1 + ~ =%
8

The divergence theorem is verified

Ex.S.11.5 : If r = xi + Yi + zk, evaluate :. IIr.n dS where S is any closed surface.


s
Sol: By the divergence theorem
:. I Ir.n dS = I I Iv." dv ,where V is the volume enclosing S.
s v
596 Engineering Mathematics - I

000
V.r = -(x) + -(y) + -(z) = 3
ox 0' oz
:. I Ir.n ds = I I I3 dv = 3v

Ex.7.11.6 : If '\jI' is a scalar function, prove that I I IV\jIdv = I I'P n ds

Sol: By divergence theorem

I I IV.Adv = I IA.n ds ..... (I)

Take A = \jIc, C = cli + c-j + c3k is any constant vector.

:.(l)=> JIIV.(lPc)dv= fI('Pc).nds ..... (2)


v s

But,

o )+-(c~)+-(c
0 0
= (V \jf).C [ ":V.c=-(c
ox l
oy- OZ3 )=0 ,
= c. V \jf cl' c2 ' c3' being constant]
and (\jIc).n = c.(\jJl1) [ ": a.b = b.a]
so that (2) becomes,

IJI(c.V'P) dv = JI{c.('Pn)}ds

=> c. JI IV'Pdv = c. JI'-IJn ds

=> JI IVlPdv = JI'Pn ds (.: c is arbitrary)


v

Ex.7.11.7 : Ifs is a closed surface enclosing a volume V and ifF = (Ix)i + (my)j + (llz)k,

I, m, n being constants, evaluate JIF.nds


Vector Differentiation 597

Sol: By Gauss' theorem

HF.nds= Hfv .Fdv

a a a
V.F = -(Ix) + -(my) + -(nz) = I + m + n
ax ry az
:. HF.nds = Hfu + m + 11) dv

=(/+m+n)V
Ex.7.11.8: IfE=curlA,evaluate HE.nds wheresisanyclosedsurface

Sol: By Gauss' theorem,


HE.n ds = Hf(div E) dv where V is the volume enclosed by s
.\' V

But Div E = div (curl A) = 0 [ See 7.5.2 (2) ]


:. HE.nds=O

Ex. 7 .11.9 : If s any closed surface and n is unit +ve normal to s, show that Hn ds = 0

Sol: Consider a constant vector


a = ali + ~ + a3k ( i.e. al' a2, a3 are constants)
Then, a[Hndv] = H(a.n)ds

= Hf(V.a)dv (by divergence theorem)

=0
a. aa aa
[ ':V.a=-) +_2 +_3 =0]
ax ay az
Thus a. a.[ Hn dv] = 0 is true where a is any arbitrary vector.

:. Hnds=O

Ex.7.11.10 : Evaluate H(F.n)ds, where s is the region bownded byy2 = lx, x = 2, Z = 0,


s
1
Z = 3, and F = 2xi + 3yj + "3 z3k, using Gauss' theorelll
598 Engineering Mathematics - I

Sol: By the divergence theorem,


ff(F .n) ds = fff(V.F)dv .... (1)
s v

V.F =~(2X)+~(3y)~~(Z3)
ax ay ay 3
= 5+z 2

From (I), ff(l1~.Il)ds = fff(5+z 2 )dv,


s

2 2.[i-; 3 3

= f f
x=O Y= 2 Ex
5z + ~
0
dydx
~--------r-----X

2 ) 2

= f 96J2xdx = 96J2 ~
x=o 0
= 192J2

Ex. 7.11.11 Evaluate ff( 2xi - 31 j + Z2 k ).n ds over the surface bounded by
s
2
x + y2 = 1, z = 0, Z = 2, using Gauss' theorem.
Sol: By the divergence theorem,
ff(F .n) ds = fff(V.F)dv ,
s
Here,F= 2xi-3y2j+Z2k

DivF =~(2x)-~(31)+~(z2)
ax ay ay
= 2-6y+2z
+1 M 2
ff(F.n)ds= f f f(2-6y+2z)dzdydx
x=ly=~O
Vector Differentiation 599

= 16[~~I-X2 +~sin-1 XJ1 =8n


2 2 .1

Ex.7.11.12: Evaluate using the divergence theorem H(F.Il)dS where S is the surface

of the sphere x 2 + I + z2 = h2 in the first octant and F = yi + zj + xk


Sol: By divergence theorem,

f
:. H F.1l d~' = HV'F dv
v
..... (i)

F = yi + zj + xk
V'.F= 0

:. H(V'.F) dv = 0 ..... (ii)


z

~--+B--Y

Let us evaluate the surface integrals over the faces OAB, OBC and OCA.
b Jj;2~)
f fF.n ds = - f fx dx(ry (,,' n = -k)
OA B x=O y=O
600 Engineering Mathematics - I

Similarly If F.ndv = If F.nds + If F.nds + If F.nds + If F.nds


DAB OAB ABC DCA ABC

3
= -Jrb + If F.nds .... (iii)
AIlC

From (i), (ii), and (iii), we get


3
0= -Jrb + If F.nds
ABC

If F.nds = Jrb 3
ABC

Ex. 7.11.13: VerifY divergence theorem for F=4xi-2y2j+z2k taken over the
region bounded by
2
x + / = 4, Z = 0 and z = 3.
Sol: By the divergence theorem, we have

Iff DivFdv = IfF.~ ds .... ( I )

(1) DivF
s

=~(4x)-~(2/)+~(Z2) =4-4y+2z
..
.,.
Ox By 8z

L.R.S. 0[(1) ~ ,t~~l,JL (4-4y+2z)dz ]dydt


~ ,Ll" [4z-4yz+z'JC dydx~ )11 (21-12y)dy Jdx
2

=42 f ~4_X2dx=84Jr
x~-2

[Do the integration w.r.t.x yourself, taking x = 2sin ()]


(2) Evaluate of surface integral IfF.~ ds
s
Vector Differentiation 601

Z=3

x
The given surface of the cylinder can be divided into 3parts, namely
(a) 8 1 : the circular surface z = 0

(b) S2: the surface z == 3 (circular) and


I

(c) S3: the cylindrical portion of 8: Xl + i = 4, z == 0, z =3


we now find JJF.~ ds over SI ,82 , S3 .If we add them, we get R.H.S of (I).

(a) on SI:Z=.O; ~=-k; F.~=-(4xi-2ij).k=0;:. JJF.~ds'=O.


s,

- - - ( ) dxdy
(b) on S2:z==3; n=k ; F.n= 4xi-2y2j+9k .k=9; d\'=-I':' -I =dxdy
n.k

:. JJF.1i (is' = JJ9dxdy = 9A , where A is the area of the circle


s, s,

x 2 + y2 = 4, = 9J'l' (22) = 36J'l'


(c) on S3: Let fjJ =x2+i-4=0;

- V fjJ 2 ( xi + yi) xi + yi ( . 2 2
n =- - = = SInce x + y = 4)
!VfjJ! 2~X2 + i 2

4 x2 -2y 3 22 3
F.n= =x-y;
2
To evaluate JJF.1i ds, take x = 2cosB,y = 2sinB,
S3
602 Engineering Mathematics - I

and ds = 2dO dz ; limits of z are 0 to 3 and those of 0 are 0 to 27r .


2lf 3

Hence fJF.~ ds = f f (8cos 0-8sin 0)2d 0 dz


2 3

S3 (}=02=0

2" 3 2"
= 16 f [( cos 0-sin O)z ]
2 3
dO = 48 f (cos 0-sin O)dB =487r
2 3

(}=o 2=0 (}=O

[ Do the integration w.r.t. e yourself]


:. R.H.Sof(I)= 0+367r+487r=847r;:. L.H.S= R.H.S
Hence the theorem is verified.

Exercise - 7K

1. Verify Gauss's divergence theorem for A = (x 2 - yz)i + (i - zx) j + (Z2 - xy)k


taken over the rectangular parallalopiped 0 ~ x ~ 2, 0 ~ y ~ 3, 0 ~ z ~ 1.[Ans:36]

2. Use the divergence theorem to find ff F.ndS, where


-- s

f = (3x + 2Z2)i - (Z2 - 2 y) j + (/ - 2z)k and S is the surface of the sphere with
centre at (2, -I, 3) and radius 2 units. [Ans:32 TC]
3. Verify the divergence theorem for the vector function,
A = (4xz)i - (i)j + (yz)k , taken over the unit cube bounded by
x = 0, x = 1, y = 0, y = 1, z = 0 and z = 1.[Ans:3/2]
4. If r = xi + yj + zk , and S is the surface of the rectangular parallelepiped bounded
by planes x = O,y = O,z = O,x = a,y = band z = c, find the value of ffr.ndS
s
using Gauss's theorem. Verity your answer by direct evaluation of the integral.
[Ans:3abc]

5. Use the divergence theorem to evaluate ffA.nds for A = (2x)i-(2y)j + (3z)k

where s is the sphere given by (x _1)2 + i + Z2 = 1 [Ans:4 7r]

6. If V=(lx)i+(my)j+(nz)k and I, m, n being constants show that

ff V.ds = 327r (l + m + n). where S IS the surface the sphere


s 3
(X-3)2 +(y-2i +(z-I)2 = 4.
Vector Differentiation 603

7.12 Stoke's Theorem


7.12.1 Stoke's Theorem
Let (1) S be an open, two-sided surface bounded by a simple closed curve C.
(2) A be a vector function having continuous derivatives

Then, fA.dr = H(Y' x A).nds = H(Y' x A).d\'


where C travels in the +ve direction and n is the unit +ve (outward drawn) normal to
S.
Proof:

Let S be the surface. Let the projections of S on the coordinate planes be regions
bounded by simple closed curves.
Let 'R' the projection ofS on xy plane be bounded by C l . (see the figure above).
Let the equation of S be z = <l>1(x, y) where <1>1 is a single valued, continuous and
differentiable function.

j
Then Y'x(Ali)=alax alay
AI 0
604 Engineering Mathematics - I

{V x (A]i}}.n ds =(8A] _ 8A] ).nds = {8A] (n.}) _ 8A3 (n.k)}ds (.)


8z 8y 8z 8y ..... 1

The position vector r of any point on S can be taken as


r = xi + y) + zk

= xi + y} + t/>/x, y)k
8r _. 0+]
and - - } + - k
ay 8y

8r
But ay being the vector tangent to S, it is .i, to n.

8r 3</>]
.. ay' n = 0 ~ n.} = - 8y (n.k)

. 8AI 84>1 8AI


:. (I) => { V x A\,)} . n ds = (---n.k--n.k)ds
8z ay 8y

..... (ii)

on S, A\(x y, z) = A\(x, y, 4>\(Y = G(x, y) (say)


8AI 8AI 8z aa
-+-.-=-
ay 8z 8y 8y
:. (ii) becomes,
aa aa
{ V x (A\i) ] .n ds = - ay (n.k) ds = - 8y dxdy r: n.k. ds = dxdy]

H(VxA\i)}.nds=
,I
Jf- aaay dxdy=fCdx-,
R ('
by Green's theorem in the plane. Now

at each point (x, y) ofC, the value ofG is the same as the value of AI at each point
(x, y, z) of C, and since dx is same for both curves, we have

I
Vector Differentiation 605

Hence, H{V x(Ali}}n ds = fAldx ..... (iii)


s c

lilly by projecting S on yz and zx planes, it can be shown that,


H{V x (A2i)}n ds = fA 2dY ..... (iv)
s c

H{V x (A3 k )}.n ds = f A 3dz ..... (v)


s c

Adding (iii), (iv) and (v), HVx A.n ds =


s
fc A. dr
(since A. dr = AI dx + A 2dy + A 3dz)
Hence the theorem is proved.

Ex.7.12.1 :(a) Express Stoke's theorem in words and (b) obtain its cartesian form.
Sol :(a) The I ine integral of the tangential component of a vector A taken around a simple
closed curve C is equal to the surface integral of the normal component of curl
A taken over a surface Shaving C as its boundary.
(b) As in 7.11.2(b)
A=A l i+Aj+A3k
n = (cosa)i + (cos~)j + (cosy)k

) k
Then, Vx A = ajax ajay ajaz
Al A2 A3

=(aA 3 _ aA2)i + (aA I _ aA3)} + (aA 2 _ aA I )k


ay az az ax ax ay

aA 3 aA 2 aA aA 3 aA 2 aA I
(V x A)Jl =( - - -)cosa + ( -I - -)cos~ +( - - -)cos y
ay az az ax ax ay
606 Engine~ring Mathematics - I

Hence the cartesian form of the Stoke's theorem can be stated as,

=fA ,dx+A 2dy+A 3dz


c
Solved Examples
Ex.7.12.3 : VerifY Stoke's Theorem for A = (x - 2y)i + yilj + y2zk, where S is the upper
half of the sphere xl + y2 + z2 = 1 and C is its boundary.
Sol: The boundary of the projection ofS in the xy-plane is a circle with centre at origin
and unit radius. Its parametric equations are x = cosS, y = sinS, Z = 0, 0:::: S < 2n
dx = (-sinS)d9, dy = (cosS)dS.

f Adr =f(x-2y)dx+ yz 2dy+ y2 zdz


c c
21t
= f[cosS - 2sinS] (- sinS )dS (.: z =0)
e~()

-_ 2~[-sin2S
21t

JI + I -cos2S]dS-
- cos2S
- - + S -sin2SI
- - -_2 n
o 2 4 2 0

i j k
Y' x A = ajax ajry ajaz = 2k
(x-2y) yz2 y2 z

H(Y' x A).nds = H2(n.k )ds = 2 Hdxdy


s R

(n.k ds = dxdy and R is the projection of S on the xy plane)

Hence Stoke's theorem is verified.


Vector Differentiation 607

Ex.7.12.4: Prove that a necessary and sufficient condition that fF.dr for every closed
("

curve C is that '\I x F =0 .

Proof: (a) The condition is necessary: Let '\IF = 0;

Then by Stoke's theorem, fF.dr = If('\I x F).n ds = 0


(" S

(b) The condition is sufficient:

Suppose that fF.dr =0 around every closed path c.


('

Assume that '\I x F "* 0 at some point P. then, assuming that '\IF is continuous,
there exists a region with P as its interior point where '\IF = 0 .Let S be surface contained
in this region and let the normal n to S at each point has the same direction as '\IF .

Then '\IF = an, (a being a +ve constant); Let C be the boundary of S.

Then by Stokes theorem, f F .dr = If ('\IF).n ds =a Ifn.n ds > 0


c s s

Which is a contradiction to the hypothesis that f F.dr =0 ; :. '\IF =0

Note: It follows that '\IF = 0 is also a necessary and sufficient condition for the line
1'2
integral f F.dr to be independent of path joining the points ~ and ~ .(see 7.6.4)
I;

Ex.7.12.S If r = xi + yj + zk , show that f r .dr = 0


c

By Stoke's theorem, Jr.dr = If(Curl r).n ds ..... (1)


c ('
608 Engineering Mathematics - I

i j k
But curl r= a/ax a/ay a/dz =0
x y z

:. (1) => Jr.dr = 0


('

Ex.7.12.6: If'f and 'g' are scalar functions, show that


Jf( grad g). dr =- Jg(grad j).dr
(' ('

Sol: By stoke's theorem,


J{grad (fg)dr = Jficurl{grad(fg)}ln ds = 0, since curl grad (fg) = 0
(' S

.. J{grad (fg)}dr =0 ..... (1)


('

But grad(fg) = f(grad g) + g(grad j) ..... (2) /


Hence the result [from (1) and (2)]
Ex.7.12.7: If A is any vector function, prove by stoke's theorem that div curl A = O.
Sol: Let V be any volume enclosed by a closed surface S. Then by Gauss' divergence
theorem. We get,
JJJV.(curl A)dv = JJ(curl A).nds ..... (1)

Divide the surface S into two portions SI and S2 by a closed curve C.


Then JJ(curl A ).n ds
= JJ(curl A).ndst + JJ(curi A).n dS 2

= JA.dr - JA.dr
c c
Vector Differentiation 609

= 0, by Soke's theorem, since the +ve directions along the boundaries ofS, andS 2
are opposite.
:. (I) ~ fffV.(clirl A)dv=O

Since this is true for all volume elements V, we have,


V . curl A = 0 ~ div(curl A) = 0
Ex.7.12.8: Use Stoke's theorem and prove that curl gradJ= 0, where 'f' is a scalar
function.
Sol: IfS is a surface enclosed by a simple closed curve C, we have, by Stoke's theorem,

H{curl(grad J)}.nd.. = f(grad J).dr .... (I)


('

Now, grad! dr

=(8f i + aJ j + 8f k ).( dx i + dy j + dz k)
ax 0' az

= aJ dx+ 8f dy+ 8f dz=dJ


ax ay az
A
:. f(grad J).dr = fdf
(' A

= f(A) - f(A) = 0, where A is any point on C


:. (1) ~ ff{curl(grad f)}nds =0

Since this equation is true for all surface elements S,


we have, curl (grad j) = 0
Ex.7.12.9: Veri1)r stoke's theorem for A = yj - 2xyj taken round the rectangle bounded
by x = b, y = 0, y = a
y
F~ __~__~y=a
______~B

x=b
x=-b

--____ ~----~--~----~~--L---~~X
D A y=O
610 Engineering Mathematics - I

i j k
Sol: curl A = ajax ajay ajaz = - 4yk
y2 -2xy 0

For the given surface S, n = k


:. (curl A). n = - 4y
Hence ff(curIA).n ds

a h a h U

= ff- 4 ydxdy= f[ f -4ydx]dy= f -4xyl dx= f- 8bydy


~h
s y=o J.=~h 0 0
a
= -4by 2/ =-4a 2b ..... (I)
o

fA.dr
C
= f+ f+ f+
DA AB BF FD
J
fA.dr = y2 dx - 2xydy

AlongDA, y = 0, dy = 0, => JA.dr=O ( .: A.dr = 0)


DA

AlongAB, x= b, dx = 0
a
2
:. f A.dr= f-2bydy=-by2/: =-a b
AB y=O

AlongBF, y=a,dy=O
-b

:. JA.dr= Ja 2
dx = -2a2b
BF b

Along FD, x = -b, dx = 0


o 0
2
:. fA.dr= f 2bydy=-bi/ =-a b
FD a a

:. fA.dr= 0 - a 2 b - 2a2 b - a 2 b = - 4a2b ..... (2)


C

From (1) and (2), fA.dr = ffcurl(A).nds


c s
Hence the theorem is verified
Vector Differentiation 611

Ex.7.12.10: Use stoke's theorem to evaluate the integral fA.dr where A = 2y.i +3xY-
('

(2x +z)k, and C is the boundary of the triangle whose vertices are (0,0,0), (2,0,0),
(2,2,0).

) k
Sol: Curl A = ajax ajay ajaz
2
2/ 3x -2x-z
= 2} + (6x - 4y)k
Since the z-coordinate of each vertex of the triangle is zero, the triangle lies in the
xy-plane.
:. n = k.
:. (curl A). n = 6x - 4y
consider the triangle in xy-plane.
Equation of the straight line OB isy = x
By Stoke's theorem,

c
f A .dr = fsf(CurlA)nds o
(0,0)
A(2,O)

I 2

= f6xy - 2/ dx = f(6x - 2X2 )dx = 4 ~


x=O 0
2
3

1= 332 .
Ex. 7.12.11 Use Stoke's theorem to evaluate ff(curlA)nds, where A = 2yi + (x- 2zx)j
+ xyk, and S is the surface of the sphere .xl + Y. + z2 = b 2 above the xy-plane.
Sol: The boundary C of the surface S is the circle.xl + y. + z2 = b2, Z = o.
The parametric equations of C are x = bcose, y = bsine, z = 0, 0 ~ e < 27t
By Stoke's theorem, we have,

fsf(CurIA).nds =cf A.dr


612 Engineering Mathematics - I

= f2ydx+(x-2zx}dy+xydz = f2ydx+xdy (0: z=O,dz=OonC)


(' ('

2"
f(2bsin8)(-bsin8)d8+bcos8.bcos8.d8 [-: x = bcos8 => dx = - bsin8 d8
o
and y = bsin8 => dy = bcos8 d8]

= h 2 ](cos 2 8 -2sin 2 8~8 = b 2 2][1 + cos 28 - (l-cos28)18


o 0 2 r
J"
h221t b2 [ 3sin28]- b2
=--.2n=-nb 2
2 t
= - r(-1+3cos28)=+- -8+
222
o

Ex.7.12.12 Apply Stoke's theorem to evaluate f A.dr .where A = (x- y)i + (2y + z}j +

(y-z)k and C is the boundary ofthe triangle whose vertices are (~, 0, 0) (O,~, 0)
(O,O,~)
z
and

Sol: LetA= (~,O,O) B= (O,~,O) C= (O,O,~).


The equation of the plane ABC is (by intercept form), y

x y z
1/6 + 1/3 + 1/2 = I =>6x+3y+2z= 1 ..... (I)

The direction ratios of the normal to (1) are 6, 3, 2


.. . 6 3 2
:. D IrectlOn cosmes are -,-,-
777

Ifn is the unit normal to the plane, n = ..i +l j +3. k


7 7 7
A = (x - y)i + (2y + z}j + (y - z)k
i j k
VxA= a/ax aj8y a/az =k
x-y 2y+z y-z

.. (Vx A}.n =3.


7
Vector Differentiation 613

:. By Stoke's theorem,

fA.dr = Ifs (VF).n ds = ~7sIfds = ~7


('
(Area of triangle ABC) ..... (2)

To find the area of triangle ABC:

AB=
(H+(H ~ ~
AC= (H+(H ~~
Direction ratios of AB are -1, !, 0 .
6 3
-1 2
Direction ratios of AC are 15' 15' 0

. . ratios
Direction _-1
. 0 f AC a r e- .~-
0- ..
flO' , flO
cos CAB = ( Ts )(~ )+ 0 +0 = )so
sin CAB = .:i.--
EO
:. Area of triangle ABC = 1. AB.AC sin CAB = 1.. 15
2 2 6
.JlO ._7_ = 2.
6 J50 72

JA.dr=~x-2..=_1 [from(2)]
(' 7 72 36
Ex.7.12.13Evaluate If(curl A).n ds taken over the portion s of the surface
s
X2+/+Z2_2/x+ft=0 above the xy plane z 0, if
A = ~::CX2 + /-z2)i and verity Stoke's theorem.
Solution: Let'S' denote the portion of the surface, x 2 + y2 + Z2 - 2ft + ft = 0 above
the xy-plane z= O.
The surface S meets the xy-plane in the circle 'C', whose equations are
2
x + / - 2ft = 0, Z = O.
614 Engineering Mathematics - I

:. The parametric equations of 'C' can be taken as

x == j + j cosS, y== jsinS, z=o (0~S<2n;)

Let S) denote the plane region bounded by C.lfS) is the surface consisting ofS and
SI' SI is a closed surface.
:. From example 7.11.8 on divergence theorem, we have

=0

i.e., f f(Curl A).2 ds + f f(Curl A).n ds = 0 [.: S) consists of Sand S)]


s

I.e., f f(Curl A).n ds - f f(curIA).kds == 0


s '"

f f(Curl A).nds = f f(curIA).kds ..... (I)


s

j k
Now, curl A= a/at a/ay a/az

= i(- 2y - 2z) + j(-2z - 2x) + k(- 2x - 2y)


:. (curl A). k = - 2(x + y)

:. From (I), ff{curIA).nds=-2 f f(x+ y}is


. " .'I'
Polar equation ofS) is r = ~cosS.

:. changing to polar coordinates,


It 2/eos9
ff(curlA).nds=-2 f f(rcosS+ rsinS)rdedr
s, 9=0 r~O
Vector Differentiation 615

8(,1t 16f31t 1t
3 4
= - 2 x -'- f(eosO + sin e )eos 8de = --3- feos OdO [.: feos 3 Osin Od8 = 0]
3 0 0 o

16f3 1t,12 4 32f3 3 I n .3


= - - x 2 feos Od8=---.-x-x-=-2nj ..... (2)
3 0 3 4 2 2

Again, fA.dr
('

= f(x 2 + i - Z2 }h: + (i + Z2 - x2~y + (Z2 + x2 -- y2 )1z


C

= f(x 2 + i }Ix + (y2 - x2)1y [.: on C z = 0, dz = 0]


C

2"
ftU + f cos 8)2 + f2 sin e](- fsine)de
o
2

21t
+ f [f2 sin 2e- (f + f cos8)21f cose)de
o

fffiki + cosS)' + sin' S}(- sinS) + {'in'S"" (I + cosS' )}coril]nIl

= f3[2I_ 2sin ede +2I_ 2sin 2ede.+ 2I_ 2cosed8 + 2I_ cos 3 eaU

o 0 0 0

21t 21t
- f(I+Cos2e)de+ fSin ecos8de
2 1 (on simplification)
o 0

= f'(-2n)=-2nf3 (all other integrals vanish) ..... (3)

from (2) and (3), we have

f f(CurIA).nd\ = fA.dr
s ('
which verifies Stoke's theorem.
616 Engineering Mathematics - I

Exercise-7(1)

1. f F=(xe )i+(3/)j-(z)dz,and
X 2
C is the X +y2=9,z=2, evaluate fF.dr
('

using Stoke's theorem.


[Ans: OJ

2. Apply Stoke's theorem to obtain the value of the integral fV.dr ,where
('

v = (3/)i + (2X2)j -(x+ 2z)k and C is the boundary of the triangle whose
veltices are (0,0,0),(1,0,0) and (I, I ,0)
[Ans: I]

3. Verify Stoke's theorem for F = (2x - y)i - (yz 2)j - (/ z)k if S is the upper halt
surface of the sphere x 2 + y2 + Z1 = I and C is the boundary.

[Ans: Jr]
4. If F = (y - z + 2)i + (yz + 4)j - (xz)k and S represents the surface of the cube
x = 0, y = 0, z = 0, x = 2, y = 2, z = 2 above the xy plane , verify that
f
ff(Curl F).ds = F.dr ,C being the boundary of S traversed in the +ve direction.
s c
[Ans: Each integral =-4]

5. Find the value of the integral f(yz )dx + (zx )dy + (xy )dz ,using Stoke's theorem
c
where C is the Curve x 2 + y2 = 4, z = y2
[Ans:O]

Verify Stoke's theorem for the function V = (3x )i + (2xy)j , integrated along the
2
6.
square x = 0, y = 0, x = I,y =) in thexy-plane.. [Ans: 0]

7. Evaluate by Stoke's theorem the integral where


c
A= (2sinz)i-(3cosx)j+(siny)k; where C is the boundary of the rectangle
o~ x ~ Jr, 0 ~ Y ~ 1, z = 0
[Ans:6]
Vector Differentiation 617

Exercise - 7(m)
1. If !(x,y,z)=x 1y lllzn-1 , find the directional derivative of f at (1,1,1) in the
direction of (i+2j+2k)
1
[Ans: -(I + 2m+ 2n)]
3
2. Find the acute angle between the surfaces

x 2 + i + Z2 = 6 and 3xyz + i Z- xy + 3 =
at (1, -I, 2)

IAns: cos'( ~}

3. If r = xi + yj + zk , and p, q are constant vectors, show that


Div{(rx p)xq} = -2(p,q)

4. If F=(x 2y)i-(iz)j+(z2x )k,findcurIFat(1,-2,3)

S. If !=xyz(x+y+z),provethat curl gradf=O

6. A fluid motion is given by V=(z3)i-(i)j+(3xz 2 )k. Show that it IS

irrotational. Find its velocity potential rjJ sllch that V = V rjJ


3
[Ans: rjJ = xz 3 _l_ + c ]
3
(2,1)

7. Evaluate J(4x -12x 2i)dx-(8x y)dy along the path x 3-3xi =2/
3 3

(0,0)

[Ans: 16]
2
8. If V=(xy)i-(yz)j+(zx )k and S is the surface of the cube bounded by
x = 0, x = 2, y = 0, y = 2, z = 0, z = 2 , evaluate JV.n ds
s
[Ans: 32/3]

9. If ! = 4x + yz , evaluate HJ!dv over the region in the first octant bounded by


v
x-, + y 2 = 1, z = 0, z = 3 [Ans: 11/2]
618 Engineering Mathematics - I

10. Express F = xi - i j + zk in (a) cylindrical polar coordinates (b) spherical polar

coordinates.

+(r sin OcosOcos 2 _,.2 sin 2 OcosBsin 3 - rsinBcosB)eo

+ (-rsinBsincos-r 2 sin 2 Bsin 2 cos)e~]

II. Iff = pz sin 2B ,find grad f in cylindrical coordinates

[(zsin 20)ep + (2zcos 2B)eo + (psin 2B)ez

12. If A = (rcosB}er -(!sinB)eo +re~, find the curlA in spherical coordinates


r

[Ans: (cotB)e r +2eo -(2sinB)e~]

13. Verify Green's theorem in the plane for f(x


('
3
- y2)dx + (x 2 - 2xy)dy ,where C is

the boundary of the squart: bounded by O:S x:S 1, O:s y:s 1 [Ans:1]

14. Using Gauss divergence theorem, prove that If F.n ds = TCr


2
P, where
s

f = (y2 Z ); + (xz)j + (z2)k ,and S is the surface bounded by

x 2 + y2 = r2 , z = 0, and z = I

15. Show that the Stoke's theorem, when restricted to the xy-p/ane, is Green's theorem

in the plane.

(Hint: In Stoke's theorem, take A = Pi + Qj; n = k; and ds = dxdy)


Vector Differentiation 619

Exercise-7(n)

I. Choose the correct answer in the following questions

The tangent vector at the point t=1 on the curve x = t + 1, y = 4t - 3, z = t 3 is


2
I.

(a) 2i-4j+3k (b) 2i+4j+3k (c) 2i-4j-3k (d) 2i+4j-3k [b]

2. The magnitude of acceleration at 0 = 0 on the curve


x = 2cos30,y = 2sin 30,z = 30 is

(a) 6 (b) 9 (c) 18 (d) 3 [c]

3. if 1= xyz, the value of Igrad II at the point (I, 2, -I) is


(a) 0 (b) I (c) 2 (d) 3 Cd]
- 4. -, The maximum rate of change of f = x/ + yz + zx 2 at the point (I, I, I) is

(a) Jli (b) 0 (c) 3 (d) none Cd]


5. The angle between the normals to the sphere x 2 + y2 + Z2 = 9 at the points (1,2,
2) and (2, 1,2)is

1C 1C
(b) - (d) - [a]
2 4
6. If a is a constant vector and r = xi + yj + zk , then '\l(a.r) is

(a) 0 (b) a (c) r (d) r [b]

7. If (x + 3y)i + (2 - 3y)j + (x + az)k is a solinoidal vector, the value of a is

(a) 0 (b) 1 (c) 2 (d) 3 [c]

8. If r = xi + yj + zk, and a = ~ r, div ii =


3

(a) 0 (b) I (c) -1 (d) 2 [b]


620 Engineering Mathematics - I

Exercise - 7(0)
I. Iff(x,yx) = X lymz"_I, find the directional derivative of fat (1,1,1) in the direction of
I
(i+2j+2k) [Ans: "3 (I + 2m + 2n)

2. Find the acute angle between the surfaces x 2 + y +:? = 6 and

3xyz + yz - xy + 3 = 0 at (1,-1,2) [Ans: cos


_,(Fs]
3]

3. If r = xi + yj + zk, and p, q, are constant vectors show that


Div {r x p} x q} =- 2(p.q)
4. If F = (ry)i - (y2z)j + (:?x)k, find curl curl F at (1,-2,3)
[Ans: 6i - 2j + 4k]
5. Iff= xyz(x + y+ z), prove that curl gradf= 0
6. A fluid motion is given by Y = (z3)i - (y2)j + (3xz 2)k. Show that it is irrotational.
Find its velocity potential +such that Y = V~

y3
[Ans: ~ = xz 3 - - +c ]
3
(2,1)
7. Evaluate 3
f{4x -12x2 y2 )Ix - {8x y}Jy
3
along the path x 3 - 3xy = 2;
(0,0)

[Ans: 16]
8. IfY = (xy)i - (yz}j + (zr)k and S is the surface of the cube bounded by x = 0, x =

2 , y=O " y=2 z=Oandz=2 , evaluate f V .nds


s
[Ans: 32/3]

9. If f = 4x + yz, evaluate f f ffdv over the region in the first octant bounded by
v

x 2 + y2 = I, z = 0, z = 3

[Ans: 1112]
Vector Differentiation 621

10. Express F = xi - y.j + zk in


a) cylindrical polar coordinates
b) spherical polar coordinates.
[Ans: a) p(cos28 - pSin38)e p - psin8cos8(1 + psin8).ea + ze z
b) (rsin 28cos2<jl- ~sin30sin3<jl + rcos20)e r + (rsinOcosOcos2<jl- ~sin20cos8sin3<jl
rsin8cos8)ea + (- rsin8sincj>coscj> - ~sin28sin2cj>coscje.]
11. If/= pzsin28, find grad/in cylindrical coordinates
[Ans: (zsin28)e p + (2xcos20)e a + (psin28)e z]

12. If A = (rcosO}er -(-;,sin 0 )eo + re4>, find curl A in spherical coordinates


[Ans: (cot8)e r + 2eo - (sin8)e.]

13. Verify Green's theorem in the plane for J(x 3 -l )tx + (x 2 - 2xy}ty, where C is the
c
boundary of the square bounded by 0 ~ x ~ I, 0 ~ Y ~ I

[Ans: 1]

14. Using Gauss' divergence theorem, prove that J


JF.nds = 1tr2[2 , where
s
F = (y2 Z} + (xz)j + (Z2 ~ , and S is the surface bounded by x2 + y2 = r2 , z =0, and
z=l
15. Show that the Stoke's theorem, when restricted to the xy plane, is Green's theorem
in the plane
[Hint: In Stoke's theorem, take A = Pi + Qj; n = k; and ds = dx dy]
"This page is Intentionally Left Blank"
8
Laplace Transforms

8.1 Laplace Transforms

Introd uction
The theory of Laplace transform is an essential part ofthe mathematical background
required by engineers, physists and mathematicians. It gives an easy and effective
means for solving certain types of differential and integral equations. It is the foundation
of the modern form of operational calculus, which was originated in an attempt to
justify certain operational methods used by an electrical engineer Oliver Heavinide, in
the latter part of the 19th century for solving equations in electromagnetic theory.
The Laplace transform reduces the problem of solving a differential equation to
an algebraic problem. It is particularly useful for solving problems where the mechanical
or electrical driving force has discontinuties, acts for a short time only or is periodic
but not merely a sine or cosine function.

8.1.1 A class of linear transformations of particular importance is that of the integral


transforms. The Laplace transformation (L.T) is a special case of an integral
transformation and is defined h~'
cO

F(s) = L [f{t)1 = Je-stf(t) dt


o
624 Engineering Mathematics - I

Def. Piecewise continuity ofa function:


A function off(t) is sectionally continuous (Piecewise continuous) in an interval
o~ t ~ b iff(t) is continuous over every finite interval 0 ~ t ~ b except possibly at a
finite number of points, where there are jump discontinuties and at which the
function approaches different limits from left and right.

f(t)

Sectionally continuous function


Def. Functions of exponential Order:-

A function f(t) is said to be of exponential order cr, as t ~ 00 if 3 M > 0


3/f(t)/ < Meat, \;f t~0 .... (1)
or equivalently, le--crtf(t)1 ~ M, \;f t ~ o.
t2
Remark: eat and sinbt are of exponential order, while e is not of exponential order since

which can be made larger than any given constant by increasing 't' indefinitely.
8.1.2 Conditions for L. T to exist
By using above remarks we can prove the conditions that are sufficient for the
existence of Laplace transforms.
Theorem: Iff(t) is piecewise continuous in every finite integral [0, N] for N > 0,
and of exponential order cr,that is If(t)1 ~ Meat, \;f t ~ 0 and M> o. (from I)
then the Laplace transform ofF(t) exists for all s> 0'

00 00 00

Proof: LetI = /F(s)1 = Je-stf(t)dt ~ ~ f(t)/ e-stdt ~ JM eate-stdt


o 0 0
=-=-s
M
cr
Here, s > 0' since I is non-negative everywhere. This shows that above conditions
are sufficient for the existence of L [f(t)].
Laplace Transforms 625

Properties of Laplace Transforms


8.1.3 Linearity property: Let f,(t) and fit) be two functions on [0, co] such that the
Laplace transforms L[f,(t)] and Lltit)] exist. If k, and k2 are two constants,
then
L[k/,(t) + klit)] = k,L[f,(t)J + k2 L[fit)J ..... (i)
This property is valid since

00 00 00

=
f[ktf; (t) + k 2 f2(t)]e-'1 dt fk\.!; (t)e-'I elt + fk2.f; (t)e -,I lit = k\L[j; (I)] + k 2 L[f2(t)]
0 0 0 ~

This result can be extended to the linear combinations of more than two functions.
8.1.4 Iff(t) is piecewise continuous on [0, N] for each N > 0, and ttt) is of exponential
order 0", that is by 8.10.1 (1), If(t)1 ::s Meat, V t ~ 0 and M > 0 and ifL [f(t)] = F(s)
then we have
(1) L [eatf(t)] = F(s-a) V s > 0" + a va> 0 ..... (i)

,
(2) L[f (tn. = e-aSF(s)

f(t -a) t > a}


where f,(t) = { 0 t < a (a>O)

and (3) L (ttat = F(~) (for any a> 0)

Proof(l)
_a_
--~------~----~~s

From definition, we have


00

L[f(t)] = fe-Mf(t) dt = F(s)


o
00 00

so that L[eatf(t)] = fe-ot[eal f(t)dt = fe-(s-a)1 f(t)dt = F(s - a)


o 0
626 Engineering Mathematics - I

Th is property shows sh ifting on the s-axis and is called the first shifting property.
This means that replacing s by s - a in the Laplace transform corresponds to
multiplying the original function by eat
00 a 00

(2) L(fl(t) = fe-Ilf; (t)dt = fe-'If; (t)dt + fe-sl.t;(t)dt


o 0 a
a 00

= fe-.II.Odt+ fe-lIf(/-a) dt from definition offl(t)


o a
00 00

= fe-llf(t-a)dt== fe-I(p+a)f(p)dp (where t = p+a)


o 0
00

= e -a.1 fe -P' f(p )dp == e -a.1 F(s)


o
This property indicates shifting on the t-axis and is called the second shifting property
00 d 00

(3) L[f(at)] = fe-'I f(at)dt == fe-I(p/a) f(p)-' [taking t = pia]


o o a

= ~ }e-IP/af(p)dP==~F(~)
a0 a a
This property is known as change of scale property.

8.1.5 Laplace Transforms of standard functions


I
(a) L (e-at ) = -- (s> a) ...... ( I)
s+a

Pro()f: we have L(e-at ) =


rJe -.1/
.e -aidt-
_ rJe -(Ha)1 d _ - 1 [ -(Ha)I]OO
t--- e 0
_
---
1
o 0 s+a s+a
. 1
Ifa=OtheL(I)= - (s> 0) ..... (2)
s
Also changing the sign of , a' we get
I
L(eat ) = -- ..... (3)
s-a
a s
(b) L(sinat) = 2 2 and L(cosat) = J 2 (s> 0)
S +a s- +a
1 s+ia
Pro()f: L(cosat+isinat) = L(e lat ) = --.- ==-)-- from (a)
S -lU s- + a 2
Laplace Transforms 627

Equating real and imaginary parts we get


a s
L(sinat) = 2 2 ,L(cosat) = 2 2 ..... (4)
S +a s +a
a s
(c) L(sinhat) =
S
2
-a
2 and L(coshat) =
S
2
-(I
2 (s> laD

Proof:
.
L(smhat)=L
(e _e-
OI
1
ol
01 11 -01
=-L(e )--L(e )
222

- - 1[1
2 s-a
-----
s+a
I] from (a)

a
Thus (sinhat) = 2 2
S -a
s
similarly we can show that L(coshat) = 2 2 . (5)
s -(I

I(n + 1)
(d) L(tn) = ----;+] [(n + I) > 0 and s> 0]
s

Proof:

OOfe-P.pnd =T(n+l)
n+1 'P 11+1
o S S

00

Since T(n+I)= fe-xxndx and T(n+ I) = n! ifn is a +ve integer


o
n!
:. L(tn) = ----;;-+J ..... (6)
s
From (6) we have
1 1
L(I)= -, L(t) = -2 ,
s s
Solved Examples

8.1.6 Find the Laplace transform of


1. 2t3 + cos4t + e-2t 4. 3t2 + e- t + sin 32t
2. e2t + 4t3 - sin2tcos3t 5. cos32t
3. cos(wt + ~)
628 Engineering Mathematics - I

6 s 1 12 s 1
Sol: I. L(2t3 + cos4t + e-2t ) = 2x-+ + - - =-+ +--
S4 S2 + 16 S + 2 S4 S2 + 16 S + 2
2. L(e2t ) + 4L(t3 ) - L(sin2tcos3t)

I 6 (5
S - 2 + 4x7 - S2 + 25 - n 2 + 1
1)
1
[.: sin2tcos3t = "2 (sin5t - sint)]
3. L cos(wt + 13) = L[coswt.cosj3 - sinwtsinj3]

=cosfJL(coswt)-sinfJL(sinwt)= cos fJ 2
S
2
.
-sm
fJ 2
w
2
+w S S +w
4. L[3t2+e-t+sin 32t] = 3L(t2 ) + L(e-t ) + L (sin 32t)
2 1 3 3
=3.-+--+ - -?--
S3 s+1 2(S2+4) 2s-+36
2 1 3 1 3 1
=-+--+--------
S3 S + 1 2 S2 + 4 2 S2 + 36
1
(.: L(sin32t) = 4" L[3sin2t - sin6t]

3 [cos6t + 3cos2t]
5. L(cos 2t) = L 4

=
1
4"
(s 3S)
S2 + 36 + S2 + 4

Exercise 8(A)
Find the Laplace transform of
8 3 1
1. 4t2 + sin3t + e-2t Ans. - + - - + - -
S3 S2 +9 s+2
1 4
2. (sin2t - cos2tf Ans. -
S S2 + 16

3. Ans. !(!+_S_)
2 S S2 -64

4.

5.
Laplace Transforms 629

6. cos 2 (2bt) 1(1


Ans. - - + ? ?
2 s s- + 16b-
s)
162
7. Ans. -----::,---------:---
(S2 -81)(.~2 -9)

a b
8. sinhat- sinbt An s. -?::-------::-)
s- -(r

S(S2 - 28)
9. Ans. -----:----::---
2
(S2 - 36)(5 - 4)

s s
10. coshat - cosbt Ans. 2 2
S -(I

8.1.7 Examples on properties 8.1.4


Find the Laplace transform of
1. e-btcosat 2. t 3 e-4t
4. e-3t sin3t 5. e-2t[2cos3t-3sin3t]
Sol.
S
1. L( cosat) =) 2
s- +0

(from 8.1.4( I) first shifting theorem)

6
2. L(t3 ) = 4
s
By shifting theorem 8.1.4(1)
6
L(e-4t. t 3) = - - -
(s + 4)4

. 3
3. L(sm3t) = -2--
S +9

by 8.1.4(1),
3
2t
L( e sin3t) = (5 _ 2)2 + 9.
630 Engil~eering Mathematics - I

4. e-3t .sin3t

L(sin3t) =- L [~(3Sin
4
1 - Sin3/)] = ~ L(sin I) - ~ L(sin 3/)
4 4
3 1 1 3

By 8.1.4(1)

-3t . - ~ I
L(e sm3t) - 4 3)2
(s+ +I
5. e-2 t(2cos3t - 3sin3t)
s
L(2cos3t) = 2.-2-
s +9
9
L(3sin3t) = ~9
8 +
By shifting theorem 8.1.4( I)
2L(e-2tcos3t) - 3L(e-2t sin3t)
2(s + 2) 9
= (s + 2)2 + 9 - (8 + 2)2 + 9
3s
8.1.8 If L[f(t)] = ~9 . find L[f(3t)]
8 +
38
SoL 'L[f(t)] = S2 +9

s
I 3") 38
By 8.1.4 (3), L[f(3t)] = 3(8)2 8
2
+27
- +9
3

Exercise 8(8)
Find the Laplace Transform of
2 8-3
1. Ans. 4cosa +sin-----,:-----
(s - 3)2 + 16 (8 - 3)2 + 16

2.

3. cosh2t.cos2t Ans ~[ 8-2 + __8_+_2_]


. 2 (8 - 2)2 + 4 (8 + 2)2 + 4
Laplace Transforms 631

a(s2 + 2( 2)
4. coshat sinat Ans.
S4 + 2a 4
S(S2 - 7)
5. cos3t.cosh4t Ans.
S4 + 625 -14s 2
2s-5
6. e- 21(2cos3t-3sin3t) Ans.
S2 +4s+ 13
S3
7. coshat cosat Ans.
S4 + 4a 4
3
8. elsin4t.cos2t Ans.
(s _1)2 +36

5s 2 -3s+2
Ans.
S3

n+ 1 6
Ans.
(8 + a)"+1 + (.\'2 + 1)(s2 + a)

8.1.9 Property (4) : Effect of multiplication by t


If L[f(t)l = F(s) then L[tf(t)] = -FI(s)
00

Proof: We know that F(s) = Je-"f(t)dt.


o
differentiating with respect to s
00 a 00

F(s)= J-[e-"f(t)]dt= Je-"'f(t)dt


o as 0

00

:. -FI(s) = Je-" [if(t)]dl = L[if(t)]


o
8.1.10 Examples

I. - -3- L(Ie -2')_


L( e-2')_ d 1 _
--------_o_ 1
s+2' ds s + 2 (s + 2)2

2,
d d ( 2
L(lcosat)=(-I)-[L(cosal)]=-- 2 = 2
2S2
2 2
s)
ds ds s + a (s + a )

3. L(t sin I) = __d


d..
(_I_) =
S2 +1
---,-2_s---,-
(S2 + 1)2
632 Engineering Mathematics - I

a
4. L(atsinat-cosat)=aL(tsinat)-L(COsat)=a[-dd ( 2-
S S +a
2)]- s 2+a 2
S

2
2as] s 2a s s
= a [ (S2 +a2f~ - (S2 +a 2) = (S2 +a 2 )2 - (S2 +l?)
5. L(1+tet)3 = L[I+3et+3t2e2t+t3e3t]
= L(I) + 3L(tet)+ 3L(t2e 2t) + L(t3e3t)
2 3
1 d 1 d 1 d ( 1 ) '1 3 6 6
=~-3 ds (s-I) +3 ds 2 (s-2)ds 3 s-2 =2+ (s-1)2 + (S-2)3 + (s-3)4

6.

d
L(te4t sin2t) = -L[e sm 2/1 = - -
41' d [ 2 ] by shifting property
7. 2
ds ds (s-4) +4

d [ 2 J 45 -16
=- ds S2 -85+20 = (S2 -8s+20)2
2 2
d d [ s -1 ]
L(t2etcos4t) = ds2 L(e cos4/) = d~2 (s _1)2 + 16 by shifting theorem
l
8.

=_~[ S2-2S-15]=
3 2
(2s -6s -90s+94)
ds (S2 - 2s + 17)2 (S2 - 2s + 17)3

Exercise 8(C)

Find the Laplace transform of the following:

1. Ie-"'sjn2. Ans. H~ (s 3)' - (X~~::15;),]


6s-18
3t
I 2. te- sin3t Ans. (S2 _ 6s + 18)2

2s+4
3. Ans, (S2 + 4s + 5)2
Laplace Transforms 633

9+ 12s -3s 2
4. 2tsin3t-3tcos3t Ans. 2 2
(s +9)

35-3
5. tetsin2tcost Ans. 2 ?
(s - 2s + 10)-

8.1.11 Property (5) If L[f(t)] = f(s), then L U.f(t)] = J/(5) ds


,
00

Proof: f(s)= fe-'I/(t)dt

Integrating bothsides w.r.t 's' between the limits s, 00 and changing the order of
integration on right hand side (R.H.S),

'1Je-.\II(t)dS]dt = J-[e~\1 l(t)]


J/(S)ds = 00 dt = Je-\I I~t) dr
"tL 0 0 0

8.1.12 Examples
Find the Laplace Transforms of the following

1. ~(1-cosal) 2. ~(e-I sinal)


I I

1 s
Sol.1. L(I-cosat) = L(l) - L(cosat) = - - 2 2
S S +a

(l I s
)K
L -(I-cosat) = - - 2 ] [
1 2
2 ds= logs--log(s2+a) ]00
1
=-Iog(2? 1
s + a-
2
t , s s +a 2 0 2 s

a
2. L(e-tsinat) = 2 2
(s+l) +a

I ISlOat)
L -(e-
[t
. ]
= oof a2 [-1
(S+I)]OO
2ds= tan - 1t
- =--tan s+1
--=cot _I(S+I)
-- -1
s (s + 1) + a a ,2 a a
634 Engineering Mathematics - I

1 1
3. L(1-e2 t) = L(I) - L(e2t) = ---
s s-2

I
L[-(1-e
t
21]
) = ----
s s-2
X
I I)ds= [logs-log(s-2)Js100 =Iog--
(s - 2)
s
s

2
4. L(e-3tsin2t) = ---::---
(S+3)2 +4

L(!e-
t
3/
Sin2t)= }
s
22
(s + 3) + 4
ds=tan- I (S+3)00
2 s

1t
=-=tan _1(S+3)
- - =cot _1(S+3)
--
2 2 2
1 I
5. L(eal ) = --, L(e ht )= - -
s-a s-b
1 al -e bl]
L [ -(e
t
) = XI - - - -1)
s-a s-b
- ds= [log(s-a)-Iog(s-bh,.
100
s

s-a s-a s-b )


= log( - )] 00
=-IOg(-)=IOg-
(
[
s-b s
s-b s-a

Exercise 8(0)

Find the Laplace transfonn ofthe following:

1.
1
-(1-cost)
t
1 (hI]
Ans. 2"log - s -

I _I
2. -e SlOt Ans. coc 1 (s+l)
t
1
3. - (-at
e - e-hI) Ans.log C+b)
--
t s+a

4. ! (e-3tsin2t) S-3)
Ans. coC I ( -2-
t

5. !
t
(sin2t) Ans.
1 (.'+4]
"4 log ~
Laplace Transforms 635

6.
I
- (coat - cosbt) I
Ans. -log (S2+b 2)
2 2
t 2 S +(1

7.
I
- (I - cos3t) I
Ans. -log (S2 +9)
--2-
t 2 S

8. ! (sin3t) Ans. 4"I [3cot


_)
S- cot
_)
"3s]
t
8.2.1 Theorem: Iff(t) is continous t ~ 0 and of exponential order, say 0" and has a
derivative ~(t) which is piecewise continuous on every finite interval [0, N] for
each N > 0, then the Lapalce transform of the derivative ~(t) exists for s > 0" and
L{~(t)} = sLf(t) - f(O)
00

Proof: L(f(t)] = je-'I f(t)dt


o
00

L(~(t)] = je-'I fl(t)dt


o
Integration by parts gives
00

L(~(t)] = [e-.'I f(1)1 + s je-'I f(t)dt


o
Since f(t) is of an exponential order, the integrand in first integral on the R.H.S. is
zero at the upper limit when s> 0' and iff(O) at the lower limit. Thus, we have
00

L( ~(t)] = 0 - f(O) + S je -;t f(t)dt =' sLf(t) - f(O)


o
If Lt f(t) = f(O+) exists, but is not equal to f(O), which mayor may not exist then
1-)0

L(~(t)] = sLf(t) - f(O+).


Note: Iff(t) and its derivatives ~(t), ~I(t) .............. fI- 1 (t) are continous t ~ 0 and are
functions of exponential order for some M > 0 and 0', and the derivative fI(t) is
Piecewise continous on every finite interval in the range t ~ 0, then the Laplace
transform offl(t) exists when s> 0' and is given by
L[fI(t)] = s"[Lf(t) - s" - If(O) - s" - 2 ~(O) ............... _tn - 1(0)]
If n = 2, L [~I(t)] = s2[L(f(t] - sf(O) -~(O) and so on.
636 Engineering Mathematics - I

8.2.2 Example

Prove that L(cosat) = S2 + a2

Sol. We can show this with the help of above theorem


Let f(t) = cosat => f(O) = 1
then [I(t) = -asint => fl(O) = 0
[l1(t) = -a2cosat
Hence, we have
L[fll(t)] = s2Lf(t) - sf(O) - fl(O)
:. L[-a 2cosat]= s2L(cosat) - s - 0 = s2L(cosat) - s
2
s2L(cosat) + a L(cosat) = s

s
:. L( cosat) = 2 2
+a S
8.2.3 Theorem: Iff is Piecewise continous on [0, N] for each N > 0 and is of exponential
I I
order cr, then L ff(u)du = -L[/(/)] v s > CT,
o s
I

Proof. Let p(t) = ff(u)du. Then we have p1(t) = f(t) and f(O) = 0;
o

L[~(t)l =s.L[~(t)l-~(O) =s[L(~(tl =SL[!f(U)du1


:. L Iff(u)du
[
o
III
= -L[I(/)] = -[L(f(/]
s s
8.2.4 Examples
Find the Laplace transforms of
I

I. fsin 2pdp
o
I

2. fp cosh p dp
o
3. r
I

o p

mp
dp
I
rp .
sm P
4. je . - - d'P
o P
2
Sol. 1. L(sin2t) = Z--4 = f(s) , say
s +
112
fsin 2pdp =- f(s) = --=-2--
o S s(s +4)
Laplace Transforms 637

2.

I 2 )
I (s + (1-)
o J
then L pcosh pdp = - I(s) =- )
8
)
(8- -a-)s

3.
.
L(smt)= ~I
I
s +
; L
(I .
-SlOt
t
)
= J-2-
00

s +1
0\
I ds

L 'JSin
-- p dOp=-Ijo() I
s =-cot -I
s
o P s s

I I
4. L(sint)= -2-1 ... L( etsint) = --.,,----
s + (s _1)2 + I

I
L [ -ee' sint) ] = ooJ ds2
t , (s -I) +1

'f sin p 1 1 1
:. L e P --dp = - I(s) = -cot- (s -1)
o P s s

Exercise 8(E)

Find the Laplace Transform of


I I
I
I. Jcos2pdp
0
3
2. Jpe- sin 4 pdp
0
3. Jp sin 3pdp
I

s 8(s + 3) I _I
Ans. I. 2. 3. -cot s
S(S2 + 4) S(S2 + 6s + 25)2 s
638 Engineering Mathematics - I

Laplace Transforms of Periodic Functions


8.2.5 A function f(t) in said to be periodic with period p>O iff(t+p) = f(t) for all t>O
Since periodic appear in many practical problems and in most cases are more
complicated than single sine or cosine functions, we find it useful to establish the
following result:

I I'
L([f(t)] = I-e- Ps fe-Slf(t)dt
o
Proof: Let f(t) be a periodic function of period 'p' so that f(t) = f(t+p) = f(t+2p) = ...... .
00 I' 21'
L[f(t)] = fe -si f(t)dt = fe -s/ f(t)dt + fe -si f(t)dt + ............... .
o o I'

writing t = u+p
21' I'
now fe-ollf(t)dt= fe-o,(u+p)f(u+ p)du
I' 0

I'
= e- Ps fe-OVlf(t + p)dt (change u to t)
o

I'
f -vlfCt )dt
= e -1" Je 0 [.: f(t+p)=f(t)]
o

31' I'
similarly, fe-oIl f(t)dt = e- 21'S fe-ollf(t)dt and so on
21' 0

I'
L[f(t)] = (1+e-ps+e-2ps + .................... 00) fe-ollf(t)
o
The expression within parantheses is a G.P. with c.r. = e-PS

:. L[f(t)] = 1_:-1'$ Je-Slf(t)dt


o
Laplace Transforms 639

Solved Problems

8.2.6 Find the Laplace Transform of


t
l. f(t) =k- for 0 < t < P and f(t) = f(t+p)
P
2. f(t) = 1 for 0 < t < a and f(t) = -1, a < t < 2a and f(t) is periodic with period 2a
Sol. Since f(t) is periodic with period p.
P P k
L[f(l)] = 1. Je-s1f(l)dt= 1 Je-"I.~dt
l-e- sp l-e-"P p
o 0

2. f(t) is peirodic with peirod 2a

L[f(t)] =
1 2a si
Je- f(t)dt =
1 [a
1 _ e- 2as
2a.
Je-sl.l.dt + Je-'I (-I)dt
1
1- e-2as 0 0 0

SI
_ 1 [{_e-SI}a {_e- }2aj_l 1 (l -a,)2
-1_e-2a, - s - 0 - - s - 0 - s l_e-2as ' -e

- 1s [11+- ee -2as
--.
-2as
]_ 1[/f - e-'T --tanh
--.
S ~
1 [-as ]
s-~ 2
1-
e 2 +e 2

8.2.7 Find the Laplace transform of the rectangular wave shown in the figure:

f(t)

1s
o b 2b 3b

Rectangular wave
640 Engineering Mathematics - I

Sol: The period of the given function is '2b'.

Hence L[f(t)]=
I 2h.
-2h, Je-Mj(/)dt=
I [ h .S1 2b
-2h.\ J1.e- j(t)dt+ J(-I)e-S1dt
1
l-e 0 l-e 0 b

hs -bs
- -
e 2 -e 2

I bs
=-tanh-
s 2
8.2.8 What is the Laplace transform of the staircase function (fig(a)) given by
f( t) = n+ I, np < t n+ 1)p for n = 0, 1, 2 ............ .
Sol. The easient way of getting the Laplace transform of the staircase function is to
consider it as the difference between the two functions shown in Fig(b) the

transform ofthe linear function (t + p) can easily be found by using first shifting,
P
second shifting and change of scale properties.
Thus, we have

f(t) f(t)

o P 2p 3p

(a) Staircse function (b) solution of the stair case function (c) Saw-tooth function.
Laplace Transforms 641

t
The function fl(t) = - is called a saw-tooth function (fig(c and its transform is
p
obtained as shown below.

1 P 1 1 [ -,I ]P
L[ft(t)] = _, fe-'I~d'= _.- _e_(_st_1)
1- e Ip p 1- e "P p S2
o 0

1- (1 + ps)e - P'
ps2(1-e-"P)

This can be simplified to

L[ I' (I)] -
JI -
~[1
P S2
+ ps - p 1
s(1-e- PS ) ..... (2)

Thus from (1) and (2), we get

L[f(t)]=~(~+P)_~(I+PS_ p- 1
p s- s P S2 s(1-e P")
1

Exercise 8(F)

1. If Lf(t) = <I>(s), Prove that


L[~II(t)] = s3<1>(s) - s2 <I>(s) - s<l>l(s) - <l>1I(s)

2. If L[~I(t)] = Tan-I (+), f(O) =,2 and ~(O) = -1 find L[f(t)]

3. A periodic function f(t) of period 2a is defind by


f(t) = t, for 0 .'S t .'S a,
= 2a - t, for a < t < 2a

show that L[f(t)] = 1 tanh 7 (as)


2
642 Engineering Mathematics - I

4. Iff(t) = t2, < t < 2 and f(t+2) = f(t)


find L[f(t)].

5. Iff(s) = L(f(t), Prove that

L[Sdl SJ'(U)duj == F~) and hence generalise the result.


o 0 s
8.2.9 Laplace transform of the unit step function
The unit step function orthe Heaviside function is defined as
H(t-a) = ift<a,
H(t-a) = lift < a
where a is a non-negative constant. In particular, when a = 0, we have
O,i/ 1 <
H(t)= [l,ifl>O

The unit step function is the basic building block of certain functions whose
knowledge greately increases the usefulness of the Laplace transformation.
It is easy to show that the Laplace transform of H(t-a) is given by
e- U \
L(H(t-a==-
s
00 a 00

L[H(t - a) == fe-vI H(t - a)dt = fe-VI O.dl + fe-\/l.d'


o 0 a
00

-e -sl e-as
s o s

. 1
In Particular, when a=O, we get L(H(t = -
s
8.2.10 Heaviside shift theorem: If Lf(t) = f(s), then L[f(t-a)H(t-a) = e-as f(s).
Proof: By defnition, we have
00 a 00

L[f(t - a)H(t - a) == fe- si f(t - a)H(t - a)dt == fe- si f(t - a)O.dt + fe- si f(t - a)l.dt
o 0 0
00 00

== fe -s(p+a) f(p )dp == e -sa fe -sp (p )dp


(consider p = t-a)
o 0
Laplace Transforms 643

8.2.1 Examples
Find the L.T. of the function

f(t) =1 } if 0 < t < 1t


1. =0 if1t<t<21t
=sint ift>21t

SoL Let us represent f(t) interms of unit step functions as flt)=H(t)-H(t-1t)+H(t-21t)


sint

1 -It' -2 It'
L[f(t)] = L[H(t)] - L[H(t-1t)] + L[H(t-21t)sint] = __ _e _ + _e_
s s s
2. Find L.T. oft2H(t-3)
Let us express <I>(t) = t 2 as a function of(t-3) by using Talor's series, which states
that

_ fI (x-a)2 II
f(x) - f(a) + (x-a) (a) + 2! f (a) + ................. + ............. .

(t 3)2
(t)=94-(t - 3).6+ .2
2!
Now, by the above result
L[f(t).H(t-a)] = L[9+6(t-3) + (t-3fH(t-3)]
= L[f(t-3).H(t-3)] = e-3S Lf(t) where f(t) = 9+6t+t2

L[t2H(t-a)] = e-3s [ -9 + -6? + 32 + ......... .]


s s- s

3. Express F(t) = (t-2)2 when t>2 and f(t) = 0 when 0<t<2 in terms of Heavis ide unit
step function and find its Laplace transform
Sol. It is easy to see that
F(t) = (t-2)2 = (t-2f.H(t-2) (since for t<2, H(t-2)=0 and for ~2, H(t-2)=1)
L[f(t).H(t-2)] = L[(t-2)2H(t-2)] = e-2s .Lf(t) where f(t) = t2

-2,\ 2!
=e .3
s
644 Engineering Mathematics - I

Exercise 8(G)

I. Find the Laplace tranform off(t) using the unit step function, where

I ifO<t<1
2 ifl<I<3
f(t) =
4 if3<t<4
-2 if 4 <I

I e-I 2e-31 6e-~1


Ans. - + - + - - - - -
J
[s s s s

2. Determine the Laplace transform of the following function:


6-2e-7t'i -4e-2""
(i) f(t) = 6-2H(t-1t) - 4H(t-21t) Ans.------
s

5 2-,
(ii) f(t) = e2I H(t-log5) Ans. - -
5-s
-(1\'
e
(iii) f(t) = (t-a)H(t-a) Ans. -2
s

e-2"(2 4s 4s 2 ) + +
(iv) f(t) = t2 H(t-2) Ans. --'-----.:3--...:...
S

1
(v) f(t) = H(t-e t ) Ans. -
s

(vi) f(t) = tH(t-2)

25 38 42 24]
(vii) f(t) = (l +2t-3t2 +4t3)H(t-2) Ans. e-2s - + - + - + -
[ s S2 S3 S4

1
(viii) f(t) = sint.H(t-1t) Ans. e-ltS --
S2 +1
Laplace Transforms 645

8.3.1 Inverse Laplace Transforms


If L[f(t)] = F(s) then f(t) is called the inverse Laplace transform of F(s) and is
denoted by L-I[F(s)] = f(t)
if fl and f2 are continuous functios on [0, 00] such that
L[fl(t)] L[f2(t)] or FI(s) = F2(s) (v- s>O) then fl(t) = fit) (v- t::::O)
=

Some of the important properties of the inverse laplace transforms are analogous to
the corresponding properties of Laplace transforms.
1. Linearity Property: If kl' k2 are any constants and F I(s) and Fis) are the Laplace
transforms of fl(t) and fit) respectives, then
L-I[kIFI(s) + k2Fis)] = klfl(t) + k/i t).
2. First shifiing property: If L- I [F(s)] = f(t), then

jor t > a
for t < a

3. Change of scale property: if L-' [F(s)] = f(t), then

4. Inverse Laplace transforms of derivatives: If L-'[F(s)] = f(t) then

5. Inverse Laplace transfonn of integrals: If L-'[F(s)] = f(t) then

6. Multiplication by's': If L-'[F(s)] = f(t) and f(O) = 0, then


L-[sF(s)] = fI(t)
7. Division by's': If L-I [F(s)] = f(t) then
646 Engineering Mathematics - I

. result can be extended to L- I ( - -


This F(S)
s"
I I I

= ff..ff(u)du".
00 0

(the proof is left to the student)


8.3.2 Table of inverse transforms

L[f(t)] L- I [F(s)]

L(1) = -
I
s
L- I (~) = 1

L(e-at) = --
s+a
1
L- I (_I
s+a
) = e-at

L(tn ) = -
S"+I
n!
1-1 ( I) + I)!
S'1+1
-
- (n
t"

if 'a' is a positive integer

L(sinat) =
S2
a
+a 2
rl ( 2
s +a
I 2) = ~a sinat
s
L(cosat) = L-I ( 2 S 2) = cosat
S2 +a 2 s +a

2)=~sinhat
a
L(sinhat) = L_ I ( 21
S2 _a 2 s -a a

s
L(coshat) = L- I ( s 2 ) coshat
S2 _a 2 S2 _a

8.3.3 Using the above results inverse Laplace transforms of some simple functions can b~
obtained as below.
Find the inverse Laplace transform of:
2s+3 3s+5 4._1_
I. 2 -- 3. 2
S2 +9 9s -25 4s+5
I 3 2s+ I s+3
5. 4s-5 6.- 7. (s + 1) 8~4
s+ 4 s +
Laplace Transforms 647

Sol. From the above table, we get

I.

2.

= 2cos3t + sin3t

3. I
L- ( 3~+5 )=L-I( 3s )+L-I( 2 5 )=~COSh(~)+~sinh(~)
9s- - 25 9s 2 - 25 9s - 25 3 3 3 3

4.

5.

6. L- (_3_)
I

s+4
= 3rl_1_ = 3e-
s+4
41

7. L-I 2s+1
1
-2L-I(_s
- 1
) + L-I(_l
2 )_" .
- .... cost+smt
(s- + 1) s- + 1 s +1

8. L-I( S+3) -I(


-1--
s- + 4
=L s ) +L
-1--
s- + 4
-I( 3 ) =cos2t+-sm21
-1--'
s- + 4
3 .
2

Exercise 8(H)

Find the invere Laplace transform of:

2 1 1 4s+ 15
1. -+-+--
S S3 S +4 5. 16s 2 _ 25
648 Engineering Mathematics - I

[2
Ans. 1.2+ - + e-4t 2. 3cos4t + sin4t 3. 2cosh3t+sinh3t
2
,4
4.1 + t 2 + -
24

Examples 8.3.4
Find the inverse Laplace transform of:
6s-4 I s-2
1. --"-2- - -
2. (s _ 3)3 3. (s _ 2)4 4. + -=-2- - -
2
s -4s+ 20 (s-l) s -4s+5

Sol.

4 L- I 1 ) + L-I (s - 2) = L-I( I ) + L- I ( (s - 2) )
. ( (S-2)4 (s2-4s+5) (s-2i s2-4s+4+1

=L -I( 1 ') ) + L
(s-2t
-l( S-2)
(S-2)2 + 12
= ')
et.t + e~t.cost

Exercise 8(1)

Find the inverse Laplace transform of:

s+4 (ii) s +6 4s+ 12


(i)---- (iii) -=----
S2 -4s+ 13 S2 -Ss +25 S2 +Ss + 16
Laplace Transforms 649

B.3.5 Method of partial fractions:


Whenever possible we exress the given function F(s) into the sum of linear or
quadratic partial fractions as
A Bs+C
F(s) = r + ? 2 r and then use the standard results to find the inverse L.T.
(s + a) (s- + a )

Examples (I) Find 4] for all s>O,


r l [ S3 +
s +s

Sol. By partial fraction expansion, we have

s+ 1 A Bs+C
---=-+---
S(S2 + I) S S2 +1
s+1 = A(s2+1) + (Bs+c)s = As2 + A + Bs2 + cs
comparing coefficients of different powers of s from both sides
O=A+B
)= c
:. I = A, B =-1
s+1 1 -s+1
. =-+--
.. S(S2 + I) S S2 + 1
By the linearity property of L-I, we have

- - = L-l(I)
L-l(S+l) - - L-1(-s- ) + L-1(-I- ) =I -cost+smt
.
S3 + s ' S S2 + 1 S2 + I

2S2 -6s+5
2. If L[f(t)] = S3 + 6s2 + lIs -6 find [f(t)]

Sol. By partial fractions we have

J(s) = 2S2 -6s+5 =_A_+_B_+_C_


S3 -6s 2 +Ils-6 (s-l) (s-2) (s-3)

:. 2S2 -6s + 5 = A(s -2)(s -3)+ B(s -l)(s -3)+C(s -l)(s -2)
put s = I we get
I =2A A= 1/2
put s = 2
B=-1
650 Engineering Mathematics - I

Similarly s = 3 gives
C = 5/2

.
.. L-I[f'( S.)]_
--I L- I - -
I - L- I ( -I- ) +-
5 L- I ( -A- )_ 1 1 -e 21 +-e
--e 5 31
. 2 (s-I) 8-2 2 8-3 2 2

3s+ I
3. If Lf(t) = ( )( 2 I ' find f(t)
s-I s + )
Sol. By partial fractions we have

j(s)=~+ Bs+C
s -1 S2 + I

:. Solving for A, B C
we have A = 2, B = -2, C = -I

L- I [j(s)] = 2C l _1__ 2CI _s_ + L- I _)_1_


S -1 S2 + I s- + I

= 2e t - 2cost + sint
s+2
4. If L[f(t)] = (s + 3)(s + 1)3 find f(t)

Sol. j(s) = s+2 =~_I__ ~_I_+~ I +1 1


(s+3)(s+I)3 8s+3 8s+1 4(8+1)2 2(8+1)3
(after expressing the function in partial fractions)

:.rl[f(8)]=~rl-I--~rl-I-+ 1 ? + I 1
8 8+38 8+14(8+1)- 2(8+1)3

=~e-31 - ~e-I -t ~/e-I +t 2e-1 = ~[(2t2 + 21 -1)e-' + e-31 ]


8 8 4 8
(S2 + 2s - 42)
5. If L[f(t)] = (S2 + 28 + 5)(8 2 + 2s + 2) find f{t)

as+b c8+d
Sol: Let f(s) = 82 + 28 + 5 + S2 + 28 + 2

:.S2 + 28 -4 =(a8 +b)(8 2 + 28 + 2)+ (cs + d)(S2 + 2s + 5)


Equating the coefficients of like powers of s we get a = 0, b = 3, c = 0, d = -2.
Laplace Transforms 651

3 2 3 2
:f(s)= s2+2s+5 s2+2s+2 (s+I)2+22 (s+I)2+12

:.L -I( f(s)]=3L -I( 12 ? ) -2L


(s+l) +2-
-I[ 12 ?
(s+l) +1-
1

2s
6. If L[f(t)] = 4s2 + 16 ' find f(t)

Sol. By change of scale property we have

L-I[ 2
2s ] =-cos
1 21 since L- I [ 2 S
s +16
] = cos4t
4s +16 2

e- 5
7. If L[f(t)] = (s _ 2)4 find f(t)

Sol. We know rl[ 1


(s - 2)4
1 L-I[_I]
= e 21
S4
= e2/.~3! = ~t3e21
6

Thus by second shifting property

L- I
[
e -5s
(S-2)4
1={Ig -(I - 5)3e2(t - 5) ift>5
if 1 < 5

Exercise 8(J)

Determine the inverse Laplace transform of each of the following functions.

10g(~)
3 s 2s+ 1
(i) (ii) (iii) (iv)
s+2 s2+2s+6 s(s + 1) s+2

s-5 e -2 .. S2
(v) (vi) (vii)
S2 +6s+ 13 s(s + 1) (S2 + 9)(S2 + 16)
652 Engineering Mathematics - I

2S2 + s -I 0
(viii) -(ix) log (I + Sl2 ) (x) 4
(s - 4)(S2 + 2s + 2) (8 + 64)

s
(xi) (xii)
s(s + I)(s + 2)(s + 3) (s4+s2+1)

2
8 + lOs + 13 s
(xiii) (xiv) 2
2 (8 + 1)(s2 +4)
8 -I 0(8 - 5s + 6)

8+29 5s+3
(xv) (xvi)
(S+4)(S2 +9) (s -1)(s2 + 2s + 5)

48+4 S2 + 1
(xvii) (xviii)
(8 _1)2(8 + 2) s3 +3s 2 +2s

Ans. . ) -1 (-2/
( IV e -e -3/)
t

(v) e-3tcos2t - 4e-3tsin2t (vi) 4(1 - e-2t)(t - 2) (vii) (~Sin4t -%Sin3t)

2 I
(viii) e4t - e-t(cos2t+2sint) (ix) -(I-cost)
t
(x) "8 sin2t sinh2t

. I
(XI)
6 2
I _/ 1 -2/ 1 -3/
-.l--e +-e --e
2 6
. J32(. TJ5 t.sln'ht]2"
(XII) Sin

(xiii) 12et - 37e2t + 25e3t (xiv) "3I (cost - cos2t) (xv) e--4t - cos3t + "35 sin3t

(xviii) .! - 2- et + ~ e-2l
2 2
Laplace Transforms 653

8.4 Convolution Theorem


If L[f(t)] = F(s) and L[g(t)] = G(s) the inverse transfonn of F(s). G(s) denoted by
H(t), is called the convolution of f and g is defind as given below.
I

H(t) = (f*g)(t) = f(t-u)g(u)dll,vtzO ..... ( I)


o
The convolution operation * has the following properties.
(f*g)(t) = (g*f)*t), v t ~ 0 (commutative)
[f*(g*h)](t) = [(f*g)*h](t) v- t ~ 0 (associative)
[1'*(g+h)](t) = (f*g)(t) + (f*h)(t) v- t ~ 0 (dstributive)
8.4.1 Theorem: Let f(t) and get) be the inverse transforms of F(s) and G(s) respectively,
and satify the hypothesis of the existence theorem (8.10.2) then the the inverse
transform h(t) of the product F(s) G(s) is the convolution of f(t) and get).
I

h(t) = (f*g)(t) = fl(t-u)g(ll)du


o
I

i.e. L-'[F(s)G(s)] = fl(t - u)g(u)du ..... (i)


o
Proof: It is sufficient if we show that

F(s) G(s) ~ L[ 1f (l- u)g(u)du 1

Now F(s)G(s) ~ [fe" f( v)dv] [V g(u)du ]

=
00
rrllo
Ile-(U+V)S J(v)g(u)dvdu = le-S(V+U) J(V)dV]g(U)dU'
where the integration is extended over the first quadrant (u ~ 0, v .:=:: 0) in the
uv-plane. We now introduce a new variable 't' in the inner integral of the last expression
by taking v = t-u so that u + v = t and dv = dt (u being fixed during this integration).
Thus

F(s)G(s) = rrllule->t J(t - U)dt]g(U)dU = Ile->t J(t -u)g(u)dtdu


0 U
654 Engineering Mathematics - I

al a

F(s)G(s)= fTIe-SI!(t-u)g(u)du]dt= fe- jlr{t-u)g(u)du}tt


s, I
00 0 0

8.4.2 Examples

(I) Evaluate ~ r' [ s2(s1+ I )2l using the convolution theorem

Sol. we have L-'C, )~I and r'((S~I)' )~/e-'


By the convolution theorem

I 2
=L- [ 2( U
1 y]= J(ue- -{t-u}du= J(ut-u )e- )du
U

s s+1 0 0

= [(ut -u2)e-u - (t-2u)e-u + (-2)(-e-U) = t-e-t + 2e- 1 + t-2

(2) ~valuate L- I
[ 2 S 2 2] using the convolution theorem.
(s +2 )

s s
Sol. Let F(s) = ~4 ' G(s) = - 2 - and Let ~s) = F(s) G(s)
s + s +4

_I s
f(t) = L-I [F(s)] = L (-2-) = cos2t
s +4

1
g(t) = L-l[G(s)] = r-I (-2-_) = ! sin2t
s +4 2

f(u).;: cos2u CIld g(t-u) = 2"1 sin2(t -u)


Laplace Transforms 655

I I
C l [tp(S)] = JCOS 2u -sin 2(1 - u) du
o 2
I

= ~ JSin 2t + sin 2(t - 2u )dll


40

=~[sin2t.U+~COS2(t-2U]1 ~tsin2u
4 2 04

(3) Evaluate L-
I
[ ~ 1using convolution theorem.
s s+4

1
Sol. Let F(s) = ~ G(s) = - and choose
vs+4 s
i...s) = F(s).G(s)

.. f(t) = L- [F(s)] =
I
r
I
~
= e-4t CI_1_ =e-41
s~
t~ = r;;
r- 1
e-
41

(s + 4) 2 2 '\I1tl

1
g(t) = L-1[G(s)] = L- 1 (-) =1
s
e --4u
f(u) = rand g(t-u) = 1
'\I1tU

By convolution theorem
1
Je-
I -4u 2!i
x2
L-1[$(s)] = Jer- du = r dx
o '\I 1tU '\I 1t 0

where 4u = .xl 2du =xdx

(where erf(t) J
I

= 2r e- x
'\I1t 0
2
dx

(erft stands for error function)


656 Engineering Mathematics - I

Exercise 8(K)

Using the convolution theorem find the inverse Laplace transforms of

(v) logs+2 (vi) 1 ?


S +1 (s + 1)(s- + 1)

1 1
(ii) "3 (2sin2t-sint) (iii) "3 tsint
-/I -211
e -e du
(iv) I
o
u
2
I f4 -cost-smt
(vi) -Le . ]

8.4.3 Applications to differential equations


Suppose we wish to find the particular solution of the differential equation
lay" + /3y' + yy = J(t) ..... (1)
which satisfiestheinitial conditionsy(O) Yo and yeO) = Yo =
Taking the Lapa1ce transform on bothsides of (l) and using its linearity, it follows
that
aL(y") + /3L(y') + yL(y) = L[J(t)]
but L(y") = s2L(y) -sy(O) _yl(O)
L(y') = sL(y) - yeO)
a[s2L(y) - sy(O) - yl(O)] + /3[sL(y) - yeO)] + yL(y) = L[f(t)]
and hence
(as 2 + /3s + y)L(y) - L[f(t)] + (as + /3)Yn + ayJ
=> L(y) = L[/(t)] + (as + /3)yo + aYb
as 2 + /3s +y

The function L[f(t)] is a specific function of s since f(t) is known; and since a, /3, y.
}'0 and y J are constants, L(y) is completely known as a function of s. The inverse
Laplace transform of L(y) will be the solution of the given diff. eqn.
(Here it is assumed that the functions f( t), y, y' and y" must have Laplace transforms).
Laplace Transforms 657

Solved Examples

8.4.4
2
' d th
Fm e soiutlOn d2
' 0f - y + -dy - 2Y = smt
' w h'IC h satls e Imtla I cond'ItlOI1S
'files t h'" , y = 0,
dt dt
y' = 0 when t = 0
Sol. Applying the Laplace transforms to both sides of the given equation and by linearity,
we have
L(yW) + L(y') - 2L(y) = L(sin t)

1
As we know that L(sint) = -2-1 ;s > 0
s +
1
[s2L(y) -sy(O) - y' (0)] + [sL(y) - yeO)] -2 L(y) = -2-1
s +

Given conditions yeO) = 0, y' (0) = 0


1
=> s2L(y) + sL(y) -2 L(y) = -2-
s +1
1
L(y)= ? =-----:---
(s- + S - 2)(S2 + 1) (s -IXs + 2)(S2 + 1)

we have the partial fraction expansion


1 A B C . +D
- - - - - - - : - - = - - + - - + -":--
(s -1)(s + 2)(S2 + I) s -I s + 2 S2 + I

where A =.! B =-=! ,C =-=! and D = - 3


6' 15 10 10
Therefore we get
1 1
yet) = L- 1 - - - - - - 1
(s -1)(s + 2)(S2 + 1)

as the solution to the given initial value problem,


658 Engineering Mathematics - I

8.4.5 Example
Solve the initial value problem

y"-3y'+2y=e 31 .y=l,y'=0 wheret=O

Sol. Applying Laplace transform to both sides of the differential equation, we get
1
[s2Ly(t) - sy(O) - y' (0)] -3 [sLy(t) - yeO)] +2 L(t) = L(e3!) = -
s-3
Applying the given conditions, we get
1
(s2 - 3s + 2) Ly(t) = s-3 + -
s-3
s-J I
:. L[y(t)] = (S2 _ 3s + 2) + (s - 3)(S2 - 3s + 2)

1 s-3
:. L(y(t)] = (s _ 3)(S2 _. 3s + 2) + (8 2- 3s + 2)

1 8-3
--------+------ .... (1)
(8-1)(s-2)(s-3) (s-I)(8-2)

To find yet), we expand each term on the right hand side of (I) in partial functions,
Thus
ABC
------=----+---
(s-I)(s-2)(s-3) s-1 s-2 s-3

1 = A(s-2) (s-3) + B(s-l) (s-3) + C(s-I)(s-2)


I
substituting s = I gives A = 2" ;similarly substituting s = 2 gets B = -1 and substituting

I
s = 3 gives c ="2 hence

- - -1- - - = 1
--- +----
(s-I)(s-2)(s-3) 2(8-1) s-2 2(8-3)
Similarly we can also write
s-3 D E
----=-+--
(s-I)(s-2) s-1 s-2
Laplace Transforms 659

s-3 = D (s-2) R(s-I)


Taking s =: 2, we get E =:-1
similarly s = I we get D = 2

I I I I I
Hence L[f(t)] = --+ +---
2(s-l) s-2 2{s-3) s-I s-2

221
- - - - + -,-----
- 2( s - I ) s - 2 2{s - 3)

Example 8.4.6
Solve yll + 3yl + 2y = 2t3 + 2t + 2 with yeO) = 2, yl(O)

Sol. Applying Laplace transform to both sides of the given equation we get
L(yll) + 3L(yl) + 2L(y) = 2L(t2) + 2L(t) + 2

? I 4 2 2
[s-Ly(t) - sy(O) - yeO)] + 3 [sL[y(t)] - yeO)] + 2Ly(t) = 3 + -2 + - .... (1)
_ s s s

using the given conditions (1) gives


3
2 3 2 () 2(2+s+s2) 2 6 2(S4 +3s +S2 +s+2)
( s + s+ )Ly t = 3 + s+ = 3
S S

By partial fractions,

yet) = 3L-I(~) -2r L\ ) 2L-ILI


1
+ 3) - L-Ie ~ 2)
yet) =3 - 2t + t 2 - e-2t
660 Engineering Mathematics - I

Exercise 8(1)

Using Laplace transform method solve the following differential equations with
the given conditions.
I. yll + 2yl - 3y = 0 at, y = 0, yl = 4 when x = 0
2. yll + 4y = 4t at y = I, yl = 0 when t = 0
3. yll - 2yl + Y = et at y = 2, yl = -I when t = 0
I
4. yll + k2y = coskt at y = 0, yl = "2 when t = 0

5. yll- y = 1 at y = -I, yl = I when t = 0


6. yll + 2yl + 3y = 3e-t sint; yeO) = yl(O) = 0
7. xii + 4xl = -8t given x(O) = 0, xl(O) = 0
8. yll + 9y = 18t given yeO) = 0, yl(O) = 0

9. yll + Y = sint given yeO) = -I, yl(O) = "21


1 I I
Ans.l. y = eX - e-3x 2. y= - t- - sint + cost 3. y = 2et - tet + -2 t2et
4 8

1. k 1 .
4. Y = 2k Sill t + "2 tSll1t 5. y = -1 + et - cosht 6. y = 3e-tsint

-I 1 I 2
7. x = 8 +"2 t - t2 + "8 e-4t 8. y = 2t - "3 sin3t

Das könnte Ihnen auch gefallen